GATE 2022 General Aptitude - Guide
GATE 2022 General Aptitude - Guide
Language : English
Editor’s Name : Vinit Garg
Copyright © : 2021 CLIP
No part of this book may be reproduced in a retrieval system or transmitted, in any form or by any
means, electronics, mechanical, photocopying, recording, scanning and or without the written
permission of the Author/Publisher.
Published by :
Career Launcher Infrastructure (P) Ltd.
A-45, Mohan Cooperative Industrial Area, Near Mohan Estate Metro Station, New Delhi - 110044
Marketed by :
G.K. Publications (P) Ltd.
Plot No. 9A, Sector-27A, Mathura Road, Faridabad, Haryana-121003
ISBN : 978-93-90820-70-2
Printer’s Details : Made in India, New Delhi.
(v)
IIT Institutes
(vii)
Preface
The Graduate Aptitude Test in Engineering (GATE) is an online exam conducted by the IITs
for admissions to PG courses in IITs, IISc Bangalore, NITs and many state run universities as
well as private universities. Also there are more than 37 PSUs that use GATE score for
recruitments. A large number of corporates are also using GATE score as a tool to screen
students for placements.
GK Publications is well known as the ‘‘publisher of choice’’ to students preparing for GATE and
other technical test prep examinations in the country. We published first set of books in 1994
when GATE exam, both objective and conventional, was conducted in the paper and pencil
environment, and used as a check point for entry to post graduate courses in IITs and IISCs.
At that time, students had little access to technology and relied mainly on instructor led learning
followed by practice with books available for these examinations.
A lot has changed since then!
Today, GATE is conducted in an online only mode with Multiple Choice Questions, Multiple
Select Questions and Numerical Based Questions. The score is valid for three years and is used
not only for post graduate courses but is also used by major PSUs for recruitment. Today’s
students have easy access to technology and the concept of a monologue within the classroom
has changed to dialogue where students come prepared with concepts and then discuss topics.
They learn a lot of things on the go with their mobile devices and practice for mock tests online.
We, as a leading publisher of GATE books, have also embraced change. Today, our books are no
more guides only but come with a fully supported mobile app and a web portal. The mobile App
provides access to video lectures, short tests and regular updates about the exam. The web
portal in additional to what is available on the App provides full length mock tests to mimic the
actual exam and help you gauge your level of preparedness. The combination of practice content
in print, video lectures, and short and full length tests on mobile and web makes this product
a complete courseware for GATE preparation.
We also know that improvement is a never ending process and hence we welcome your
suggestions and feedback or spelling and technical errors if any. Please write to us at
[email protected]
We hope that our small effort will help you prepare well for the examination.
We wish you all the best!
(viii)
About GATE
The Graduate Aptitude Test in Engineering (GATE) conducted by IISc and IITs has emerged as one of
the bench mark tests for engineering and science aptitude in facilitating admissions for higher education
(M.Tech./Ph.D.) in IITs, IISc and various other Institutes/Universities/Laboratories in India. With the
standard and high quality of the GATE examination in 27 disciplines of engineering and science,
Humanities and Social Sciences subjects, it identifies the candidate's understanding of a subject and
aptitude and eligibility for higher studies. During the last few years, GATE score is also being used as
one of the criteria for recruitment in Government Organizations such as Cabinet Secretariat, and
National/State Public Sector Undertakings in India. Because of the importance of the GATE examination,
the number of candidates taking up GATE exams has increased tremendously. GATE exams are conducted
by the IITs and IISc as a computer based test having multiple choice questions and numerical answer
type questions. The questions are mostly fundamental, concept based and thought provoking. From
2017 onwards GATE Exam is being held in Bangladesh, Ethiopia, Nepal, Singapore, Sri Lanka and
United Arab Emirates. An Institute with various nationalities in its campus widens the horizons of an
academic environment. A foreign student brings with him/her a great diversity, culture and wisdom to
share. Many GATE qualified candidates are paid scholarships/assistantship, especially funded by Ministry
of Human Resources Development, Government of India and by other Ministries. IIT, Kharagpur is
the Organizing Institute for GATE 2022.
Why GATE?
Admission to Post Graduate and Doctoral Programmes
Admission to postgraduate programmes with MHRD and some other government scholarships/
assistantships in engineering colleges/institutes is open to those who qualify through GATE. GATE
qualified candidates with Bachelor’s degree in Engineering/Technology/Architecture or Master’s degree
in any branch of Science/Mathematics/Statistics/Computer Applications are eligible for admission to
Master/Doctoral programmes in Engineering/Technology/Architecture as well as for Doctoral
programmes in relevant branches of Science with MHRD or other government scholarships/
assistantships. Candidates with Master’s degree in Engineering/Technology/Architecture may seek
admission to relevant Ph.D programmes with scholarship/assistantship without appearing in the GATE
examination.
Financial Assistance
A valid GATE score is essential for obtaining financial assistance during Master’s programs and direct
Doctoral programs in Engineering/Technology/Architecture, and Doctoral programs in relevant
branches of Science in Institutes supported by the MHRD or other Government agencies. As per the
directives of the MHRD, the following procedure is to be adopted for admission to the post-graduate
programs (Master’s and Doctoral) with MHRD scholarship/assistantship. Depending upon the norms
adopted by a specific institute or department of the Institute, a candidate may be admitted directly
into a course based on his/her performance in GATE only or based on his/her performance in GATE
and an admission test/interview conducted by the department to which he/she has applied and/or the
(ix)
candidate’s academic record. If the candidate is to be selected through test/interview for post-graduate
programs, a minimum of 70% weightage will be given to the performance in GATE and the remaining
30% weightage will be given to the candidate’s performance in test/interview and/or academic record,
as per MHRD guidelines. The admitting institutes could however prescribe a minimum passing
percentage of marks in the test/interview. Some colleges/institutes specify GATE qualification as the
mandatory requirement even for admission without MHRD scholarship/assistantship.
To avail of the financial assistance (scholarship), the candidate must first secure admission to a program
in these Institutes, by a procedure that could vary from institute to institute. Qualification in GATE
is also a minimum requirement to apply for various fellowships awarded by many Government
organizations. Candidates are advised to seek complete details of admission procedures and availability
of MHRD scholarship/assistantship from the concerned admitting institution. The criteria for
postgraduate admission with scholarship/assistantship could be different for different institutions.
The management of the post-graduate scholarship/assistantship is also the responsibility of the
admitting institution. Similarly, reservation of seats under different categories is as per the policies
and norms prevailing at the admitting institution and Government of India rules. GATE offices will
usually not entertain any enquiry about admission, reservation of seats and/or award of scholarship/
assistantship.
PSU Recruitments
As many as 37 PSUs are using GATE score for recruitment. It is likely that more number of PSUs
may start doing so by next year. Below is the list of PSUs:
MDL, BPCL, GAIL, NLC LTD, CEL, Indian Oil, HPCL, NBPC, NECC, BHEL, WBSEDCL, NTPC,
ONGC, Oil India, Power Grid, Cabinet Secretariat, Govt. of India, BAARC, NFL, IPR, PSPCL, PSTCL,
DRDO, OPGC Ltd., THDC India Ltd., BBNL, RITES, IRCON, GHECL, NHAI, KRIBHCO, Mumbai
Railway Vikas Corporation Ltd. (MRVC Ltd.), National Textile Corporation, Coal India Ltd., BNPM,
AAI, NALCO, EdCIL India.
Important :
1. Admissions in IITs/IISc or other Institutes for M.Tech./Ph.D. through GATE scores shall be
advertised separately by the Institutes and GATE does not take the responsibility of admissions.
2. Cabinet Secretariat has decided to recruit officers for the post of Senior Field Officer (Tele) (From
GATE papers of EC, CS, PH), Senior Research Officer (Crypto) (From GATE papers of EC, CS,
MA), Senior Research Officer (S&T) (From GATE papers EC, CS, CY, PH, AE, BT) in the
Telecommunication Cadre, Cryptographic Cadre and Science & Technology Unit respectively of
Cabinet Secretariat. The details of the scheme of recruitment shall be published in National
Newspaper/Employment News by the concerned authority.
3. Some PSUs in India have expressed their interest to utilize GATE scores for their recruitment
purpose. The Organizations who intend to utilize GATE scores shall make separate advertisement
for this purpose in Newspapers etc.
(x)
Eligibility Criteria for GATE 2022
Expected
Description of
Degree/Program Qualifying Degree/Examination Year of
Eligible Candidates
Completion
In case a candidate has passed one of the qualifying examinations as mentioned above in 2020 or
earlier, the candidate has to submit the degree certificate / provisional certificate / course completion
certificate / professional certificate / membership certificate issued by the society or institute. In
case, the candidate is expected to complete one of the qualifying criteria in 2022 or later as mentioned
above, he/she has to submit a certificate from Principal or a copy of marks card for section A of AMIE.
(xi)
Certificate From Principal
Candidates who have to submit a certificate from their college Principal have to obtain one from his/
her institution beforehand and upload the same during the online submission of the application form.
GATE Structure
Structure of GATE
GATE 2022 will be conducted on 27 subjects (papers). Table below shows the list of papers and paper
codes for GATE 2022. A candidate is allowed to appear in ANY ONE or UP TO TWO papers of the
GATE examination. However, note that the combination of TWO papers in which a candidate can
appear MUST be from the pre-defined list as given in Table. Also note that for a paper running in
multiple sessions, a candidate will be mapped to appear for the examination in one of the sessions
ONLY.
List of GATE Papers and Corresponding Codes
Note: Environmental Science and Engineering (ES) and Humanities and Social Sciences (XH) are two
new papers introduced in GATE 2022.
(xii)
*XE Paper Sections Cod **XH Paper Code ***XL Paper Code
e Sections Sections
Engineering
A Reasoning and B1 Chemistry P
Mathematics Comprehension (Compulsory)
(Compulsory) (Compulsory)
(15 marks) (25 marks)
(25 marks)
Any TWO optional Any ONE optional Any TWO optional
Sections Section Sections
(2x35 = 70 marks) (60 marks) (2x30 = 60 marks)
Atmospheric and H
Oceanic Sciences
*XE (Engineering Sciences), **XH (Humanities & Social Sciences), ***XL (Life Sciences), papers are
of general nature and will be comprised of Sections listed in the above table
Note: Each subject/paper is of total 100 marks. General Aptitude (GA) section of 15 marks is common
for all papers. Hence remaining 85 marks are for the respective subject/paper code.
Combination of Two Papers Allowed to Appear in GATE 2022
(subject to availability of infrastructure and schedule)
Code of The First Codes of Papers Allowed as The Second
(Primary) Paper Paper
AE XE
AG ES
AR CE
BM BT / XL
BT BM / XL
CE AR / ES
CH CY / PE / XE
CS MA
CY CH / XL
EC IN / PH
EE IN
ES AG / CE
EY XL
GG MN / PE / PH
IN EC / EE / PH
MA CS / PH / ST
ME XE
MN GG / XE
MT PH / XE
PE CH / GG / XE
PH EC / GG / IN / MA / MT / ST
PI XE
ST MA / PH
TF XE
XE AE / CH / ME / MN / MT / PE / PI / TF
XH ----
XL BM / BT / CY / EY
(xiii)
General Aptitude Questions
All the papers will have a few questions that test the General Aptitude (Language and Analytical
Skills), apart from the core subject of the paper.
(xiv)
Question Papers other than GG, XE, XH and XL
These papers would contain 25 questions carrying 1-mark each (sub-total 25 marks) and 30 questions
carrying 2-marks each (sub-total 60 marks) consisting of some MCQ type questions, while the remaining
may be MSQ and / or NAT questions.
(xv)
XL Paper (Life Sciences)
A candidate appearing in the XL paper has to answer the following:
• GA – General Aptitude carrying a total of 15 marks.
• Section P–Chemistry (Compulsory): This section contains 15 questions carrying a total of
25 marks: 5 questions carrying 1-mark each (sub-total 5 marks) and 10 questions carrying
2-marks each (sub-total 20 marks). Some questions will be of MSQ and/or numerical answer type
while remaining questions will be MCQ type.
• Any two of XL Sections Q to U: The choice of two sections from Q to U can be made during the
examination after viewing the questions. Only TWO optional sections can be answered at a time.
A candidate wishing to change midway of the examination to another optional section must first
choose to deselect one of the previously chosen optional sections (Q to U). Each of the optional
sections of the XL paper (Sections Q through U) contains 20 questions carrying a total of
30 marks: 10 questions carrying 1-mark each (sub-total 10 marks) and 10 questions carrying
2-marks each (sub-total 20 marks). Some questions will be of MSQ and/or numerical answer type
while remaining questions will be MCQ type.
GATE Score
After the evaluation of the answers, the actual (raw) marks obtained by a candidate will be considered
for computing the GATE Score. For multi-session papers (subjects), raw marks obtained by the
candidates in different sessions will be converted to Normalized marks for that particular subject.
Thus, raw marks (for single session papers) or normalized marks (for multi-session papers) will be
used for computing the GATE Score, based on the qualifying marks.
Mtg – Mqg
Mij ( Mij – Miq ) Mqg
M ti – Miq
where
Mij : is the actual marks obtained by the j th candidate in ith session
Mtg : is the average marks of the top 0.1% of the candidates considering all sessions
Mqg : is the sum of mean and standard deviation marks of the candidates in the paper
considering all sessions
Mti : is the average marks of the top 0.1% of the candidates in the ith session
Miq : is the sum of the mean marks and standard deviation of the ith session
(xvi)
Calculation of GATE Score for All Papers
For all papers for which there is only one session, actual marks obtained by the candidates will be
used for calculating the GATE 2022 Score. For papers in multi-sessions, normalized marks will be
calculated corresponding to the raw marks obtained by a candidate and the GATE 2022 Score will be
calculated based on the normalized marks.
The GATE 2022 score will be computed using the formula given below.
( M – Mq )
GATE Score = Sq (St – Sq )
( Mt – Mq )
where
M : marks obtained by the candidate (actual marks for single session papers and
normalized marks for multi-session papers)
Mq : is the qualifying marks for general category candidate in the paper
Mt : is the mean of marks of top 0.1% or top 10 (whichever is larger) of the candidates
who appeared in the paper (in case of multi-session papers including all sessions)
Sq : 350, is the score assigned to Mq
St : 900, is the score assigned to Mt
In the GATE 2022 the qualifying marks (Mq) for general category student in each subject will be
25 marks (out of 100) or , whichever is larger. Here is the mean and is the standard
deviation of marks of all the candidates who appeared in the paper.
After the declaration of results, GATE Scorecards can be downloaded by the GATE qualified candidates
ONLY.
The GATE 2022 Committee has the authority to decide the qualifying mark/score for each GATE
paper. In case of any claim or dispute with respect to GATE 2022 examination or score, the Courts
and Tribunals in Mumbai alone will have the exclusive jurisdiction to entertain and settle them.
(xvii)
GATE Syllabus
Verbal Aptitude
Basic English Grammar: tenses, articles, adjectives, prepositions, conjunctions, verb-noun agreement, and
other parts of speech.
Basic Vocabulary: words, idioms, and phrases in context, Reading and comprehension, Narrative sequencing.
Quantitative Aptitude
Data Interpretation: data graphs (bar graphs, pie charts, and other graphs representing data), 2-and
3-dimensional plots, maps, and tables.
Numerical Computation and Estimation: ratios, percentages, powers, exponents and logarithms,
permutations and combinations, and series, Mensuration and geometry, Elementary statistics and probability.
Analytical Aptitude
Logic: Deduction and induction, Analogy, Numerical relations and reasoning.
Spatial Aptitude
Transformation of shapes: translation, rotation, scaling, mirroring, assembling, and grouping Paper folding,
cutting, and patterns in 2 and 3 dimensions.
(xix)
Verbal Aptitude
1
C HAPTER English Grammar
ERRORS IN USE OF ARTICLES Use of ‘The’
ARTICLES 1. When we speak of a particular person or thing
already referred to.
Indefinite Definite e.g. I dislike the follow.
(A, An) (The) 2. When a singular noun represents a whole class.
Use of ‘An’
e.g. The mango is considered the king among fruits.
1. Before words beginning with vowel sounds
[a, e, i, o, u are called vowels, others are consonants]. 3. With name of
e.g. an apple, an egg, an owl. (i) gulfs, rivers, oceans, islands and mountains e.g.
the Himalayas, the Indian ocean, the Persian
2. Before words beginning with silent ‘h’ but sounds
Gulf, the Red sea, the Andaman islands, the
as vowel.
Brahmaputra river.
e.g. an hour, an honourable man, an heir, an
honest man. (ii) Certain books
3. F, H, L, M, N, R, S, X are letters that are not e.g. the Vedas, the Puranas, the Bible, the
vowels but begin with vowel sound ‘M’ has the Ramacharitmanas.
sound of ‘em’. So, ‘an’ is used before abbreviations (iii) Musical instruments
beginning with vowels of these letters. e.g. the flute, the violin, the tabla, the trumpet.
e.g. an M.L.A., an R.A.F., an N.C.C. officer, (iv) The inventions
an F.I.R., an X-ray, an H.E. school, an S.P.
e.g. I hate the telephone for its constant ringing.
Use of ‘A’
(v) Parts of body
1. In the sense of one.
e.g. He was wounded in the leg.
e.g. He couldn't speak a word to save himself.
(vi) Religious groups
With ‘one’ (since ‘one’ begins with sound of ‘w’)
e.g. the Sikhs, the Hindus, the Parsees.
e.g. a one-man show, a one-rupee note.
2. Before words beginning with consonant sound (vii) Names enforcing law
e.g. a boy, a box, a dog. e.g. the Police, the Navy, the Air Force.
3. With Vowel letters having consonant value. (viii)Political parties
e.g. a university, a unique article, a euphenism, e.g. the Congress, the Janata Pary, the B.J.P.
a unit, a European language (ix) Aeroplanes, ships, trains etc
4. With units and rate (per). e.g. the Makalu (aeroplane), the Vikrant (Ship),
e.g. He earns rupees five hundred a month. the Rajdhani express (train).
5. In exclamatory expressions before singular (x) Before names of an empire, dynasty or
countable nouns. historical event
e.g. What a pretty girl ! e.g. the Gupta dynastry, the Old Stone Age, the
First World War, the American Revolution.
6. When two subjects or articles are thought of as a
single unit. (xi) Clubs, foundations etc.
e.g. He was ready with a cup and saucer. e.g. the Lion's Club, the Ford Foundation.
7. With certain expressions of quantity. (xii) Before common nouns denoting unique things
e.g. a lot of , a dozen, a great deal of, a couple. e.g. the sun, the sky, the earth, the worl.
8. With a person's name to indicate that the person (xiii) With superlatives
is perhaps unknown to the person addressed. e.g. He is the best boy in the class.
e.g. A Mr. roy is at the door. (xiv) With ordinals
9. With a special meal (to celebrate something or in e.g. He took the first taxi that came his way.
someone's honour). (xv) Before the comparative degree
e.g. I called my friends to a lunch to celebrate my success.
e.g. The more they get, the more they want.
10. To make a common noun of a proper noun.
(xvi) Before an adjective when the noun is
e.g. This man is ‘a second Newton’. understood
(This phrase means ‘a philosopher as great as e.g. The poor would favour him.
Newton’)
1.2 English Grammar
8. Reflexive pronouns are never used with verbs keep, PARTS OF SPEECH
conceal, quality, spread, rest, stay.
e.g. I stayed away from my class. The parts of speech explain how a word is used in a
He qualified in the test. sentence.
9. When first, second and third person singular There are eight main parts of speech (also known as
pronouns (I, you and He) are used together, they word classes): nouns, pronouns, adjectives, verbs,
are placed in the order : You, he and I. adv erbs, prepositions, conju nctio ns and
e.g. You he and I are neighbours. interjections.
In case of plural pronouns, ‘we’ comes first, then Most parts of speech can be divided into sub-classes.
‘you’ and then ‘they’.
Prepositions can be divided into prepositions of time,
e.g. We, you and they must work together. prepositions of place etc. Nouns can be divided into
But if we have only two persons including first, proper nouns, common nouns, concrete nouns etc.
then first person pronoun is written first.
e.g. I and Sanjeev have done this job. It is important to know that a word can sometimes be
in more than one part of speech. For example with
10. ‘Who’ denotes subject and ‘whom’ denotes object.
the word increase.
e.g. Who do you think did the job ?
11. ‘Whose’ is used for persons and ‘which’ for lifeless Increase can be a verb e.g. Prices increased
objects. and increase can also be a noun e.g. There was an
e.g. This is the table which I was talking about. increase in the number of followers.
12. ‘Which’ conveys additional information and ‘that’ The eight main parts of speech in English are:
explains a certain thing.
e.g. I will tell you the first thing which I remember.
1. NOUN
13. The following expressions usually take ‘that’ in place A noun is the name of a person, place, thing, or
of ‘who’ or ‘which’. idea.
Only, Any, It is, All, Superlatives. man... Butte College... house... happiness
e.g. He is the only man that can do it. A noun is a word for a person, place, thing, or idea.
Any man that listens to you is a fool. Nouns are often used with an article (the, a, an), but
14. ‘Each other’ is used for two; ‘one another’ for more not always. Proper nouns always start with a capital
than two. letter; common nouns do not. Nouns can be singular
e.g. Rahul and Renu love each other. or plural, concrete or abstract. Nouns show possession
15. The complement of the verb to be, when it is by adding ’s. Nouns can function in different roles
expressed by a pronoun, should be in Nominative within a sentence; for example, a noun can be a
case.
subject, direct object, indirect object, subject
e.g. It was he who did it.
complement, or object of a preposition.
16. When the same person is the subject and object, it
is necessary to use reflexive pronouns. Examples of nouns: Daniel, London, table, dog,
e.g. I cut me shaving this morning. (×) teacher, pen, city, happiness, hope
I cut myself shaving this morning. () Example sentences:
17. When a pronoun is the object of a verb or Steve lives in Sydney. Mary uses pen and
preposition it should be in objective case. paper to write letters.
e.g. These books are for you and I. (×) The young girl brought me a very long letter
These books are for you and me. () from the teacher, and then she quickly
Between him and me there is an understanding. disappeared. Oh my!
18. The Relative Pronoun should be placed as near as
possible to the antecedent. 2. PRONOUN
e.g. I have read Shakespeare's works who was a A pronoun is a word used in place of a noun.
great dramatist. (×) She... we... they... it
I have read the works of Shakespeare who was
A pronoun is a word used in place of a noun. A pronoun
a great dramatist.()
is usually substituted for a specific noun, which is
19. The case of the pronoun following ‘than’ and ‘as’ is
decided by mentally supplying the verb and called its antecedent. In the sentence above, the
completing the sentence. antecedent for the pronoun she is the girl. Pronouns
e.g. She is taller than I (am) are further defined by type: personal pronouns refer
I love more than (I love) him. to specific persons or things; possessive pronouns
indicate ownership; reflexive pronouns are used to
1.4 English Grammar
emphasize another noun or pronoun; relative pronouns answers the questions of when, where, how, why,
introduce a subordinate clause; and demonstrative under what conditions, or to what degree. Adverbs
pronouns identify, point to, or refer to nouns. often end in -ly.
Examples of pronouns: I, you, we, they, he, she, it, Examples: slowly, quietly, very, always, never, too,
me, us, them, him, her, this, those well, tomorrow, here
Example sentences: Example sentences:
Mary is tired. She wants to sleep. I want her I am usually busy. Yesterday, I ate my lunch
to dance with me. quickly.
The young girl brought me a very long letter The young girl brought me a very long letter
from the teacher, and then she quickly from the teacher, and then she quickly
disappeared. Oh my! disappeared. Oh my!
3. VERB 6. PREPOSITION
A verb expresses action or being. A preposition is a word placed before a noun or
jump... is... write... become pronoun to form a phrase modifying another
word in the sentence.
The verb in a sentence expresses action or being. There
is a main verb and sometimes one or more helping by... with.... about... until
verbs. (“She can sing.” Sing is the main verb; can is (by the tree, with our friends, about the book, until
the helping verb.) A verb must agree with its subject in tomorrow)
number (both are singular or both are plural). Verbs A preposition is a word placed before a noun or pronoun
also take different forms to express tense. to form a phrase modifying another word in the
Examples: go, speak, run, eat, play, live, walk, have, sentence. Therefore a preposition is always part of a
like, are, is prepositional phrase. The prepositional phrase almost
Example sentences: always functions as an adjective or as an adverb. The
following list includes the most common prepositions:
I like Woodward English. I study their charts
and play their games. Examples: at, on, in, from, with, near, between, about,
under
The young girl brought me a very long letter
from the teacher, and then she quickly Example sentences:
disappeared. Oh my! I left my keys on the table for you.
4. ADJECTIVE The young girl brought me a very long letter
from the teacher, and then she quickly
An adjective modifies or describes a noun or
disappeared. Oh my!
pronoun.
pretty... old... blue... smart 7. CONJUNCTION
An adjective is a word used to modify or describe a A conjunction joins words, phrases, or clauses.
noun or a pronoun. It usually answers the question of and... but... or... while... because
which one, what kind, or how many. (Articles [a, an, A conjunction joins words, phrases, or clauses, and
the] are usually classified as adjectives.) indicates the relationship between the elements
Examples: big, happy, green, young, fun, crazy, three joined. Coordinating conjunc tions conn ect
Example sentences: grammatically equal elements: and, but, or, nor, for,
so, yet. Subordinating conjunctions connect clauses
The little girl had a pink hat.
that are not equal: because, although, while, since,
The young girl brought me a very long letter etc. There are other types of conjunctions as well.
from the teacher, and then she quickly
Examples: and, or, but, because, so, yet, unless, since,
disappeared. Oh my!
if.
5. ADVERB Example sentences:
An adverb modifies or describes a verb, an I was hot and exhausted but I still finished
adjective, or another adverb. the marathon.
gently... extremely... carefully... well The young girl brought me a very long letter
An adverb describes or modifies a verb, an adjective, from the teacher, and then she quickly
or another adverb, but never a noun. It usually disappeared. Oh my!
English Grammar 1.5
ERRORS IN SUBJECT–VERB AGREEMENT But these words are singular when they refer to a
1. Singular subject must have singular verb. study, science or practice. If these words have
modifiers with them, they become plural and hence
e.g. He writes; I write.
take plural verbs.
2. Plural subject must have plural verb. e.g. His politics are somewhat divided.
e.g. They write; We write. 14. Title of the books need singular verbs.
3. Two subjects joined by and will always take a plu- e.g. Great Expectations is a good book.
ral verb.
15. Some nouns in the plural form represent an
e.g. The doctor and nurse work together. amount, a fraction, or an element of time. These
The doctor and nurses work together. nouns are considered singular and hence take sin-
The doctors and nurse work together. gular verbs.
4. Two singular subjects joined by or or nor will take e.g. Sixty minutes is enough to finish this work.
a singular verb. 16. If two subjects are joined together by as well as
e.g. A doctor or a nurse is working in the hospital. the verb will act according to the first subject.
5. A singular subject and a plural subject joined by or e.g. Students as well as the teacher are playing.
or nor will take a singular or plural verb depending 17. The subject Many a ..... is always followed by the
on which subject is nearer the verb. singular verb.
e.g. Neither Deepak nor his friends are joining the e.g. Many a man was drowned in the sea.
tour. 18. If two subjects are joined together by with the
Neither his friends nor Deepak is joining the verb will act according to the subject.
tour. e.g. The principal together with his students was
6. If the subject is singular and the predicate is plural, seeing the final match.
the verb must agree with its subject and not its 19. If subject is The number of ... use a singular verb.
predicate. e.g. The number of books is very small.
e.g. Physical conditioning and mental attitude are 20. If the subject begins with A number of ....
winning combination. (= many), use a plural verb.
The winning combination is physical e.g. A number of books are missing.
conditioning and mental attitude.
21. If two subjects express one idea, use a singular
7. Indefinite pronouns such as verb.
someone, somebody, each, nobody, anyone, e.g. Bread and butter is wholesome food.
anybody, one, no one, everyone, everybody, either,
neither, etc. always take a singular verb. 22. When adjectives such as much, less, little and
more are used as nouns, they must have a
e.g. Each of my friends calls me once a month. singular verb.
8. Indefinite pronouns which indicate more than one e.g. A little of good habits makes our life happy.
(several, few, both, many) always take plural verbs
ERRORS IN THE USE OF TENSES
e.g. Both of the books required careful reading.
The changed forms of a verb that indicate time of the
9. Collective nouns (fleet, army, committee, crowd) action are called tenses of the verb.
are singular when the group works together as a
unit and hence take singular verbs. 1. When the verb in the Principal Clause is in the
Past tense, the verbs of the Subordinate Clauses
e.g. The jury has reached its verdict. should be in the Past tense.
10. Collective nouns are plural when the members of e.g. He said that he had finished his work.
the group are acting individually and hence take
plural verbs. 2. But a past tense in the Principal Clause may or
may not be follwoed by the Past tense in the sub-
e.g. The jury have argued for five hours. ordinate clause if the latter expresses unviersal
11. Some words (such as news, measles, mumps etc.) or habitual truth.
end in —s but represent a single thing. These words e.g. The teacher said that the earth revolves round
need singular verbs. the sun.
e.g. The 7 o'clock news is about to begin. 3. Any tense may be used in the sub-ordinate clause
12. Some words (such as scissors, trousers, spectacles, if it gives a comparison by using the word than.
shorts etc.) end in —s and seem to represent a e.g. The teacher liked Anil better than he liked me.
single thing, but they are two parts to that single
thing. These words take plural verbs. 4. Any tense can be used when the subordinate clause
is in a quotation.
e.g. The scissors are on the table.
e.g. I said, “I am going to Delhi today”.
13. Words (such as politics, ethics, athletics etc.) that
end in —ies are usually singular and hence take 5. The Present Perfect Tense (subject + has/have
singular verbs. + V3) cannot be used when an expression of Past
time (yesterday, last night, ago etc.) is used.
e.g. Mathematics is his favourite subject.
e.g. We wrote to you yesterday about his mother.
English Grammar 1.9
5. Should is the past form of shall. It is used in sub- But if who, what or which is the object of the
ordinate clauses after in case and sometimes after sentence, do is used
if. e.g. Who did you see ?
e.g. I shall get some money in case brother comes. Which department do you want ?
It is also used in past sentences with so that and Do is also used with an —ing form when we want
in order that : to talk of activity that takes a certain time or that
e.g. He turned the stereo down very low so that he is repeated. In this case we find determiners like
should not disturb him. —the, my, some, much, etc.
Should can also be used in sub-ordinate clauses e.g. I usually do most of my washing on Sundays.
when we are expressing the idea that something
must be done or is important. The fact is that this ERRORS IN USE OF ADJECTIVES
happens after verbs like command, order, request, Word qualifying a noun or pronoun is called an adjective.
insist, suggest, advise etc. & after adjectives like
1. The adjectives ending in —ior (prior, junior, senior,
important, vital, essential, necessary, eager,
anxious, concerned etc. superior, inferior, posterior) take ‘to’ and not ‘than’
e.g. I am anxious that nobody should be hurt. after them.
Should is also used in sub-ordinate clauses in e.g. He is senior to me.
sentences where we express personal reactions to 2. Some adjectives like unique, ideal, perfect, extreme,
events. We express our reactions with words like— complete, universal, infinite, perpetual, chief,
amazing, interesting, shocked, sorry, normal, entire, round, impossible are not compared.
natural, it's a shame etc.
e.g. I am sorry you should think I did it on purpose. e.g. It is the most unique book. (×)
6. Must is followed by the infinitive without to. It is a unique book ()
e.g. I must get up at five tomorrow. 3. Comparative degree is used in comparing two things
Must is used to give strong advice or orders : or persons.
e.g. I really must stop drinking. e.g. It is the better of the two books.
Must is used to say that we are sure about Superlative degree is used in comparing more than
something : two things or persons.
e.g. I am in love—that must be nice.
e.g. He is the best of the three boys.
Must is used after a past reporting verb.
4. Double comparatives and double superlatives must
e.g. I felt there must be something wrong.
not be used.
7. Should and Ought have similar meanings, but
ought is followed by to. Ought to has a more e.g. He is more wiser than his brother. (×)
objective force and is used when we are talking He is wiser than his brother. ()
about laws, duties and regulations.
5. When we compare two qualities in the same person
e.g. We ought to see her tomorrow. or thing, the comparative ending —er is not used.
Should and ought to can also be used to talk
about strong probability. e.g. You are wiser than old. (×)
e.g. He has bought thirty pints of whisky—that You are more wiser than old. ()
ought to be enough. 6. When two adjectives in superlative or comparative
To talk about things which did not happen, although degree are used together, the one formed by adding
they were supposed to, we use ‘more’ or ‘most’ must follow the other adjective.
should and ought to with the perfect infinitive : e.g. He is more intelligent and wiser than his
e.g. The taxi should have arrived at 8.30. brother. (×)
8. Do is used to make question and negative forms
He is wiser and more intelligent than his
of ordinary verbs.
brother. ()
e.g. Do you know Rajat ?
Do is used in question-tags and short answers 7. When two adjectives with differing degrees of
e.g. Does he know I am here ? —Yes, he does. comparison are used they should be complete in
Do is used before an imperative to make the themselves.
request more persuasive e.g. He is as wise, if not wiser than his brother. (×)
e.g. Do accompany us. He is as wise as, if not wiser than his
Do is not used in questions which have who, what brother. ()
or which as their subject 8. When two changes happen together, comparative
e.g. Who said that ? degree is used in both.
What happened ? e.g. The higher you go, the cooler you feel.
Which one's that ?
English Grammar 1.11
9. When comparative degee is used in superlative 8. Latter means the second of two things.
sense, it is followed by ‘any other’. e.g. Keats and Byron are romantic poets, but I prefer
e.g. Kapil is better than any bowler. (×) the latter.
Kapil is better than any other bowler. () Later refers to time.
10. Compound adjective formed by adding ‘worth’ is e.g. She came to school later than I.
placed after the noun it qualifies. ERRORS IN USE OF ADVERBS
e.g. This is a worth seeing sight. (×) Adverbs are words that add information about
This is a sight worth seeing. () the verb.
11. When two or more comparatives are joined by ‘and’, 1. Adverbs of manner, place and time are usually
they must be in the same degree. placed after the verb or object.
e.g. Russel was one of the wisest and most learned e.g. He was running slowly.
men of the world. Reena does her work carefully.
12. When there are two objects of comparison, then to 2. Adverbs of frequency (e.g. never, often, usually,
avoid repetition of noun, ‘that’ is used for singular always, rarely, etc.) and other adverbs (like already,
noun and ‘those’ for plural noun. almost, just, quite, nearly, hardly) are normally
e.g. The climate of Ranchi is better than Gaya. (×) put between subject and verb. If there is more than
The climate of Ranchi is better than that of one word in the verb, they are put after first word.
Gaya. () e.g. He often goes to Delhi.
13. If comparison is made by using ‘other’, ‘than’ is I quite agree with you.
used instead of ‘but’. But if verb is ‘am’, ‘is’ and ‘are’, adverb is placed
e.g. He turned out to be no other than my old friend. after the verb.
14. Likely, certain and sure are followed by ‘to’. e.g. I am never late for school.
e.g. He is likely to win. 3. The adverb enough is placed after the adjective.
CONFUSED ADJECTIVES e.g. She is cunning enough to tackle him.
1. Beautiful is used for woman; handsome for man. 4. ‘Ever’ is sometimes incorrectly used for ‘never’.
e.g. He is a handsome youth. e.g. He seldom or ever tells a lie. (×)
She is a beautiful girl. He seldom or never tells a lie. ()
2. Less refers to quantity, fewer denotes number. 5. Adverb ‘not’ shouldn't be used with words having
e.g. He takes no less than a litre of milk. negative meaning.
They have fewer books than I have. e.g. The teacher forbade me not to go. (×)
3. Last is the final one; Latest is last upto the present. The teacher forbade me to go. ()
e.g. Z is the last letter of the alphabet. 6. The word ‘only’ should be placed immediately before
This is the latest edition of the book. the word it modifies.
4. Each is used for one of two or more things; every e.g. Hari answered only two questions.
is used for more than two things, taken as a group. 7. An adverb should not be used before an infinitive.
e.g. Each of the two boys was wrong. e.g. He quickly did the job. (×)
He read every book I gave him. He did the job quickly. ()
5. Older refers to persons or things. 8. The auxiliaries have to and used to come after the
e.g. This tree is older than that. adverb.
Elder refers to persons only. e.g. He often used to go to cinema.
e.g. He is my elder brother. SOME MORE TIPS OF USAGE
6. Little means ‘not much’. 1. Avoid the use of lots, a lot and a whole lot in the
A little means ‘at least some’. sense of much or a great deal.
e.g. He slept little. e.g. He expects to earn a lot of money on his sale
He slept a little. of farm products. (×)
7. Farther means ‘more distant’. He expects to earn a great deal of money on
Further means ‘additional’. his sale of farm products. ()
e.g. Mumbai is farther than Delhi. 2. Plenty is a noun and is always followed by of.
I shall get further information. e.g. He has plenty of room in his old house.
1.12 English Grammar
3. Due to always modifies a noun and not a verb. Hence, 14. When two comparatives are used in a sentence for
no sentence should begin with due to, it must be proportion, the is used before both of them.
used only after some form of the verb to be. e.g. The higher we go, the cooler it is.
e.g. His death was due to natural causes. 15. Do the needful is incorrect, write do what is
4. Alright is incorrect, use all right. necessary.
e.g. I think it is a quite alright if you stay. (×) 16. Never say family members, say members of the
I think it is quite all right if you stay. () family.
5. If the gender is not determined, use the pronoun e.g. His family members have gone to Mumbai. (×)
of the masculine gender. The members of his family have gone to
e.g. If anybody has got the book let her return it (×) Mumbai ()
If anybody has got the book let him return it () 17. Else is always followed by but and never by than.
6. When two nouns are closely connected, apostrophe e.g. It is nothing else but your pride which makes
—'s will be written after second only. you say such a thing.
e.g. Ravi's and Shikha's mother is ill. (×) 18. All of is used in conversation, not in standard
Ravi and Shikha's mother is ill. () written English.
7. Both and as well as cannot be used together in the e.g. She gave all of her antiques to the museum (×)
same sentence because both convey the same sense. She gave all her antiques to the museum ()
e.g. Both Pinki as well as Pooja are beautiful (×) 19. Talking terms is wrong, use speaking terms.
Both Pinki and Pooja are beautiful. () e.g. He is not on talking terms with his brother (×)
8. Supposing and if cannot be used together in the He is not on speaking terms with his
same sentence. brother ()
e.g. Supposing if he fails, what will he do ?(×) 20. Verbs like resemble, recommend, comprise, order,
If he fails, what will he do ? () accompany, reach, join, shirk, attack, emphasise,
discuss, fear, succeed, resist, request, assist, ben-
9. Endure suggests suffering, usually in silence and
efit, afford etc. are not followed by any preposition
tolerate, means to allow it with some degree of
when used in the active voice.
approval.
e.g. He emphasised on the need for discipline. (×)
e.g. He endured the pain without complaint.
He emphasised the need for discipline. ()
He will not tolerate laziness.
She will accompany with you to Chennai. (×)
10. Avoid the use of had with ought.
She will accompany you to Chennai. ()
e.g. You had not ougth to do it. (×)
21. The phrase type of, sort of and kind of do not follow
You ought not do it. ()
a or an.
11. As is used when one compares things or persons
e.g. What kind of a friend are you ? (×)
of equal or about equal size or quality, so is used
when one compares things or persons which are What kind of friend are you ? ()
unequal. 22. A full hour number follows o'clock, but in fraction
e.g. He weighs as much as his father. (×) it does not follow.
He does not weigh so much as his father. () e.g. He left this place by 10.40 o'clock. (×)
12. Avoid using the double comparative and double He left this place by 10.40. ()
superlative. 23. Yet means up to the present time something that
e.g. This pen is the most costliest in this store. (×) has not happened.
This pen is the costliest in this store. () e.g. His brothers are not married even yet.
13. Cent per cent and word by word are wrong uses.
The real terms are hundred percent and word
for word.
English Grammar 1.13
EXERCISE
MCQ TYPE QUESTIONS 10. (a) Have you a pencil ? I have not got one
(b) Is he coming ? Yes I think so
Directions (Q. 1 – 20)
(c) He enjoyed during the holidays
In this section you are required to spot errors in
sentences. Read each sentence to find out whether there (d) I asked for my pencil but he did not give it to me
is any error in any of the parts. Errors, if any, are only (e) No error.
in one of the parts. No sentence has more than one 11. (a) The boy who does best he will get a prize
error. When you detect an error, in any of the parts of (b) Whoever does best will get a prize
the sentence, choose the corresponding alphabet.
(c) Who did this ? I
1. (a) The scenery here is not good
(d) He and I are brothers
(b) I have lost my furnitures
(e) No error.
(c) We have received no information
12. (a) I went for a walk with some friends
(d) He told his mother this news
(b) He is wiser than I
(e) No error.
(c) The master tested the boy if he could read
2. (a) Please excuse the trouble
English
(b) He took pains over his work
(d) I shall see whether the brakes work well
(c) In India there are many poor
(e) No error.
(d) Scouts wear shorts
13. (a) Everyone is frightened when he sees a tiger
(e) No error.
(b) None of us has seen him
3. (a) I spent the holidays with my family
(c) People strave when they have no money
(b) Gone him some blotting paper
(d) The size of the shoe should be the same as
(c) Write this in your note book this shoe
(d) Gone my kind regards to all
(e) No error.
(e) No error.
14. (a) Everyone knows this
4. (a) We had a good play of football
(b) Every man knows this
(b) We saw a play
(c) These all mangoes are ripe
(c) We like acting
(d) He hold the book in both hands
(d) He is a tall man
(e) No error.
(e) No error.
15. (a) I have no any friends
5. (a) The boy was wearing a new suit
(b) Neither man has come
(b) He took offence at this
(c) All idle man should do some work or other
(c) Please put your sign here
(d) Shakespeare is greater than any other poet
(d) Please put your signature here
(e) No error.
(e) No error.
16. (a) Open your book at page six
6. (a) He is my cousin brother
(b) He is in class ninth
(b) We are all brothers
(c) This article costs ten rupees
(c) I spent the middle of the day working
(d) He had a large amount of money (d) He came a second time
(e) No error. (e) No error.
7. (a) He got into bad company 17. (a) King George VI
(b) One of my servant tells me (b) He is older than I
(c) Some of my servants tell me (c) He is better than I
(d) The front of the house (d) Raipur is Hotter thanSimla
(e) No error. (e) No error.
8. (a) I had a bathe in the sea 18. (a) He is worse than I
(b) I have hurt a toe (b) A horse is more useful than a car
(c) Each of these days play games (c) He gets a small salary
(d) None of us went (d) I our library the number of books is less
(e) No error. (e) No error.
9. (a) A man should work hard 19. (a) He is the more clever of the two
(b) A man should not waste his time (b) He is the cleverer of the two
(c) A boy should not waste his time (c) From the three he is more clever
(d) Here is my cup; please fill it (d) Of the two plans this is the better
(e) No error. (e) No error.
1.14 English Grammar
20. (a) He is becoming strong 27. That man is aggressive by nature is a hard fact of
(b) There is a very good teacher in that class life and no one can deny it.
(c) He will spend the rest of his life here (a) That man is aggressive by nature is a hard
(d) This is a sight worth seeing fact of life which none can deny.
(e) No error. (b) That man is aggressive by nature is a hard
fact of life and no one can deny it.
Directions (Q. 21 – 30)
(c) That man is aggressive by nature is a hard
Each question below has a sentence, from the choices fact of life and not one can deny.
provided, identify the one which best restates the given
sentence and mark its number as the answer. (d) That man is aggressive by nature is hard for
anyone to deny.
21. Uneasy lies the head that wears a crown.
28. He is as old as I.
(a) Uneasily lies the head that wears a crown.
(a) He is as old as me.
(b) Uneasy is the head wearing a crown.
(b) He and I are equally old.
(c) The head which wears the crown lies uneasy.
(c) He and I are equally older.
(d) The head which is wearing the crown lies
uneasy. (d) Both he and I are of the same age.
22. Had I been a little early, I would not have missed 29. A man becomes wiser with age and experience.
the train. (a) A man as he is aged and experienced becomes
(a) Had I been a little early, I would not have wiser.
missed the train. (b) A man with age and experience becomes wiser.
(b) Had I been a little early than now, I would not (c) A man as he is aged and experienced becomes
have missed the train. the wiser.
(c) Had I been a little earlier, I would not have (d) With age and experience, a man becomes wiser.
missed the train. 30. Hazards of life cannot be negated but they can be
(d) Had I only been a little early than now, I would quite effortlessly evaded.
not have missed the train. (a) Hazards in life can be quite effortlessly evaded
23. She is as intelligent as her sister if not more. and completely nullified.
(a) She is more intelligent than her sister. (b) Hazards of lilfe cannot be avoided but they
(b) She is as intelligent as her sister if not more can be made ineffective.
intelligent. (c) To be made ineffective, hazards of life must
(c) She and her sister are equally intelligent. be avoided.
(d) She is less intelligent than her sister. (d) It may not be possible to nullify the hazards
of life but they can quite easily be avoided.
24. You will not succeed unless you are optimistic.
Directions(Q. 31 –40)
(a) You will not be successfull unless you are
optimistic. Each sentence is broken into four parts a, b, c, d.
Mark the part which has an error. Ignore errors of
(b) You will not succeed until you are an optimist. punctuation.
(c) You will not succeed unless you don't be 31. (a) Every man, woman
optimistic.
(b) and child
(d) You will not succeed unless you don't be an
optimistic. (c) in the house on fire
25. The students were advised to follow the (d) have been saved.
instructions of the examiner. 32. (a) One of the
(a) The instructions of the examiner were (b) best lawyers in town
followed by the students. (c) have been
(b) The students followed the examiner 's (d) hired.
instructions. 33. (a) I request you (b) kindly to
(c) The examiner's instructions were advised to (c) come to me (d) immediately.
be followed by the students.. 34. (a) My friend's mother
(d) It has been advised to the students that they (b) is the principal
should follow the examiner's instructions.
(c) of a
26. Despite being ill, he attended the classes.
(d) girl's college.
(a) Despite of being ill, he attended the classes.
35. (a) To succeed in these tests
(b) Despite his illness, he attended the classes. (b) it is absolutely necessary
(c) Inspite of his being ill, he attended classes. (c) for us
(d) The classes were attended by him inspite of (d) to aim for speed and accuracy.
being ill.
English Grammar 1.15
36. (a) The aeroplane is a 43. Although most hotels in Bhutan has a distinctly
(b) powerful instrument of war local flavour, some pander with the perceived tastes
(c) and their peacetime role of Western tour groups - and can be rather bland.
(d) is just as important. 44. The crustaceans find sanctuary in the jords, often
clinging to the walls in large, gregarious groups.
37. (a) The depletion of ozone in the atmosphere
45. In these tropical lowlands the Maya built an
(b) is a reminder to all of us agrarian civilization that eventually supported
(c) both in the developed and developing world one of the highest population densities in the pre
(d) that they cannot continue to use contraptions - industrial America.
which give out noxious gases. Directions (Q. 46–50)
38. (a) Scotsmen are notorious Each of the following questions has two sentences A
(b) for their thrift and B.
(c) and he is the butt Mark (a), if you think sentence A has an error.
(d) of many exaggerated jokes. Mark (b), if you think sentence B has an error.
39. (a) A mixed economy Mark (c), if you think both sentences A and B have
(b) is it in which errors.
(c) the public sector Mark (d), if you think neither sentence has an error.
(d) and the private sector co - exist. 46. A. The boss himself or his secretary answer the
40. (a) Although the beaver's hind feet phone on Saturday.
(b) are webbed for swimming B. Neither the quality nor the prices have
changed.
(c) their front feet
47. A. The members re-commended that all
(d) are small and handlike. delinquents be fined.
Directions(Q. 41 – 45) B. A stone lying in one position for a long time
Given below are five sentences, each of which may or may gather moss.
may not have errors. 48. A. Until I received that letter, I was hoping to
Mark (a), if there is only one error. have had a visit from Krishnan.
Mark (b), if there are two errors. B. Follow the main road for a mile; then you need
Mark(c), if there are more than two errors. to take the next road at the left.
Mark (d), if there is no error. 49. A. The people to watch closely are the ones ruling
41. ‘‘This feels like one big dream,’’ says a weary behind the political scene.
American woman to her husband, as they sit in a B. Give the tools to whoever can use them best.
plane flying home. 50. A. The magazine has been published continuous
42. The worst mistake a non - profit organization can since 1951, and it has the funniest cartoons
make is to take all the money it is given and be you can possibly imagine.
beholden to doing things on someone else's terms. B. I feel glumly when I read his nightmarish tales.
ANSWERS
MCQ Type Questions
1. (b) 2. (a) 3. (e) 4. (a) 5. (c) 6. (a) 7. (b) 8. (c) 9. (e) 10. (c)
11. (a) 12. (c) 13. (d) 14. (c) 15. (a) 16. (b) 17. (d) 18. (d) 19. (c) 20. (a)
21. (c) 22. (a) 23. (c) 24. (a) 25. (d) 26. (b) 27. (a) 28. (d) 29. (d) 30. (d)
31. (d) 32. (c) 33. (b) 34. (d) 35. (d) 36. (c) 37. (d) 38. (c) 39. (b) 40. (c)
41. (a) 42. (d) 43. (b) 44. (d) 45. (d) 46. (a) 47. (d) 48. (c) 49. (d) 50. (c)
1.16 English Grammar
EXPLANATIONS
MCQ TYPE QUESTIONS 29. Choice (a) and (b) mean that the prerequisite for
wisdom is age and experience, which is incorrect.
21. Uneasily does not denote a state, which is required
Just by growing old and working for a long period,
in the context. ‘Wearing a crown' denotes that
a person cannot improve his wisdom. ‘The wiser'
the head is doing it, ‘is wearing' denotes a
is used only when we compare wisdom with some
continuous action, which is impossible, as the
other quality. Choice (d) is the correct restatement
head cannot do it. Hence choice (c) restates the
of the given one. Hence correct choice is (d).
given sentence correctly.
30. The given statement says that one cannot
22. All the statements have the correct combination
completely nullify the hazards of life but they can
of tenses. So, we need not check it. Let us find
be evaded quite easily. Choice (a) which states
the errors in other statements. ‘Early than now'
that hazards cannot be avoided is wrong (b) states
and ‘earlier' suggest that I reached early; in that
that hazards can be made ineffective and hence
case, I did not miss the train. So, choices (b), (c),
this is also wrong (c) does not convey the meaning
and (d) do not state the same idea as in the question
as stated and hence it too is incorrect. (d) correctly
statement. Hence correct choice is (a).
restates the idea and is our answer.
23. The given statement states that both she and her
31. Every man refers to a singular noun, therefore....
sister are equally intelligent. This is given in
has been saved.
statement. Hence correct choice is (c).
32. has
24. ‘Unless' denotes condition, ‘until' denotes time.
Choice (b) has adverb clause of time.With unless, 33. drop ‘kindly'
another negative is not used. ‘Unless ..... don't' is 34. Girl's implies possessive noun, whereas the
wrong. So, Choices (c) and (d) are incorrect. Hence college is meant for girls.
correct choice is (a). 35. aim at, aim for is wrong usage.
25. Choices (a) and (b) state the same idea [(a) – 36. delete their - add : its
passive voice; (b) Active voice] that the students 37. delete they - add : we
followed the instructions, which differs from the
38. delete he is - add : they are.
question statement. Instructions cannot be
advised. So, choice (c) is meaningless. Choice (d) 39. delete it - add : that
states the context of the given statement aptly. 40. delete their - add : its.
26. Despite means inspite of. Despite of is a wrong 41. One sits ‘on' a plane.
usage. ‘His' need not be used in choice (c), as the 42. There are no errors.
sentence is about one person. Choice (d) means
43. Hotels ‘have' and some pander ‘to....'
that the classes were ill. Hence correct choice
is (b). 44. There are no errors in this sentence.
27. Choice (a) best restates the given sentence, 45. There are no errors.
though choice (b) is the repetition of the given 48. (A) hoping to have a visit;
statement. ‘Not one can deny' in choice (c) and (B) .... then take the next road to
for any one to deny' in choice (d) make them 50. (A) continuously; (B) glum.
incorrect. Hence correct choice is (a).
28. In this type of comparison, the pronouns are in
subjective case 'Me', which is in objective case is
wrongly used in choice (a). Equally is not used
with ‘old'. Of the same age is the correct
expression. Hence correct choice is (d).
2
CHAPTER Sentence Completion
A Sentence Completion question consists of an incomplete sentence. Moreover the question
incomplete sentence, which has one or two blanks. sentence also may have difficult words in itself,
From the answer choices given, you are asked to knowing the meaning of which is essential for
choose the appropriate word / words that can take understanding the sentence holistically and thus
the place of the blanks in order to best complete the to choose the appropriate word(s).
meaning of the given sentence. Look at the following question:
Directions: The department can use information received
Each of the sentences below has one or two blanks; from dissatisfied employees and __ to launch an
each blank indicates omission of a word or set of investigation against the company.
words. Beneath the sentence are given four answer A. pettifoggers
choices, each containing a word or a set of words.
B. buccaneers
Choose the word or set of words for each blank that
gives the best meaning to the sentence as a whole. C. ambassadors
A thinking man resists the vulgar prosperity that D. whistleblowers
______ ever to ______ by preserving and The knowledge of the meaning of the words in
communicating heroic sentiments. choices is essential for you to select the answer
A. disintegrates ... banality to this question with confidence. Let's analyze
B. transforms... neutrality . . meaning of these words.
C. decays... luxury A Pettifogger is one who practices chicanery or
unlawful business. Can this word fit the sentence?
D. retrogrades ... barbarism
No.
E. level ...hurtly
A Buccaneer is a pirate; again not a word that
The theory underlying a sentence completion stimulus would go with dissatisfied employees to supply
is that the meaning and the tone of a sentence can be
information about a company.
gathered without the missing word/ words. If so, it is
possible for you to supply the appropriate word(s) to A whistleblower is one who informs about
complete the sentence. corruption or wrongdoing. Yes, this fits the
sentence.
Invariably, in every sentence completion question,
there will be a clue or two that could be used as lead(s) An Ambassador is not one to do anything with a
toward identifying the correct word(s). company. In fact this is the first choice you could
eliminate.
These clues could be structural (syntax/grammar
clues) or semantic (meaning clues). The word(s) that ii. Skill of semantics and syntax
you choose should agree and merge with the sentence Every sentence has a semantic dimension
structurally and functionally or semantically. (meaning) and a syntactical dimension (word
Every sentence completion question can be analyzed arrangement). Knowledge of these two is a
for the negative or positive nature or tone of words to mandatory condition for you to be able to choose
be filled in. the appropriate word. You must understand the
What is measured by a Sentence completion meaning of a sentence in order to identify the
question? appropriate choice word that fills the blank, and
completes the sentence coherently.
i. Skill of vocabulary
You should have a good understanding of the
Sentence completion question is a test of your
functions of structural words in a sentence such
word power and diction. It is only if you know the
as unless, rather, yet, while unfortunately, in
meaning of word(s) that you will be able to choose
confidently, the appropriate one(s) to fill the contrast, despite.
2.2 Sentence Completion
Look at the following question: part. And both words must fit into the meaning of the
The meaning of words is destroyed if you change sentence; therefore read your choice into the sentence
them, whereas the meaning of a thought______ supplying and evaluating both words.
whatever word you think proper to use. Each sentence contains a few crucial clues that
A. alters B. transforms determine the answers, and finding these clues will
guide you to the correct answer. Look for what is
C. remains D. settles
directly stated. In sentence completion question, you
E. Ruins may also find poetic and literary language, which tests
When you read the sentence you must recognize your understanding of figurative usage. The correct
the function of ‘whereas’ in the sentence. It signals answer is the one that is directly implied in the
a shift in the meaning of the sentence. sentence.
‘One thing is destroyed whereas the other is not' Determine where the sentence is heading, using
the answer thus is (C) remains. structural clues.
The semantic feature of a sentence refers to its
Anticipate answers in your words.
meaning, and how the meaning of the given words
in a sentence helps us to identify the word to Look for answers that are similar to yours.
complete the sentence. Fix the words in the sentence to see if they get
Study the following sentence: with the whole sentence.
A cult brand _____ the imagination of a small Sentence completion strategy in a nutshell.
group who spreads the word, makes converts,
Strategy 1
helps turn a fringe product into a ______ name.
Read the entire sentence.
A. ceases... redoubtable
In both one- and two-blank sentences read the
B. elevates ... dynamic
entire sentence before you inspect the choices.
C. fires... dream An overall understanding of the sentence is
D. seizes... mainstream essential.
The first clue is the word Imagination; hence Strategy 2
options (A) and (D) it, ceases and seizes can easily
Identify structural clues to determine the nature
be eliminated. You have to work for fires and
of the word(s) required.
elevates. To fire One's Imagination is a standard
idiomatic usage. Moreover the meaning of the You can determine the positive or negative tone
sentence implies turning a fringe (peripheral) of the expected word using structural clues. And
product into (the opposite of it) a dream product. identify the words whose synonym, antonym,
Hence, answer is (C). parallel word or cause/effect you have to supply.
iii. Skill of identifying clues Strategy 3
A sentence completion question is not merely a Substitute your own words.
test of your vocabulary. It also tests your ability Place your own words in the blanks assuming
to analyze the given sentence and to identify the that answer choices are not given, going by the
structural and semantic clues that decide on the clues you have identified.
meaning and nature of the word(s) to be chosen.
Strategy 4
You will find one or two clues in a sentence
completion question. Common key words such Identify the words similar to the ones you
as and or but are all more valuable for us in substituted.
sentence completion. These are structural clues. If the word you have substituted goes with the
(These are dealt with in detail later) requirements of the sentence, you are sure to
find, a word(s) similar in meaning and tone to
Sentence Completion Strategies
the ones you have substituted.
In order to answer sentence completion question
correctly, you should first read the entire sentence as Strategy 5
you see it, without trying to fill the word(s). Desist Read the sentence again with word(s) filled in.
from working on each blank separately. The second Now read the entire sentence with the words filled
part of the sentence may have a bearing on its first in to see if the word(s) fit literally and figuratively.
Sentence Completion 2.3
Similarly Although
Because While
And Unfortunately
Since Nonetheless
Thus Ironically
In fact Paradoxically
Moreover On the other hand
Indeed
Contradictorily
In addition
Contrastively
Consequently
Despite
Study the following sentences:
1. The film script showed depth and maturity; Even though
likewise the treatment showed profundity. Even if
2. His argument was convincing and persuasive. Illustrations:
3. Shakespeare lives forever; in fact he is immortal. His argument was flawed but persuasive.
B. PARALLEL IDEA CLUES
There is no cynicism in the way the young deal with
These clues are similar to the thought continuity clues. issues; on the contrary they are optimistic toward life.
The word that completes the sentence is in the same
tone, as in the word already given. The word(s) that The emphasis will not be on physical assets but on
complete the sentence complements a word or phrase intangibles.
already given. In the sentences above the word in italics are
i. All that the young ask of the political class is structural clues, which indicate that the words to be
compassion and concern/ sympathy/ care. filled in are opposite in meaning to the words, that
ii. On question about political and electoral reforms, are already given in the sentences.
the respondents showed great forethought and D. CAUSE EFFECT CLUES
vision/rationality. While studying a sentence completion question, you
In the above sentences you may observe that the should be able to gauge as to which direction the
words given and the words that have to be filled in sentence is heading. If the sentence has an idea, it is
are parallel/ complementary or in the same tone. possible to infer what would logically follow the causal
C. THOUGHT REVERSAL CLUES phenomenon. In other words, we can anticipate the
type of effect that the given cause would lead to.
These terms change the direction of the sentence.
Similarly by analyzing the nature of an effect
They make one part of the sentence contradictory to
(sentences in which effect is given), we can also infer
the other part. The word or phrase that completes
the plausible causal factor that would've contributed
the sentence is opposite in meaning to a word or
to the type of effect given. The easiest way is to
phrase already given. The positive or negative charge
2.4 Sentence Completion
identify the tone of the cause or effect to be supplied E. WORD DEFINATION CLUES
into the sentence-negative or positive Yet another common clue found in sentence
Let's work on sentences with cause effect relationship. completion stimuli is the meaning or definition of the
Torture is so prevalent and _________of the helpless word to be filled given as part of the incomplete
so common that these helpless beings are certainly sentence itself. It is similar to the thought continuity
______ clue, except that in case the meaning of the word or
When they pair out their story of agony. definition of the word to be selected is already given.
(1) indemnifications... unhinged Let us study an example :
(2) felicitation... believable The great irony of Angle Computers is that very
(3) disparaging ...credulous posture that won Steve jobs’ a cult following creating
(4) victimization ... credible an exclusive group of ______ technicians who ______
refused to bow to IBM, hobbled his company.
In the above sentence you observe that there are two
clues: parallel and causal. Signaled by the word and iconoclastic……stubbornly
the first blank needs a word parallel to torture i.e. conservative……invariably
victimization. The intensity of these two results in rebellious…….timidly
the possibility of the stories believable or credible. traditional……..out rightly
Look at another example: In the sentence above you would’ve already observed
Higher education in India can no longer be allowed to the words ‘refused to bow’, which signals the word to
remain as a/ an _____ instrument that crushes the be placed in the second blank. This phrase clearly
spirit of the individual while goading him into a defines the word stubbornly.
________ and consumerist mode. Another example :
(1) innerving... boisterous The crisis in modern education is not merely a
(2) emasculating... materialist constraint of ____ of funds but of _______ practices,
unsuited for contemporary system.
(3) invigorating ... pragmatic
scarcity……doctrinaire
(4) relegating ... humanistic
fulsomeness……authoritative
An instrument that crushes the spirit of the individual
cannot be innerving or invigorating (both means restrictiveness…….autocratic
energizing) nor can it be riveting; since rivet means paucity…….anachronistic
to hold firmly and relegate means to lower in rank is In the above sentence the key to the letter word is
closer but the latter word cannot be humanistic (3) ‘unsuited for contemporary education’, you can now
The word that is parallel to consumerist is materialist. easily eliminate unsuitable choices and arrive at
In a similar mode identify the cause effect ideas in choice D, i.e., anachronist.
the incomplete sentences in the following drill and
choose appropriate words to complete them.
Sentence Completion 2.5
EXERCISE
MCQ TYPE QUESTIONS 7. Home is ____________ one has yet to improve.
(a) a discovery over which
Directions for Q. 1 to 41 : In each of the following
sentences, a part/parts of the sentence is left (b) a certain over which
unfinished. Beneath each sentence, four different (c) an invention on which
ways of completing the sentence are indicated. Choose (d) an institution upon which
the best alternative among the four. 8. The highest reward for a man's toil is not what
1. For some of our ruling politicians, social justice he gets for it but what ___________.
has apparently come to mean that ____________. (a) he makes out of it.
(a) they have a right to plunder public money
(b) he gets for others.
from the treasury
(b) they have a right to the plunder of public (c) he has overcome through it.
money from the treasury (d) he becomes by it.
(c) theirs is the right to plunder public money 9. An act of justice closes the book on a misdeed;
from the treasury an act of vengeance ___________.
(d) the right is theirs to plunder public money (a) reopens the first chapter
from the treasury (b) writes an epoilogue
2. Some critics believe that Satyajit Ray never quite (c) writes one of its own
came back to the great beginning he made in his
(d) opens new books
path-breaking film Pather Panchali. ____________
have endured decades of well-travelled bad prints 10. The trail is the thing , not the end of the trail.
to become a signpost in cinematic history. Travel too fast and you miss __________.
(a) The bizarre history of its misty origins (a) all you are travelling for.
(b) Its haunting images (b) all the sights you are supposed to see.
(c) Its compelling munificence (c) the very excitement of your travel.
(d) The breathtaking awe it inspires (d) all the enjoyments of travel.
3. ____________ , the more they remain the same. 11. ________________ that in this apparent mess,
(a) The less the dynamism two things not be interfered with.
(b) The more things change (a) It is important.
(c) The more pronounced the transformation (b) it is of cardinal important.
(d) The more the merrier
(c) It should be urgently under-stood.
4. Revenues are likely to register a dramatic
(d) It cannot be emphasised.
increase if ____________ on the hundred million
people who are said to comprise the risingIndian 12. A country's wealth is its people. But instead of
middle class. drawing out the strengths of the people, instead
(a) a flat tax is collected of drawing out their talent, this use of religion
debases, degrades and depresses than
(b) a flat tax is placed ____________.
(c) taxes at a flat rate are charged (a) in greater and greater measure
(d) tax is imposed at a flat rate (b) further
5. I am an entertainer. ____________ I have to keep (c) beyond reasonable limit
smiling because, deep in my heart, laughter and
sorrow have an affinity. (d) more and more
(a) Even if I have tears in me 13. Wines that yielded a good commercial profit
____________ in the same limited areas of France
(b) Despite conditions of extreme adversity
as of now.
(c) Inspite of misery around me
(a) seem to have been produced
(d) Although I have yet to make it big
(b) appear to have a remarkable semblance
6. The stock market is probably ____________ And
the way the markets are plunging says a lot about (c) bear a significant similarity in terms of
investor confidence. production to those grown
(a) an ideal indication of the health of public (d) appear to have been similarly produced
sentiment 14. This is about ____________ a sociological analysis
(b) the least imperfect mechanism for judging the can penetrate.
quantity of the sentiment of the public. (a) the other limits that
(c) the best indicator of public sentiment (b) just how far into the subject that
(d) the best barometer to assess the sentiment of (c) the relative distance that
the public (d) as far as
2.6 Sentence Completion
30. Many people suggest __________and still others 34. As navigators, calendar makers, and other ____of
would like to convince people not to buy pirated the night sky accumulated evidence to the
cassettes. contrary, ancient astronomers were forced
(a) to bring down audio-cassette prices to reduce to______that certain bodies might move in circles
the incidence of music piracy, others advocate about points, which in turn moved in circles about
strong legal action against the offenders,
the earth.
(b) bringing down audio-cassette prices to reduce
the incidents of music piracy, others are (a) scrutinizers; believe
advocating strong legal action against (b) observers; agree
offenders, (c) scrutinizers; suggest
(c) bringing down audio-cassette prices to reduce
(d) observers; concede
the incidence of music piracy, others advocate
strong legal action against offenders, 35. Every human being, after the first few days of his
(d) audio-cassette prices to be brought down to life, is a product of two factors: on the one hand,
reduce incidence of music piracy, others there is his _______ endowment; and on the other
advocate that strong legal action must be taken hand, there is the effect of environment, including
against offenders, _______.
31. The ancient Egyptians believed_________so that (a) constitutional; weather
when these objects were magically reanimated
(b) congenital; education
through the correct rituals, they would be able
to function effectively. (c) personal; climate
(a) that it was essential that things they portrayed (d) genetic; pedagogy
must have every relevant feature shown as 36. Exhaustion of natural resources, destruction of
clearly as possible individual initiative by governments, control over
(b) it was essential for things they portray to have man’s minds by central ______ of education and
had every relevant feature shown as clearly propaganda are some of the major evils which
as possible, appear to be on the increase as a result of the
impact of science upon minds suited by _______
(c) it was essential that the things they portrayed
to an earlier kind of world.
had every relevant feature shown as clearly
as possible, (a) tenets; fixation
(d) that when they portrayed things, it should (b) aspects; inhibitions
have every relevant feature shown as clearly (c) institutions; inhibitions
as possible (d) organs; tradition
32. Archaeologists believe that the pieces of red-ware 37. But _________________ are now regularly written
pottery excavated recently near Bhavnagar and not just for tools, but well-established practices,
_________shed light on a hither to dark 600-year organisations and institutions, not all of which
period in the Harappan history of Gujarat. seem to be _________________ away.
(a) estimated with a reasonable certainty as being (a) reports, withering (b) stories, trading
about 3400 years old, (c) books, dying (d) obituaries, fading
(b) are estimated reasonably certain to be about 38. The Darwin who _______________ is most
3400 years old remarkable for the way in which he
(c) estimated at about 3400 years old with _________________ the attributes of the world
reasonable certainty, class thinker and head of the household.
(d) estimated with reasonable certainty to be (a) comes, figures
about 3400 years old, (b) arises, adds
33.The genocides in Bosnia and Rwanda, apart from (c) emerges, combines
being mis-described in the most sinister and
(d) appeared, combines
_______manner as ‘ethnic cleansing’, were also
blamed, in further hand-washing rhetoric, on 39. Since her face was free of ______________ there
something dark and interior to _________ and was no way to ___________ if she appreciated
perpetrators alike. what had happened.
(a) innovative; communicator (a) make-up, realise
(b) enchanting; leaders (b) expression, ascertain
(c) disingenuous; victims (c) emotion, diagnose
(d) exigent; exploiters (d) scars, understand
2.8 Sentence Completion
40. In this context, the __________ of the British The key point here is to view the fired employees as
labour movement is particularly ___________. a “failed product” and to ask how the process (47) _____
(a) affiar, weird such a phenomenon in the first place.
(b) activity, moving 45. (a) dismissing (b) punishing
(c) experience, significant (c) firing (d) admonishing
(d) atmosphere, gloomy 46. (a) resolve (b) thwart
41. Indian intellectuals may boast, if they are so (c) defeat (d) close
inclined, of being _________________ to the most 47. (a) derived (b) engineered
elitist among the intellectual _________________ (c) produced (d) allowed
of the world. Directions for Q. 48 to 57 :
(a) subordinate, traditions Fill up the blanks, numbered [48], [49]....up to [57], in
(b) heirs, cliques the two passages below with the most appropriate word
(c) ancestors, societies from the options given for each blank.
(d) heir, traditions “Between the year 1946 and the year 1955, I did not
Directions for Q. 42 to 47. file any income tax returns.” With that [48] statement,
Fill the gaps in the passages below with the most Ramesh embarked on an account of his encounter with
the Income Tax Department. “I originally owed
appropriate word from the options given for each gap.
Rs. 20,000 in unpaid taxes. With [49] and [50], the
The right words are the ones used by the author. Be 20,000 became 60,000. The Income Tax Department
guided by the author’s overall style and meaning when then went into action, and I learned first hand just
you choose the answers. how much power the Tax Department wields. Royalties
Von Nuemann and Morgenstern assume a decision and trust funds can be [51]; automobiles may be [52],
framework in which all options are thoroughly and auctioned off. Nothing belongs to the [53] until
considered, each option being independent of others, the case is settled."
with a numerical value derived for the utility of each 48. (a) devious (b) blunt
possible outcome (these outcomes reflecting, in turn, (c) tactful (d) pretentious
all possible combinations of choices). The decision is 49. (a) interest (b) taxes
then made to maximize the expected utility. (c) principal (d) returns
(42) _____, such a model reflects major simplifications 50. (a) sanctions (b) refunds
of the way decisions are made in the real world. (c) fees (d) fines
Humans are not able to process information as quickly 51. (a) closed (b) detached
and effectively as the model assumes; they tend not (c) attached (d) impounded
to think (43) _____ as easily as the model calls for;
52. (a) smashed (b) seized
they often deal with a particular option without really
(c) dismantled (d) frozen
assessing its (44) _____, and when they do assess
alternatives, they may be extremely nebulous about 53. (a) purchaser (b) victim
their criteria of evaluation. (c) investor (d) offender
42. (a) Regrettably (b) Firstly At that time the White House was as serene as a
resort hotel out of season. The corridors were [54].
(c) Obviously (d) Apparently In the various offices, [55] gray men in waistcoats
43. (a) quantitatively (b) systematically talked to one another in low-pitched voices. The only
(c) scientifically (d) analytically colour, or choler, curiously enough, was provided by
44. (a) implications (b) disadvantages President Eisenhower himself. Apparently, his [56]
(c) utility (d) alternatives was easily set off; he scowled when he [57] the
corridors.
In a large company, (45) _____ people is about as
common as using a gun or a switch-blade to (46) _____ 54. (a) striking (b) hollow
an argument. As a result, most managers have little (c) empty (d) white
or no experience of firing people, and they find it 55. (a) quiet (b) faded
emotionally traumatic; as result, they often delay the (c) loud (d) stentorian
act interminably, much as an unhappy spouse will
56. (a) laughter (b) curiosity
prolong a bad marriage. And when the firing is done,
it’s often done clumsily, with far worse side effects (c) humour (d) temper
than are necessary. 57. (a) paced (b) strolled
Do the world-class softward organizations have a (c) stormed (d) prowled.
different way of firing people ? No, but they do the
deed swiftly, humanely, and professionally.
Sentence Completion 2.9
ANSWERS
MCQ Type Questions
1. (a) 2. (b) 3. (b) 4. (d) 5. (b) 6. (c) 7. (d) 8. (d) 9. (c) 10. (c)
11. (a) 12. (d) 13. (a) 14. (d) 15. (a) 16. (d) 17. (b) 18. (a) 19. (d) 20. (c)
21. (3) 22. (c) 23. (d) 24. (b) 25. (b) 26. (d) 27. (a) 28. (c) 29. (d) 30. (c)
31. (c) 32. (d) 33. (c) 34. (d) 35. (d) 36. (d) 37. (d) 38. (c) 39. (b) 40. (c)
41. (d) 42. (d) 43. (d) 44. (d) 45. (c) 46. (a) 47. (d) 48. (b) 49. (a) 50. (d)
51. (c) 52. (b) 53. (d) 54. (c) 55. (a) 56. (d) 57. (d)
EXPLANATIONS
MCQ TYPE QUESTIONS (b) A man’s toil may secure a lot of things for
his children or friends . That too is not
1. They in choice (a) means ‘some of our ruling something great.
politicians’ (the subject). Also grammatically (c) A man’s toil may help him to overcome
‘they have a right to plunder .........’ is correct many difficulties, but that does not give him
wheras’...... right to the punder ......’ is incorrect. much pleasure.
2. ‘Bizarre' means ‘weird'. Hence, choice (a) is So, choice (d) is the best answer. What one
ruled out. "Its haunting images have endured ' becomes by one’s toil is greater than what one
fits syntactically. choice (c) can also be ruled gets.
out as ‘munificence' means generosity and this 9. (a) ‘Reopens the first chapter’ means begins
is not intended in the sentence. Also, the word from the very start.
‘prints' in the given sentence gets with the word (b) ‘Writes an epilogue’ means ends the book.
‘images' of choice (b).
(c) ‘Opens new books’ means creates new
3. ‘They’ in the part after the comma must refer problems.
to somebody or some-thing. Hence, only
choice (b) will fit as ‘they' in that case refers The above three are only partly correct. An act
to ‘things'. of justice pardons for ever an act of vengeance
4. The right word to precede the proposition ‘on' creates new problems. It is a saying. “An act of
is imposed. Tax is not collected on ....... people. justice closes the book on a misdeed; an act of
Tax is not placed on people, Tax is not charged vengeance writes one of its own.”
on people. Hence, choices (a), (b) and (c) are not 10. Here, the trail is the important thing and hence
the answers. Tax is imposed ..... on ..... . the excitement of the trail is what you will be
5. Choice (d) is ruled out as it does not logically looking for.
fit. Choice (a) is also ruled out because of the 11. (b) The expression must be “it is of cardinal
expression ‘tears in me'. importance”.
6. The key words are ‘plunging' and ‘investor (c) There is no urgency in understanding, “It
confidence'. If markets are plunging, investor should be understood” is enough.
confidence is low. This aspect is brought out (d) Instead of “it cannot be,’ the expression should
best by choice (c). be it must be’.
7. (a) ‘Discovery’refers to something that existed So choice (a) is the best, It is important that
previously and came to knowledge later. the two things are not to be interfered with.
Home is not a discovery.
13. (b) It is not mentioned with what do wines
(b) Home is not a certain. It is a set up which bear semblance.
comes into being as a result of a certain
composition of various factors. (c) The ‘similarity’ is only in terms of
production. The similarity in production
(c) Home is not a new invention of anyone at does not achieve commercial profit.
any stage of development.
(d) For the same commercial profit wines need
So choice (d) is the best answer. Home is an not be produced in the same of similar manner.
institution just like ‘marriage. Home is a reality
only when its members accept certain principles. So, choice (a) is the best. It does not mention
that the wines of similar profit are similarily
8. (a) A man’s toil may help to make out wealth, produced or have semblance with one another.
but it is not the ultimate aim.
2.10 Sentence Completion
14. (a) A sociological analysis does not stop at the 46. An argument is always ‘resolved’, it is never
outer limits of the subjects. thwart, defeat or closed.
(b) No one can indicate how far into the subject 47. Going by options makes choice easier. Only “how
can a sociological analysis go. the process allowed.......” suits logically.
(c) ‘Relative distance’ refers to the distance of 48. ‘Blunt’ means `very direct’ with respect to a
something else which is not mentioned. remark. ‘Devious’ in choice (d) means ‘dishonest’
So, choice (d) is the best. This indicates the and is incorrect as the statement is a frank one.
probable distance to which the sociological For the same reason ‘tactful’ in choice (c) is also
analysis can go to. not appropriate.
30. Choices (a) and (d) are ruled out because ‘suggest’ 49. ‘Interest’ is the most appropriate work because
does not go with ‘to+ verb’ form. Choice (2) cannot the original amout he owed to the department
fit in the blank because of the error in accumulated to the said figure. ‘Taxes is
parallelism. The appropriate structure is ‘‘Many redundant while. ‘principal’ usually refers to a
people suggest.....and others advocate loan (in this case it isn’t). ‘Returns’ is what one
........’’. gets rather than pays.
31. The tense in choices (a) and (b) is not consistent. 50. ‘Sanctions is used for an official permission or
Moreover, choice (a) uses both ‘essential’ and order that limits trade and is rarely used for an
‘must’. thus rendering either one of these words amout. ‘Fees’ are paid for a service and ‘refunds’
redundant. In choice (d) ‘things’ the plural form are what one gets back. ‘Fines is the best choice
cannot take it’. because of the penalty component.
32. Choice (d) fits into the blank in choice (a) the
51. ‘Attach’ is the word used for officially taking
usage of the indefinite article is erroneous.
something away. ‘Impound’ means ‘confiscate’
Choices (b) and (c) are erroneous due to the
incorrect ordering of words. and is generally used in the context of illegal
goods or contraband. Other choices are in no
33. ‘Disingenuous’ means ‘insincere’ and this best
way comparable.
fits the context as the phrase ‘ethnic cleansing’
is marked by apostrophes indicating a 52. ‘Smashed in choice (a) and ‘dismantled’ in choice
misrepresentation. Also, the word ‘victims’ is (c) are far from being logical. ‘Frozen is a wrong
best in contrast to ‘perpetrators’. Hence, option choice. ‘Frozen is used in the context of making
(c) is correct. bank accounts inoperable etc. Automobiles are
36. ‘Navigators, calendar makers’ are best clubbed seized.
under the category of ‘observers’ rather than 53. Clearly the topic is about a person who has
‘scrutinizers’, as the latter term is more used to violated incom tax regulations and he should be
refer to specialists in a particular field. ‘Students’ called an ‘offender’ of the law.
can also fit to a certain extent and this leaves 55. Choices (c) and (d) have similar meaning and
us with options (b), (d). ‘Concede’ has more to are misnomers in the given context, as the
do with defeat or revision which is more apt in keywords ‘lowpitched voices’ require a word that
this context than ‘agree’ or ‘conclude’. Hence,
is synonymous with ‘quiet’ or ‘calm’. The usage
option (c) is correct.
of the adjective ‘faded’ in the sentence to describe
42. Going by options, ‘regrettably’ doesn’t fit as there men is untenable.
is nothing to regret. ‘Firstly’ is no choice and
nothing ‘secondly’ is mentioned. Paragraph has 56. The given sentence depicts the President as a
nothing aggresive tone so ‘obviously’ does not very angry person as suggested by the keyword
suit. ‘Apparently’ is only and most suitable option. ‘scowled’ Choices (a), (b) and (c) are either
43. All options are suitable for the space but to positive or neutral and cannot fit in the sentence.
choose one, the best, is required here 57. ‘Paced’ in choice (a) is not consistent with the
‘Analytically’ appears to be best. It expresses to mood of the President as depicted in the first
analyse something using statistically proven part of the sentence. It means ‘to walk fast’.
methods to reach a decision. ‘Strolled’ in (b) means ‘Walk casually’ and hence
44. It is a very obvious choice determined from the is discarded. ‘Stormed’ in choice (c) refers to
paragraph giving it an access. ‘attack a place suddenly’ or ‘say something in a
45. ‘Firing’ comes as best suited option as paragraph loud, angry way’. But here, neither of them fits
explains it as a subject of discussion. in. ‘Prowled’ refers to moving in a restless
manner and hence fits.
3
CHAPTER Synonyms
INTRODUCTION
A Synonym may be defined as a word that has the same meaning or the same general meaning as a particular
word in the same Language or is in some applications a more or less satisfactory application of it. Thus the
synonym of a word can be an exact equivalent in meaning or should atleast convey its meaning in one way or
the other or should show in what sense it can best be used.
EXERCISE
MCQ TYPE QUESTIONS 12. Abut
Directions: Each of the questions below consists of a (a) stimulate (b) grasp
word followed by four lettered words or phrases. Choose (c) oppose (d) adjoin
the lettered word or phrase that is most nearly same
13. Abysmal
in meaning to the word.
(a) bottomless (b) eternal
1. Abase
(c) meteric (d) diabolic
(a) incur (b) tax
14. Accede
(c) estimate (d) humiliate
(a) fail (b) compromise
2. Aberration
(c) correct (d) consent
(a) deviation (b) abhorrence
15. Acclivity
(c) dislike (d) absence
(a) index
3. Abet
(b) report
(a) conceive (b) wager
(c) upslope of a hill
(c) encourage (d) evade
(d) negotiator
4. Aabeyance
16. Astute
(a) obedience (b) discussion
(a) sheer (b) noisy
(c) excitement (d) suspended action
(c) astral (d) clever
5. Abjure
17. Atrocity
(a) discuss (b) renounce
(a) endurance (b) fortitude
(c) run off secretly (d) perjure
(c) session (d) heinous act
6. Ablution
18. Atrophy
(a) censure (b) forgiveness
(a) capture (b) waste away
(c) mutiny (d) washing
(c) govern (d) award prize
7. Abnegation
19. Attenuate
(a) blackness (b) self-denial
(a) appear (b) be absent
(c) selfishness (d) cause
(c) weaken (d) testify
8. Aborigine
20. Atypical
(a) first design
(a) superfluous (b) fortitude
(b) absolution
(c) unusual (d) clashing
(c) finale
21. Audacity
(d) primitive inhabitant
(a) boldness (b) asperity
9. Abortive
(c) strength (d) stature
(a) unsuccessful
22. Augment
(b) consuming
(a) make noble (b) anoint
(c) financing
(c) increase (d) harvest
(d) familiar
23. Auxiliary
10. Abstinence
(a) righteous (b) prospective
(a) restrained (b) vulgar display
(c) assistant (d) archaic
(c) deportment (d) reluctance
24. Avarice
11. Abstruse
(a) easiness (b) greed
(a) profound (b) irrespective
(c) statement (d) invoice
(c) suspended (d) protesting
Synonyms 3.5
ANSWERS
MCQ Type Questions
1. (d) 2. (a) 3. (c) 4. (d) 5. (b) 6. (d) 7. (b) 8. (d) 9. (a) 10. (a)
11. (a) 12. (d) 13. (a) 14. (d) 15. (d) 16. (d) 17. (d) 18. (b) 19. (c) 20. (c)
21. (a) 22. (c) 23. (c) 24. (b) 25. (c) 26. (d) 27. (c) 28. (c) 29. (b) 30. (d)
31. (b) 32. (b) 33. (d) 34. (b) 35. (b) 36. (a) 37. (d) 38. (b) 39. (c) 40. (d)
41. (b) 42. (a) 43. (a) 44. (c) 45. (d) 46. (d) 47. (d) 48. (c) 49. (c) 50. (d)
51. (a) 52. (d) 53. (b) 54. (d) 55. (a) 56. (d) 57. (c) 58. (b) 59. (a) 60. (b)
61. (b) 62. (a) 63. (a) 64. (a) 65. (c) 66. (c) 67. (d) 68. (d) 69. (b) 70. (b)
71. (d) 72. (d) 73. (a) 74. (b) 75. (a) 76. (d) 77. (d) 78. (d) 79. (a) 80. (a)
81. (b) 82. (c) 83. (b) 84. (c) 85. (c) 86. (b) 87. (d) 88. (a) 89. (a) 90. (d)
91. (a) 92. (c) 93. (d) 94. (d) 95. (b) 96. (c) 97. (d) 98. (c) 99. (d) 100. (a)
101. (d) 102. (c) 103. (a) 104. (b) 105. (a) 106. (b) 107. (b) 108. (b) 109. (d) 110. (a)
111. (a) 112. (c) 113. (c) 114. (c) 115. (d) 116. (d) 117. (a) 118. (a) 119. (b) 120. (b)
121. (b) 122. (c) 123. (c) 124. (c) 125. (d) 126. (c) 127. (b) 128. (d) 129. (d) 130. (d)
131. (a) 132. (b) 133. (d) 134. (c) 135. (a) 136. (d) 137. (b) 138. (d) 139. (d) 140. (b)
141. (b) 142. (c) 143. (d) 144. (a) 145. (d) 146. (b) 147. (d) 148. (c) 149. (a) 150. (c)
151. (a) 152. (b) 153. (b) 154. (b) 155. (a) 156. (a) 157. (c) 158. (c) 159. (c) 160. (b)
161. (d) 162. (d) 163. (d) 164. (a) 165. (b) 166. (a) 167. (b) 168. (d) 169. (a) 170. (b)
171. (d) 172. (c) 173. (a) 174. (b) 175. (d) 176. (b) 177. (a) 178. (d) 179. (b) 180. (c)
181. (d) 182. (b) 183. (b) 184. (d) 185. (c) 186. (c) 187. (b) 188. (a) 189. (c) 190. (c)
191. (a) 192. (b) 193. (d) 194. (c) 195. (a) 196. (b) 197. (a) 198. (a) 199. (d) 200. (a)
201. (b) 202. (d) 203. (d) 204. (b) 205. (c) 206. (c) 207. (b) 208. (a) 209. (d) 210. (d)
4
C HAPTER Antonyms
The basic idea of a antonym question is to find an 6. Sometimes the test writer selects a word you are
opposite for a word. Antonym items are first and likely to be familiar with, but sets up the question
foremost a test of vocabulary. to test a meaning you do not ordinarily associate
If you have no idea of the meaning of the given word, with the word. Here is an example:
there’s not much you can do. When you don’t know the e.g. precipitous
meaning of an antonym, don’t waste a lot of time trying (a) pleasantly sweet
to fiture it out. In other words, once you recognize that (b) overly ambitious
you are out of ammunition, just do a random guess, (c) agreeably situated
confirm it, and move on the next item.
(d) gently sloping
Each antonym starts with a capitalized word, which
is followed by four words of phrases in lowercase The best choice is (d). We most often use the word
letters. The task is to find the word or phrase that is precipitous to mean rash or foothardy, but its
most nearly opposite in meaning to the bold word. central meaning is related to precipice, a sharp
drop-off.
Plan to Answer Antonym Questions
7. Eleminate non-answers
1. Define the capitalized word.
The simple way to eleminate answer choices is
2. Think of a meaning that is opposite to this word. to toss out any words that don’t have opposite
3. Read all the answer choices. Eliminate those that meanings.
do not relate to the meaning you thought of. If 8. Concentrate on answer choices
only one choice remains, mord it and go on. Words can have different meanings based upon
4. If more than one choice remains, go back and their part of speech. If you are uncertain about
refine your thinking about the bold word. the part of speech of the bold word, just check
5. Pick the answer choice that is most nearly opposite the naswer choices. If they’re all verbs, for
to the bold word. example, so is the bold word.
Tedious 9. Play around with the part of speech of a
(a) unlimited (b) confined word
(c) enthralling (d) appetizing You can alter the part of speech of the bold word
and answer choices in your own mind. Sometimes
(i) The capitalized word means “boring” or
an antonym will use a word you know but as a
“tiresome”
part of speech that is unfamiliar to you.
(ii) The opposite of “boring” would be something e.g. sublimity
like exciting or interesting.
(a) erosion (b) baseness
(iii) Looking at all the answer choices, you can
immediately eliminate (a), and (b) as having (c) conciseness (d) insistence
nothing to do with excitement. But that leaves The best choice is (b). You may know the word
(c) enthralling and (d) apperizing. sublimity better as the adjective sublime, meaning
(iv) Of the two remaining choices, enthralling, “lofty, high, or noble”. So you may find it easire
which means “captivating” or fascinating,” is to think about the antonym by changing sublimity
to the more familiar form sublime”. As you think
more nearly opposite in meaning to tedious
about each answer choice, you would then change
than appetizing, which means “savory,” or
it in your mind to an adjective Baseness, therefore,
“delicious”.
would become base; and base is an opposite of
(v) Mark (c), enthralling, as the correct choice. sublime.
Note : 10. Use word connotations to eliminate answer
(i) Makes sure to read all the answers : To choices
answer antonym questions you’ll sometimes Even if you don’t know the exact meaning of a
have to distinguish fine shades of meaning. word, you may have a vague recollection of the
That’s why you have to consider very answer context in which you first encountered it. So you
choice. may know whether the word has positive
(ii) Verb or Noun? If the part of speech of the overtones or negative ones. This recollection may
capitalized word is ambiguous, check the be sufficient to get a correct answer.
answer choices.
4.2 Antonyms
EXERCISE
MCQ TYPE QUESTIONS
Directions : Each question below consists of a word printed in capital lettes, followed by four lettered words of
phrases. Choosed the lettered word or phrase that is most nearly opposite in meaning to the word in bold letters.
1. Abrogate 12. Aloof
(a) transgress (b) signify (a) triangular (b) gregarious
(c) allevate (d) ratify (c) comparable (d) honorable
2. Admonition 13. Amalgamate
(a) premonit ion (b) hallucination (a) equip (b) separate
(c) escape (d) commendation (c) generate (d) materialize
3. Adulation 14. Ambiguous
(a) youth (b) purity (a) salvageable (b) corresponding
(c) brightness (d) criticism (c) responsible (d) clear
4. Advocate 15. Amble
(a) define (b) oppose (a) befriend (b) hasten
(c) remove (d) inspect (c) steal (d) browse
5. Affable 16. Ambulatory
(a) rude (b) ruddy (a) convalescent (b) valedictory
(c) needy (d) useless (c) bedridden (d) emergency
6. Affecfted 17. Ameliorate
(a) weary (b) unfriendly (a) make slow (b) make sure
(c) divine (d) unfeigned (c) make young (d) make worse
7. Affluence 18. Amicable
(a) poverty (b) fear (a) penetrating (b) compensating
(c) persuasion (d) consideration (c) unfriendly (d) zigzag
8. Agility 19. Amoral
(a) awkwardness (b) solidity (a) unusal (b) unfriendly
(c) temper (d) harmony (c) ethical (d) suave
9. Alacrity 20. Amorphous
(a) slowness (b) plenty (a) nauseous (b) obscene
(c) fifth (d) courtesy (c) providential (d) definite
10. Alleviate 21. Amplify
(a) endure (b) worsen (a) distract (b) infer
(c) enlighten (d) maneuver (c) publicize (d) decrease
11. Allure 22. Analogous
(a) hinder (b) repel (a) not comparable (b) not capable
(c) ignore (d) leave (c) not culpable (d) not corporeal
Antonyms 4.3
23. Anthematize 41. Bigotry
(a) locate (b) deceive (a) arrogance (b) approval
(c) regulate (d) bless (c) mourning (d) tolerance
24. Anemic 42. Bizarre
(a) pailid (b) cruel (a) roomy (b) veiled
(c) red-blooded (d) ventilating (c) subduced (d) normal
25. Animated 43. Blanch
(a) worthy (b) dull (a) bleach (b) scatter
(c) humorous (d) lengthy (c) darken (d) analyze
26. Animus 44. Bland
(a) pterodactyl (b) bastion (a) caustic (b) meager
(c) giraffe (d) favor (c) soft (d) uncooked
27. Anomaly 45. Blase
(a) desperation (b) requisition (a) fiery (b) clever
(c) registry (d) regularity (c) intriguing (d) ardent
28. Anonymous 46. Bleak
(a) desperate (b) signed (a) pale (b) sudden
(c) defined (d) expert (c) dry (d) cheerful
29. Antediluvian 47. Blithe
(a) transported (b) subtle (a) spiritual (b) profuse
(c) isolated (d) modern (c) cheerless (d) hybrid
30. Antipathy 48. Capacious
(a) profundity (b) objection (a) warlike (b) cordial
(c) willingness (d) foundness (c) curious (d) not spacious
31. Antithesis 49. Capricious
(a) velocity (b) maxim (a) satisfied (b) insured
(c) similarity (d) acceleration (c) photographic (d) steadfast
32. Aphasia 50. Captious
(a) volubility (b) necessity (a) tolerant (b) capable
(c) pain (d) crack (c) frivolous (d) winning
33. Baroque 51. Carnal
(a) polished (b) constant (a) impressive (b) minute
(c) transformed (d) simple (c) spiritual (d) actual
34. Beatific 52. Carnivorous
(a) glorius (b) dreadful (a) gloomy (b) tangential
(c) theatrical (d) crooked (c) productive (d) vegetarian
35. Belittle 53. Carping
(a) disobey (b) forget (a) rapid (b) uncritical
(c) magnify (d) extol (c) unintellectual (d) illegal
36. Bellicose 54. Castigation
(a) peaceful (b) naval (a) commendation (b) patience
(c) amusing (d) piecemeal (c) generosity (d) understatement
37. Benign 55. Categorical
(a) tenfold (b) peaceful (a) negative (b) ironic
(c) blessed (d) malignant (c) impartial (d) qualified
38. Benison 56. Catholic
(a) curse (b) bachelor (a) religious (b) pacific
(c) wedding (d) orgy (c) narrow (d) weighty
39. Berate 57. Celerity
(a) grant (b) praise (a) assurance (b) state
(c) refer (d) purchase (c) acerbity (d) delay
40. Bestial 58. Celibate
(a) animated (b) noble (a) investing (b) married
(c) zoological (d) clear (c) retired (d) commodious
4.4 Antonyms
59. Censure 77. Debonair
(a) process (b) enclose (a) awkward (b) windy
(c) interest (d) praise (c) balmy (d) strong
60. Centrifugal 78. Derogatory
(a) centripetal (b) ephemeral (a) roguish (b) immediate
(c) lasting (d) barometric (c) openionated (d) praising
61. Cessation 79. Desecrate
(a) premium (b) gravity (a) desist (b) integrate
(c) beginning (d) composition (c) confuse (d) consecrate
62. Chaffing 80. Despicable
(a) achieving (b) serious (a) streeing (b) worthy of esteem
(c) caitalistic (d) sneezing (c) inevitable (d) featureless
63. Coy 81. Destitute
(a) weak (b) airy (a) affluent (b) dazzling
(c) brazen (d) old (c) stationary (d) characteristic
64. Cozen 82. Devoid
(a) amuse (b) treat honestly (a) latent (b) eschewed
(c) prate (d) shackle (c) full of (d) suspecting
65. Craven 83. Devout
(a) desirous (b) direct (a) quiet (b) dual
(c) bold (d) civilized (c) impious (d) straightforward
66. Crux 84. Diabolical
(a) affiliction (b) spark (a) mischievous (b) lavish
(c) events (d) trivial point (c) seraphic (d) redolent
67. Cryptic 85. Diatribe
(a) tomblike (b) futile (a) mass (b) range
(c) famous (d) candid (c) eulogy (d) elegy
68. Cupidity 86. Diffidence
(a) anxiety (b) tragedy (a) sharapness (b) boldness
(c) generosity (d) entertainment (c) malcontent (d) dialogue
69. Curtail 87. Dilate
(a) mutter (b) lengthen (a) procrastinate (b) contract
(c) express (d) burden (c) conclude (d) participate
70. Cynical 88. Dilatory
(a) trusting (b) effortless (a) narrowing (b) prompt
(c) conclusive (d) gallant (c) enlarging (d) portentous
71. Dank 89. Diminution
(a) dry (b) guiltless (a) expectation (b) context
(c) warm (d) babbling (c) validity (d) appreciation
72. Dapper 90. Din
(a) unintelligent (b) untidy (a) lightness (b) safety
(c) uncertain (d) ungrateful (c) silence (d) hunger
73. Dauntless 91. Disabuse
(a) stolid (b) cowardly (a) crash (b) violate
(c) irrelevant (d) peculiar (c) renege (d) deceive
74. Dearth 92. Disconsolate
(a) life (b) abundance (a) unprejudiced (b) thankful
(c) brightness (d) terror (c) threatrical (d) joyous
75. Debacle 93. Enervate
(a) progress (b) refusal (a) strengthen (b) sputter
(c) masque (d) cowardice (c) arrange (d) scrutinize
76. Debilitate 94. Enhance
(a) bedevil (b) repress (a) degrade (b) doubt
(c) strengthen (d) animate (c) scuff (d) gasp
Antonyms 4.5
95. Ennui 113. Execrable
(a) hate (b) excitement (a) innumerable (b) philosophic
(c) seriousness (d) humility (c) physical (d) excellent
96. Enunciate 114. Excrate
(a) pray (b) request (a) decipher (b) sadden
(c) deliver (d) mumble (c) integrate (d) admit
97. Ephemeral 115. Exhume
(a) sensuous (b) passing (a) decipher (b) sadden
(c) popular (d) eternal (c) integrate (d) inter
98. Equable 116. Exodus
(a) flat (b) decisive (a) neglect (b) consent
(c) stormy (d) dishonest (c) entry (d) gain
99. Equanimity 117. Exonerate
(a) agitation (b) stirring (a) forge (b) accuse
(c) volume (d) identity (c) record (d) doctor
100. Equilibrium 118. Exorbitant
(a) imbalance (b) peace (a) moderate (b) partisan
(c) inequity (d) directness (c) military (d) barbaric
101. Equitable 119. Extemporaneous
(a) able to leave (b) able to learn (a) rehearsed (b) hybrid
(c) unfair (d) preferable (c) humilating (d) satetesmanlike
102. Equivocal 120. Extraneous
(a) mistaken (b) quaint (a) modern (b) decisive
(c) azure (d) clear (c) essential (d) effective
103. Erratic 121. Extrinsic
(a) unromantic (b) free (a) reputable (b) inherent
(c) popular (d) steady (c) swift (d) ambitious
104. Erroneous 122. Extrovert
(a) accurate (b) dignified (a) clown (b) hero
(c) curious (d) abrupt (c) ectomorph (d) introvert
105. Erudite 123. Grandiose
(a) professorial (b) stately (a) false (b) ideal
(c) short (d) ignorant (c) proud (d) simple
106. Ethereal 124. Gratuitous
(a) long-lasting (b) earthy (a) warranted (b) frank
(c) ill (d) critical (c) ingenuous (d) frugal
107. Eulogistic 125. Gregarious
(a) pretty (b) critical (a) antisocial (b) anticipatory
(c) brief (d) stern (c) glorious (d) horrendous
108. Euphonious 126. Grisly
(a) strident (b) lethargic (a) suggestive (b) doubtful
(c) literary (d) significant (c) untidy (d) pleasant
109. Evasive 127. Gullible
(a) frank (b) correct (a) incredulous (b) fickle
(c) empty (d) fertile (c) tantamount (d) easy
110. Exasperate 128. Gusto
(a) confide (b) formalize (a) noise (b) panic
(c) placate (d) betray (c) atmosphere (d) distaste
111. Excoriate 129. Ghusty
(a) scandalize (b) encourage (a) clam (b) noisy
(c) avoid (d) praise (c) fragrant (d) routine
112. Exculpate 130. Hackneyed
(a) blame (b) prevail (a) carried (b) original
(c) acquire (d) ravish (c) banned (d) timely
4.6 Antonyms
131. Haggard 149. Hypocritical
(a) shrewish (b) inspired (a) sincere (b) narrow-minded
(c) plump (d) maidenly (c) shameful (d) amiable
132. Halcyon 150. Hypothetial
(a) wasteful (b) prior (a) rational (b) fantastic
(c) subsequent (d) martial (c) wizened (d) axiomatic
133. Haphazard 151. Ignoble
(a) safe (b) indifferent (a) produced by fire (b) worthy
(c) deliberate (d) tense (c) given to questioning(d) huge
134. Hapless 152. Illusive
(a) cheerful (b) consistent (a) not deceptive (b) not certain
(c) fortunate (d) considerate (c) not obvious (d) not choherent
135. Heed 153. Irksome
(a) ignore (b) hope (a) interesting (b) lazy
(c) overtake (d) nurture (c) tireless (d) devious
136. Heretic 154. Irrelevant
(a) sophist (b) believer (a) lacking piety (b) fragile
(c) interpreter (d) pacifist (c) congruent (d) pertinent
137. Heterogeneous 155. Irreparable
(a) orthodox (b) pagan (a) legible (b) correctable
(c) unlikely (d) similar (c) proverbial (d) concise
138. Hibernal 156. Irtreverent
(a) musical (b) summerlike (a) related (b) mischievous
(c) local (d) seasonal (c) respective (d) pious
139. Hilarity 157. Jaded
(a) gloom (b) heartiness (a) upright (b) stimulated
(c) weakness (d) casualty (c) aspiring (d) applied
140. Hirsute 158. Jaundiced
(a) scaly (b) bald (a) whitened (b) inflamed
(c) erudite (d) quiet (c) quickened (d) unbiased
141. Hortatory 159. Jaunty
(a) inquiring (b) denying (a) youthful (b) ruddy
(c) killing (d) dissuading (c) strong (d) sedate
142. Hovr 160. Jeopardy
(a) commence (b) soothe (a) patience (b) courage
(c) leave (d) transform (c) safety (d) willingness
143. Hunbbub 161. Jettison
(a) calm (b) fury (a) salvage (b) submerge
(c) capital (d) axle (c) descend (d) decelerate
144. Hummock 162. Jocular
(a) ummusical (b) scorn (a) arterial (b) bloodless
(c) wakefulness (d) vale (c) verbose (d) serious
145. Husbandry 163. Judicious
(a) sportsmanship (b) dishonesty (a) punitive (b) unwise
(c) wastefulness (d) friction (c) criminal (d) licit
146. Hybrid 164. Kindle
(a) productive (b) special (a) dislike (b) quench
(c) purebred (d) oafish (c) gather (d) sparkle
147. Hyperbole 165. Lachrymose
(a) velocity (b) climax (a) cheering (b) smooth
(c) curve (d) understatement (c) passionate (d) curt
148. Hypercritical 166. Lackadasical
(a) tolerant (b) false (a) monthly (b) possessing time
(c) extreme (d) inarticulate (c) ambitious (d) pusillanimous
Antonyms 4.7
ANSWERS
MCQ Type Questions
1. (d) 2. (b) 3. (d) 4. (b) 5. (a) 6. (d) 7. (a) 8. (a) 9. (a) 10. (b)
11. (b) 12. (b) 13. (b) 14. (d) 15. (b) 16. (c) 17. (d) 18. (c) 19. (c) 20. (d)
21. (d) 22. (a) 23. (d) 24. (c) 25. (b) 26. (d) 27. (b) 28. (b) 29. (d) 30. (d)
31. (c) 32. (a) 33. (d) 34. (b) 35. (d) 36. (a) 37. (d) 38. (a) 39. (b) 40. (b)
41. (d) 42. (d) 43. (c) 44. (a) 45. (d) 46. (d) 47. (c) 48. (d) 49. (d) 50. (a)
51. (c) 52. (d) 53. (b) 54. (a) 55. (d) 56. (c) 57. (b) 58. (a) 59. (b) 60. (a)
61. (c) 62. (b) 63. (c) 64. (b) 65. (c) 66. (d) 67. (d) 68. (c) 69. (b) 70. (a)
71. (a) 72. (b) 73. (b) 74. (b) 75. (a) 76. (c) 77. (a) 78. (d) 79. (d) 80. (b)
81. (a) 82. (c) 83. (c) 84. (d) 85. (c) 86. (b) 87. (b) 88. (b) 89. (d) 90. (c)
91. (d) 92. (d) 93. (a) 94. (a) 95. (b) 96. (d) 97. (d) 98. (c) 99. (a) 100. (a)
101. (c) 102. (b) 103. (d) 104. (a) 105. (d) 106. (b) 107. (b) 108. (a) 109. (a) 110. (c)
111. (d) 112. (a) 113. (d) 114. (b) 115. (d) 116. (c) 117. (b) 118. (a) 119. (b) 120. (c)
121. (b) 122. (d) 123. (d) 124. (a) 125. (a) 126. (d) 127. (a) 128. (d) 129. (d) 130. (b)
131. (c) 132. (d) 133. (c) 134. (c) 135. (a) 136. (b) 137. (d) 138. (b) 139. (a) 140. (b)
141. (d) 142. (c) 143. (a) 144. (d) 145. (c) 146. (c) 147. (d) 148. (a) 149. (a) 150. (d)
151. (b) 152. (a) 153. (a) 154. (d) 155. (b) 156. (d) 157. (b) 158. (d) 159. (d) 160. (c)
161. (a) 162. (d) 163. (a) 164. (b) 165. (a) 166. (c) 167. (d) 168. (b) 169. (a) 170. (d)
171. (d) 172. (c) 173. (b) 174. (d) 175. (d) 176. (d) 177. (c) 178. (b) 179. (a) 180. (d)
181. (d) 182. (a) 183. (c) 184. (b) 185. (a) 186. (c) 187. (c) 188. (d) 189. (b) 190. (c)
191. (c) 192. (a) 193. (c) 194. (d) 195. (c) 196. (b) 197. (d) 198. (d) 199. (d) 200. (b)
201. (a) 202. (c) 203. (a) 204. (c) 205. (b) 206. (d) 207. (d) 208. (b) 209. (a) 210. (a)
211. (c) 212. (d) 213. (b) 214. (d) 215. (c) 216. (d) 217. (b) 218. (a) 219. (a) 220. (c)
221. (c) 222. (c) 223. (b) 224. (c) 225. (b) 226. (d) 227. (c) 228. (a) 229. (d) 230. (a)
231. (c) 232. (d) 233. (d) 234. (c) 235. (c) 236. (d) 237. (d) 238. (c) 239. (a) 240. (a)
241. (c) 242. (b) 243. (a) 244. (a) 245. (c) 246. (d) 247. (b) 248. (a) 249. (d) 250. (d)
251. (c) 252. (b) 253. (b) 254. (d) 255. (d) 256. (b) 257. (a) 258. (d) 259. (a) 260. (b)
261. (c) 262. (d) 263. (c) 264. (b) 265. (a) 266. (d) 267. (a) 268. (b) 269. (b) 270. (b)
271. (a) 272. (d) 273. (b) 274. (c) 275. (d) 276. (b) 277. (a) 278. (a) 279. (c) 280. (a)
281. (b) 282. (c) 283. (d) 284. (d) 285. (d) 286. (d) 287. (d) 288. (c) 289. (b) 290. (d)
291. (a) 292. (d) 293. (d) 294. (a) 295. (d) 296. (b) 297. (c) 298. (a) 299. (a) 300. (c)
301. (b) 302. (c) 303. (d) 304. (a) 305. (d) 306. (b) 307. (d) 308. (d) 309. (a) 310. (a)
311. (b) 312. (c) 313. (d) 314. (b) 315. (b) 316. (a) 317. (a) 318. (d) 319. (d) 320. (d)
321. (c) 322. (b) 323. (d)
5
CHAPTER
Miscellaneous
Bacchanalian : Orgy; referred to a festival which
VOCABULARY FOR READING was celebrated with dancing, song and revelry.
COMPREHENSION Blasphemy : The act of insulting or showing
Not appreciating the subtle differences between words contempt or lack of reverence towards God.
like - sarcastic and sardonic, ironical, cynical,... Example : Blasphemy and independent thinking are
(the list goes on), many threaten your chances of a very often confused.
high score in R.C. (especially in inferential and true
tone passages). Empiricism : The practice of relying on observation
and experiment especially in the natural sciences.
The beauty of English gets more enhanced if
understood in the right perspective. On the other hand, A theory that all knowledge originates in experience.
one word misunderstood many stand you in for a Example : An empirical analysis of the situation
comical interpretation of a serious passage. The yields different results.
‘Vocabulary for R.C.' section becomes a unique section, Critical (for somebody/something) : It means
as it tries to encompass a kaleidoscope of tones, indicating faults in someone.
moods, opinions, characteristics, analogies which have
statistically found a higher representation in the R.C. The jehadis were quite critical of Pakistan supporting
passages of management entrance exams. US against Afghanistan.
Anarchist : A person who does not believe in any Critique : Critical analysis
kind of system, law and order, governance and goes Example: Presented in this column is a critique of
an extra mile to prove his view on political disorder. the govt's economic policies.
Agnostic : A person who is unsure of the existence Cynic : A person who believe that people do not do
of God. Unlike atheists he is a little flexible with things for good, sincere or noble reasons, but only for
respect to his belief about God, but still unsure. their own advantage: concerned only with one's own
Atheism/atheist : A belief in the non-existence of interests.
God. Example : Stop giving these cynical smiles .
Authoritarian : A person following complete Example : Women in Afghanistan had over a period
obedience to authority. Hitler is a good example of time started viewing the cynical Taliban regime with
the kind. much apprehension.
Example : The authoritarian regime of Taliban finally Example : His adventurism was very close to being
met its nemesis.
termed as a cynical disregard for the safety of others.
Analytical : Characterized by logical and reasonable
Contemplative : Pondering over or considering
basis. Analytically sound managers - need of the hour.
thoughtfully.
Critic : A person who shows faults in something and
Example : Contemplate changing careers.
usually expresses a base/low opinion.
Commendation : Praise or approval.
Example : Saurav Ganguly's critics have suddenly
found a new voice, after his litany of failures with Castigating : To chastise, punish or to criticize
shortpitch ball. A person who judges the quality of severely.
something especially works of art, literature, music, Example : Hansie Cronje's castigation by the sports
etc. media.
Example : He is a music critic, i.e. he judges Disparagement : The art or process of speaking in
Callous : Having or showing no sympathy for other a belittling way; decrying, or reducing someone in
people's feelings or suffering. rank or esteem.
Example : A callous disregard for the feeling of
others.
5.2 Miscellaneous
Example : Disparaging someone's character or Example : To call CAT exam, a catwalk would be a
achievements. misnomer.
Disinterested (noun) : Freedom from bias or self- Mystic : Something/someone which has a hidden
interest, impartiality meaning or spiritual power and arouses feeling of deep
Example : A disinterested act of kindness. Different respect; and wonder.
from uninterested being without an interest. Example : Mystical writings.
Disillusion (ed) : To destroy the but mistaken beliefs Mysticism : To believe that knowledge of God and a
or ideals. real truth may be reached by directing one's mind
Example : Disillusioned voters want an alternative through spiritual insight.
to the two parties.
Disingenuous : Not straightforward; crafty.
NARRATIVE SEQUENCE
Exploratory : To investigate systematically; to Narrative sequence is used to identify what happens
examine. at the beginning, middle and end of a sample
Example : Explore every suggestion given. paragraph and also identify the transition word in a
given paragraph. Places of these identify words in
Ecclesiastical : Of or relating to a church, as an
given sentences used for to create a story.
organized institution.
Example : The period witnessed strong anti- When we read, it is important to be able to retell the
ecclesiastical view from the masses. events in a story. Those events need to be retold in
the correct order for the story to make sense. Here,
Eclectic : Selecting what appears to be the best form
diverse sources systems, or styles. An eclectic we will learn how to sequence events in a narrative.
approach. Notes:
Example : Different from the non-holistic:
emphasizing the importance or the whole and the
interdependence of its parts.
Frivolous : Something which demands lesser
important or someone who is lacking in seriousness.
Example : The Zimbabweans are not frivolous when
it comes to playing them in their own soil.
Indignation : Anger aroused by something unjust,
unworthy, or mean.
Example : Kapil Dev's indignation is just.
Inquisitive : Someone who has the propensity to ask
questions.
Sequence of Events
Example : My first class in the college saw many
Have you ever seen a really good movie? Maybe it
inquisitive students trying to unnerve me.
was an action-packed superhero movie or a fantasy
Ironic : Something which happens in the opposite adventure to a far-off land. When a director creates a
way to what expected and often causes amusement new film, he or she has to make sure the scenes help
because of this. the movie make sense from the opening scene until
Example : It is ironic that he died of AIDS, though the credits start rolling.
he was the one who discovered the first AIDS vaccine. Likewise, as a reader, it’s important for you to identify
Myth : It is something untrue. the sequence of events in a story. The sequence of
Mystify : To puzzle someone/something. events is the order of events as they occur in a story.
Morose : Unhappy and bad tempered; but not saying Just like when the heroes can’t stop the world-ending
doomsday machine before it’s been built in a superhero
much.
movie, the sequence of events in story occurs in a
Example: Morose mannerisms. precise order so that it makes sense to the person
Moron : A very stupid person. reading it.
Misnomer : A wrong use of a name, word or So how do you identify the sequence of events? Get
description. your popcorn because it’s showtime!
Miscellaneous 5.3
Organize the Story and easy to remember. Choose at least three stories
Example Sequence of Events Chart and divide them into their beginning, middle, and end
sections. Check back on this lesson if you get stuck.
The Boy Who Cried Wolf
When you feel confident that you’ve divided the stories
Beginning Middle End properly, get a family member or teacher to check
your work.
A wolf was The towns A wolf really did
eating the people came to approach the boy Story Scramble: Medium Level
sheep of a help and found There is more to a story than just beginning, middle,
town out the boy was
and end. Many events can happen in a single story,
lying.
just like in the example of the tortoise and the hare
The boy was The boy did this The boy cried for in this story. Think of a book that you’ve recently
bored of a couple more help, but no one read. On one piece of paper, write a clear list of events
watching the time. came because in the story. Now, cut them up into separate sentences
sheep they thought he
and mix them all up. Can you put them back together
was lying again
again in the right order? Make sure that the end result
He decided to The town’s The wolf of your activity makes logical sense.
pretend the people were attacked the boy
Write Your Own: Hard Level
wolf was upset. and the sheep
attacking the If you feel really confident in your understanding of
sheep. narrative structure, try writing your own short story.
Think carefully about the order of events as you write
Movie directors often use storyboards to help them them. What does the audience have to be aware of for
visualize what will happen in each scene. Storyboards the events to make sense? What is your story’s
show what will happen in each camera shot just like beginning, middle, and end? Consider these questions
the panels of a comic book tell a story. As a reader, as you write.
you can also think of the sequence of events like a Narrative Sequencers:
storyboard.
Narrative Sequencer is nothing but a word which tells
After you’ve read a story, you can organize the story
you where in the paragraph transition words will take
into three sections and ask yourself the following
place.
questions:
Steps of sequencers:
a) What happens at the beginning of the story?
1. First
b) What happens in the middle of the story?
2. Next
c) What happens at the end of the story?
3. Then
The beginning of the story usually introduces the main
characters, setting, and problem faced by or purpose 4. After that
of the characters. The middle part of the story usually 5. Finally
focuses on the characters as they try and solve the Example 1:
problem. The end of the story focuses on how the Add sequencers in the correct spot in the following
problem is resolved, or the end of the characters’ sentence:
journey.
I’ll buy coffee and go to the movie.
It may help to create a chart, like this one, to help
(a) I’ll buy first coffee and go then to the movie
you organize the different events that make up the
beginning, middle, and end of a story. (b) Then, I’ll buy coffee and first go to the movie
Sequence of Events: (c) First, I’ll buy coffee and then go to the movie
Here we taught the importance of understanding the (d) I’ll buy coffee and first go to the movie
events of a story in order. Anser : (c)
Structure of a Narrative: Easy Level Example 2:
As this lesson explained, there are certain things that Andrea loves to play soccer, but she broke her arm
usually happen at the beginning, middle, and end of and cannot play for a while. Since breaking her arm,
a story. Think of some stories that you are very Andrea has been very sad. A friend of hers suggested
familiar with. Nursery rhymes and fairy tales are a that Andrea find another hobby to take her mind off
great place to start because they tend to be simple of her injury. Andrea took her friend’s advice and tried
5.4 Miscellaneous
knitting. She loved it! Now, Andrea spends her time admonition, evidence, prosecution, advocate,
knitting while waiting for her arm to heal, and she forgery, reprieve, allege, gibbet, scourage,
isn’t sad anymore. attorney, incarceration, scoundrel, barrister,
(a) Andrea broke her arm and cannot play soccer indictment, sentences, borstal, judicial, surety,
for a while. circumstantial, culpable, perjury, vault, culprit
(b) Andrea’s friend suggested that she find another (2) Words related to death:
hobby, so she tried knitting bier, decease, requiem, burial, epitaph, rites,
cenotaph, obituary
(c) Andrea isn’t sad anymore.
(3) Words related to Human Nature:
(d) Andrea’s arm healed and she started playing
soccer again. arrogant, foppish, obsequious, beneficients,
garrulous, peculiar, benefactor, haughty,
Answer (b) propensity, benevolent, impudent, reckless,
SPEAKERS courteous, insolent, rogue, cringing, knave,
sheer, dauntless, malevolent, sober, deficient,
Banal :A person whose speech is dull, flat, boring,
malicious, suave, enthusiastic.
unimaginative
Mendacious, taunt, finical, obstinate, tantalise
Cogent : A person who can express his views
brilliantly & convincingly (4) Words related to Energy and Electricity:
Garrulous : A person who goes on speaking a lot, all aerial, crystal, galena, atmospheric, detector,
meaningless insulate, battery, distortion, leads, bias, ebonite,
neutralise, capacity, emitter, oscillate,
Grandiloquent : A person who always uses heavy
phenomenon, circuit, energy, condenser, ether,
words
radiating, current, frequency, radio, receiver.
Inarticulate : A person who is very awkward while Telephone, sensitiveness,
speaking
(5) Words related to Automobile:
Laconic : A person saying few words full of deep
accelerator, differential, piston, armature,
meaning
dynamo, radiator, axle, exhaust, silencer,
Loquacious : A person who speaks just too much, sparking plug, bonnet, gear, bearings, generator,
quantity wise sump, carburetor, ignition, switch, clutch,
Taciturn : An extremely quite person speaking very induction, tyres, cylinder, magneto, valves,
little Syn : Reticent/ Reserved/Close- lipped decarbonise, petrol, windscreen
Verbose : A person who repeats the same idea again (6) Words Related to Games and
and again Entertainment:
Ventriloquist : A person who can throw his voice aquatic, cycling, rehearsal, arena, hockey,
creating an illusion that it is coming from another sculling, athletics, opera, tennis, chess,
source pantomime, theatre, croquet, regatta, whistle
Voluble : A very fluent and rapid speaker (7) Words Related to Geography:
Vociferous : A very noisy speaker agriculture, frigid, plantation, arable, frontier,
Magniloquent : A person who talks big sedimentary, atoll, furrow, solar, barrier, glade,
survey, cascade, insular, swamp, climate,
Eloquent : A person who uses great, artistic or
inundation, temperature, confluence, irrigation,
emotional language
tornado, continent, knoll, torrential, contour,
Soporific : A person whose speech puts the audience mound, tillage, crater, meteor, typhoon,
to sleep. configuration, monsoon, vegetation, cyclone,
Egotist : A person who constantly boasts of his own oasis, verdure, eclipse, peninsular, zephyr,
achievements. expanse, planetary, zone
Soliloquist : A person talking to oneself. (8) Words related to Aviation:
JARGONS accessory, aviation, monoplane, aerodrome,
aviator, propeller, aeroplane, hydroplane,
These lists will help you remember groups of words pancake, rigid, skid, volplane, rudder, stunt
and their interconnections. This will help you solve
questions based on verbal analogies etc. (9) Words related to Metallurgy:
(1) Words Related to law: amalgam, filament, platinum, brazen, flux,
radium, copra, galvanise, shellac, concrete, kiln,
Miscellaneous 5.5
Malapropism Mrs Malaprop from Sheridan's play Bowdlerize (from Dr Bowdler, an art critic)
'theRivals' gained immortality by To make a book readable by
her verbal mistakes, a hilarious deleting offensive portions
misuse of words Boycott (from Captain Boycott) Ostracize,
Pickwickian Dicken's character of ' Tickwick break connections with
Papers”: Pickwickian words are Braille (from Louis Braille) A system
used in an esoteric special way (not of printing for the blind
in accordance with their usual Chauvinism (from Nicholas Chauvin)
meaning) Exaggerated loyalty or
Pooh-bah In Gilbert and Sullivan's opera The patriotism
Mikado' was the Lord', any Chimerical (from Greek monster
politician who holds several offices Chimaera) Ideal but impractical
is a pooh-bah
Cologne (from German city Cologne)
Quixotic Don Quixote was the lovable Famous as "Eau de cologne
impractical old man in Cervante's
Derrick (from a hangman Derrick) A
novel with the same title. Quixotic
hoisting apparatus
schemes are unrealistic
Fahrenheit (from D.G. Fahrenheit) Unit of
Robot Czech playwright karel Capek's
temperature measurement
play titled R. U. R (Rossum's
universal Robots); a robot is anyone Frankenstein (from the person Frankenstein)
who works without thought A horrible monster
Scrooge The penny-pinching miser of Galvanize (from Dr Galvani) To electrify;
Christmas Carol (Dicken's); now to stir into action;
any grasping, covetous person Galvanization = electrolytic
without kindness technique
Simon Legree Stowe's character in 'Uncle Tom's Gargantua (from Gargantua) Anything on
Cabin', the character was a slaver; a large scale; massive
anyone who works others Gordian Knot (from king Gordian) A very
mercilessly tough problem
Uncle Tom In uncle Tom's Cabin, Uncle Tom Guy (from Guy Fawkes, a drama
was saintly old black. Today a black character) Ridiculous name for
who is seen as quick to co-operate a man
with whites Hector (from Greek hero Hector) To
Uriah Heep From Dicklen's David Copper field, quarrel, tease, bully someone
a term to describe someone Herculean (from Roman Hercules) Very
hypocritical. heavy, very difficult
Walter Mitty Thurber' s lovable character from Hobson's Choice (from Thomas Hobson) Really
'The secret life of Walter Mitty; the no choice at all
embodiment of fantasies. Jingoism (from Jingo) Feeling of
belligerence towards foreigners
WORDS FROM PEOPLE'S &
Lilliputian (from Lilliput) Very small,
PLACE NAMES Miniature, Diminutive
People’s Name Lothario (from Lothario, a character) A
flirt; seducer of women
America (from the sailor Amerigo
Vespucci) The two great Macadam (from John Macadam) Roads
continents made of broken stones
Ampere (from Andre Ampere) Unit of Malapropism (from Mrs. Malaprop) A
electric current contusion of words; verbal
blunder
Atlas (from Greek demigod Atlas) A
man holding the globe on his Martinet (from Gen. Martinet) A person
back; a collection of maps ; very strict about discipline
Miscellaneous 5.11
The symbol (::) stands for ‘in the same way as' or base pair depicts a relationship between two nouns,
simply ‘as'. the correct answer will also consist of two nouns.
In working out an analogy, you must proceed by first If, moreover, the base pair is made up of two plurals,
determining the nature of relationship that exists the correct answer will be made up of two plurals.
between the two words in the base pair. In this Sometimes a quick scrutiny of options will help you
example, analysis indicates that orchestra is a large immediately discard the choices that do not have the
group of people playing various musical instruments same form as the base pair.
together. Strategy For Analogy
The next step is to study the relationship between (i) Identify the relationship between the words in
the words in the pairs of answer choices. In our base pair by linking them using a sentence,
example, a study of the first pair indicates that a trio (ii) Look out for contextual meaning of words,
is not a large group performing harmonic music; in
(iii) The relationship between the words could be
choice 2, a ‘choir' is a large group performing vocal
vertical or horizontal.
music. In choice (3) Band is a group but is not known
to perform martial music. (iv) The sides should match; if the ‘parent' is on the
left side and the ‘child' on the right in the question
Similarly, in choice (4) pianist is not a group. Of all
pair, then the correct answer choice should also
the options given, the words in the second pair match
follow the same order in sense.
most closely with the relationship expressed in the
base pair. Consequently, option (b) provides the best CAUTION: Do read the instruction given in a test
answer. closely; sometimes the direction can be to select the
option that LEAST similar to the question words.
IMPORTANT: You must remember that the pair of
words of the answer choice must match the base pair Word analogies frequently illustrate common
in ‘form' as well as ‘relationship'. If, for example, the relationships. The following are worth noting.
EXERCISE
MCQ TYPE QUESTIONS
Word Frequency (II) We tried t... down several passing cars in the
hope of getting a lift.
Directions for (Que. 1 - 30): Read the information
carefully and answer the questions given below it. (III) Showing disrespect to the national... is an
act to treason.
Each of the following questions has a set of four
sentences, each containing a blank. (IV) The hurricane was on the... when we entered
the city limits.
Each set is followed by four words. Find out the
maximum number of times that any of the words fits (a) wane (b) flag
each set. (c) wave (d) emblem
Use this number to indicate you answer. 5. (I) The leaning tower or Pisa is a(n)... structure.
For example, if any one of the words fits into all four (II) Of the two living dodos, one died and made
sentences, your answer will be (d) However, if it fits the other...
only into three sentences (and that is also the highest (III) Tigers are protected in order to prevent them
frequency for any of the four words) your answer will from becoming...
be 3 and so on.
(IV) The... details of his escapade were very
1. (I) This pole, erected for its aesthetic appeal, has
interesting.
turned into a popular ...for birds.
(a) extinct (b) unique
(II) I did not expect that old cupboard to... such a
fancy price at the auction. (c) rare (d) astounding
(III) The... was rusty and creaked whenever the 6. (I) The first day's play was washed out due to
door was closed or opened. inclement...
(IV) The dog was trained to... the newspaper every (II) After the accident, his thinking has become
morning. very...
(a) hinge (b) perch (III) His success has not come easily, he has had
to... many a storm in his career.
(c) bring (d) fetch
(IV) Because of the incessant... we could not go
2. (I) The ... adjourned all business for the day as
a mark of respect for the departed leader. out.
(II) For several centuries before it was proved (a) rain (b) cloudy
that the earth was round, scientists believed (c) weather (d) turmoil
that the earth was ... 7. (I) When the effect of the painkiller wore off,
(III) What is referred to as an apartment in he was racked by... pain.
America, is called a... in England. (II) In any right angled triangle, at least two of
(IV) Driving on a(n)... road is easier than driving the angles are...
on an uneven one. (III) The philanthropist was moved by the ...
(a) even (b) flat poverty he saw around him.
(c) house (d) oral (IV) The lawyer 's commitment was... and
3. (I) The troops were ordered to... the forests to unqualified.
locate the deserter. (a) total (b) prevalent
(II) Since the heavy duty equipment failed, the (c) complete (d) acute
stones had to be moved by..... .means 8. (I) His... in the subject stood him in good stead.
(III) The PM had to... to popular pressure in the (II) A mere... to do well will not do; it must backed
face of documented evidence. by will power and hard work.
(IV) This... explains how to assemble, use and (III) In driving a car, you have to change gears to
maintain the system. negotiate a/an ...
(a) physical (b) comb (IV) The RBI has decreased lending ... rates by
(c) bow (d) manual 2%,
4. (I) It is important that your concentration (a) interest (b) inclination
should not... particularly in the reading
(c) support (d) curve
comprehension section.
Miscellaneous 5.19
9. (I) Why should he apologise? He has not... any (IV) The tidal ware... down on the ship with
mistake. relentless fury.
(II) He has risen to the top only because he has (a) barrel (b) bore
been ... to his profession. (c) crashed (d) tear
(III) Since the supply situation is unclear, I hope 14. (I) His insightful... on juvenile rehabilitation has
you have not... any dates to the client. become the foundation for several laws.
(IV) Sharma's... was there for all to see; she (II) No sooner did he hear the gun's ... than he
worked beyond the call of duty. collapsed.
(a) diligence (b) capacity (III) The value of this... is inestimable.
(c) committed (d) confirmed (IV) The company's ... into software exports has
10. (I) Because of the transporter's strike and the proved profitable.
consequent delay in material supply, Harsha's (a) essay (b) report
work schedule wasway off the (c) article (d) study
(II) ... my words; this boy is destined for 15. (I) In this era of liberation, one popular, request
greatness. from local entrepreneurs is for a level
(III) This single failure continues to be the only... playing... .
on his impeccable resume. (II) Some of the greatest scientist have had little
(IV) Manners are the... of a gentleman. formal training in their...
(a) 37mark (b) stain (III) The scandal-ridden Pakistan team had to...
one of its weakest teams in recent years.
(c) blot (d) impression
(IV) In the absence of insightful reporters, the
11. (I) When the machinery in the... broke down, the senators had a... day at the press conference.
production manager was summoned
(a) ground (b) field
immediately.
(c) area (d) enter
(II) This... is a rare one because it does not
produce either flowers or seeds. 16. (I) Without the... that normally surrounded him,
the musician was simple human being with
(III) This puzzle requires more than three hours his own share of weaknesses and
to put the ... together. eccentricities. :
(IV) The teacher must... the idea of lateral thinking (II) It is amazing that the... on the vase remained
in her student even after centuries.
(a) parts (b) unit (III) This text is tautological and you can grasp
(c) encourage (d) plant its meaning even if you... over it.
12. (I) Because of the wide spread protests the (IV) The assassin was hard to trace because of
government was forced to ... the law. the... that surrounded him.
(II) In the bilateral contest, one contestant polled (a) gloss (b) glaze
29% of the votes, giving the other an ... (c) web (d) skim
majority. 17. (I) The miner used his... to open up the tunnel.
(III) This countryside... with natural minerals. (II) Prince William had to ... at the vines to get
(IV) In relative terms, your salary may not be to the castle.
much, but in... terms, Rs.2,00,000 per month (III) This horse is a ... and cannot be used for
is a sizeable amount. racing purposes.
(a) rescind (b) abounds (IV) The brick maker took the dried bricks out of
(c) absolute (d) financial the... and put them on the truck.
13. (I) When Rip Van Winkle woke up, the world (a) pick (b) hack
around him ... no resemblance to the one he (c) thoroughbred (d) chop
had known. 18. (I) The... that we normally talk about in a pencil
(II) This gun is a Smith and Wesson with a 0.38 is actually graphite.
... (II) Sherlock Homes is known for his ability to
(III) We avoided Ram in the office because he was follow even a simple... to unravel acomplex
such a... problem.
5.20 Miscellaneous
(III) General Mac Arthur is a man who... by 23. (I) The ... on the newly asphalted road had not
example. dried and stuck to the wheels of our car.
(IV) It is always a difficult task for the... speaker (II) The diva's voice surprisingly cracked when
in a debate. she moved into the song at a high ...
(a) clue (b) first (III) The cricket match had to be called off because
(c) lead (s) (d) principle the vandals had damaged the ...
(IV) The advertising agency spent many days in
19. (I) As the best lawyer specializing in air disaster
developing its ... for the new business.
cases, he was without ...
(a) pitch (b) proposal
(II) During adolescence... pressure plays a major
role in molding the value system. (c) level (d) tar
(III) A... resembles an apple, though it is different 24. (I) The ... has come to be synonymous with the
in taste. psychiatrist's profession.
(IV) Sam asked Anitha to... out of the window and (II) Despite his age, Raman had a(n)... posture.
see if the guests had arrived. (III) During the last year, the government has
(a) social (b) peer sanctioned Rs. 1,000 to... new structures.
(c) look (d) equal (IV) The committee has taken steps to... charges
of misconduct and bias.
20. (I) This is not a matrix because it has several
(a) avoid (b) erect
columns but only one...
(c) renovate (d) couch
(II) The sergeant ordered the cadets to stand in
single... at shoulder lengths. 25. (I) Frazier was out for the... and lost the match
by K.O. to Muhammed Ali.
(III) The... containing the papers on the merger
was lost in transit. (II) While coarse yams are not widely exported,
yams of finer... are very much in demand.
(IV) The mechanic is looking for a... to even out
the rough surface. (III) Dracula was, in terms of social hierarchy, a
...
(a) row (b) order
(IV) To pass the bill in the assembly, the ruling
(c) file (d) hammer party had to ,.. on the support of at least 10
21. (I) Because of the strong winds, the cargo in the opposition members.
ship moved unevenly and caused the ship to (a) depend (b) vampire
...
(c) count (d) grade
(II) This Ayurvedic drug is popular for its ability
26. (I) Trade in waste metal or... is carried out
to... asthmatic disorders. ;
through a well organized network,
(III) Despite the trainer's best efforts, the dog
(II) Following wide spread copying and paper
would not come to... ;
leakage, the board decided to ... the test.
(IV) The new pair of socks was small and would
(III) Tilak is a belligerent person and picks up a
not fit ,at the... . ... with almost everyone he meets.
(a) tilt (b) heel (IV) The child cut out the picture of the dinosaur
(c) foot (d) alleviate for the use in his ... book.
22. (I) The annual boat race is in the month after (a) scrap (b) perch
April, that is in..... (c) bring (d) fetch
(II) The rate of interests works out to a/an ... 8% 27. (I) Dr. Shekar is our... representative in
of given the highest rate of 10% and the Bangalore.
lowest pf 6%.
(II) Most dental treatment calls for only...
(III) His academic record is no ... of his anesthesia.
intellectual caliber. (III) Once the foreigners left, only the... people
(IV) Not helping his friend who was indisposed, remained.
was a ... act. (IV) The invaders wrested...
(a) average (b) may (a) area (b) local
(c) measure (d) mean (c) primary (d) control
Miscellaneous 5.21
28. (I) She was a key... for the prosecution. 36. Cosmology: Universe :: Morphology:____
(II) You should not... a book by its cover. (a) Structure (b) Function
(III) Thousand of people gathered to... fireworks (c) Radio (d) Size
display. 37. Answer : Test :: Denouement : _______
(IV) Because he was personally involved, the (a) Symphony
lawyer could not take up the... (b) Mystery story
(a) case (b) judge (c) Horse race
(c) witness (d) court (d) Complete understanding
29. (I) You must ...more time if you want to make a 38. Astronauts: Space :: Argonauts :_________
considered decision.
(a) Fire (b) Ship
(II) Batsmen wearing rubber soles get little
(c) Birds (d) Sea
...when running on the pitch.
39. Snake : Reptilian : : Fox : ____________
(III) She want to the market to... some grocery.
(a) Leotard (b) Vulpine
(IV) The.., department in an organization is
normally responsible for vendor (c) Tiger (d) Feline
development. 40. Genuine: Simulated:: Unaffected: _______
(a) buy (b) purchase (a) Elevated (b) Bombastic
(c) finance (d) grip (c) Dynamic (d) Emulated
30. (I) A...is an accepted of male office wear. Directions for(Que) 41 - 70): Read the information
(II) Of the two living dodos, one died and made carefully and answer the questions given below it
the other... In each of the following questions identify the pair of
(III) A sailor's knot is one of the most difficult words, which has a relationship similar to the pair in
knots to... question
(IV) "... your gun from its holster", Adams said. 41. Wool : Acrylic
(a) draw (b) bow (a) Rayon : Silk (b) Winter : Spring
(c) shirt (d) tie (c) Cotton : Terylene (d) Rubber : Plastic
42. Bouquet : Flower
Word Analogy
(a) Skin : Body (b) Chain : Link
Directions for (Que. 31-40): Read the information
carefully and answer the questions given below it. (c) Page : Book (d) Product : Factory
Each question below consists of a pair of words that 43. Germ : Disease
are related. Complete the blank with suitable word from (a) Man : Woman (b) War : Destruction
the answer choices given to form an analogous pair.
(c) Doctor : Medicine (d) Owner : Shop
31. Pediatrician : _____ :: Dermatologist: skin
44. Ignominy : Disloyalty
(a) Children (b) Feet
(a) Fame: Heroism (b) Derelict : Fool
(c) Plant (d) Hair
(c) Death : Victory (d) Martyr : Man
32. Blandish : Coax : : Asseverate :_________
(a) Affirm (b) Repeat 45. Gazelle : Swift
(c) Complain (d) Twist (a) Horse : Slow (b) Swan : Graceful
33. Exigency: Emergency:: Expansive :______ (c) Lion : Roar (d) Lamb : Bleat
(a) Hostile (b) Friendly 46. Ascetic : Luxury
(c) Proactive (d) Clumsy (a) Capitalist : Communist
34. Motif : decoration : : Eulogy : ___________ (b) Teacher : Classroom
(a) Diatribe (b) Lament (c) Misogynist : Women
(c) Praise (d) Satire (d) Scholar : Knowledge
35. Illustration : Clarity :: Constriction : _____ 47. Augur : Future
(a) Wide (b) Upright (a) Knight : Medieval (b) Post : Century
(c) Swiftness (d) Narrow (c) Historian : Past (d)Commuter : Present
5.22 Miscellaneous
71. (a) Incapacitate (b) Debauch 92. (a) Nylon (b) Silk
72. (a) Scepter (b) Throne 93. (a) Coral (b) Amethyst
73. (a) Minaret (b) Dome 94. (a) Pedagogue (b) Scientist
74. (l) Moat (b) Tower 95. (a) Febrile (b) Fecund
75. (a) Portrait (b) Caricature 96. (a) Pulpit (b) Pew
76. (a) Decipher (b) Destroy 87. (a) Virus (b) Bacteria
77. (a) Collie (b) Pekinese 98. (a) Debutante (b) Decent
78. (a) Cantata (b) Oratorio 99. (a) Galley (b) Deck
79. (a) Lobby (b) Portico 100. (a) Editor (b) Author
80. (a) Beans (b) Peas 101. (a) Poise (b) Polish
81. (a) Succinct (b) Terse 102. (a) Attire (b) Dress
82. (a) Pink (b) Red 103. (a) Poetry (b) Haiku
83. (a) Bugle (b) Trumpet 104. (l) Dine (b) Dime
84. (l) Tennis (b) Rugby 105. (a) Triangle (b) Rectangle
85. (a) Deleterious (b) Pernicious 106. (a) Botany (b) Sericulture
86. (l) Arrow (b) Javelin 107. (a) Paradigm (b) Carpet
87. (l) Soft (b) Strident 108. (a) Cow (b) Mare
88. (a) Corporeal (b) Somatic 109. (a) Poise (b) Balance
89. (a) Snide (b) Sneer 110. (a) Keen (b) Plexus
ANSWERS
MCQ Type Questions
1. (b) 2. (b) 3. (b) 4. (c) 5. (b) 6. (b) 7. (c) 8. (c) 9. (c) 10. (c)
11. (c) 12. (b) 13. (d) 14. (c) 15. (d) 16. (b) 17. (d) 18. (d) 19. (b) 20. (b)
21. (b) 22. (b) 23. (d) 24. (b) 25. (d) 26. (d) 27. (c) 28. (b) 29. (c) 30. (c)
31. (a) 32. (a) 33. (b) 34. (c) 35. (d) 36. (a) 37. (b) 38. (d) 39. (d) 40. (a)
41. (c) 42. (b) 43. (b) 44. (a) 45. (b) 46. (c) 47. (c) 48. (d) 49. (d) 50. (b)
51. (a) 52. (d) 53. (a) 54. (c) 55. (b) 56. (b) 57. (b) 58. (b) 59. (a) 60. (a)
61. (c) 62. (b) 63. (b) 64. (a) 65. (b) 66. (d) 67. (c) 68. (b) 69. (b) 70. (a)
71. (b) 72. (d) 73. (c) 74. (d) 75. (d) 76. (b) 77. (a) 78. (c) 79. (d) 80. (c)
81. (c) 82. (a) 83. (d) 84. (a) 85. (c) 86. (a) 87. (a) 88. (d) 89. (c) 90. (a)
91. (b) 92. (a) 93. (a) 94. (d) 95. (a) 96. (d) 97. (d) 98. (a) 99. (d) 100. (d)
101. (d) 102. (d) 103. (a) 104. (b) 105. (a) 106. (d) 107. (b) 108. (c) 109. (d) 110. (b)
111. (c) 112. (d) 113. (a) 114. (c) 115. (d) 116. (b) 117. (d) 118. (a) 119. (a) 120. (c)
121. (b) 122. (c) 123. (b) 124. (a) 125. (d) 126. (b) 127. (c) 128. (c) 129. (b) 130. (a)
131. (a) 132. (b) 133. (c) 134. (d) 135. (c) 136. (a) 137. (c) 138. (a) 139. (b) 140. (d)
Analytical Aptitude
1
C HAPTER Logical Reasoning Ability
REASONING certain pattern throughout. In the questions we have
to identify last one or two terms to continue the series
Reasoning is the process of drawing conclusions from
or to find a missing term in between given terms to
facts. These conclusions must follow inevitably from
continue the sequence followed in the question. There
the facts from which they are drawn. Reasoning is not
is no set pattern and each question may follow a
concerned with some conclusion that has a good chance
different pattern or sequential arrangement of letters
of being true when facts are true. Indeed, reasoning
or digits, which have to detect using common sense
as used here refers to logical reasoning, not of
and reasoning ability.
commonsense reasoning or probabilistic reasoning. The
only conclusions that are acceptable are those that Types of series completion Questions
follow logically from the supplied facts. Mainly following five types of question are asked.
Types of Reasoning I. Alphabet series
Reasoning can be classified in two major parts : In this series, given alphabets follow a particular
(i) Verbal Reasoning sequence or order. We have to detect the pattern
from the given alphabets and find missing alphabet
(ii) Non-verbal Reasoning
or the next alphabet to continue the pattern.
Non-verbal reasoning involves the ability to understand There are no set rules.
and analyze visual information and solve problems
There can be omission of alphabets in an order.
using visual reasoning. This reasoning test will test
your non-verbal reasoning ability as the questions Alphabets may also be omitted in an increasing/
appear in diagrammatic and pictorial form. Such test decreasing order, which may be direct increase
is also called diagrammatic or abstract reasoning tests, or decrease.
this test discussed in next chapter of spatial aptitude. There can also be alternate order.
For example: identifying relationships, similarities and There may also be alternate sequences
differences between shapes and patterns, recognizing There may be several other patterns in the
visual sequences and relationships between objects, letter series.
and remembering these.
To tackle letter series questions, vary position
VERBAL REASONING of the alphabet and its position number in both
forward and backward sequences.
Verbal reasoning tests words, letters and numbers,
and require logical reasoning and a reasonable To continue the series after Z, we again begin
knowledge of the English Language. It is also with A. In other words, the sequence is kept in
necessary to be familiar with simple manipulations a circular order.
with figures, like addition, subtractions division and In solving these questions pattern of the
multiplication. The problems of number in Test of alphabet series should be noted.
Reasoning test how logical we are i.e., how well we Some Skipping Patterns.
reason and think while carrying out simple arithmetic (i) Regular Order : Number of alphabets
manipulations. skipped remains the same.
CLASSIFICATION OF VERBAL Example. A, D, G, J, ...?
REASONING Ans. M
1. SERIES COMPLETION (ii) Increasing Order : Each time the number
In verbal series, words, letters or digits are given of alphabets skipped increases in a given
in a specific sequence/order and we have to find pattern.
out next word, letter or digit to complete the given Example. A C F J O ?
series. Ans. U
Numbers or alphabetical letters, are generally Here, each time number of letters skipped
called terms of the series. These terms follow a increases by one.
1.2 Logical Reasoning Ability
(iii) Decreasing Order : Each time the number Last space must be filled in by ‘a'.
of letters skipped decreases in a given pattern. Thus, we have two possible answers : ‘baa'
Example. A G L P S ? and ‘bba'.
Ans. U But, only ‘baa' appears in the alternatives.
Here number of letters skipped decreases by So, the answer is (a).
one each time, i.e., first 5, then 4, then 3, and Note : In case, we had both the possible
so on. answers in the alternatives, we would have
(iv) Interlinked Series : In this two or more chosen the one that forms a more prominent
different series are attached together. These pattern, which is aabb/ aaabbbaa. Thus, our
different series follow their own different answer would have been “bba”.
rules. Example 1 : Find the next letter in the series
Example. A D F J M R ? D, G, J, M, P, ______
Ans. V (a) Q (b) R
Here, there are two interlinked series. (c) S (d) T
II. Letter Series Solution (c)
This type of questions usually consist of a series Three letters are added to each letter to get
of small letters which follow a certain pattern. the next letter in the series
However, some letters are missing from the series.
i.e. D + 3, G + 3, J + 3, M + 3, P + 3, S and P = 16
Then these missing letters are given in a proper
+ 3 = 19 and the 19th letter in the alphabet is S.
sequence as one of the alternatives.
Example 2 : Find the next letter in the series
In these type of questions we have letters of the
alphabet given here. We have to first identify the A, B, D, H, ______
patterns that the series of letters follow. Then we (a) L (b) N
have to find the missing letter based on the (c) R (d) P
pattern already identified. In number series we
Solution (d)
saw different patterns that the numbers in the
series can follow- like squares, cubes. In letter Each letter in the given series is multiplied
series, obviously, patterns like squares, cubes will with 2 to get the next letter in the series. A 2
not be possible. In letter series, in increasing or 1 2 = 1 And the 2nd letter is B, B 2
decreasing difference. The position of the letters 2 2 = 4 and the 4th letter is D.
in the English alphabet is considered to be the Similarly H 2 8 2 = 16 and the 16th letter
value of the alphabet in questions on letter series. is P.
Also when we are counting after we count from A
Example 3 : What is the next letter in the
to Z we again start with A, i.e. we treat the letters
series?
as being cyclic in nature like in Number Series.
In this type of Letter Series also, we can have a B, D, G, K, P, ___________
“combination “ of series , i.e. two series are (a) S (b) V
combined and given . We need to identify the (c) W (d) X
patterns in the two series to find out the missing
Solution (b)
letter. Sometimes, there will be some special types
of series also Let us look at a few examples to B + 2, D + 3, G + 4, K + 5, P + 6,_______
understand question on Letter Series. P + 6 = 16 + 6 = 22 and the 22nd letter is V
Example. aaa... b...a... Example 4 : I, X, J, W, K, V, L, _____
(a) baa (b) abb (c) bab (d) aab (e) bbb (a) M (b) U
Ans. (a) (c) S (d) T
Method : Solution (b)
First blank space should be filled in by ‘b', so The given series is an alternate series, I + 1,
that we have two a's followed by two b's. J + 1, K + 1, L is one series and X – 1, W – 1,
Second blank space should be filled in either by V – 1, ________is the other series .
‘a', so that we have four a's followed by two b's,
X – 1 = W, W – 1 = V and V – 1 = 22 – 1 = 21 and
or by ‘b' so that we have three a's followed by
the 21st letter is U
three b's.
Logical Reasoning Ability 1.3
Consider the series 2, 6, 12, 20, 30…. followed by its square 9 and so on. Hence the
next number in the series is 5 and the one
after that is square i.e. 25.
Similarly each term could be the square root
This can be looked at a series of increasing of its predecessor, For example, in the series
differences. The difference of consecutive 81, 9, 64, 8, 49, 7, 36, … 81 is the square
pairs of terms are 4 (between 2 and 6), 6 root of 9, 64 is the square root of 8, and so
(between 6 and 12), 8 (between 12 and 20), 10 on. Therefore the next number which follow
(between 12 and 20), 10 (between 20 and 30) in the series should be the square root of
and so on. Hence, the difference between 30 36 ie. 6.
and the next term should be 12 and so the The terms of the series could be the squares
next terms will be 42. But this series can also of natural numbers increased or reduced by
be looked at as a product series certain number. For example, in the series 3,
8, 15, 24….
2 6 12 20 30
We have {Squares of natural numbers-1} as
the terms. The first term is 22 – 1; the second
12 33 34 45 56 term is 32 – 1, the third term is 42 – 1 and so
The first term is the product of 1 and 2; the on. Hence, the next term will be 62 – 1 i.e. 35
second term is the product of 2 and 3; the [Please note that the above series can also be
third terms is the product of 3 and 4; the fourth looked at as a series with increasing
term is the product of 4 and 5 and the fifth differences.]
term is the product of 5 and 6. Hence the next The difference between the 1st & 2nd term,
term will be the product of 6 and that is 42. the 2nd & 3rd terms, and on are 5, 7, 9, and so
(c) Squares/Cubes Series on. Hence the next difference should be11
There can be series where all the terms are giving us the next term as 35. There could
related to the squares of numbers or cubes of also be series with {squares of natural
numbers. With squares/ cubes of numbers numbers + some constant}.
as the basis, there can be many variations in Like we have seen with squares of numbers.
the patterns of the series. Let us look at various We can have similar series with cubes of
possibilities of series based on squares/ cubes numbers. For example take series 1, 8, 27, 64…
Each term of the series may be the square of 1 8 27 64
a natural number such as 1, 4, 9, 16….
1 4 9 6 13 23 33 4 3
Here, all the terms are cubes of natural
numbers So, the next terms will be 53, i.e.
12 22 32 4 2
125.
The numbers are squares of 1, 2, 3, 4… Consider the series 2, 9, 28, 65….
respectively. The number which follows 16
(which is the square of 4) will be 25 (which is 2 9 28 68
the square of 5).
The terms of the series may be the squares 13 1 23 1 33 1 4 3 1
of odd numbers (for example, 1, 9, 25, 49….) Here, the terms are {Cubes of natural
or even numbers (for example, 4, 16, 36, 64…) numbers +1}. The first terms is 13 + 1, the
The terms of the series could be such that a second term is 23 + 1, the third term is 33 + 1
number and its square are both given one and so on. Hence the next term will be 53 + 1
after the other and such pairs are given in i.e. 126.
some specific patterns. For example, take the (d) Combination Series
series 2, 4, 3, 9, 4, 16 …
A number series which has more than one
type of (arithmetic) operation performed or
more that one series combined together is
combination series. The series that are
combined can be two series of the same type
Here, 2 is followed by its square 4; then comes or could be different type of series as described
the number 3 (which is one more than 2) above. Let us look at some example.
Logical Reasoning Ability 1.5
First let us look at those series which are a series of natural numbers. The second series
formed by more than one arithmetic operation is the squares of natural numbers. Hence, the
performed on the terms to get the subsequent next term is 5.
terms. Consider the series: 1, 1, 4, 8, 9, 27 …… Here
Consider the series: 2, 6, 10, 3, 9, 13, 4, 12, … we have to find out the term that should be in
Here the first term 2 is multiplied by 3 to get place of the question mark. The odd terms
the second term, and 4 is added to get the third forms one series 2,5,9,14,20 …. Where the
term. The next term is 3 (one more than the difference is increasing The difference are
first term 2) and it is multiplied by 3 to get 9 3,4,5,6…. This series is combined with the
(which is the next terms) and then 4 is added series of even terms 4,9,16... where the terms
to get the next term 13. The next term 4 (which are squares of numbers 2,3,4.… Hence, the
is one more than 3) which is multiplied by 2 to term that should come in place of the question
get 12. Then 4 is added to this to get the next mark is the next term of the second series
number 16. which is 52, ie., 25.
Consider the series: 1, 2, 6, 21, 88.…. Here we (e) Miscellaneous Series
can observe that 88 is close to 4 time s 21. It is There are series that do not come under the
in fact 21 4 + 4. So, if we now look at the other patterns and are of general nature but
previous term 21, it is related to the previous are important and are fairly common. Even
term 6 as 6 3 + 3. Now we get the general here, sometimes there can be specific patterns
patterns: to get any term, multiply the previous in some cases.
term with k and then add k where k is a natural Take the series 3, 5, 7, 11, 13 ….. This is a
number with values in increasing order from 1. series of consecutive PRIME NUMBERS. It
So, to get the second term, the first term has to is an important series and the student should
be multiplied with 1 and then 1 is added. To get look out for this as one of the patterns. The
the third term, the second term is multiplied next term in this series is 17.
with 2 and then 2 is added and so, on Hence
There can also be variations using prime
after 88 the next term is 88 5 + 5 i.e. 445.
numbers. Take the series 9, 25, 49, 121… In
Now, let us look at a series that is formed by this series, the terms are squares of prime
combining two (or more) series can be of the numbers. Hence the next term is 132 i.e. 169.
same type or of different types described above.
Take the series 15, 35, 77…. The first term is
Consider the series 8, 12, 9, 13, 10, 14….. Here 3 5; the second term is 5 7; the third term
the 1st, 3rd, 5th… terms which are 8, 9, 10... form is 7 11; here the terms are the product of two
one series whereas the 2nd ,4th ,6th etc. terms consecutive product of 11 and 13 i.e. 143.
which are 12, 13,14 forms another series. Here
Take the series 8, 24, 48, 120, 168…. Here the
both series that are being combined are two
2nd term is 3 times the first term and the 3rd term
simple constant difference series. Therefore the
is 2 times the 2nd term, but after that it does not
missing number will be the next term of the
follow this pattern any more, if you look at the
first series 8, 9, 10 … which is equal to 11.
terms carefully ,you will find that the terms are
Consider the series 0, 7, 2, 17, 6, 31, 12, 49, {one less that squares of prime numbers}. Hence
20…. Here the series consisting of 1st, 3rd, 5th the next term will be 172 – 1 i.e. 288.
…. terms (i.e., the series consisting of the odd
Consider the series 1, 4, 9, 1, 6, 2, 5, 3.… At
terms) which is 0, 2, 6, 12, 20,… is combined
first sight there is nothing we can say about
with another series consisting of 2nd 4th 6th ….
the series. This is actually a series formed by a
terms(i.e., the series consisting of the even
squares is in two or more digits, each of the
terms) which is 7, 17, 31, 49…The first series
digits is written as a separate term of the series.
has the differences in increasing order 2, 4, 6,
Thus the first terms are 1, 4 and 9 the squares
8, 10 and so on. The second series also has the
of 1, 2 and 3 respectively. After this we should
difference in increasing order 10, 14, 18… Since
get 16 (which is the square of 4) since this
the last term 20 belongs to the first series a
different terms 1 and 6 in the series. Similarly,
number from the second series will be obtained
the next square 25 is written as two different
by adding 22 to 49 that is 71.
terms 2 and 5 in the series. So, the next square
Consider the series 1, 1, 2, 4, 3, 9, 4, 16,… Here 36 should be written as two terms 3 and 6. Of
one series consisting of odd terms, which is 1, these 3 is already given, so the next terms of
2, 3, 4….is combined with the series of even the series is 6.
terms which is 1, 4, 9, 16….. The first series is
Consider the series 1, 1, 2, 3, 5, 8…
1.6 Logical Reasoning Ability
(d) Fish : Shark : Water Choose the best answer. If none of the choices
(e) Car : Engine : Circle fits exactly, choose the one that works best.
Solution : (a) Plan to Solve Analogy Questions
Pen contains ink and pencil belongs to the same (i) Make your Sentence More Precise
category as pen i.e. stationery. Similarly, orange The analogies get more difficult as we work our
contains juice and banana belongs to the same way through each group. Use the common cat-
category as orange, i.e. fruits.
egories as a starting point, but be prepared to
(vi) Number Analogy refine the relationship by making your sentence
It includes deals two types of questions : more precise.
(i) Choosing a similarly related pair as the given Example. Grain : Silo ::
number pair on the basis of relation between (a) Pilot : plane (b) judge : courtroom
the numbers in each pair.
(c) water : reservoir (d) clock : time
Example. 582 : 194 :: 258 : ?
If we apply the “place where” idea without
(a) 82 (b) 86
thinking, here is what happens.
(c) 92 (d) 58
A silo is a place where we would find grain.
Solution : (b)
(a) A plane is a place where you would find a
Just as 582 is related to 194 as 582 is 3 times pilot.
194, in the same way 258 is 3 times the 86.
(b) A courtroom is a place where you would find
(ii) Choosing a number similar to a group of
a judge.
numbers on the basis of certain common
properties that they possess. (c) A reservoir is a place where you would find
water.
Example.Which number belongs to given set
of numbers? (d) A clock is a place where you would find time.
2, 3, 5, 7, 11, 13, 17, 19, 23...... We can eliminate (d), but that still leaves you
(a) 15 (b) 9 (c) 21 (d) 29 (e) 27 with four possible answers. Now is express the
relationship between silo and grain more pre-
Solution : (d)
cisely?
Given set of numbers belong to the prime
A silo is a place where grain is stored.
numbers. In the given option, only prime
number is 29. (a) A plane is a place where a pilot is stored.
(vii) Alphabet Analogy (b) A courtroom is a place where a judge is
In this type of questions, two groups of alphabets stored.
related to each other in same way, are given and (c) A reservoir is a place where water is stored.
we are required to find out this relationship and (d) A highway is a place where automobiles are
choose a group of alphabets which is related in the stored.
same way to a third group provided in the question.
(ii) Analogies Work only if there’s a clear cut
Example. B C D : P Q R : : X Y Z : ? connection
(a) FGH (b) LMN (c) RST (d) TUV (e) STU An analogy depends upon a necessary
Solution : (b) connection between pairs of words based
Each letter of the first group is moved fourteen on the meaning of the words. This clear
steps forward to obtain the corresponding letter of connection must exist for both the original
the second group. A similar relationship will exist capitalized word pair and the correct answer
between the third and fourth groups. choice, i.e. we can eliminate any answer choice
How to Answer Analogy Questions for which we cannot describe a necessary
Figure out how the capitalized words are related. relationship between the words. Use the “clear
connection” test to rule out answer choices even
Create a sentence that expresses that connection.
when we don’t know one of the capitalized
Test the choices with your sentence and eliminate words.
the ones that don’t work.
Example. Letter : Alphabet : :
If we are left with more than one answer—or no
answer at all—go back and refine your sentence. (a) note : scale
(b) ocean : merchandise
Logical Reasoning Ability 1.11
pairs may have the same property or may be Thus, in FREEZE, F will be coded as E, R as Q,
related to each other according to the same rule. E as D and Z as Y. So, the code becomes
We are is required to choose the odd pair/group. EQDDYD.
Example. Choose the numeral pair/group which (ii) Number Coding
is different from others. In these questions, either numerical code
(a) 83 – 75 (b) 58 – 50 (c) 49 – 42 values are assigned to a word or alphabetical
code letters are assigned to the numbers and
(d) 25 – 17 (e) 131– 123
we are is required to analyse the code as per
Ans. (c) the directions.
In each of the pairs, first number is eight more Example. If CHINA is written as 38126 and
than the second. NEPAL is 25769, how is PLAINE is coded ?
(v) Choosing the Odd letter group Solution : Clearly Alphabets are coded as
In this type of questions, usually four groups of shown:
letters are given. Three of them are similar to C H I N A E P L
each other in some manner while one is different 3 8 1 2 6 5 7 9
and we have to the answer.
Hence, PLAINE is coded as 796125.
Example. Choose the group of letters which is
Example. In a certain code ‘37’ means ‘which
different from others. class’ and ‘583’ means ‘caste and class’. What is
(a) SU (b) PN the code for ‘caste’ ?
(c) IK (d) BD (a) 3 (b) 7 (c) 8
Ans. (b) (d) Either 5 or 8
All other are two alternate letters Solution : (d)
Given, which class’ ‘37’ ...(i)
4. CODING-DECODING
and‘caste and class’ ‘583’ ...(ii)
A CODE is a ‘system of signals'. Therefore, coding
From (i) and (ii), the common word ‘class’ is
is a method of transmitting a message between the
coded as ‘3’.
sender and the receiver without a third person
knowing it. From (ii), ‘caste’ is coded as ‘5’ or ‘8’.
Coding and Decoding Test is set up to judge the (iii) Substitution
candidate's ability to decipher the rule that codes a In this type of questions, some particular
particular word/ message and break the code to objects are assigned code names. Then a
decipher the message. question is asked that is to be answered in the
code language.
Types of Coding Decoding Questions
Example. If cook is called butler, butler is
(i) Letter Coding called manager, manager is called teacher,
In these questions, real alphabets in a word are teacher is called clerk and clerk is called
replaced by certain other alphabets according principal, who will teach in a class ?
to a specific rule to form its code and we are (a) Cook (b) Butler (c) Manager
required to detect the common rule and answer (d) Teacher (e) Clerk
the questions accordingly.
Solution : (e)
Example. If TAP is coded as SZO, then how is
A ‘teacher' teaches in a class and as given
FREEZE coded?
‘teacher' is called ‘clerk'. So, a ‘clerk' will teach
(a) EQDFYG (b) ESDFYF (c) GQFDYF in the class.
(d) EQDDYD (e) EPDEZD (iv) Mixed letter Coding
Solution : (d) In this type of questions, three or four complete
Each letter in the word TAP is moved one step messages are given in the coded language and
backward to obtain the corresponding letter of the code for a particular word is asked. To
the code. analyse such codes, any two messages bearing
a common word are picked up. The common
S Z O
code word will mean that word. Proceeding
–1 similarly by picking up all possible combinations
T A P of two, the entire message can be analysed.
Logical Reasoning Ability 1.13
Example.If ‘tee see pee' means ‘Drink fruit Mother's or father's father — Grand father
juice'; ‘see kee lee'means ‘Juice is sweet' and Mother's or father's mother — Grand mother
‘fee ree mee' means. “He is intelligent', which
Son's wife — Daughter-in-Law
word in that language means ‘sweet' ?
Daughter's husband — Son-in-Law
(a) see (b) kee (c) lee
Husband's or wife's sister — Sister-in-Law
(d) pee (e) None of these
Solution : (b) Husband's or wife's brother — Brother-in-Law
In first and second statements, common word is Brother's son — Nephew
‘Juice' and common code word is ‘see'. So, ‘see' Brother's daughter — Niece
means ‘Juice'. In second and third statements, Uncle or aunt's son or daughter— Cousin
common word is ‘is' and common code is lee'. Sister's husband — Brother-in-Law
So, ‘lee' means ‘is'. Thus, in second statement,
remaining word ‘sweet' is coded as ‘kee'. Brother's wife — Sister-in-Law
(v) Mixed number Coding Grandson's or Grand daughter's daughter
In this type of questions, a few groups of — Great grand daughter
numbers each coding a certain short message, Types of Blood Relation Questions
are given. Through a comparison of the given (i) Deciphering Jumbled up descriptions
coded messages, taking two at a time, the
In this type of questions, a round-about
candidate is required to find the number code
description is given in the form of certain small
for each word and then formulate the code for
relationships and direct relationship between
the message given.
the persons concerned is to be deciphered.
Example. In a certain code, ‘786' means ‘study
very hard', ‘958' means ‘hard work pays' and ‘645' Example. Pointing to a photograph, a man
means ‘study and work'. Which of the following said, “I have no brother or sister but that man's
is the code for ‘very' ? father is my father's son.” Whose photograph
was it ?
(a) 8 (b) 6 (c) 7 (d) 5
(a) His own (b) His son's
Solution : (c)
(c) His father's (d) His nephew's
In first and second statements, common word is
Solution : (a)
‘hard'and common code digit is ‘8’. So, ‘8' means
‘hard’. Since man has no brother, his father's son is
he himself.
In the first and third statements, common word
is ‘study' and the common code digit is ‘6'. So, ‘6' (ii) Relation Puzzle
means ‘study'. In this type, mutual blood relations or other
Thus, in the first statement, ‘7' means ‘very'. informations of more than two persons are
mentioned and information about any two is
(vi)Deciphering individual letter codes by
mentioned.
analysis
In this type of questions, certain sample words Example. A and B are brothers. C and D are
are given along with their codes. The candidate sisters. A’s son is D's brother. How is B related
is required to decipher individual codes for to C.
different letters by comparing, taking two words (a) Father (b) Brother (c) Grandfather
at a time, and then answer the given questions (d) Uncle (e) None of these
accordingly. Solution : (d)
5. BLOOD RELATIONS B is the brother of A; A's son is D's brother.
In these tests, the success of a candidate depends This means D is the daughter of A. Since C
upon the knowledge of the blood relations. and D are sisters, C is also the daughter of A.
So, B is the uncle of C.
Following Blood relations help solve these
Questions : (iii) Coded Relations
Mother's or father's son — Brother In such questions, relationships are
represented by certain codes or symbols such
Mother's or father's daughter — Sister
as +, –, , ,*, etc. Then relationships
Mother's or father's brother — Uncle between certain persons, given in the form of
Mother's or father's sister — Aunt these codes, are to be analysed.
1.14 Logical Reasoning Ability
Example. If A + B means A is the sister of B; In addition, remember the word spouse which means
A – B means A is the brother of B; either husband or wife.
A B means A is the daughter of B, which of Grandfather and grandmother will come in the first
the following shows the relation that E is the stage. Mother, father uncle and aunt will come in
maternal uncle of D? the second stage: sister brother and cousin will come
(a) D + F E (b) D – F E (c) D F + E at the third stage: son daughter, niece and nephew
(d) D F E (e) None of these. will come in the fourth stage and finally,
granddaughters and grandsons will come. The above
Solution : (c)
stage are made from the point of view of an
E is the maternal uncle of D means D is the individual.
daughter of the sister (say F) of E i.e.
In type I questions, the relationship between two
D F + E.
people is given through a roundabout way of
The easiest and non-confusing way to solve these relating them through other people. We have to go
types of problems would be to draw a family tree through the series of relationship can be given I in
diagram and increase the levels in the hierarchy the questions. The relationship can be given as a
as shown below: simple statements or as a statement made by a
1st GRAND PARENTS Grandfather, Grandmother,
person. In the first example given below, a person is
Granduncle, Grandaunt involved in making a statement whereas in the
2nd PARENTS & Father, Mother, Uncle, Aunt, second question, there is no person involved in
IN-LAWS Father- in-low, Mother in law making a statement.
3rd SIBLINGS, SPOUSE Brother, Sister, Cousin, Wife, Example 1 : A’s father’s mother-in law only
& IN-LAWS Husband, Brother in law, Sister
daughter’s son is b. How is A related to B?
in law
Example 4 : The person travelling to Delhi went Reena and Manju. Who would be second from the
by which of the following modes ? left in the photograph ?
(a) Bus (b) Train (a) Reena (b) Shikha (c) Manju
(c)Aeroplane (d) Car (d) Rita (e) Either Shikha or Reena
(e)Boat Solution : (a)
Shikha is to the left of Reena and Manju is to her
Example 5 : Who among the following travelled
right.
to Delhi ?
Rita is between Reena and Manju.
(a) R (b) S
Hence order is : Shikha, Reena, Rita, Manju.
(c)T (d) Data inadequate
In the photograph, Reena will be second from left.
(e)None of these
(iii) Comparison Type Questions
Solutions : Given information can be analysed as In this type of questions, clues are given
follows : regarding comparisons among a set of persons or
Mode of Transport : things with respect to one or more qualities and
R travels by Car, Q by Aeroplane, S by Boat and T we are required to analyse the whole information,
by Train. form a proper ascending/descending sequence and
then answer the given questions accordingly.
Now, only P remains. So, P travels by Bus.
Directions (1 to 3) : Read the information given
Place of Travel : below and answer the questions that follow :
R goes to Bangalore, Q to Calcutta. (i) There is a group of five girls.
Now, bus transport is not available for Delhi or (ii) Kamini is second in height but younger than
Chennai. So, P who travels by Bus goes to Rupa.
Hyderabad. (iii)Pooja is taller than Monika but younger in
S travels by boat and hence, S did not go to Delhi. age.
So, S goes to Chennai. (iv) Rupa and Monika are of the same age but
Now, only T remains. So, T goes to Delhi. Rupa is tallest between them,
(v) Neelam is taller than Pooja and elder to Rupa.
Person Place Mode
Example 1 : If they are arranged in the
P Hyderabad Bus ascending order of height, who will be in third
Q Calcutta Aeroplane position ?
(a) Monika (b) Rupa
R Bangalore Car
(c) Monika or Rupa (d) Data inadequate
S Chennai Boat
(e) None of these
T Delhi Train Example 2 : If they are arranged in the
1. (e). Incorrect combination is T — Aeroplane descending order of their ages, who will be in
fourth position ?
2. (c). Correct combination for S is Chennai —
Boat (a) Monika or Rupa (b) Kamini
(c) Monika (d) Data inadequate
3. (a). Incorrect combination is Delhi — Bus
(e) None of these
4. (b). T travelled to Delhi by Train
Example 3 : To answer the question “who is
5. (c). T travelled to Delhi the youngest person in the group”, which of
(ii) Seating / Placing Arrangements the given statements is superfluous ?
In this type of questions, some clues regarding (a) Only (i) (b) Only (ii)
seating or placing sequence (linear or circular) (c) Only (v) (d) Either (i) or (iv)
of some persons or items is given. The candidate (e) None of these
is required to form the proper sequence using Solution : First find sequence of heights
these clues and answer the questions
By (iii), we have M<P
accordingly.
By (v), we have P < N.
Example. Four girls are sitting on a bench to
be photographed. Shikha is to the left ofReena. Now, Rupa is tallest and Kamini is second in
Manju is to the right of Reena. Rita is between height, hence sequence of heights is :
M < P < N < K < R.
Logical Reasoning Ability 1.17
Example 4 : Starting from his house, Sachin represents a group as shown in the examples. The
walks a distance of 8 m towards north, then he area common to two or more figure represent
turns left and walks 6m, then walks 3m towards those elements which are common to two or more
south an finally travels 6m towards west to reach groups. There are various models in Venn
his office. What is the distance between his house Diagrams which we will discuss with examples.
and office and also find in which direction is his Venn Diagram Type I
office situated with respect to his house.
In these kind of questions. There are many
Solution : geometrical figures representing different groups.
C 6m B Let’s discuss this type with the help of the
following example.
6m 3m
F8 m
E D
2 Actors
A Singers
Let A be the Sachin’s house and E his office. The 1 4
distance between A and E 7 5
6
EF AF
2 2
3
Dancers
ED CB AB FB
2 2
This diagram consists of three groups –Singers,
ED CB AB CD
2 2
Actors and Dancers represented by a triangle, a
rectangle and a circle respectively. There are
122 52 13m seven regions represented by numbers from 1 to
Example 5 : Starting from her house, Nisha 7 where each region represents a different group
traveled 8 m towards west then turned right and Region 1 Represents only Singers
travelled 15 m. She then traveled 20 m towards Region 2 Represents only Actors
east, followed by 20 m to south to reach a hostel. Region 3 Represents only Dancers
How far is her house from the hostel and in which
Region 4 Represents only Singers & Actors
direction?
Region5 Represents only Actors & Dancers
Solution :
Region6 Represents only Dancers & Singers
C 20 D
Region7 Represents Singers, Actors & dancers.
15 Hence the various areas as shown represent
20 different groups i.e. region 6 represents those
B 8 A Singers who are only Dancers as region 6 is the
intersection of thee triangle and the circle, but
F not the square. You may be triangle and the circle,
Let A and f be the initial and final positions. but not the square. You may be asked questions
like “Which region represents
Now AF AE2 EF 2
Dancers who are only Singers?” The answer to
AE CD BA 20 8 12m this question, as explained above will be region 6.
EF DF DE CB 20 15 5m Venn Diagram Type II
AF 144 25 169 13m
Her house is towards north- west from the hostel A B
8. LOGICAL VENN DIAGRAMS a d b
This type of questions aim at analysing ability r
to relate a certain given group of items and f e
illustrate it diagrammatically.
Venn Diagrams are diagrammatic representation c
of sets, using geometrical figures like circles, C
triangles rectangles etc. Each geometrical figure n
1.20 Logical Reasoning Ability
Solution : (c) ‘less than', ‘greater than', ‘equal to', ‘not equal to',
A number which comes after a given number etc.
is said to follow it while the one which comes are represented by symbols, different from the
before the given number precedes it. Thus, usual ones.
numbers satisfying the given conditions, can The questions involving these operations are set
be shown as follows. Hence there are three using artificial symbols. The candidate has to
such numbers. substitute the real signs and solve the questions
(ii)Ranking Test accordingly, to get the answer.
In this type of test, generally ranks of a Type of Problems in Mathematical
person both from top and bottom are Operation
mentioned and total number of persons is
asked. However, sometimes this question is (i) Solving by Substitution
put in the form of a puzzle of interchanging In this type of questions, we are provided with
seats by two persons. substitutes for various mathematical symbols,
Example : Anil ranked ninth from the top followed by a question involving calculation of
and thirty eighth from the bottom in a class. an expression or choosing the correct/ incorrect
How many students are there in the class? equation. We are required to put in the real
(a) 45 (b) 46 (c) 47 signs in the given equation and then solve the
questions as required.
(d) 48 (e) 49
While solving a mathematical expression,
Solution : (b)
proceed according to the rule BODMAS
Whole class consists of :
i.e., Brackets, Of, Division, Multiplication,
(i) 8 students who have a rank higher than Addition, Subtraction.
Anil
(ii)Deriving appropriate Conclusions
(ii) Anil; and
In such questions, the usual algebraic signs and
(iii) 37 students who have rank lower than mathematical symbols may be given different
Anil i.e.,(8 + 1 + 37) = 46 students. meanings or may even be replaced by new
(iii) Time Sequence Test symbols. For instance (÷) may denote addition,
Example : Raman remembers that his (×) may denote subtraction (<) may denote
brother's birthday is after fifteenth but before division and so on. The simplest way to
eighteenth of February whereas his sister solve such an equation is to rewirite the
Kamla remembers that her brother's birthday problem replacing the given symbols with the
is after sixteenth but before nineteenth of signs they are supposed to stand for. For
February. On which day in February is instance, if :
Raman's brother's birthday ? + means – means +
(a) 16th (b) 17th (c) 18th ‘V’ means < – means >
(d) 19th (e) None of these means = > means
Solution : (b) &, < means
According to Raman, brother's birthday is on
Then equation, 65 < 5 2.5 — 15 will be
one of the days among 16th and 17th February
rewritten as 65 5 + 2.5 > 15, and problem may
According to Kamla, the brother's birthday is be solved.
on one of the days among 17th and 18th
In yet another type of questions included under
February. Hence Raman’s brother's birthday
this system, even the digits (instead of the
is on the day common to both the above groups
symbols)
i.e., 17th February.
0, 1, 2, .... 8, 9, may be replaced by alphabets
11. MATHEMATICAL OPERATIONS and the candidate may then be asked to perform
This type questions are based on simple calculations and solve the given problem. For
mathematical operations. instance, codes may be used as (a = 1, b = 2, c =
Here, four fundamental operations : 3 etc.) or in reverse (i.e., z = 1, y = 2, x = 3 and
so on).
Addition, subtraction, multiplication and division
and statements such as
Logical Reasoning Ability 1.23
Following question employs such a code. Example 2 : Arrange the following in a logical
Example 1 : If y w = s and u — v = z, what order :
is r/x ? 1. Euphoria 2. Happiness
(a) y (b) z (c) s (d) x 3. Ambivalence 4. Ecstasy
Solution : If we count backwards from z, we get 5. Pleasure
y as the 2nd letter, w as the 4th letter, and s as (a) 4, 1, 3, 2, 5 (b) 3, 2, 5, 1, 4
the 8th letter. In this order u is the 6th, v is the
(c) 2, 1, 3, 4, 5 (d) 1, 4, 2, 5, 3
5th and z the 1st letter of the code.
(e) 5, 4, 3, 2, 1
So, 2 4 = 8 and 6 – 5 = 1
Solution : (b)
Hence, r = 9 and x = 3.
All the given words stand for ‘Joy', but the
r 9
So = 3. So the answer is (d) x. intensity increases in the order —
x 3 Ambivalence, Happiness, Pleasure, Euphoria,
In certain cases, arithmetical operations may not Ecstasy. Thus, the correct order is 3, 2, 5,
be defined. Consider the following question. 1, 4.
Example 2 : If 8 + 12 = 2, and 7 + 13 = 3, then
what is the value of 10 + 18 ? 13. DECISION MAKING
(a) 9 (b) 6 (c) 4 (d) 7 It with questions in which we have to decide the
course of action to be taken upon a candidate
12 – 8 13 – 7 who has applied for a vacancy or allotment or
Solution : = 2, and = 3. So, code
2 2 membership to an institution, keeping in mind
employed here is special. the essential requisites and the data given for
the candidate.
y–x
In x + y, ‘+’ stands for Type I Decision Test
2
The decision about each candidate has to be made
18 – 10 from amongst the five choices named (a), (b), (c),
10 + 18 = = 4, so the answer is (c).
2 (d), (e) which state the courses of action to be
taken as per the candidate's potentials.
12. LOGICAL SEQUENCE OF WORDS Direction (1 to 5) : Read the following
In this type of questions, a group of words is information to answer the given questions
given. The candidate is required to arrange these Following are the criteria for selecting &
words in a meaningful order such as the sequence marketing officer by a company. The candidate
of occurrence of events, sequence from a part to must :
the whole, sequence of increasing/decreasing size,
(1) be a graduate with at least 50% marks.
value, intensity etc., and then choose the correct
sequence accordingly. (2) have secured at least 40% marks in the
written test.
Example 1 : Arrange the following in a
meaningful sequence : (3) not be less than 24 years and more than 29
years as on 10th October, 2007.
1. Consultation 2. Illness
(4) should have work experience of at least two
3. Doctor 4. Treatment
years as an officer. However, if a candidate:
5. Recovery
(5) fulfils all other criteria except at (4) above
(a) 2, 3, 1, 4, 5 (b) 2, 3, 4, 1, 5 but has a diploma in Marketing Management,
(c) 4, 3, 1, 2, 5 (d) 5, 1, 4, 3, 2 his/her case is to be referred to General
(e) 2, 3, 4, 5, 1 Manager, Marketing.
Solution : (a) (6) fulfils all other criteria except at (3) above
but has worked as Marketing Officer at least
Illness occurs first. One then goes to the
for three years, his/her case is to be referred
doctor and after consultation with him,
to Director, Marketing.
undergoes treatment to finally attain
recovery. Thus, the correct order is 2, 3, 1, Based on the above criteria and information given
4, 5. in each of the following questions, we have to
take the decision in regard to each case. We are
1.24 Logical Reasoning Ability
not to assume anything. These cases are given provided. Five alternative comments on these
as on 10th October, 2007. are given and the correct one is to be chosen.
Mark answer (a) if candidate is to be appointed; Direction (1 and 2) : For the Assertions (A) and
Mark answer (b) if the candidate is not to be Reasons (R) below, choose the correct alternative
appointed; from the following:
Mark answer (c) if the data given are not sufficient (a) Both A and R are true and R is the correct
to take decision; explanation of A.
Mark answer (d) if to be referred to General (b) Both A and R are true but R is NOT the
Manager - Marketing; and correct explanation of A.
Mark answer (e) if to be referred to Director – (c) A is true but R is false.
Marketing. (d) A is false but R is true.
Example 1 : Amit, born on 5th June, 1983, has (e) Both A and R are false.
done his post-graduation in Marketing Example 1 : Assertion (A) : Moon cannot be
Management with first class. He has secured 50% used as a satellite for communication.
marks in the written test. He has been working
Reason (R) : Moon does not move in the
in an organisation as a Marketing Officer for the
equatorial plane of the earth.
last four years.
Solution (a) :
Example 2 : Rohit has been working in an
organisation as Officer for the last ten years. His Since R explains A.
date of birth is 17th February, 1964. He has Example 2 : Assertion (A) : Salt is added to
secured 60% marks in the degree examination cook food at higher altitudes.
and 40% marks in the written test. Reason (R): Temperature is lower at higher
Example 3 : Manju is a first class graduate and altitudes.
has done a diploma in Marketing Management. Solution (b) :
She has secured 50% marks in the written test. Since both statements A and R are separately
She was 23 years old as on 5th September, 2006. true but R does not explain A.
Example 4 : Nitin was born on 25th August,
1985. He has secured 60% and 50% marks in Type III Situation Reaction Test
graduation and in the written test, respectively. In this test, certain situation is described and
He has been working in an organisation as Officer the candidate is required to choose the most
for the last four years. suitable reaction to the given situation from
Example 5 : Suman is a graduate with first class amongst the alternatives provided. This test
and has secured 60% marks in the written test. judges reasoning power of the candidate and his
She has been working as an Officer for the last ability to act correctly and promptly to a situation
three years. She was born on 20th May, 1982. that may arise in emergency.
Solutions Direction (1 to 3) : While travelling in your
car, certain persons stop you on the way asking
1. (a). All conditions of eligibility are satisfied.
you to take an injured child to the hospital. You
2. (e). Condition (3) is violated but it can be would :
waived by condition (6).
(a) ask them to leave your way and then drive
3. (d). The candidate satisfies all conditions away.
except (4). But he fulfils condition (5) so
(b) ask them to first call the police.
that (4) is waived.
(c) immediately take the child to hospital.
4. (e). The candidate satisfies condition (2)
instead of condition (3). (d) get out of the car and ask some other person
to help them.
5. (a). All conditions of eligibility are satisfied.
(e) None of these
Type II Assertion and Reason Solution (c)
This test is meant to judge the candidate's Situation described above demands that the
knowledge and his ability to reason out correctly. person should immediately render the help asked
In this test, two statements referred to as the for and take the child to the hospital.
Assertion (A) and Reason (R) respectively are
Logical Reasoning Ability 1.25
Example 1 : While playing cricket in the school, (i) Number of cuts given to a cube is given, find the
suddenly when you hit the ball, it strikes your maximum number of identical pieces that can be
classmate on the forehead and blood starts oozing produced.
out. You would : (ii) Number of pieces a cube is cut into is given, find
(a) run away from the field. the least number of cuts required to produce these
(b) start fighting with the boy why he came in many pieces.
the way. (iii) Miscellaneous types.
(c) blame somebody else for the accident. It must be remembered that in order to obtain
(d) take the boy to the first aid room. the maximum number of pieces, the cuts given
to a large cube must be divided as equally as
(e) None of these
possible in three different directions.
Solution (d)
Let us discuss each of these types with the help
In the above situation, urgent need is to provide of examples.
first aid to the boy so that the bleeding may
(i) Number of cuts = given;
stop.
Maximum number of pieces =?
Example 2 : You are visiting a place for the first
time and are travelling in a bus. Suddenly you (a) Whenever the number of cuts made to a
realise that the driver is taking the bus to a lonely cube is a multiple of 3.
place with no right intentions. You would : If ‘n’ is the number of pieces along each
(a) with the help of some other passengers, try edge, and
to baffle the driver and take over the bus. Total cuts
n 1, then
(b) sit and wait to face the repercussions. 3
(c) jump out of the running bus. Maximum number of identical pieces = n3
(d) console the worried passengers. Number of pieces with 3 faces visible = 8
Solution. (a) Number of pieces with 2 faces visible
When a wrong doing is expected, immediate = 12(n – 2)
action to prevent it is the need. Number of pieces with 1 face visible
Example 3 : You have gone to enjoy a Diwali = 6(n – 2)2
Mela organised by a club. Suddenly you come
Number of pieces with no face visible
across a lost child crying desperately.
= (n – 2)3
You would :
Let us take an example where 30 cuts
(a) neglect and walk away.
are made to a large cube to get the
(b) ask the child to find his parents. maximum number of pieces.
(c) ask him to stop crying and wait patiently for
his parents. 30
Here, the value of ‘n’ will be +1= 11
(d) contact with the club authorities and make 3
an announcement for the parents. Then,
Solution (d) Maximum number of pieces
Immediate need is to find the child's parents and = n3 = 113 = 1331
for this, best way is to announce the child's name Number of 3 – face visible pieces = 8.
and appearance so that his parents might know Number of 2 – faces visible pieces
where the child is.
= 12(n – 2) – 12(11 – 2)
14. CUBES = 108.
A cube is a solid which has 6 faces, 8 corners and Number of 1 – face visible pieces
12 edges. – 6(n – 2)2 = 6(11 – 2)2 = 486.
Following are some of the important types based Number of no face visible pieces
on which questions have been asked in various
= (n – 2)3
aptitude tests.
= (11 – 2)3 = 729.
1.26 Logical Reasoning Ability
(b) Whenever the number of cuts made to the Directions (1 to 5) : Read the data given below
large cube is NOT a multiple of 3. carefully and answer the questions.
In such a case, there is no direct formula Seven persons Paul, Queen, Rax, Sam, Tom, Unif
for finding out the 2 - faces visible, 1 - face and Vali are sitting in a row facing us. Rax and
visible or no face visible pieces, but the Sam sit next to each other. There must be exactly
maximum number of pieces produced can four persons between Queen and Vali. Sam sits
be found out. Let us consider that a large to the immediate right of Queen.
cube is given 4 cuts. Now 4 cuts are divided
into three directions as 1 cut, 1 cut, 2 cuts. Example 1 : If Paul and Tom are separated
We know that the number of pieces along exactly by two persons, then who sits to the
each edge is one more than the number of immediate left of Vali?
cuts along that edge (when all cuts are made (a) Paul (b) Tom
in the same direction - parallel to the same (c) Unif (d) Rax
pair of faces). We can represent the above
Example 2 : If Queen is not sitting at either
as shown below:
extremes of the row, then who among the
1 1 2
following has as many persons on his left as on
+1 +1 +1
his right ?
2 2 3 12 pieces.
(a) Sam (b) Unif
Similarly, 11 cuts will give us maximum 100
pieces, as shown below: (c) Rax (d) Vali
3 4 4 Example 3 : If Queen sits at one extreme, then
+1 +1 +1 who is at the other extreme?
4 5 5 100 pieces. (a) Paul (b) Tom
(ii) Number of pieces = given, (c) Vali (d) Cannot determined
Least number of cuts =? Example 4 : Tom sits to the right of Queen, and
We use the reverse process. To get the least Paul is separated from Tom by exactly three
number of cuts to produce 100 pieces, factorise persons. Then, who is sitting to the immediate
100 into 3 factors with least difference between left of Vali?
them, and then subtract one from each of them, (a) Sam (b) Unif
as shown below:
(c) Tom (d) Rex
100 4 5 5
Example 5 : In how many different ways can the
1 1 1
seven persons sit in a row?
3 4 4 11 cuts
(iii) Miscellaneous types: (a) 3 (b) 2
Apart from the above two types, a cube (c) 10 (d) 12
can be coloured with one, two or more colours, Solution:
and then is given some number of cuts. Let us write down the conditions given in short
Let us study and practice each of the above types form and then represent them pictorially Also,
with the questions given in the exercise. let us treat the left of the persons sitting as “left”
and their right as “right” for interpreting the
15. ARRANGEMENTS conditions
Linear Sequencing
Rax and Sam sit next to each other
Linear sequencing is essentially arranging the
RS or SR.
items in a sequence (in a single line.) The questions
of this type are also referred to as “Seating There are exactly 4 persons between Queen and
Arrangement”. The word “Seating Arrangement” Vali
should not be misconstrued- it should not be treated Q ___ ____ ____V or V ___ ____ ____ Q
as questions involving only persons sitting a Sam sits to the immediate right of Queen SQ.
specified conditions. Essentially, these questions
involve arranging subjects (people or things) Now let us analysis the data /conditions that we
according to the given conditions. The arrangement are given and then put the three conditions
is done only on one “axis” and hence, the positions together. Let us number the steps from our left
of the subjects assume importance in terms of to right as Seat 1 to Seat 7.
order like first position second position, etc.
Logical Reasoning Ability 1.27
Mithra sits at the extreme left –next to Kishan Once you read the data, first draw the shape
MK ____ ___ ____ _____ specified in the data and then draw the slots in
Shyam sits to the immediate left of Vardhan and the seating arrangement.
third to the right of Rajan Rn ___ ____ S Va Six people around a circular table Eight people
Putting both the above condition together Va can around a cirucular table
go only to the extreme right postion. Thus, We have
the arrrangement as M K Rn ___ ___ S Va.
Raj and Vimal occupy the two vacant seats between
Rajan aand Shyam.
6. Solution (d)
From the seating arrangement figure above
Vardhan is to the immediate right of Shyam Statement like “A and B are sitting farthest from
each other” or “A and B sit across the table” imply
7. Solution (d)
that A and B sit opposite each other.
If Kishan and Vardhan exchange places as can be
On the other hand, you should remember that
seen from the arrangement, the person to the
unlike in straight –line arrangement, the words
immediate left of Rajan will be Vardhan
“immediately” and “directly” do not play any role
8. Solution (c) in circular arangement. In general, there is no
If Shyam sits between Raj and Vardhan then the left side or right side (unless we are talking of
seating arrangement is as follows. Mithra, Kishan, ‘immediate right’ or ‘immediate left’).
Rajan, Vimal, Raj, Shyam, Vardhan. Then, Vimal So, if it is given that C sits to right of B, then it is
will be exactly in the middle of the row. clear that C must be to the immediate right of B. Go
9. Solution (d) “anti- clockwise” if anybody’s right has to be located
Statement (a) gives the arrangement as Mithra, and go “clockwise” if somebody left has to be located.
Kishan, Rajan, Raj, Vimal, Shyam, Vardhan
16. CLOCKS
Statement (b) gives the seating arrangement as
Mithra, Kishan, Rajan, Raj, Vimal Shyam, Vardhan The hour hand and the minute hand of a clock
move in relation to each other continuously and
Statement (c) gives the seating arrangement as:
at any given point of time, they make an angle
Mithra, Kishan, Rajan, Vimal, Raj, Shyam, Vardhan
between 0 and 180 with each other.
So only statement (d) cannot makes the seating
If the time shown by the clock is known, the angle
arrangement unique while others can.
between the hands can be calculated. Similarly,
10. Solution (c) if the angle between two hands is known, the
The arrangement is M K Rn R/Vi Vi/R S Va time shown by the clock can be found out.
Rajan exchange his place with Mithra and Vimal When we say angle between the hands, we
with Vardhan then we have the following normally refer to the acute/obtuse angles (upto
arrangement Rn K M R/Va Va/R S Vi 180) between the two hands and not the reflex
While we still do not know the exact position of angles (> 180).
Vardhan (or which place Vimal sits), we can see that For solving the problems on clocks, the following
there are five persons between Rajan and Vimal points will be helpful.
Circular Arrangement (1) Minute hand covers 360 in 1 hour, ie. in 60
Questions on circular arrangement involve seating minutes. Hence Minute Hand Covers 6
people around a table or arrangement of things in Per Minute.
a circular manner (for example, different colors (2) Hour hand covers 360 in 12 hours. Hence
beads string to form a necklace). In case of people hour hand covers 30 per hour. Hence, Hour
sitting around a table, the table could be of any Hand Covers 1/2 Per Minute. The
shape – rectangular, square, circular or any other. following additional points also should be
remembered, in a period of 12 hours, the
The data given in such sets of questions specify the
hands make an angle of
positions of some or all of the individuals (or things)
in the arrangement. The positions are specified (a) 0 with each other (ie. they coincide with
each other) 11 times.
through conditions involving specified persons
sitting (or not sitting) opposite each other or a (b) 180 with each other (i.e. they point
paritcular person sitting to the right or left of exactly in opposite directions) 11times.
another person etc. (c) 90 or any other angle with each other
22 times.
Logical Reasoning Ability 1.29
NOTE (d) are on the same straight line but are facing
(1) We can also solve the problems on clocks using opposite directions.
the method of “Relative Velocity”. Solution
(2) In 1 minute Hand covers 6o and Hour hand (1) In the formula
covers1/2
11
(3) Therefore, Relative Velocity = 6 – 1/2 = 5½ = m 30h
per minute. Alternately, in 1 hour hand covers 2
5 minute divisions. = 60 and h = 2
Relative Speed = 60 – 5 = 55 minutes per
11
60 = m 30 2
hor. However, adopting the approach of actual 2
angles covered is far simplest and does not 11
create any confusion. m = 120
2
(4) Any angle other than (0 and 180) is made 22
240 9
times in a period of 12 hours. m= 21 min past2
(5) In a period of 12 hours there are 11 11 11
coincidences of the two hands, when the two 11
(or) 60 = 30 2 m
hands are in a straight line facing opposite 2
directions. 11
m =0
(6) The time gap between any two coincidences 2
is 12/11 hours or 655/11 minutes m=0
(7) If the hands of a clocks (which do not show Therefore, the angle between the hour hand
the correct time) coincide every ‘p’ minutes. and the hour hand and the minute hand is at
Then, If 655/11 then the watch is going slow or
losing time. 9
2 O’clock and at 21 minutes past 2O’clock.
If then the watch is going fast or gaining time. 11
(8) To calculate the angle ‘’ between the hands (2) When the two hands overlap, the angle between
of a clocks, we use the following formula them is
(where m = minutes and h = hours) 11
= m 30h
11 11 2
(i) θ m 30h when m 30h = 0 and h = 2
2 2
0 = 11 m 30 2
11 11
(ii) θ 30h m when30h m 2
2 2 120 10
m= 10 min past2.
Example 1 : What is the angle between the minute 11 11
hand and the hour hand of a clock at 3 hour 40 (3) When two hands are perpendicular = 90 and
minutes? h=2
(a) 20 (b) 70 11 11
= m 30h or 30h m
(c) 90 (d) 130 2 2
11
Solution (d) 90 = m 30 2
2
The angle between the hands can be calculated by 11
11 m = 150
θ m 30h where m is minutes and h is hours. 2
2 300 3
m= 27 minutes past2
Here, m = 40 and h = 3 11 11
11 11
40 30 3 = 220 – 90 = 130 (Or) 90 = 30 2 m
2 2
The angle between the two hands is 130 11
m = 30
Example 2 : Find the time between 2 and 3 O’clock 2
at which the minute hand and the hour hand As m cannot be negative, this case is not
(a) Make an angle of 60 with each other possible. So, the hands are perpendicular to each
3
(b) Overlap. other only once i.e. at 27 minutes past
11
(c) are perpendicular to each other. 2 O’clock.
1.30 Logical Reasoning Ability
(4) When two hands are pointing opposite = 2 24 odd days + 1 76 odd days
directions and are on a straight line the angle = 124 days
between them would be 180 ie. = 180 and
= 17 weeks + 5 days
h=2
The extra 5 days are the odd days.
11
180 = m 3h So, 100 years contains 5 odd days.
2
Similarly, for 200 years we have10 extra days (1 week
11
m = 180 + 60 = 240 +3 day)
2
200 years contains 3 odd days.
480 7
m = 43 Similarly 300 years contains 1 odd day and 400 years
11 11
7 contains 0 odd days.
So, at 43 minutes past 2 O’clock the hands
11 Counting of Number of Odd Days, When
will be at 180. only one Date is Given
17. CALENDARS Here we take January 1st AD as the earlier date and
Suppose you are asked to fine the day of the week we assume that this day is a Monday. We take is its
on 30th June 1974 it would be a tough job to find previous day i.e. Sunday as the reference day. After
because you do not know the method. The this the above mentioned method is applied to count
method of finding the day of the week lies in the the number of odd days and find the day of the week
number of “odd day” for the week for the given date.
NOTE Counting Number of Odd Days, When Two
Every 7th day will be the same day count wise, Dates Are Given
i.e. if today is Monday then 7th day counting from Any month which has 31 days has 3 odd days ( 31 7
Tuesday onwards will once again be Monday. Odd leaves 3 as reminder) and any month which has 30
days are the days remaining after completion of days has 2 odd days (30 7 leaves 2 as reminder).
an exact number of weeks. An Odd day is the Then, the total numbers of odd days are calculated by
reminder obtained on dividing the total number adding the odd days for each month. The value so
of days with seven. obtained is again divided by 7 to get the final number
Example: 52 days 7 = 3 odd days of odd days. The day of the week of the second date is
obtained by adding the off days the day of the week of
Leap and Non-Leap Year the earlier date.
A non –leap year has 365 days whereas a leap year Example 1 : If you were born on 14th April 1992 which
has one extra day because of 29 days in the month was a Sunday then on which day of the week does
of February .Every year which is divisible by 4 is your birthday fall in 1993?
called leap year. Leap year consists of 366 days,
(a) Monday (b) Tuesday
(52 complete weeks +2 days). The extra two days
are the odd days. So, a leap year has two odd days. (c) Wednesday (d) cannot be determined
A non-leap year consists of 365 days (52 complete Solution (a)
weeks +1 day). The extra one day is the odd day. 14th April 1992 to 14 the April 1993 is a complete year,
NOTE which has 365 days.
Every century, year which is a multiple of 400, is Hence the number of odd days from 14th April 1992 to
a leap year. A century year which is not divisible 14th April 1993 is 1.
by 400 is a non-leap year Hence 14th April 1993 is one day after Sunday ie.
Example: Monday
400,800, 1200, 1600…are leap years Example 2 : If 1st Jan 1992 is a Tuesday then on which
500,700, 900, 1900.. are non-leap years day of the week will 1st Jan 1993 fall?
(a) Wednesday (b) Thursday
Counting the Number of Odd Days
(c) Friday (d) Saturday
100 years consist of 24 leap years +76 ordinary
Solution (b)
years. (100 years when divided by 4 we get 25.
But at the 100th year is not a leap year, hence Since 1992 is a leap year there are 2 odd days. Hence
only 24 leap year). 1st January 1992 is two days after Tuesday i.e. Thursday.
Logical Reasoning Ability 1.31
Example 3 : If 1st April 2003 was Monday, then which 18. DEDUCTIONS
day of the week will 25th December of the same year be? In number of competitive exams there are a few
(a) Tuesday (b) Wednesday questions on “deductions”. Typically here, each
(c) Thursday (d) Friday question consists of two statements–on the basis
of which a deduction has to be made. The answer
Solution (b)
has to be chosen from the given four (or five)
The number of days from 1st April to 25th December choices and that will be the deduction made.
(29 + 31 + 30 + 31 + 31 + 30 + 31 + 30 + 25) days = 268 days. (If no conclusion can be drawn or the answer is
268 not obtained then the choice which is normally
= 38 2odd days. worded as “none of the above” has to be marked).
7
Hence 25th December is two days after Monday ie. These questions can be answered by representing
Wednesday the given statements by Venn Diagrams.
However, here we will look at arriving at the
Example 4 : On which day of the week does 4th June deduction by using some simple rules.
2001 fall?
First let us look at some basic terms used in the
(a) Monday (b) Tuesday rules and understand what they mean. The two
(c) Wednesday (d) Thursday statements given in the question are called
Solution (a) ‘premises’ & the answers are conclusion.
4th June 2001 (2000) year+1st January to 4th June Eg. All dogs are cats—(i)
2001 All cats are pigs —(ii)
We know that 2000 years have zero odd days. The The premises normally start with the words All,
number of odd days from 1st January to 4th June 2001 No Some and Some –not
Month: Jan + Feb + Mar + Apr + May + June The word “All” has its synonyms as –Every, Any,
Each, whereas the word “Some “ can also be
Odd day: 3 + 0 + 3 + 2 + 3 + 4
replaced by Many, Few, A little, Most of, Much
15 of, More etc.
1odd day.
7 These words are referred to as qualifiers (also
Hence 4th June 2001 was a Monday. termed as quantifiers).
Example 5 : Which year will have the same calendar A premise consists of a subject and a predicate
as that of 2005? wherein the first term [e.g. “dogs” in statement
(a) 2006 (b) 2007 (i)] is the subject and the second term [e.g. “cat”
in statement (i)] the predicate. Similarly, in
(c) 2008 (d) 2011 statement (ii),”cats” is called the subject and “pigs”
Solution (d) is the predicate.
Year: 2005 + 2006 + 2007 + 2008 + 2009 + 2010 The word that occurs in both the premises is
Odd days: 1 + 1 + 1 + 2 + 1 + 2 + 1 known as the ‘middle term’ (“cat” in the example,
give above). The answer or “conclusion” should
Total numbers of odd days from 2005 to 2010 are 7 = 0
consists of the other two words (“dog and “pig’s
odd days.
in the above example) and the middle term should
Hence 2011 will have the same calendar as that of not appear in the answer.
2005
The premises can be divided into
Example 6 : What day of the week was 18th April 1901?
(a) Universal statements and
(a) Monday (b) Tuesday
(b) Particular statements.
(c) Wednesday (d) Thursday
This classification of the premises into the above
Solution (d) categories is dependent on the qualifier used in
18th April 1901 (1600 + 300) years + 1st January to the premise. For example, the statements where
18th April 1901 “All” is used are called Universal statements and
1600 years have = 0 odd days the statements where “Some” is used are called
Particular statements where “Some” is used are
300 years have – 1 odd day. The number of days from called Particular statements. Premises can also
1st January 1901 to 108 days 3 odd days be divided into
Total number of odd days = 3 + 1 = 4 (i) Positive (affirmative) statements and
Hence 18th April 1901 is Thursday (ii) Negative statements)
1.32 Logical Reasoning Ability
If there is a negative term like “not” or “no” in the second statement is also Universal affirmative, the
statement it is called a negative premise. Otherwise it subject cat is distributed () and predicate pigs is not
is called a positive premise or an affirmative statement. distributed (). The above answer/ logic is arrived at
The combination of the two different categories of on the basis of Table II.
classification leads to four different premises as given
in Table I below. The middle term (“cat is the middle term as it occurs
in both the premises) is distributed once in the
Table I premises. Hence it satisfies Rule[2] As “dogs” is
Affirmative Negative distributed in the premise and “pigs” is undistributed
Universal All A No E in the deduction also, they should appear accordingly.
Particular Some many l Some not many O The type of statement that satisfies both of them is
Universal affirmative statement i.e. a statement with
The subject and the predicate are either distributed “All” Hence the answer will be
() Or not distributed () depending on what kind of a
“All dogs are pigs”
statement it is (particular affirmative etc.)
The answer cannot be “All pigs are dogs”, because Rule
Table II shows the distribution Pattern of the subject
and the predicate. [7] states that no term can be distributed in the
conclusion if it is not distributed in the premises. As
Table II “pigs” is not distributed in the premise. It cannot be
Subject Predicate distributed in the conclusion (because if we take “All
Universal affirmative √ x pigs are dogs”, then the subject “pigs” will be
distributed). Hence the conclusion “All pigs will dog”
Universal negative √ √
is wrong.
Particular affirmative x x
Example 2 :
Particular negative x √
All cats are dogs — (i)
Note: () indicate distributed () indicates
All cats are pigs - - (ii)
undistributed.
Statement (i) is Universal affirmative and hence the
Rules for Deductions subject “cats” is distributed and the predicate “dogs “is
(1) Every deduction should contain three and only not distributed as per Table II.
three distinct terms. Statement (ii) is also Universal affirmative and hence
(2) The middle term must be distributed at least once the subject “cats” is distributed and thee predicate “pigs”
in the premises. is not distributed as per Table II.
(3) If one premise is negative, then the conclusion Here, the middle term “cats” (“cats” is the middle term
must be negative. as it is occurring in both the premises) is distributed;
(4) If one premise is particular then the conclusion hence we can draw a conclusion.
must be particular The answer should contain the terms “dogs” and
(5) If both the premises are negative, no conclusion “pigs” and both the terms are not distributed Referring
can be drawn to Table II ,we find that this is possible only in
Particular affirmative [the conclusion cannot start with
(6) If both the premises are particular, no conclusion
qualifier ‘All’ as the subject in “All” should be
can be drawn
distributed]. According to Rule 7 a term cannot be
(7) No term can be distributed in the conclusion, if it distributed in the conclusion if it is not distributed in
is not distributed in the premises. the premises. So the answer will be
We take example of each type and look at them in “Some dogs are pigs” Or “Some pigs are dogs”
details.
Example 3 :
Example 1 :
All dogs are cats (i)
All dogs are cats — (i)
All pig are cats (ii)
All cats are pig (ii)
Statements (i) is a Universal affirmative and hence
As the first statement is a Universal affirmative the subject “dogs” is distributed and the predicate “cats”
statement. The subject (dog) has to be distributed () is not distributed. In statement (ii) which is also a
and the predicate (cats) is not distributed (). As the Universal affirmative the subject “Pigs” is distributed
Logical Reasoning Ability 1.33
and the predicate “cats” is not distributed this is arrived In the above given example no conclusion can be drawn
at one the basis of Table II. , as rule [2] states that the middle term(“cats” in the
The middle term “cats” [“cats” is the middle term as it above example as it occurs in both the premises) should
occurs in both the statements]is not distributed in be distributed at least once in the premises. This is
either of the two statements From Rule [2] which states not satisfied.
that the middle term should be distributed at least Example 7 :
once in the premises for drawing a conclusion we All cats are dogs (i)
cannot draw any conclusion in this case.
Some cats are not pigs (ii)
Example 4 :
The first statement is a universal affirmative and hence
All cats are dogs. (i) “cats” is distributed and “dogs” is not distributed The
Some cats are pigs (ii) second statement is a Particular negative and hence
The first statement is a Universal affirmative premise “cats” is not distributed and “pigs” is distributed (as
and hence the subject “cats” is distributed and per Table II)
the predicate “dogs” is not distributed(x). The second Here, the middle term (cats) is distributed and hence
statement is Particular affirmative and hence both the we can draw a conclusion.
subject “cats” and the predicate “pigs” are not The conclusion should be particular negative as Rule
distributed (x)as per Table II. As we have a particular [3] states that if a premise is negative. Also Rule[4]
premise, the conclusion. Should also be a particular state that if a premise is Particular, the conclusion
one as per Rule [4]. The middle term is distributed should also be Particular. Hence the conclusion should
hence we can draw a conclusion .So the answer be a particular negative.
will be
In particular negative, we know that the subject is not
“Some dogs are pigs” Or “Some pigs are dogs” distributed and the predicate is distributed.
Example 5 : The terms “dogs” and “pigs” should come in the
All dogs are cats (i) conclusion. Also since “dogs” is not distributed in the
No cats are pigs (ii) premise, it cannot be distributed in the conclusion, as
per Rule [7]
As the first premise is a Universal affirmative, the
subject (dogs) is distributed and the predicate (cats) is As per the above reasoning only “pigs” can be the
not distributed. In the second premise which is predicate in the conclusion and hence “dogs” will be
a Universal negative, the first term (cats) and the the subject
second term (pigs) are both distributed (as per Table Thus the answer will be “Some dogs are not pigs”
II) As the middle term is distributed at least once in Example 8 :
the premises. Rule [2] is satisfied and hence we can
No dogs are cats (i)
draw a conclusion.
No cats are pigs (ii)
From Rule 3] which states that if one of the premises
is negative the conclusion should be negative, the We cannot draw any conclusion, as Rule [5] states that
conclusion should be negative and as both the terms if both premises are negative, we cannot draw any
“dogs” and “pigs” are distributed, the conclusion should conclusion.
be a Universal negative statement. Hence the answer Example 9 :
will be No dogs are cats
“No dogs are pigs” Or “No pigs are dogs” Some cats are not pigs.
Example 6 : As both the premises are negative hence as per Rule
All dogs are cats (i) [5] we cannot draw any conclusion (Please note that
Some cats are not pigs (ii) the first premise is a Universal negative and hence
the subject (dogs) is distributed and the predicate (cats)
Since the first statement is a Universal affirmative
is also distributed as per Table II.
“dogs” is distributed and “cats” is not distributed.
Since the second statement is a Particular negative, The second statement is a Particular negative and
“cats” is not distributed and “pigs” is distributed (as hence the subject (cats) is not distributed and the
per Table II) predicate (pigs) is distributed a per Table II)
1.34 Logical Reasoning Ability
Flow Chart
start
Mark choice Mark choice Mark choice Mark choice Mark choice
(a) (b) (a) (c) (d)
as your as your as your as your as your
answer answer answer answer answer
EXERCISE
MCQ TYPE QUESTIONS 5. What was the colour of the saree that Ms. Uttar
Pradesh was wearing?
1. Given two numbers x and y, define (a) White (b) Green
Addition A(x, y) = x + y (c) Red (d) Yellow
Subtraction S (x, y) = x – y Directions Q. 6 – 7 :
Multiplication M (x, y) = x, y A, B, C, D, E and F are a group of friends from a club.
Division D (x, y) = x/y There are two housewives, one lecturer, one architect,
one accountant and one lawyer in the group. There
S[M(D(A(a, b), 2), D(A(a, b), 2)), are two married couples in the group. The lawyer is
M(D(S(a, b), 2), D(S(a, b),2))] is equal to
married to D who is a housewife. No lady in the group
(a) (a – b)2 (b) ab is either an architect or an accountant C. The
(c) (a + b)2 (d) None of these accountant, is married to F who is a lecturer. A is
Directions Q. 2 – 5 : married to D and E is not a housewife.
Kamal Babu came home just after judging a beauty 6. What is E ?
contest where there were four semi-finalists,
(a) Lawyer (b) Architect
Ms. Andhra Pradesh, Ms. Uttar Pradesh,
Ms. Maharashtra and Ms. West Bengal. His wife (c) Lecturer (d) Accountant
was very keen on knowing who the winner was and 7. How many members of the group are males?
Kamal Babu replied immediately that it was the one (a) 2 (b) 3
wearing the yellow saree. When his wife asked for
more details, he gave the following information: (c) 4 (d) none of these
* The four girls were wearing sarees of different Directions Q. 8 – 9 :
colours (yellow, red, green, white) and the runner- Seven university cricket players are to be honoured
up was wearing green. at a special luncheon. the players will be seated on
* The four girls were sitting in a row, and Ms. West the dais along one side of a single rectangular table.
Bengal was not sitting at either end. A and G have to leave for the lunch early and must
* There was only one runner-up and she was sitting be seated at the extreme right end of the table which
next to Ms. Maharashtra. is closest to the exit.
* The girls wearing yellow and white sarees B will receive the Man of the Match award and must
occupied the seats at either end. be in the center chair.
* Ms. West Bengal was nither the winner nor the C and D, who are bitter rivals for the position of
runner-up. wicket keeper, dislike each other and should be
* Ms. Maharashtra was wearing white. seated as far apart as possible.
* The winner and the runner-up were not sitting E and F are best friends and want to sit together.
next to each other.
8. Which of the following may not be seated at either
* The girl wearing the green saree was not end of the table ?
Ms. Andhra Pradesh.
(a) C (b) D
Answer the following questions based on the above
information: (c) G (d) F
2. What was the colour of the saree that 9. Which of the following pairs may not be seated
Ms. Andhra Pradesh was wearing? together ?
(a) White (b) Yellow (a) E & A (b) B & D
(c) Red (d) Cannot be determined (c) C & F (d) G & D
3. Between which two was Ms.West Bengal sitting? Directions Q. 10 –13 :
(a) Ms. Andhra Pradesh and Ms. Maharashtra A study was conducted to ascertain the relative
(b) Ms. Andhra Pradesh and Ms. Uttar Pradesh importance that employees in five different countries
(c) Ms. Uttar Pradesh and Maharashtra assigned to five different traits in their Chief Executive
(d) Cannot be deternmined Officers. The traits were compassion (C), decisiveness
4. Who was wearing the red saree? (D), negotiation skills (N), public visibility (P), and
(a) Ms. Andhra Pradesh vision (V). The level of dissimilarity between two
countries is the maximum difference in the ranks
(b) Ms. Uttar Pardesh
allotted by the two countries to any of the five traits.
(c) Ms. West Bengal
The following table indicates the rank order of the
(d) Cannot be determined five traits for each country.
Logical Reasoning Ability 1.37
Country Raj said his son could not come because of his exams.
RANK INDIA CHINA JAPAN MALAYSIA THAILAND 14. Which woman arrived third ?
1 C N D V V (a) Shanthi (b) Sridevi
2 P C N D C (c) Anita (d) Joya
3 N P C P N
15. Name the correct pair of husband and wife ?
4 V D V C P
(a) Raj and Shanthi (b) Sunil and Sridevi
5 D V P N D
(c) Anil and Sridevi (d) Raj and Anita
10. Which of the following pairs of countries are most 16. Of the following pairs, whose daughters go to the
dissimilar? same school ?
(a) China and Japan (a) Anil and Raman (b) Sunil and Raman
(b) India and China (c) Sunil and Anil (d) Raj and Anil
(c) Malaysia and Japan 17. Whose family is known to have more than one
(d) Thailand and Japan kid for certain ?
11. Which of the following countries is least dissimilar (a) Raman's (b) Raj's
to India? (c) Anil's (d) Sunil's
(a) China (b) Japan Directions Q. 18 – 20 :
(c) Malaysia (d) Thailand Answer the following questions based on the
12. Which amongst the following countries is most statements given below:
dissimilar to India? (i) There are three houses on each side of the road.
(a) China (b) Japan (ii) These six houses are labeled as P, Q, R, S, T and
(c) Malaysia (d) Thailand U.
13. Three of the following four pairs of countries have (iii) The houses are of different colours, namely, Red,
identical levels of dissimilarity. Which pair is the Blue, Green, Orange, Yellow and White.
odd one out? (iv) The houses are of different heights.
(a) Malaysia and China (v) T, the tallest house, is exactly opposite to the
(b) China and Thailand Red coloured house.
(c) Thailand and Japan (vi) The shortest house is exactly opposite to the
(d) Japan and Malaysia Green coloured house.
Directions Q. 14 – 17 : (vii) U, the Orange coloured house, is located between
P and S.
Four families decided to attend the marriage ceremony
of one of their colleagues. One family has no kids, (viii)R, the Yellow coloured house, is exactly
while others have at least one kid each. Each opposite to P.
family with kids has at least one kid attending the (ix) Q, the Green coloured house, is exactly opposite to
marriage. U.
Given below is some information about the families, (x) P, the White coloured house, is taller than R,
and who reached when to attend the marriage. but shorter than S and Q.
The family with 2 kids came just before the family 18. What is the colour of the house diagonally opposite
with no kids. to the Yellow coloured house?
Shanthi who does not have any kids reached just before (a) White (b) Blue
Sridevi's family. (c) Green (d) Red
Sunil and his wife reached last with their only kid. 19. Which is the second tallest house?
Anil is not the husband of Joya. (a) P (b) S
Anil and Raj are fathers. (c) Q (d) cannot be determined
Sridevi's and Anita's daughters go to the same school. 20. What is the colour of the tallest house?
Joya came before Shanthi and met Anita when she (a) Red (b) Blue
reached the venue. (c) Green (d) Yellow
Raman stays the farthest from the venue.
1.38 Logical Reasoning Ability
Directions for questions 21 to 25: 30. How many small cubes have exactly two faces
Select the correct alternative from the given choices painted pink?
21. If five cuts are made on a cube, what is the (a) 36 (b) 48
minimum number of pieces obtained? (c) 64 (d) 80
(a) 18 (b) 6 31. How many small cubes have exactly three faces
(c) 16 (d) 25 painted blue?
22. If six cuts are made on a cube, what is the (a) 8 (b) 4
maximum number of identical pieces obtained? (c) 2 (d) 6
(a) 16 (b) 18 Directions for questions 32 to 34:
(c) 36 (d) 27 These questions are based on the following information
23. If two, three and four cuts are made parallel to A cube is painted in black and green, each on three
different faces of a cube, then what is the number faces such that any two faces with same color are
of identical pieces obtained? adjacent to each other. Now this cube is cut into 60
(a) 60 (b) 30 identical pieces using 2, 3, and 4 cuts parallel to
(c) 48 (d) 24 different faces.
24. What is the minimum number of cuts required 32. How many smaller pieces have exactly two faces
to cut a cube into 216 identical pieces obtained? painted in black color?
(a) 36 (b) 18 (a) 5 (b) 9
(c) 15 (d) 12 (c) 18 (d) 27
25. What is the minimum number of cuts required 33. How many smaller pieces have both
to cut a cube into 24 identical pieces? the colors on them?
(a) 6 (b) 8 (a) 9 (b) 18
(c) 9 (d) 12 (c) 6 (d) 24
Directions for questions 26 to 28: 34. How many smaller pieces have no face painted?
These questions are based on the following information (a) 6 (b) 9
A cube, painted on all its faces, is cut into 125 identical (c) 11 (d) 1
smaller cubes.
Directions: Q. No 35 to 39: These questions are
26. How many smaller cubes have no face painted? based on the information given below:
(a) 100 (b) 81 Four ladies A, B, C and D and four gentlemen E, F, G and
(c) 64 (d) 27 H are sitting in a circle round a table facing each other.
27. How many smaller cubes have only one faced (a) No two ladies or two gentlemen are sitting side by
painted? side.
(a) 36 (b) 54 (b) C who is sitting between G & E is facing D.
(c) 64 (d) 108 (c) F is between D and A is facing G.
28. How many smaller cubes have exactly two faces (d) H is to the right of B
painted? 35. Who is sitting to the left of A?
(a) 36 (b) 54
(a) E (b) F
(c) 48 (d) 60
(c) G (d) h
Directions for questions 29 to 31:
36. E is facing whom?
These questions are based on the following information
(a) F (b) B
Each of 216 small identical cubes are painted blue on
(c) G (d) None of these
all faces and all these cubes are arranged to form a
large cube. Now all the faces of the large cube are 37. Who are immediate neighbor of B?
painted pink. (a) G and H (b) E and F
29. How many small cubes have only one color on (c) E and H (d) F and H
them? 38. Who is sitting between E and G?
(a) 96 (b) 125 (a) A (b) B
(c) 64 (d) 48 (c) C (d) D
Logical Reasoning Ability 1.39
39. Who is sitting to the right of D? 46. Which of the following cannot be the correct seating
(a) F (b) A arrangement of the five persons in either the
clockwise direction or the anti-clockwise directon ?
(c) E (d) C
(a) P,Q,R,S,T (b) PS,R,T,Q
Directions Q. No 40 to 44
(c) P,Q,S,R,T (d) P,T,R,S,Q
Study the following information carefully and
answer the question given below: A, B, C, D, E, 47. If S is not sitting next to Q, who sitting between
F, G and H are sitting around a circle facing the Q and S?
centre. B is second to the right of D who is third (a) R (b) P
to the right of F, C is second to the left of A who (c) T (d) Both R and P
is second to the left of F, G is third to the right 48. If a new person U joins the group such that the
of E. intial conditions for the seating arrangement
40. In which of the following combinations is the first should be observed and also a new condition that
person sitting between the second and the third U does not sit next to R be satisfied then which
persons? of the following statements is true?
(a) GCD (b) FGH (a) U sits to the immediate right of S
(c) EFH (d) ABE (b) U sits to the immediate left of T
41. Who is third to the right of H? (c) U sits to the immediate left of P.
(a) Data inadequate (d) P is to the right of Q
(b) D 49. If a new person U joins the group such that the
initial condtions for the seating arrangement
(c) C
should be observed and also a new condition that
(d) G U does not sit next to P,S or T be satisfied then
42. Who is the immediate right of A? who will be the neighbours of P (one either side)?
(a) Data inadequate (a) S and T (b) S and Q
(b) E (c) T and R (d) R and Q
(c) F Directions for questions 50 to 59:
(d) B Select the correct alternative from the given choices
43. What is H’s position with respect to B? 50. What is the angle covered by the minute hand in
22 minutes?
(a) Fifth to the right
(a) 66 (b) 110
(b) Third to the left
(c) 132 (d) 220
(c) Third to the right
51. By how many degrees does an hour hand move
(d) None of these in one quarter of an hour?
44. Who is the immediate left of G? (a) 5 (b) 7.5
(a) H (c) 10 (d) 12.5
(b) F 52. By how many degrees will the minute hand move,
(c) Data inadequate in the same time, in which the hour hand moves ?
(d) D (a) 54 (b) 84
Directions for questions 45 to 49 : (c) 72 (d) 60
Read the following information and answer the 53. What is the angle between the hands of the
questions that follow. clocks, when it shows 40 Minutes past 6?
P,Q,R,S and T sit around a table. P sits two seats to (a) 40 (b) 70
the left of R and Q sits two seats to the right of R. (c) 80 (d) 90
45. If S sits in between Q and R, who sits to the 54. When the clocks shows 3 hours 14 minutes what
immediate right of P? is the angle between the hands of the clocks?
(a) T (b) S (a) 10 (b) 12
(c) Q (d) R (c) 13 (d) 14
1.40 Logical Reasoning Ability
55. What is the angle between the two hands 61. How many odd days are there in 352 days?
of a clock when the time is 25 minutes past (a) 1 (b) 2
7 O’clock?
(c) 3 (d) zero
(a) 66½ (b) 66½
62. The number of odd days for the year 1996 is
(c) 72½ (d) 69½
(a) 0 (b) 1
56. When the clock shows 20 minutes past 11 O’clock,
what is the angle between the two hands of the (c) 2 (d) 3
clocks? 63. Which among the following years is a leap year?
(a) 220 (b) 120 (a) 3000 (b) 3100
(c) 230 (d) 140 (c) 3200 (d) 3300
57. At the time between 9 and 10 O’clock, will both 64. What will be next leap year after 2096?
the hands of the clocks coincide? (a) 2100 (b) 2101
3 (c) 2104 (d) 2108
(a) 43 Minutes past 9 O’clock
11 65. If 5th January 2001 was a Friday then 25th
December 2001 was a
6
(b) 45 Minutes past 9 O’clock (a) Monday (b) Tuesday
11
(c) Wednesday (d) Thursday
6
(c) 45 Minutes past 9 O’clock st
66. If 21 march 2000 was on Monday, what day of
11
the week will be 21st March 2003?
6 (a) Tuesday (b) Friday
(d) 49 Minutes past 9 O’clock
11 (c) Thursday (d) Wednesday
58. At what time between 4 and 5 O’clock are the 67. If 14th February 2001 was a Wednesday then
hands of a clock in the opposite directions? what day of the week will be 14th February 2101
3 (i.e. after a century)?
(a) 52 minutes past 4 O’clock
11 (a) Friday (b) Saturday
6 (c) Sunday (d) Monday
(b) 54 minutes past 4 O’clock
11 68. If in a calendar year, there are 541 days and 10
6 days a week then how many odd days will be there
(c) 51 minutes past 4 O’clock
11 in that year?
9 (a) 1 (b) 2
(d) 53 minutes past 4 O’clock
11 (c) 3 (d) 4
59. The angle between the hands of a clock is 20 69. If 8 February 1995 was a Wednesday then 8th
th
and the hour hand is in between 2 and 3. What is February 1994 was on which day?
the time shown by the clock?
(a) Wednesday (b) Thursday
3 (c) Tuesday (d) Monday
(a) 2 hours 7 minutes
11 th
70. If 11 August 1985 was a Sunday then which day
6 of the week was 13th August 1986?
(b) 2 hours 14 minutes
11
(a) Tuesday (b) Wednesday
5
(c) 2 hours 15 minutes (c) Thursday (d) Friday
11 st
71. If 1 January 2012 is a Sunday then which days
(d) Both (a) and (b)
of the week will the New Year is celebrated in
Directions for questions 60 to 72: 2016?
Select the correct alternative from the given choices. (a) Friday
60. The number of odd days in non-leap year is
(b) Sunday
(a) 0 (b) 1
(c) Wednesday
(c) 2 (d) 3
(d) Saturday
Logical Reasoning Ability 1.41
72. If holidays are declared only on Sundays and in a 76. Which batches meet for the same number of
particular year 12th March is a Sunday, is 23rd classes during the week?
September in that year a holiday? (a) B, A, F only (b) E, B, C only
(a) Yes (c) E, F, A only (d) None of these
(b) No 77. If a certain class of D is scheduled on the same
(c) Yes, if it is a leap yea today as that of B batch, then how many batches
(d) No, if it is a leap year meet on Friday?
Direction for Q. 73 to 75: (a) 1 (b) 2
Read the following information and answer the (c) 3 (d) 4
questions that follow. 78. If there is one such day that all batches are to meet,
Abhishek, Bharat, Chetan, Dinesh Eklvya and then which batch would gain an additional day?
Fraklin are friends married to Richa, Stuti, Urmila, (a) A (b) B
Varsha, Trupti and Wanda not necessarily is same
(c) D (d) E
order.
79. Which two batches can never be conducted on
Following facts are known about them-
the same day?
(1) Richa and stuti are Abhishek’s sisters
(a) C and D (b) C and E
(2) Neither Richa nor trupti are wives of Chetan
(c) A and D (d) None of these
(3) Wanda is wife of Eklavya and Varsh is wife of Bharat
Directions for Qu. 80 to 82
(4) Dinesh is not married to Richa, stuti or trupti
Read the following information and answer the
73. Who is Abhishek’s wife?
questions that follow.
(a) Richa (b) Urmila
Abhita is bright girl who has a CD box on her study
(c) Trupti (d) cannot be determined table. The box has three horizontal row each can be
74. Richa is wife of filed with 4 CD cases. She has also put her “Jewellery
(a) Chetan (b) Franklin box” in the CD box which consume the space of two
(c) Bharat (d) Dinesh CD ceases.
75. Dinesh is married to The following information is known
(a) Urmila (b) Barsha (1) The jewellery box is in the bottom row.
(c) Trupti (d) Richa (2) The CD cases with “Window XP” and “Windows
2000” are horizontally adjacent to each other.
Directions for Qu. 76 to 79
(3) The CD case with Linux is in the bottom row.
Read the following information and answer the
questions that follow. (4) “Utility tools” is placed horizontally adjacent to
“Game1” CD.
XYZ, an institute involved in training the students for
MBA, MCA, MMS and other entrance exams runs six (5) A vertical columns containing “Linux” and “music”
batches A, B, C, D, E and F, each of which is scheduled also contains “Movies”
of at least once a week. (6) “Net tools” is just below the “Windows XP” and
(1) Sunday is a holiday. “CS II” is just above “Game 2”
(2) The B batch meets all days except Friday and (7) “Net tools” is not at the corner.
Saturday. 80. Which CD is just below “Net tools”?
(3) The C batch meets 4 days in succession. (a) Utility tools (b) Games 1
(4) The F batch meets only from Monday to Thursday. (c) CS II (d) None of these
(5) The E batch is scheduled every day, but not on 81. If “Movie” is in second row, then how many CDs
Thursday and Saturday. are between “Movies” and “Game2”?
(6) On alternate days, the A batch is scheduled. (a) 0 (b) 1
(7) The A and D batches never meet on the same days. (c) 2 (d) Cannot be determined
(8) The C batch does not meet on Mondays and 82. If “Music is in top row, how many CDs are
Tuesday. between “Music” and “Utlity tools”.
(9) The D batch is scheduled only once a week on
(a) 0 (b) 1
either Wednesday or Friday.
(c) 2 (d) Cannot be determined
1.42 Logical Reasoning Ability
ANSWERS
MCQ Type Questions
1. (b) 2. (b) 3. (b) 4. (c) 5. (b) 6. (b) 7. (b) 8. (d) 9. (a) 10. (a)
11. (b) 12. (d) 13. (d) 14. (a) 15. (b) 16. (c) 17. (b) 18. (d) 19. (d) 20. (b)
21. (b) 22. (d) 23. (a) 24. (c) 25. (a) 26. (d) 27. (b) 28. (a) 29. (c) 30. (b)
31. (a) 32. (b) 33. (b) 34. (a) 35. (b) 36. (d) 37. (a) 38. (c) 39. (a) 40. (d)
41. (a) 42. (a) 43. (d) 44. (c) 45. (a) 46. (a) 47. (b) 48. (c) 49. (a) 50. (c)
51. (b) 52. (c) 53. (a) 54. (c) 55. (c) 56. (d) 57. (c) 58. (b) 59. (d) 60. (b)
61. (b) 62. (c) 63. (c) 64. (c) 65. (b) 66. (c) 67. (d) 68. (a) 69. (c) 70. (b)
71. (a) 72. (b) 73. (c) 74. (b) 75. (a) 76. (d) 77. (b) 78. (a) 79. (c) 80. (d)
81. (c) 82. (b) 83. (a) 84. (a) 85. (b) 86. (d) 87. (b) 88. (c) 89. (a) 90. (c)
91. (c) 92. (b)
Numerical Type Questions
1. 70 2. 105 3. 2 4. 2 5. 11 6. 7920 7. 12870 8. 35 9. 10
1.44 Logical Reasoning Ability
EXPLANATIONS
MCQ TYPE QUESTIONS 11. From above calculations, highest level of
dissimilarity is 4 which is for Japan.
1. S[M(D(A(a, b)2, D(A(a, b)2)), M(D(S(a,b)2)) 12. The level of dissimilarity between
D (s(a,b)2)]
China and Japan is for the parameter D
=S
LM M a b , a b M a b ; a b OP (4 – 1) = 3
N 2 2 2 2 Q India and China is for the parameter N
F a bIJ – FG a bIJ = ab
= G
2 2 (3 – 1) = 2
H 2 K H 2 K Malaysia and Japan is for the parameter V
(4 – 1) = 3
6 – 7.
Thailand and Japan is for the parameter D
A Lawyer Male
(5 – 1) = 4
B Housewife Female
13. The level of dissimilarity between
C Accountant Male
D Housewife Female Malaysia and China is for the parameter V
E Architect Male (5 – 1) = 4
F Lecturer Female China and Thailand is for the parameter V
Married Couple : AD & CF (5 – 1) = 4
8—9. The seating arrangement is shown below: Thailand and Japan is for the parameter D
Left Right (5 – 1) = 4
7 6 5 4 3 2 1 Japan and Malaysia is for the parameter V
D/C E/F F/E B C/D G/A A/G (4 – 1) = 3
10 – 13. 14 to 17 :
Level of Dissimilarity is equal to Maximum The given set of questions is based on distribution
difference in ranks alloted to any two countries on and sequencing.
any of the five traits. Firstly, let us put the names of women and men,
For example, dissimilarity between India and China. Women : Shanthi, Sridevi, Joya, Anita
Trait India’sRank China’s Rank Difference Men : Sunil, Anil, Raj, Raman
C 1 2 1 It is given that(2 kids family) (No
came before
P 2 3 1 kid family)
N 3 1 2
V 4 5 1 Also Shanti (0 kids) came
before
Sridevi
D 5 4 1 As Sunil and his wife came with their only kid,
From the above table, maximum difference is 2. hence Sunil cannot be the husband of Shanthi.
So dissimilarity level between India and China is 2. As Anil and Raj are fathers, hence neither of
10. Calculating the level of dissimilarity for India them is the husband of Shanthi. Hence Raman
Vs remaining countries. is the husband of Shanthi,
China : For the parameter N (3 – 1) = 2 Also, Joya came before Shanthi and met Anita
Japan : For the parameter D (5 – 1) = 4 on the venue, this gives the following order of
Malaysia : For the parameter C (4 – 1) = 3 their reaching the venue. [Anita, Joya, Shanthi,
Thailand : For the parameter V (4 – 1) = 3 Sridevi]
Hence least level of dissimilarity is 2 which is As Sunil reached last, Sunil is the husband of
for China. Sridevi. This means that Raj is Joya’s husband
and Anil is Anita’s husband.
Following table gives the desired details
Order Women Men No. of Kids
1 Anita Anil at least 1
2 Joya Raj 2
3 Shanthi Raman 0
4 Sridevi Sunil 1
Logical Reasoning Ability 1.45
33. Except two corners, all the corners have both 45. If S sits between Q and R, then the arrantgements is
the colors 6 pieces. as follows
On the edges the number of pieces having both the S
colors according to the different cuts in each plane. R
Q
Required number of pieces = 6 + 12 = 18
34. The number of pieces with no face painted = 1
(along the plane with 2 cuts) +2 (along the plane
with 3 cuts) + 3 (along the plane with 4 cuts)
T
Solution for questions 35 to 39:
All information can be drawn in a circle P
A As can be seen from the diagram,T is to the
F immediate right of P.
E
46. We will take each choice and see whether it fits
in the arrangement that we represented through
C D
a diagram in the analysis of the data (the same
diagram is reproduced below).
G H
S or T
B R
Q
35. F is sitting the left of A.
36. E is facing H
37. G and H are immediate neighbors’ of B
38. C sitting between E and G T or S
39. F is sitting to the right of D.
p
Solution for questions 40 to 44:
We can see that the arrangement given in
All information can be drawn in a circle
choice (a) is not possible and hence the answer
G
choice is (a)
H
D 47. If S is not next to Q, then the seating arrangement
is fixed as follows
C F
T
R
B E Q
A
40. A is sitting between B and E.
41. C is thief to the right of H.
S
42. E is to the immediate right of A.
43. H is to the fourth to the left of B. p
44. H is the immediate left of G. Now, P between Q and S.
Solution for questions 45 to 49: 48. On the basis of the diagram that we drew , we
P sits two seats the left of R, and Q sits two seats to find that to accommodate U we have to create a
the right of R. We can represent this information in new space between P and Q
the diagram below. S or T
R
S or T Q
R
Q
U T or S
T or S p
49. We create a new slot for the sixth person B. But As angle is more than 180 the angle must be
since U will not sit next to P, S or T, he will have 360 – 220 = 140
to sit between R and Q. The arrangement will
57. When hands coincide with each other the angle
then look as follows:
between them is 0. Therefore angle between
U hands is given by
R
Q 11 11
θ = 30 – m Q30h > m here h = 9
2 2
2
270 m
T or S 11
T or S
1
p m = 49 = 12 minutes past 9 hours.
As we can see from the diagram, the neighbors of 11
P will be T and S. 58. When hands of a clock are in opposite direction
the angle between them is 180
50. The angle covered by the minute hand in 22
minutes is 22 6 = 132 11
Therefore θ = m – 30h where
51. The hour hand covers 360 in 12 hours. 2
1 = 180 and h = 4
It covers in one minute. In quarter of an
2 11
hour i.e., in 15 minutes the hour hand will 180 = m – 120
2
1
move 15 7.5 11
2 m = 300
2
52. The hour hand will move by 6 in 12 minutes, so
minute hand will move 12 6 = 72 in 12 600 6
m= m = 54 minutes
minutes, as the minute had moves by 6 in one 11 11
minute So, at 54 minutes past 4 the hands are in opposite
11 direction.
53. Angle will be θ = m – 30 h
2 59. Given = 20 and h = 2
11 11 11
2 20 30 6 40 θ= m – 30h or 30h – m
2 2
54. The angle between the hands will be 11 11
20 = m – 30 2; m = 80
11 2 2
θ= m – 30 h Here h = 3 and m = 14
2 160 6
m= = 14 minutes
11 11 11
θ= 14 – 30 3
2 11 11
or 20 = 30 2 – m; m = 40
θ = 77 – 90 = 13 2 2
55. Angle between two hands is given by 80 3
m= = 7 minutes
11 11 11
θ= m – 30h Here h = 7 and m = 25
2 Therefore the angle between the hands will be
11 275 210 145 1 6 3
25 30 7 72 20 at hors 14 minutes past 2 and 7 minutes
2 2 2 2 11 11
past 2.
56. Angle between the two hands is given by
60. Total numbers of days in an ordinary year are
11 365. Number of odd days
θ= m – 30h here m = 20 and h = 11
1
2 365
= 52 weeks + 1 odd day
11 7
20 30 11 220 Therefore number of odd days is 1.
2
1.48 Logical Reasoning Ability
61. The number of days in 352 days 71. The total number of years from 2012 to 2016 is
352 four out of which 2013, 2014 and 2015 are non
= leap years. Hence there is 1 odd day in each of
7
these years 2012 is a leap year , hence it has 2
= 50 weeks + 2 odd days.
Hence, the total number of odd days is 2 odd days.
62. The year 1996 is a leap year. Total number of The total number of odd days in these four
days in a leap year are 366 days. Number of odd years is 5. 1st Jan 2016 is five days to Sunday
i.e. Friday
366
days 52 weeks + 2 odd days. 72. The total number of odd days from 12th March to
7
23rd September
63. Century years which are divisible by 400 are
leap years. As 300 and 3300 are not divisible by Months : M + A + M + J + J + A + S
400 they are not leap years, but 3200 is a leap Odd day : 5 + 2 + 3 + 2 + 3 + 3 + 2 = 20days.
year. 20
= 6 odd day
64. For a century year to be a leap year it should 7
be divisible by 400. As 2100 is not divided by Hence 23rd September is 6 days to Sunday ie.
400 it is not a leap year. The next leap year is Saturday, so 23rd September is not a holidays.
2104
Solution for 73 to 75:
65. Total numbers of odd days from 5th January 2001
From the given information we can conclude Richa
to 25th Dec 2001 are
and Stuti are not Abhishek’s wives Further Wanda
Months: Jan + Feb + Mar + Apr + May + June + and Varsha are wives of Eklavya and Bharat
July + Aug + Sep + Oct + Nov + Dec respectively. In this manner we analyses the
Odd days: 26 + 0 + 3 + 2 + 3 + 2 + 3 + 3 + 2 information.
+ 3 + 2 + 25 =74
Wife
74
10 Weeks + 4 odd days Richa Stuti Trupti Urmila Varsha Wanda
7 Husband
4 days from Friday is Tuesday. Abhishek × ×
66. 21st March, 2000 is Monday and the year is a leap Bharat
year. So, none of the next 3 year is the leap year. Chetan × ×
So the day of the week will be 3 days beyond Dinesh × × ×
Monday i.e. Thursday Eklavya
67. A century has 5 odd days Franklin
5 day beyond Wednesday, Thursday, Friday, Now as Varsha and Wanda are married to Bharat and
Saturday, Sunday, Monday. Eklavya they cannot be wives of anyone else Further,
Hence 14th February 2101 will be on Monday as every husband has one wife.
68. To find the number of odd days, we have to find Dinesh is married to Urmila, Chetan, is married to
541 stuti, Abhishek is married to Trupti and Franklin is
remainder of the reminder is one. Hence married to Richa.
10
there is one odd day. Wife
Richa Stuti Trupti Urmila Varsha Wanda
69. 1994 is not a leap year. Husband
It has only 1 odd day. Abhishek × × × × ×
8th Feb 1995 is one day before Wednesday. Bharat × × × × ×
Hence 8th Feb 1994 is a Tuesday Chetan × × × × ×
th
70. It is given that 11 August 1985 was Sunday Dinesh × × × × ×
then 13th August 1985 is Tuesday. 1985 is not a Eklavya × × × × ×
leap year hence it has only one odd day. So 13th Franklin × × × × ×
August 1986 is one day after Tuesday ie. Now all the question can be answered
Wednesday
Logical Reasoning Ability 1.49
5. Total fruits in the basket = 11 position, does not create any restriction on the
6. Total number of four letter password formed vowel is position 1.
= 11C4 × 4 = 11 × 10 × 9 × 8 = 7920 e n
7. Required number of passwords 3rd 2nd 1st
st
Hence, 1 position can be occupied by any of the
= 26C3 × 3 – 15C3 × 3
5 vowels.
= 26 × 25 × 24 – 15 × 14 × 13 = 12870
Hence, 5 different ways ...........(ii)
8.
When 2nd position is filled by p,
1
CHAPTER
Spatial Aptitude
TRANSFORMATIONS
In a plane, you can slide, flip, turn, enlarge, or reduce
figures to create new figures. These corresponding
figures are frequently designed into wallpaper borders,
mosaics, and artwork. Each figure that you see will
corresopnd to another figure. These corresponding
figures are formed using transformations.
A transformation maps an initial image, called a
preimage, onto a final image, called an image.
The geometric transformation is a bijection of a set
that has a geometric structure by itself or another
set. If a shape is transformed, its appearance is This type of transformation has an object about a fixed
changed. After that, the shape could be congruent or point without changing its size or shape.
similar to its preimage. The actual meaning of
transformations is a change of appearance of
something.
Types of Transformations
When the size of a shape is increased or reduced then
the image of the shape will be similar to the pre-image.
The similar figures have dimensions equal in
proportion.
In the above figure, you can see, that the shape is
But in the case of congruent, the transformation of rotated to form its image.
objects is done by using rotation, reflection or
To describe you need:
translation. The shape is turned or flipped to
transform into another shape. direction (clockwise or counterclockwise)
There are four major types of transformations namely: degree
Rotation center point of rotation (this is where
Translation compass point goes)
Dilation Translation
Reflection This type of translation is defined as moving the object
Rotation in space by keeping its size, shape or orientation
constant. In a translation, each point of the shape
The rotation transformation leaves the transformed
must be moved in the same direction and for the same
object the same size, but turned relative to the original
distance. When you are doing a translation, the
position.
primary object is called the pre-image, and the object
The rotation is controlled by a base point (which does after the translation is called the image.
not move) and an angle. As illustrated in the figure, if
the base point is away from the object being
transformed, the effect of the rotation may well include
“movement” of the original shape.
In three dimensional rotations, the angle is often (but
not always) measured in one of the primary coordinate
system planes.
Remember, when performing multiple rotations, that
Rotation “A” followed by Rotation “B” does not produce
the same result as Rotation “B” followed by Rotation
“A”.
1.2 Spatial Aptitude
To describe you need: The following diagrams show the triangle ABC dilated
direction (left/right/up/down) with different scale factors. Scroll down the page for
magnitude (number of units) more examples and explanations of dilations.
Coordinate Notation:
(x, y) (x a, y b)
Dilation
A dilation is a transformation that produces an image
that is the same shape as the original, but is a different
size. (The image is similar to the original object).
Dilation is a transformation in which each point of an
object is moved along a straight line. The straight
line is drawn from a fixed point called the center of
dilation. The distance the points move depends on
the scale factor. The center of dilation is the only
invariant point.
This type of translation expands or contracts the object
by keeping its orientation or shape the same. This is
also known as resizing.
R Q
R’ Q’
Spatial Aptitude 1.3
NOTES:
Rigid transformations preserve lens measures,
perimeter, and area. The image and preimage are CO.
These transformations include:
Rotations
Reflections
Translations
Non-Rigid transformations preserve angle measure
In a mirroring or reflection operation, a shape is only. The side lengths and perimeter are not equal,
“flipped over” the mirror line (2D) or plane (3D). Each but are in proportion. The image and preimage are
point in the original is repositioned relative to the similar. This transformation is a:
mirror line/plane, ending up an equal distance away,
but on the opposite side. Dilation
The easiest mirror operation uses one of the primary GEOMETRICAL TRANSFORMATIONS
coordinate system axes as the plane and simply Conversion of One Shape to Another.
multiplies all x coordinates (or all y-coordinates, or Assembling
z-coordinates) by –1. Grouping
More general mirroring operations usually rely on Conversion from one Shape to Another
the user selecting or indicating a line (or plane) in
Each and every solid that exists occupies some volume.
space. Because a line is somewhat easier to establish
When you convert one solid shape to another, its
than a plane, a line is sometimes used, assuming that
volume remains the same, no matter how different
the mirror plane is perpendicular to the line, passing
the new shape is. In fact, if you melt one big cylindrical
through its midpoint.
candle to 5 small cylindrical candles, the sum of the
Topologically, mirroring turns objects inside-out, so volumes of the smaller candles is equal to the volume
programs maintaining consistently ordered surface of the bigger candle.
orientations must compensate for the inversion by
Hence, when you convert one solid shape to another,
reversing the ordering of the faces which have been
all you need to remember is that the volume of the
mirrored.
original, as well as the new solid, remains the same.
To describe you need: Let us discuss some examples to understand this
the equation of a line better.
Example 1: Conversion of Shapes Examples
Directions: In each of the following questions, there Example 1:
are four alternatives, which follow the word or number An iron ball of radius 21 cm is melted and recast
marked as a question. You have to select one alternative, into 27 spherical balls of the same radius. Find
which exactly matches with the mirror of the word/ the radius of each spherical ball.
number in the question.
Solution:
Which among the following illustrations specifies the
correct mirror image of P R E C A R I O U S? 4
Volume of the iron ball = r3
(a) (b) 3
(c) (d)
4 22
Ans.: (a) = × × 21 × 21 × 21
3 7
Example 2:
Directions: In each of the following questions, there = 38,808 cm3
are four alternatives, which follow the word or number Let us assume that the radius of the smaller
marked as a question. You have to select one alternative, balls is r.
which exactly matches with the mirror of the word/
number in the question. 4
So volume of 27 smaller balls = 27 × r3
Which among the following illustrations specifies the 3
correct mirror image of P E R F E C T I O N? = 36r3
(a) (b) Thus we have, the volume of the big iron ball
(c) (d) = volume of 27 smaller balls
Ans.: (c) 38,808 cm3 = 36r3
Spatial Aptitude 1.5
r3 = 343 cm3
1
So r = 7 cm r2 =
900
Therefore, the radius of each spherical ball is
7 cm. 1
r=
Example 2: 30
How many cylindrical candles, 1.75 cm in Hence, the thickness of the wire should be the
diameter and of thickness 2 mm, must be diameter of the cross-section of the new wire.
melted to form a cuboid candle of dimensions
5.5 cm × 10 cm × 3.5 cm? 1 1
Thickness = × 2 = cm.
Solution: 30 15
Thus, the thickness of wire is approximately
1.75
Radius of cylindrical candle = = 0.875 cm equal to 0.067 cm.
2
Assembling:
Volume of one cylindrical candle = r2h
Assembling is a process of finding the combined image
= × (0.875)2 × (0.02) cm3 with the help of visualization of it’s different-different
= × 0.0153125 shapes.
= 0.048125 cm3 Example 1:
Volume of cuboid candle = 5.5 × 10 × 3.5 (a) (b) (c) (d)
= 192.5 cm3
Thus, number of cylindrical candles
Example 5: Example 2:
(a) (b) (c) (d) Directions: There are two classes of three figures each.
Class ‘A’ figures differ in a certain way from the figures
in class ‘B’. Which of the four answer figures belong to
class ‘A’?
Problem Figure:
Answer. (d)
Grouping of Figures
In Grouping of Figures, different figures are put together
on the basis of a certain rule, pattern or property. Here
in this section on the grouping of figures, we will solve
many questions that are similar to the previous year
questions and concepts. In these questions, a given set
of figures will be featured. Some of these figures can be
grouped together while as others can’t be. The task is to
find out the rules and/or similarities between each
individual set.
In this type of questions, you are given a set of usually Answer Figures:
6, 7 or 9 etc. figures, which are numbered. The candidate
is required to analyses these figures and classify them
into groups consisting of figures having more or less the
same properties.
The candidate is required to analyses these figures and
classify them into groups consisting of figures having more
or less than the same properties. The best answer is to
be selected from a given set of fairly close alternatives.
(a) Both (1) and (3)
Example 1:
(b) (1) and (2) both
Directions: In the following question, group the given
figures into three classes using each figure only once. (c) Both (2) and (4)
Problem Figure: (d) (2) and (3) only
(e) None of the above
Answer:
The answer here is very simple. Each figure in class A
consists of two similar closed figures, which are placed
one inside the other. Therefore the answer is that (2)
and (3) both are correct and the correct option is (d). (2)
and (3) only.
Example 3:
In the following question, group the given figures into
(a) (7, 8, 9) (2, 4, 3) (1, 5, 6) three classes using each figure only once.
(b) (1, 3, 3) (4, 5, 7) (6, 8, 9) Problem Figure:
(c) (1, 6, 8) (3, 4, 7) (2, 5, 9)
(d) (1, 6, 9) (3, 4, 7) (2, 5, 8)
(e) none of the above
Answer:
The figures (1), (6) and (9) are all triangles while as the
figures (3), (4) and (7) are all quadrilaterals. Also, the
figures (2), (5) and (8) are all pentagons. So these groups
are identical and the correct answer is thus D.
Spatial Aptitude 1.7
(a) (1, 5, 6) (2, 3, 4) (7, 8, 9) Thus, these figures indicate the sequence in which the
(b) (1, 2, 4) (3, 5, 8) (6, 7, 9) paper is to be folded. The designs from the cut will appear
(c) (5, 6, 7) (1, 2, 4) (3, 8, 9) on each one of the folds made on the paper. In questions
based on paper folding and cutting it can be asked to
(d) (1, 2, 4) (3, 5, 7) (6, 8, 9)
find either the folded or unfolded pattern of the sheet.
(e) none of the above Let us see some examples below. We also have some
Answer: really tricky problems that have been put under the
In the question figure we see that the figures that we higher order thinking section.
label 1, 2 and 4 all consist of three lines. Also, the figures Example 1:
3, 5 and 8 all consist of four lines each. Similarly, we can A square transparent sheet with a pattern is given. Figure
see that the figures 6, 7 and 9 all consist of five lines. out from among the four alternatives as to how the
Therefore this gives us the grouping of the figures. Thus pattern would appear when the transparent sheet is
the correct option is (b). folded at the middle line shown?
Example 8:
In the following question, group the given figures into
three classes using each figure only once.
Problem Figure:
Answer:
If you look at the options, you would see that the option
(a) can’t be the answer because if the sheet was folded at
the middle the triangles will be joined at the vertex.
Similarly, you can discard all the other options except
(d). The correct option is (d).
Example 2:
A piece of paper is folded and punched as shown below
in the question figures. from the given answer figures,
indicate how it will appear when opened?
(a) (1, 7, 8) (2, 6, 5) (3, 4, 9) Problem Figures
(b) (1, 8, 9) (2, 3, 5) (4, 6, 7)
(c) (2, 3, 5) (1, 7, 8) (4, 6, 9)
(d) (2, 6, 7) (1, 3, 4) (5, 8, 9)
(e) none of the above
Answer Figures
Answer:
As is clear from the question figure, the figures 2, 3 and
5 have one point or in other words, we can say that they
are a one-pointed system. Also, we can see that the
figures 1, 8 and 9 are made up of four lines. Below we
have some practice problems on the Grouping of Figures.
Answer:
PAPER FOLDING AND CUTTING
In the last problem figure, we see that the paper is folded
The Paper Folding and Cutting involves a process in in the form of a right triangle and the cut is made near
which a transparent sheet is folded and then some cuts the midpoint of the hypotenuse inside the triangle.
and/or folds are made. In the questions based on Paper
Unfolding the triangle would give us another right
Folding and Cutting a few figures are given showing
triangle which has marks near the two sides apart from
the way in which a piece is to be folded and then cut the hypotenuse. At this point, we can say that the answer
from a particular section. The dotted line is the reference could be either (a) or (d). Similarly, if we keep unfolding,
line along which the paper is to be folded and the arrow we will see that the correct answer is (d).
indicates the direction of the fold.
Spatial Aptitude 1.9
Answer:
The right half of the transparent sheet is being folded
along the dotted line and placed on the left half of the Answer:
sheet. The figure thus obtained resembles the answer As we have done with the other questions, we will begin
figure (c). with the last problem figure and move on to the first
Example 4: one, while tracing out all the spots on each step. Whence
Directions: In the following question, a piece of paper we do that we will see that out of the four options, the
is folded and cut as shown below in the question figures. only one that represents the answer figure is the one in
From the given answer figures, indicate how it will appear option (a).
when opened? PATTERNS IN 2 AND 3 DIMENSIONS
Problem Figures 2D Shape’s Pattern:
In geometry, a shape or a figure that has a length and a
breadth is a 2D shape pattern. In other words, a plane
object that has only length and breadth is 2 dimensional.
Straight or curved lines make up the sides of this shape.
Also, these figures can have any number of sides. In
general, plane figures made of lines are known as
polygons. For example, triangle and square are
Answer Figures
polygons.
Examples of 2D shapes
Rectangle, circle, square, triangle, quadrilateral and
pentagon are some examples of 2D shapes.
Initial angle of the object O with respect to = Xold × sin + Yold × cos
origin =
= 4 × sin30° + 4 × cos30°
Rotation angle =
New coordinates of the object O after rotation 1 3
= (Xnew, Ynew) =4× +4×
2 =2+2 3
2
3 1
=4× – 4 × = 2 3 – 2
2 2
= 2( 3 – 1) = 2(1.73 – 1)
= 1.46
Ynew
Spatial Aptitude 1.11
Xnew = Zold × sin + Xold × cos Xnew = Xold × cos – Yold × sin
Ynew = Yold = 1 × cos90° – 2 × sin90°
= 1 × 0 – 2 × 1 = –2
Znew = Yold × cos – Xold × sin
For Z-Axis Rotation- Ynew = Xold × sin + Yold × cos
This rotation is achieved by using the following rotation = 1 × sin90° + 2 × cos90°
equations- =1×1+2×0=1
Xnew = Xold × cos – Yold × sin Znew = Zold = 3
Hence, New coordinates after rotation = (–2, 1, 3).
Ynew = Xold × sin + Yold × cos
Nets for Building 3-D Shape Pattern:
Znew = Zold
A geometric net is a two-dimensional ‘pattern’ for a three-
Example:
dimensional object that is unfolded along its edges. It
Given a homogeneous point (1, 2, 3). Apply rotation represents each face of the figure in two dimensions. In
90 degree towards X, Y and Z axis and find out the other words, a net is a pattern made when the surface
new coordinate points. of a three-dimensional figure is laid out. Thus, showing
Solution: each face of the figure. A solid may have different nets.
Given- For example, a box is solid. It’s a 3D object with the
Old coordinates = (Xold, Yold, Zold) = (1, 2, 3) shape of a cuboid. Below is a net pattern for a box. Copy
an enlarged version of the net and try to make the box
Rotation angle = = 90° by folding and gluing the faces together. You may use
For X-Axis Rotation- suitable units.
Let the new coordinates after rotation = (Xnew, Ynew,
Znew).
1.12 Spatial Aptitude
By Euler’s Formula:
For Polyhedron
F+V=E+2
Where,
F = Number of faces of polyhedron
V = Number of vertices of polyhedron
E = Number of Edges of polyhedron
A net is a 2- dimensional representation of a 3-
dimensional figure that is unfolded along its edges so
that each face is in 2- dimension. So, a given polyhedron
can have different nets.
Polyhedrons are straight-sided solids, which has the
Furthermore, different shapes have different nets. following properties:
It should have flat sides are called the faces
It must have the corners, called vertices
Polyhedrons should have straight edges.
Like polygons in two-dimensional shapes, polyhedrons
are also classified into regular and irregular
polyhedrons and convex and concave polyhedrons. The
most common examples of polyhedral are cube, cuboid,
pyramid, prism. Some other examples of regular
polyhedr on are tetrahedron, octahedr on,
dodecahedron, icosahedron, and so on. These regular
polyhedrons are also known as the platonic solids,
whose faces are identical to each face.
For example, the most commonly used example of a
Example of 3D Shape Pattern:
polyhedron is a cube, which has 6 faces, 8 vertices,
Polyhedron and 12 edges.
A solid shape bound by polygons forms a polyhedron. In a polyhedron, three or more edges meet at a point to
The word polyhedral is the plural of word polyhedron. form a vertex. Some examples of polyhedrons are cuboid,
cube, pyramid, and triangular pyramid.
Curved Solids
Apart from polyhedrons, there are 3D shapes with
curved shapes such as the sphere, cone, cylinder, etc.
For example, cones have a circular base that narrows
smoothly from the circular base to the point called
the apex. All these shapes have curved faces, and hence
they are called curved solids.
Example:
Which shape in Group 2 corresponds to a shape in
Group 1?
This is very much a speed test, as you will be given answer ‘jumps out’ at you. The problem with this is
far more of these questions than you can comfortably that if the answer doesn’t ‘jump out’ fairly quickly
answer in the given time. then panic sets in and you usually resort to the
These questions use a large number of shapes that systematic approach anyway.
are presented close together. Some people find this When you are confronted with a question like this
very distracting and find it easier to work through that asks you to compare several different shapes,
the shapes in the second group systematically, rather the other answer options can be distracting. It is much
than trying to look at the whole group at once. easier and less mentally tiring to cover the other
In some questions of this type, there may not be a shapes and concentrate only on the two shapes that
one-to-one match and some of the shapes in the first you are comparing.
group may not appear in the second. The way that 3. Group Rotation – Two Dimensional pattern:
the question is worded will make this clear. This is slightly more complex than the rotations in
You should be especially careful to look out for the two-dimensional shape matching question.
reflections in the second group as these will be Example:
incorrect. These are often put in by the test designers Which of the answer figures is a rotation of the
to trap the unsuspecting. question figure?
For example:
Example: Answer: D
Which of the complete shapes can be made from the First, check the answer options to see if any elements
components shown? have changed position. If so, you can eliminate these
answers. This will eliminate answer A straight away
as the square and arrow have swapped places.
Then look for any elements which have been
manipulated differently. Answer E appears to be a
mirror image, but the triangles have been incorrectly
inverted.
Answer C is an example where the square and arrow
have changed position, but the triangles are also
incorrectly inverted. Therefore, this can not be a
reflection and can be eliminated.
Answer B is exactly the same as the question image,
it hasn’t been reflected at all. This answer can also be
eliminated.
6. Two-Dimensional Maps
Spatial Ability Map questions will assess your ability to follow basic
Answer: B instructions and visualise a route. Normally, you will
The best strategy for answering these questions is to be given a two-dimensional plan or map and a series
look at the complete shapes and see if there are any of directions via two or three questions that all relate
distinct features that would make it impossible to to the same map.
construct such a shape from the components. You just need to use the instructions to navigate the
Sometimes there are one or two shapes that can be map. As long as you have a basic sense of direction,
immediately discounted based on size alone. this will seem relatively straightforward, but you will
One thing to remember is that if the complete shapes need to do so quickly and accurately.
don’t have any bits sticking out (they usually don’t) Example:
then the components must fit together so that sides
of the same length are together. This reduces the
number of combinations considerably.
5. Two-Dimensional Mirror Reflections:
These questions will assess how you view two-
dimensional objects from a different perspective. You
will be presented with an image and will be asked to
identify its mirror image.
Example:
Which answer shows a reflection of the image below?
Answer: A
The key to these questions is to remember that only
three faces of the cube can be shown in the illustration,
this means that you n eed only consider the
relationship between the three visible elements on
each cube and see if the same relationship exists in Answers:
the pattern.
The best strategy for this type of question is to call
one face of the cube the ‘front’ and then name the
other faces of the cube in relation to it.
Spatial Aptitude 1.17
Answer: D Answer: D
Some people seem to have a natural talent for Shape A can be eliminated because it shows an
imagining objects in three dimensions and find these unshaded face below a triangular face, both of these
questions straightforward. However, if you’re not one faces (below the triangular face) are shaded on the
of them and you find thinking in three dimensions pattern. Shape C can be eliminated for the same
difficult, there are other ways to get to the answer. reason.
In the question above, for example, you can simply Shape B can be eliminated as it has a shaded roof
use a process of elimination. If you can see a symbol above the shaded side, which does not appear on the
next to the ‘X’ on the cube, then it cannot be opposite. pattern.
The second and third cubes eliminate A, B and C. This Once again, you can reduce these problems to the
leaves only D and ‘other’ as possibilities. D has edges relationship between the visible elements on the
shared with A and B, which would be consistent with three-dimensional picture. This makes things easier
the third cube illustrated. Therefore, D is correct. because even though the solid shape may have more
Although it is not usually specified in the instructions, faces than a cube, it is unusual for more than four
it is almost always true that in these questions each faces to be shown. This means that you need to consider
symbol is used only once. This means that even in the relationship between the four visible faces, paying
cases where elimination is not possible, it is sometimes particular attention to shading or other patterns on
quite easy to see the solution without mentally them.
manipulating the cube too much. 10. Block Counting in Three Dimensions Pattern:
In the example above, you can simply compare the In these questions, you will be given a three-
first and third illustrations. The third illustration dimensional drawing of a shape formed from a group
shows a 90-degree anticlockwise rotation (looking at of blocks, some of which may be hidden.
the cube from above) of the first illustration. Therefore, You will be asked to identify how many blocks have
D must be opposite the ‘X’. been used to make up the shape. You will need to
9. Other Solids in Two- and Three-Dimension’s visualize the blocks you cannot see.
Pattern: Example:
These questions are similar to the cube questions How many blocks make up the shape below?
above but rather than cubes, they use other solid
shapes which may be irregular.
In some respects, these questions are easier than the
cube questions as there are more relationships to work
with. In other words, each face of the solid shape has
a shape of its own rather than just being square.
Example:
Which of the solid shapes shown could be made from
the pattern?
Answer: E
Try to break up the blocks in your mind. For example,
in the exercise above, there is a block of three 5-cube
pillars (which together makes 15 blocks), a group of
three 3-cube pillars (which together makes 9 blocks),
and two single blocks.
Adding together 15 + 9 + 2 gives 26
This is quicker and produces easier sums, leading to
more robust answers.
1.18 Spatial Aptitude
EXERCISE
dots are placed in the same corners of the objects
MCQ TYPE QUESTIONS
as in the model.
ROTATION
1. Below are two separate objects, each has a dot
placed in one corner, and each has five possible
rotations. Choose the answer option in which the
dots are placed in the same corners of the objects
as in the model.
(a) (b)
(a) (b)
(c) (d)
4. In the figures shown below, one of the shapes
(a-d) is identical to the given figure but has been
rotated.
Which figure is identical to the given figure?
(c) (d)
2. Below are two separate objects, each has a dot
placed in one corner, and each has five possible
rotations. Choose the answer option in which the
dots are placed in the same corners of the objects
as in the model.
(a) (b)
(c) (d)
3. Below are two separate objects, each has a dot
(a) (b) (c) (d)
placed in one corner, and each has five possible
rotations. Choose the answer option in which the
Spatial Aptitude 1.19
6. In the figures shown below, one of the shapes 9. Which figure is a rotation of the object?
(a-d) is identical to the given figure but has been
rotated.
Which figure is identical to the given figure?
(a) (b)
(a) (b)
(a) (b) (a) S'(–1, 5), Q'(2, 4), R'(3, 7), E'(0, 8)
(b) S'(–2, 5), Q'(2, 4), R'(3, 7), E'(1, 8)
(c) S'(–1, 5), Q'(2, 4), R'(1, 7), E'(2, 8)
(d) None of these.
16. Find the translation rule for TRI to T'R'I'. Look
(c) (d)
at the movement from T to T'. The translation
rule is
TRANSLATION
I
13. Triangle ABC has coordinates A(3, –1), B(7, –5) 8
and C(–2, –2). Translate ABC to the left 4 units 6 R
4 I
and up 5 units. Determine the coordinates of
T 2 R
A'B'C'.
–2 2 4 6
T
I 8
6 R
4 I
T 2 R
–4
–2 2 4 6
+6 T
17. Find the translation rule for: 20. Find the translation rule for:
10 A B
6 A D B C
4
D C
2
x
(a) (x, y) (x – 3, y – 5). 0 2 4 6 8 10
(b) (x, y) (x + 7, y + 9).
(c) (x, y) (x + 9, y – 7). (a) x 2, y 2 b) x 2, y 3
(d) None of these. (c) x 3, y 2 d) x 3, y 3
1.22 Spatial Aptitude
C
–5
Fig. (a) Fig. (b)
(a) x 2, y 3 (b) x 2, y 2 (a) 320, 136.25 (b) 420, 236.25
(c) x 3, y 2 (d) None of these (c) 520, 336.25 (d) None of these
24. As given, the area of shaded portion of image (a) 28. As given, for image (a); small side length l = 8,
is 24m2 and scale factor is 3, then find the area of large side length l = 10, and scale factor is 0.75 ,
image (b). then find the area of image (b).
31. Choose the alternative which is closely resembles 35. Choose the alternative which is closely resembles
the Mirror image of the given figure. the Mirror image of the given figure.
51.
(c) (d)
43. R E S T 52. ×
o
(a) T E (b) T
(c) (d) T S E R
44. 1 5 8 2 4
× × × ×
(a) (b) 8 1 o o
8 o
(c) 8 (d) 4 2 8 5 1 o
47.
25
(a) (b) (c) (d)
Spatial Aptitude 1.25
(a) 1, 5, 8; 2, 4, 7; 3, 6, 9
54. (b) 1, 4, 7; 2, 5, 8; 3, 6, 9
(c) 1, 5, 7; 2, 4, 9; 3, 6, 8
(d) 1, 5, 7; 3, 4, 9; 2, 6, 8
59. Group the following figures into three classes
with identical perpoerties and select the correct
alternative.
1 2 3
AAA
4 5 6
55. ×××
S W
7 8 9
E K
(a) 1, 2, 3; 6, 5, 4; 7, 9, 8
(b) 1, 6, 7; 2, 5, 9; 3, 4, 8
(c) 1, 6, 7; 2, 5, 8; 3, 4, 9
(d) 1, 3, 7; 2, 5, 9; 4, 6, 8
60. Choose the correct sequence which are the group
of identical image.
56.
Options
(a) (b) (c) (d)
(a) 2, 3; 4, 6; 5, 1 (b) 1, 5; 2, 3; 4, 6
(c) 1, 4; 3, 2; 5, 6 (d) 1, 3; 4, 2; 5, 6
57. 61. Choose the correct sequence which are the group
of identical images.
GROUPING
58. Group the following figures into three classes
with identical properties and select the correct
alternative Options
(a) 1, 5, 8; 2, 6, 7; 3, 4, 9
1 2 3 (b) 1, 2, 3; 4, 5, 6; 7, 8, 9
(c) 4, 5, 6; 7, 8, 9; 1, 2, 3
4 5 6
(d) 1, 4, 7; 2, 5, 8; 3, 6, 9
7 8 9
1.26 Spatial Aptitude
62. Choose the correct sequence which are the group 65. Choose the correct sequence which are the group
of identical images. of identical images.
Options Options
(a) 1, 2; 3, 4; 5, 6 (b) 1, 4, 6, 3; 2, 5 (a) 3, 2, 4; 1, 5, 6; 7, 8, 9
(c) 1, 3; 4, 5; 2, 6 (d) 1, 5; 2, 3; 4, 6 (b) 1, 2, 3; 4, 5, 6; 7, 8, 9
63. Choose the correct sequence which are the group (c) 7, 1, 2; 3, 4, 5; 6, 8, 9
of identical images.
(d) 3, 8, 9; 1, 2, 7; 4, 5, 6
66. Group the given figures into three classes using
each figure only once.
1 2 3
4 5 6
7 8 9
(a) 1, 5, 7; 2, 4, 6; 3, 9, 8
(b) 1, 5, 7; 2, 4, 8; 3, 6, 9
(c) 1, 4, 7; 2, 5, 8; 3, 6, 9
(d) 1, 7, 9; 3, 5, 8; 2, 4, 6
67. Group the given figures into three classes using
Options each figure only once.
(a) 1, 3, 5; 2, 4, 6; 7, 8, 9
(b) 1, 4, 8; 2, 5, 9; 3, 6, 7
1 2 3
(c) 1, 2, 3; 4, 6, 7; 8, 9, 5
(d) 1, 6, 7; 2, 3, 4; 5, 8, 9 4 5 6
69. Identify the figure that completes the pattern. 73. Choose the image that completes the pattern:
A B
(a) (b)
(a) A (b) B
(c) Both A & B (d) None of these.
71. Choose the image that completes the pattern:
(c) (d)
(a) (b)
(c) (d)
(a) (b)
72. Choose the image that completes the pattern:
(a) (b) 75. Choose the image that completes the pattern:
(c) (d)
Which of the shapes below replaces the missing
box above?
1.28 Spatial Aptitude
(a) (b)
(c) (d)
(a) (b)
(c) (d)
(a) (b)
(c) (d)
(a) (b)
(a) (b)
(c) (d)
(c) (d)
Spatial Aptitude 1.29
++ + + +
++ ++ +
+ + +
(a) (b) (c) (d)
Answer Figures 85. Question Figures
Answer Figures
(a) (b) (c) (d)
Answer Figures
Answer Figures
Answer Figures
Answer Figures
+ + + + + +
+
(a) (b) (c) (d) (a) (b) (c) (d)
90. Question Figures 95. Question Figures
Answer Figures
Answer Figures
P P P P
Answer Figures
K K K K K K
K K K K
Answer Figures
B B B B
B B B B B B
(c) (d)
102. Question Figures 106. Which cube can be formed by folding the given
shape?
M
V L
Answer Figures
M M M M M
VL V VL V VL V VL V
M M
Answer Figures
(c) (d)
P P P P
R R R R
R
ASSEMBLING
Directions 107-138: During the following spatial
reasoning question your task is to look at the given
shapes and decide which of the examples match the (a) (b)
shape when joined together by the correspoding letters.
107.
(c) (d)
(a) (b)
110.
(a) (b)
(c) (d)
(c) (d)
108.
111.
(a) (b)
(a) (b)
(c) (d)
109.
(c) (d)
Spatial Aptitude 1.33
114.
112.
(a) (b)
(a) (b)
(c) (d)
115.
116.
(a) (b)
(a) (b)
(c) (d)
(c) (d)
1.34 Spatial Aptitude
117. 120.
(a) (b)
(a) (b)
(c) (d)
(c) (d)
121.
118.
(a) (b)
(a) (b)
(c) (d)
122.
(c) (d)
119.
(a) (b)
(a) (b)
(c) (d)
(c) (d)
Spatial Aptitude 1.35
123. 126.
(a) (b)
(a) (b)
(c) (d)
(c) (d)
124.
127.
(a) (b)
(c) (d)
(a) (b)
(c) (d)
(c) (d)
1.36 Spatial Aptitude
128.
(c) (d)
131.
(a) (b)
(c) (d)
(a) (b)
129.
(c) (d)
(a) (b)
132.
(c) (d)
130.
(a) (b)
(c) (d)
(a) (b)
Spatial Aptitude 1.37
133.
(a) (b)
134.
(c) (d)
(c) (d)
135.
138.
(a) (b)
(a) (b)
(c) (d)
(c) (d)
136.
1.38 Spatial Aptitude
139.
(c) (d)
142.
(a) (b)
(a) (b)
(c) (d)
140.
(c) (d)
143.
(a)
(c)
(c) (d)
144.
(d)
141.
(a) (b)
(a) (b)
(c) (d)
Spatial Aptitude 1.39
145. 146.
(a) (b)
(a) (b)
(c) (d)
(c) (d)
ANSWERS
MCQ Type Questions
1. (d) 2. (d) 3. (c) 4. (c) 5. (b) 6. (a) 7. (d) 8. (b) 9. (b) 10. (d)
11. (c) 12. (a) 13. (a) 14. (a) 15. (a) 16. (c) 17. (b) 18. (c) 19. (a) 20. (c)
21. (a) 22. (a) 23. (b) 24. (a) 2. (a) 26. (c) 27. (b) 28. (c) 29. (d) 30. (b)
31. (b&c) 32. (c&d) 33. (b) 34. (c) 35. (b&d) 36. (c&d) 37. (c) 38. (b) 39. (c&d) 40. (c&d)
41. (c) 42. (a) 43. (b) 44. (c) 45. (d) 46. (a) 47. (b) 48. (c) 49. (a) 50. (d)
51. (c) 52. (c) 53. (a) 54. (c) 55. (a) 56. (a) 57. (c) 58. (c) 59. (b) 60. (a&b)
61. (a) 62. (b) 63. (b) 64. (d) 65. (b) 66. (b) 67. (a) 68. (d) 69. (c) 70. (b)
71. (d) 72. (b) 73. (d) 74. (c) 75. (b) 76. (c) 77. (d) 78. (a) 79. (b) 80. (a)
81. (b) 82. (c) 83. (b) 84. (a) 85. (d) 86. (c) 87. (d) 88. (c) 89. (a) 90. (b)
91. (d) 92. (a) 93. (b) 94. (c) 95. (a) 96. (c) 97. (d) 98. (a) 99. (a) 100. (b)
101. (c) 102. (a) 103. (a) 104. (b) 105. (c) 106. (d) 107. (b) 108. (d) 109. (a) 110. (d)
111. (d) 102. (b) 113. (a) 114. (c) 115. (a) 116. (c) 117. (d) 118. (b) 119. (d) 120. (a)
121. (c) 122. (d) 123. (d) 124. (a) 125. (c) 126. (b) 127. (d) 128. (d) 129. (c) 130. (c)
131. (d) 132. (b) 133. (d) 134. (d) 135. (d) 136. (a) 137. (d) 138. (c) 139. (d) 140. (b)
141. (c) 142. (b) 143. (a) 144. (d) 145. (d) 146. (a)
1.40 Spatial Aptitude
EXPLANATIONS
MCQ TYPE QUESTIONS 47. Mirror image of 2 is and of 5 is
Hence, answer is .
43.
58. Here, figures 1, 5, 7 contain similar porperties.
Fig.2, 4 and 9 are similar as they are made of
four straight lines. Similarly, Figures 3, 6, and 9
contain a circle as a base.
Hence, option (b) is correct.
44. Hence, (c) in the answer.
59. Here we observe that the figure 1, 6, 7 have only
elements, the figures 2, 5 and 9 have two
elements while figures 3, 4 and 8 consist of three
elements.
Hence, option (c) is correct. Hence, (b) is the correct alternative.
45.
70. When you look at the shape from above or below,
you will see a shadow identical to image B.
When you look at the shape from the side, you
will see a shadow in the form of a dark square
with lit triangles in it (B.N. the lit triangle is not
Hence, option (d) is correct. identical to the one shown in shape itself!).
46. This case is like water-image. Hence, option (a)
is correct.
Quantitative Aptitude
Numbers, Algebra and
1
CHAPTER
Data Interpretation
n 6 n
LCM of Fractions = 3 2
HCF of Denominators 4
4 8 2 6
e.g. Find HCF and LCM of , , , ALGEBRAIC FORMULAE AND THEIR
5 35 15 25
APPLICATION
HCF of 4, 8, 2, 6
HCF = An equation is a statement that two algebraic
LCM of 5, 35, 15, 25 expressions are equal. If an equation is satisfied by
2 2 any value of the variable, then equation is said to be
= = an identity.
5 7 3 5 525
1. (a + b)2 = a2 + 2ab + b2 = (a – b)2 + 4ab
LCM of 2, 8, 4, 6 24 2. (a – b)2 = a2 – 2ab + b2 = (a + b)2 – 4ab
LCM = =
HCF of 5, 35, 15, 25 5 3. (a + b)2 + (a – b)2 = 2(a2 + b2)
PROGRESSIONS 4. (a + b)2 – (a – b)2 = 4ab
Arithmetic Progression (A.P.) 5. (a + b + c)2 = a2 + b2 + c2 + 2(ab + bc + ca)
Let ‘a’ be the first term of an arithmetic progression; 6. (a + b + c + d)2 = a2 + b2 + c2 + d2 + 2a(b + c + d) + 2b
‘d’ the common difference and ‘n’ the number of terms (c + d) + 2cd
in the progression. 7. (a + b)(a – b) = a2 – b2
Tn = nth term = a + (n – 1)d 8. (x + a)(x + b) = x2 + (a + b)x + ab
n 9. (x + a)(x + b)(x + c) = x3+(a + b + c)x2
Sn = Sum of n terms = [2a + (n – 1)d]
2 + (ab + bc + ca) x + abc
Progression can be represented as a, a + d, a + 2d, 10. (a + b)3 = a3 + 3ab(a + b) + b3
........, [a + (n – 1) d]. Here, quantity d is to be added to 11. (a – b)3 = a3 – 3ab(a – b) – b3
any chosen term to get next term of the progression. 12. a3 + b3 = (a + b)3 – 3ab(a + b) = (a + b)(a2 – ab + b2)
When there are n terms in an arithmetic progression, 13. a3 – b3 = (a – b)3 + 3ab(a – b) = (a – b)(a2 + ab + b2)
a is first term and {a + (n – 1) d} is last term.
14. a3 + b3 + c3 – 3abc = (a + b + c)(a2 + b2 + c2 – ab – ac – bc)
n If a + b + c = 0 then a3 + b3 + c3 = 3abc
Then Sn = [First Term + Last Term]
2 15. an – bn = (a – b)(an–1 + an–2 b + an–3 b2 + ... + bn–1) for all n.
Geometric Progression (G.P.) 16. an + bn = (a + b)(an–1 – an–2 b + an–3 b2 – ... – bn–1)
Geometric progression can be represented as a, ar, if n is even
ar2, .... where a is first term and r is common ratio of 17. an + bn = (a + b)(an–1 – an–2 b + an–3 b2 + ... + bn–1)
the geometric progression.
if n is odd.
nth term of the geometric progression = ar n–1.
18. a4 + a2b2 + b4 = (a2 + ab + b2)(a2 – ab + b2)
Sum to n terms =
a 1 rn or
a rn 1 POLYNOMIALS
1r r 1 Factors of polynomials: An expression is said to be
resolved into factors when expressions of its product
r last term First term are found.
=
r 1 HCF of polynomials: When two or more polynomials
are factorized, then product of all the common factors
is HCF of the polynomials.
1.4 Numbers, Algebra and Data Interpretation
LCM of polynomials: When two or more polynomials 9. If a, b, c are positive and not all equal, then
are factorized, then product of all the factors with (a+b+c) (ab+ bc + ca) > 9abc
highest power is LCM of the polynomials. and, (b + c) (c + a) (a + b) > 8abc.
INEQUATIONS ax a
Properties 10. If x is positive and a < b, then
bx b
1. An inequality will still hold after each side has been
ax a
increased, diminished, multiplied or divided by the If x is positive and a > b, then
same positive quantity. bx b
i.e., if a > b and c > 0 ax a
11. If x is positive and a > b > x, then
a+c>b+c bx b
ac > bc ax a
If x is positive and x < a < b, then
a b bx b
c c a c e ...
12. is less than greatest and greater than
2. In an inequality, any term may be transposed from b d f ...
one side to the other if its sign is changed. a c e
i.e., if a – c > b, then a > b + c or c < a – b least of the fractions , , , ...
b d f
3. If sides of an inequality be multiplied by same 13. For positive numbers, if a > x, b > y, c > z, then a
negative quantity, then sign of inequality must be + b + c + ... > x + y + z + ... and abc ... > xyz ...
reversed.
14. a2 + b2 + c2 bc + ca + ab
i.e., if a > b and c < 0, then ac < bc
15. (n!)2 > nn, for n > 2.
1 1
4. If a > b, a, b 0 then an > bn and n n , or a–n < b–n ; 1
n
a b 16. For any positive integer n, 2 1 3
if n is a positive quantity. n
5. Square of every real quantity is positive and 17. a2b + b2c + c2a 3abc
therefore must be greater than zero a b c d
i.e., for a b, (a – b)2 > 0; a2 + b2 > 2ab; 18. 4
b c d a
Similarly, if x > 0, y > 0, then 19. a4 + b4 + c4 + d4 4abcd
xy
xy QUADRATIC EQUATION WITH APPLICATION
2 TO INEQUALITIES
Hence, Arithmetic mean of two positive quantities In a quadratic equation ax2 + bx + c = 0, if b2 – 4ac > 0,
is greater than or equal to their Geometric mean. then
6. If sum of two positive quantities is given, their ax2 + bx + c = a(x – )(x – ), where and are two
product is greatest when they are equal; and if roots of the equation with < .
product of two positive quantities is given, their
sum is least when they are equal. (x – )(x – ) is always positive when x < or x > i.e.,
x does not lie between and .
7. If a, b, c.... k are n unequal quantities, then
n (x – ) (x – ) is always negative when < x or x >
a b c ... k i.e., x lies between and .
> a b c d ... k
n DATA INTERPRETATION
a b c ... k Data can be organized in a number of ways so that
i.e., > (a b c ... k)1/n
n larger volume of data can be presented in a more
Note: Arithmetic mean of any number of positive compact and precise form. Data thus presented has
quantities is greater than their Geometric mean. to be deciphered correctly by the user of the data.
8. If a and b are positive and unequal, then The deciphering of the data from its compactly
a m bm a b
m presented form is called Data INTERPRETATION.
2
except when m is a Representation of Data
2
positiveproper fraction. Numerical data can be presented in one or more of
If m is a positive integer or any negative quantity, the following ways
m
a m bm a b (1) Data Table
2
then
2 m (2) Pie Chart
a m bm a b
If m is positive and less than 1, then (3) Line Graph
2 2
If there are n positive quantities a, b, c ... k, then (4) Bar Chart
a m bm c m ... k m a b c ... k m (5) Others
n n
unless m is a positive proper fraction.
Numbers, Algebra and Data Interpretation 1.5
The “Others” category covers miscellaneous form like (1) Total number of students appeared and qualified
descriptive case format etc. customized for the from the given districts in each of the years.
situation. Data can also be presented by using a (2) Percentage increase in the number of students
combination of two or more of the above forms. appeared or qualified in the district over the years.
While some data can be presented in many different (3) Average number of students appeared or qualified
forms, some other may be amenable to be presented
Pie-Charts
only in few ways. In real life situation, the style of
data presentation is based on the end-objective. In This is probably the simplest of all pictorial forms of
certain situation data has to be presented as a data presentation. Here, total quantity to be shown is
combination of two or more forms of data presentation. distributed over one complete circle or 360 degree. In
Let us understand each of the above form of data pie-charts, data is essentially presented shares of
presentation with an example. various elements as proportion and percentage of the
total quantity. Each element or group in a pie-chart
Data Table
is represented in the terms of quantity (or value, as
Here data is presented in the form of table. Where the case may be) or as the angle made by the sector
any type of data can be presented in tabular form, representing the elements or as a proportion of the
that too is a very accurate manner, interpreting the total or as a percentage of the total.
data in table form becomes more difficult and time
Chart 1 gives distribution of sales of different
consuming than the other modes, all of which are
companies.
basically pictorial or graphical in presentation.
CHART – 1
A B Total sales Rs. 6,000 crores
Year Year
Appeared Qualified Appeared Qualified
2002 250 720 2002 1750 460
Others Bharat
2003 2750 810 2003 1860 490
10% Petroleum
2004 3000 890 2004 2000 520
18%
2005 3250 910 2005 2100 640
Hindustan
2006 3720 1050 2006 2400 830
Petroleum
18%
C D
Year Year
Appeared Qualified Appeared Qualified
2002 100 120 2002 800 120
IOC
2003 1120 200 2003 1000 220
Castrol India 37%
2004 1250 300 2004 1200 300
17%
2005 1500 600 2005 1210 340
2006 1650 780 2006 1440 480
From the above pie chart, we can calculate the
E following;
Year
Appeared Qualified (1) Total sales of each of the companies.
2002 2000 370 (2) Sales of a company as a percentage of the other.
2003 2200 420 (3) Conversion of these percentage values into angles
2004 2500 510 for each zone.
2005 2750 680 Pie-chart are also very frequently used in combination
2006 3440 960 with other forms of data or along with other pie-charts.
Data tables can be of a number of types. They can be Two-Dimensional Graphs
of a single –table variety or combination of tables. This is essentially used for continuous data but can
One such example of table is given above. also be used for depicting discrete data provided we
The above table shows the number of students understand the limitation. Also known as Cartesian
appeared and qualified in an entrance test from five Graphs, they represent variation of the one parameter
districts A, B, C, D and E of a state. with respect to another parameter each shown on a
From the above table, we can obtain the following different axis. These types of graphs are useful in
data; studying the rate of change or understanding the
trends through extrapolations.
1.6 Numbers, Algebra and Data Interpretation
These graphs can be of various types and a few of Chart 4 shows import and export of a company over
them are shown below: different years. From this graph we can obtain the
CHART – 2 following:
(1) Percentage contribution of imports or exports to
70
the company’s total trade for different years.
Profit (in Rs. Lakhs)
60
(2) Relative increase or decrease in the share of
50
imports or exports
40
(3) Percentage growth/average annual growth in
30
imports or exports during given period.
20
10
Three-Dimensional Graph
0 The data in a triangular graph are given on each side
2000 2001 2002 2003 2004 2005 2006 of the triangle. Each point represents three different
values, one each in each direction.
The graphs in Chart 2 show the changes in the profit CHART 5
of the company during a period time. One can find out
trends and the growth in the profit over the years. Students Percentage
A 82 66 59 76 62 65
B 76 72 65 84 74 75
C 56 78 71 66 86 70
D 64 80 68 72 66 80
E 48 68 83 88 56 60
F 60 74 79 64 80 85
1000
800
600
400
200
0
2001 2002 2003 2004 2005 2006 2007
Import Export
Numbers, Algebra and Data Interpretation 1.7
EXERCISE
MCQ TYPE QUESTIONS 9. Which of the following is true?
(a) Sum of four consecutive even numbers is always
1. If x = b + c, y = c – a, z = a – b, then
divisible by 8.
x2 + y2 + z2 – 2xy – 2xz + 2yz is equal to
(b) Sum of four consecutive odd numbers is always
(a) a + b + c (b) 4b2 divisible by 8.
(c) abc (d) a2 + b2 (c) Product of any n consecutive natural numbers
2. HCF of y – 4y and 4y (y3 + 8) is
3
may not be divisible by n!.
(a) y(y + 2) (b) y – 2 (d) Product of 4 consecutive odd numbers is
(c) 2(y + 2) (d) (y + 2)(y2 – 2y + 4) always divisible by 15.
3. A tiled floor of a room has dimensions 10. If a and b are prime numbers, which of the
m m sq.m. Dimensions of the tile used are following is true?
n n sq.m. All tiles used are green tiles except I. a2 has three positive integer factors.
diagonal tiles which are red. After some years II. ab has four positive integer factors.
some green tiles are replaced by red tiles to form
III. a3 has four positive integer factors.
an alternate red and green tile pattern. How
many green tiles are removed? Codes :
(m n and total number of tiles are odd). (a) I and II only (b) II and III only
(c) All of these (d) None of these
m 2 4mn 2n 2 m 2n 2 n2 11. If 7x + 6y = 420, x and y are natural numbers,
(a) (b)
2n 2 2n2 then what can be said about x?
m 4mn n 2
2
m 4mn 2n 2
2 (a) x is always odd
(c) (d) (b) x is always even.
2n 2 2n 2
(c) x is even only if y is odd.
x4 2
4. If then (d) x is odd if y is even.
x 6 x 6
2 2
12. If a, b, c, d, p and q are non-zero, unequal integers
(a) x 2 (b) x 2
(c) x 6 (d) x 6 a bi p a 2 b2
and , then 2 equals
5. Which of the following is true? c di q c d2
(a) 2 5 7 (b) 2 5 7 p p2
(a) (b)
(c) 2 5 7 (d) 2 5 7 q q2
y (c) 1 (d) None of these
6. If 6 x –2 and 4 y – 4, then limits for ,
x 13. An Egyptian fraction has a numerator equal to 1,
where x and y are non zero integers, is
and its denominator is a positive integer. What is
y y 2 y 2 y the maximum number of different Egyptian
(a) 2, (b) , 2
x x 3 x 3 x fraction such that their sum is equal to 1, and
y 2 y 1 y y their denominators are equal to 10 or less?
(c) , (d) – 4, 4 (a) 3 (b) 5 (c) 7 (d) 9
x 3 x 4 x x
14. What is the least number which must be
2n n 1
n
7. What are the limits of ; where n is a subtracted from 1936 so that the remainder when
nn divided by 9, 10, 15 will leave in each case the
positive integer? same remainder 7?
2n n 1
n (a) 32 (b) 53
(a) 2 3 (c) 46 (d) 39
nn
15. What is the greatest number consisting of six
2n n 1
n
digits which on being divided by 6, 7, 8, 9, 10 leaves
(b) 2n+1 3.2n 4, 5, 6, 7, 8 as remainders respectively?
nn
(a) 997920 (b) 997918
2n n 1
n
(c) 2n 3.2n (c) 999999 (d) 997922
nn 16. What is the least number which on being divided
(d) None of these by 5, 6, 8, 9, 12 leaves in each case a remainder
8. If g = 10100 and H = 10g, then in which interval 1 but when divided by 13 leaves no remainder?
does g! = 1 . 2 . 3 .... 10100 lie? (a) 2987 (b) 3601
(a) 10H < g! < H (b) H < g! < 10H (c) 3600 (d) 2986
H
(c) 10H < g! < 10 (d) 10H < g!
1.8 Numbers, Algebra and Data Interpretation
17. A group of four numbers has only one prime such a way that ratio of the number of members
number amongst them. Which of the following in a month to the preceding month bear a ratio
must be true about the group? equal to b. On May 1, 2003, both organizations
I. HCF of the four numbers of the group is ei- had the same number of members. If a = 20x, then
ther 1 or equal to that prime number. b will be
II. LCM of the four numbers of the group is same (a) 2 (b) 3 (c) 2.5 (d) 3.5
as product of the prime number and LCM of 23. What is the ratio of common differences d1 and d2 of
the remaining three numbers. two arithmetic progressions if respective nth terms
III. Product of four numbers is equal to product are in the ratio of 2n + 3 : n – 11?
of the prime number HCF of the group (a) 1 : 2 (b) 2 : 3
LCM of the group. (c) 2 : 1 (d) 1 : 3
(a) I only 24. What is the sum upto 20 terms of the series
2 8 26 80
(b) II only ...... ?
(c) I and II only 3 9 27 81
(d) All of these 19.320 1 19.320 1
(a) (b)
18. If (a, n)! is defined as product of n consecutive 320 2.320
numbers starting from a, where a and n are both
natural numbers, and if H is the HCF of 19.320 1 39.320 1
(c) (d)
(a, n)! and n!, then what can be said about H? 2.320 2.320
(a) H = a! (b) H = n! 25. What is the sum to infinity of the series,
(c) H n! (d) H a n 3 + 6x2 + 9x4 + 12x6 + ... given x < 1?
19. HCF and LCM of two numbers is given. It is
possible to find out the two numbers uniquely if 3 3
1 x
(a) (b)
1 x
2 2
I. either sum or difference between the two 2
numbers is known.
II. HCF of two numbers = LCM of two numbers. 3
3
1 x
(c) (d)
1 x
2 2
LCM 2
III. = Prime number..
HCF
(a) I and II only 26. A school has 5 divisions in a class IX having 60,
50, 55, 62 and 58 students. Mean marks obtained
(b) II only
in a History test were 56, 64, 72, 63 and 50 by
(c) II and III only each division respectively. What is overall average
(d) I, II and III of the marks per students?
20. A ball is dropped from a height of 12 m and it (a) 56.8 (b) 58.2
rebounds 1/2 of the distance it falls. If it continues (c) 62.4 (d) 60.8
to fall and rebound in this way, how far will it
27. Average age of a committee of seven trustees is
travel before coming to rest?
the same as it was five years ago, a younger man
(a) 36 m (b) 30 m having been substituted for one of them. How
(c) 48 m (d) 60 m much younger was he then trustee whose place
21. There are N questions in an exam. he took?
For i = 1, 2, ....., N, there are 2N–1 students who (a) 32 years (b) 35 years
answered 1 or more questions wrongly. If total (c) 33 years (d) 34 years
number of wrong answers is 8191, then N will be 28. Six men A, B, C, D, E, F agree with a seventh
(a) 12 (b) 11 (c) 10 (d) 13 man G to provide a sum of money among them.
22. Two new charity organizations C1 and C2 were A, B, C, D, E, F are to subscribe `10 each, and G
formed, with x members each, on January 1, 2003. is to pay `3 more than the average of the seven.
On first day of each, subsequent month, in C1, What is the whole sum to be provided?
number of members increases by a certain number (a) `73.50 (b) ` 74
a, while in C2, number of members increases in (c) ` 73 (d) ` 72.50
Numbers, Algebra and Data Interpretation 1.9
49. What is the ratio of the total production of number of rupees is _______
companies D and E to the total sale of the same 10. The remainder when 2050 2071 2095 is divided
companies? by 23 is _______
(a) 28 : 15 (b) 9 : 5
11. The smallest number which when increased by 3
(c) 15 : 11 (d) None of these is divisible by 27, 35, 25 and 21 is ______
12. _______is the least number which on being
NUMERICAL TYPE QUESTIONS divided by 12, 21 and 35 will leave in each case
1. If x – y = 1, then x3 – y3 – 3xy equals to _______ the same remainder 6
2. When x + y + z = 9 and xy + yz + zx = 11, then 13. A gardener had a number of shrubs to plant in
x3 – y3 – z3 – 3xyz equals to _______ horizontal rows. At first he tried to plant 5 shrubs
in each row, then 6, then 8 and then 12, but had
x 2 y 2 z2
3. If x + y + z = 0, then equals always 1 left. On trying 13, in one row he had
yz zx xy
to _______ none left. The smallest number of shrubs that he
could have had is ______
4. In an election for the President, if 261 valid votes
are cast, for the 5 contestants then least number 14. Sum of an infinite geometric series is 3 and sum
of votes a candidate requires to receive to win of an infinite geometric series formed from
the election are is _______ squares of the terms of the original series is 6.
First term of the first series will be____
5. A number when divided by sum of 555 and 445
gives two times their difference as quotient and 15. The maximum sum of the series 60, 58, 56, 54,
30 as remainder. The number is _______ 52, .... is_______
6. Sum of three prime numbers is 100. If one of them 16. On an average my income for 15 days ` 7, average
exceeds another by 36, then one of the numbers for the first 5 days was ` 6 and average for the
is _______ last 9 days was ` 8. The income on the sixth day
was ________
7. Sum of all odd numbers up to 100 is_______
17. The harmonic mean of two numbers whose
8. A = 0. a1a1a1 ... and B = 0.a2a2a2, where a1 and a2
geometric mean and arithmetic mean is 8 and 5
are multiples of 3 and also, a1 and a2 are distinct
respectively is ________
integers from 0 to 8. Then value of A + B
is _______ 18. Average marks of 15 students in a class is 145,
maximum marks being 150. If two lowest scores
9. Anil wants to divide `100 into a number of bags
are removed, the average increases by 5. Also,
so that one can ask for any amount between `1
two lowest scores are consecutive multiples of 9.
and `100, he can give the proper amount by giving
The lowest score in the class is _______
certain number of these bags without taking out
the amount from them. The minimum number
of bags he will require if each bag has whole
1.12 Numbers, Algebra and Data Interpretation
ANSWERS
MCQ Type Questions
1. (b) 2. (a) 3. (b) 4. (d) 5. (d) 6. (d) 7. (b) 8. (c) 9. (b) 10. (c)
11. (b) 12. (b) 13. (a) 14. (d) 15. (b) 16. (b) 17. (a) 18. (b) 19. (d) 20. (a)
21. (d) 22. (b) 23. (c) 24. (d) 25. (b) 26. (d) 27. (b) 28. (a) 29. (b) 30. (d)
31. (c) 32. (b) 33. (d) 34. (c) 35. (c) 36. (d) 37. (d) 38. (d) 39. (a) 40. (b)
41. (c) 42. (d) 43. (c) 44. (d) 45. (b) 46. (c) 47. (a) 48. (c) 49. (d)
Numerical Type Questions
1. 1 2. 432 3. 3 4. 53 5. 220030 6. 67 7. 2500 8. 1 9. 7 10. 6
11. 4722 12. 426 13. 481 14. 12/5 15. 930 16. 3 17. 12.8 18. 108
EXPLANATIONS
MCQ TYPE QUESTIONS x4 2
i.e.
1. x2 + y2 + z2 – 2xy – 2xz + 2yz = (x – y – z)2
x2 6 x 6 2
2 2
= (b + c – c + a – a + b) = 4b
i.e. x – 4 2 ...(cancelling a negative term
2. y3 – 4y = y(y2 – 4) reverses sign of the inequality)
= y(y – 2)(y + 2) i.e. x 6.
4y(y3 + 8) = 4y(y + 2)(y2 – 2y + 4) 2
5. 2 5 = 2 2 10 5
HCF = y(y + 2).
m = 7 2 10
3. Total red tiles initially = 2 1
n 2 5 7
2n n 1
n n n
m2 n 1 1
= 2n = 2n 1
n
Total number of tiles = 7.
n2 nn n
=
m 2n2 n2 On the other hand,
2n2 10H = 10 10g = 10g+1 < g!
x4 2 (most factors are much bigger than 10)
4.
x 6 2
x2 6 10H < g! < 10H
Numbers, Algebra and Data Interpretation 1.13
1 1 1 1 1 abc
= 20 ... II. AM =
3 3 2 3
3 4
3 320 3
Sum of the numbers = a + b + c = 3AM.
1
20
1 Thus, II is true.
1 3
= 20
3
3 1 III. GM = abc .
1
3
abc
But AM = , which cannot be found out.
3
320 1 Thus, III is false.
= 20
2.320 30. I. Average of a set of numbers is greater than
smallest and smaller than the greatest
40.320 320 1 39.320 1 number of the set. Thus, I is true.
= =
2.320 2.320 II. Consider five numbers a, b, c, d and e whose
25. Let s = 3 + 6x2 + 9x4 + 12x6 + ... abcde
2
xs = 3x2 + 6x4 + 9x6 + 12x6 + ... average is
5
s – x2s = 3 + 3x2 + 3x4 + 3x6 + ...
Now if each of them is increased by k, then
s(1 – x2) = 3 [1 + x2 + x4 + x6 + ...] we have average
1
s(1 – x2) = 3 x 1 ak b k ck dk e k
1 x2 =
5
3
s = a b c d e 5k
1 x 2 2 =
5
5
= old average + k.
Let the age of the person who returns be x years. 41. Number of students in Management discipline
484 x 10
Then new average = 4800 1008
14 100
Now x lies between 20 and 36 (both inclusive) Number of girls in Management discipline
484 20 8
New average min = = 36 years 2000 160
14 100
Number of boys = 480 – 160 = 320
484 36 Number of students in Science discipline
And New average max =
14
37
= 37.14. years. 4800 1776
100
33. Total income of A = 43.3 lakh
Number of girls in Science discipline
43.3
Monthly income of A lakh 32
7 12 2000 640
100
= Rs. 51547 Rs. 51245
Number of boys 1776 – 640 = –1136
425 5 Hence, required ratio = 320 : 1136
34. Required ratio = i.e.5 : 6
510 6 = 20 : 71
35. Required difference 42. Number of girls in catering discipline
= (36.565 – 30.2) lakh – Rs. 636500
10
36. Required percentage 2000 200
100
6425
100 7 Hence, required percentage
47810
200
= 94% 100 4.1%
4800
37. Average annual income of F
43. Required average
1 3.50 3.65 3.75 4
lakh 1
7 4.25 4.80 5.10 3.5 4 5 3 6.5
5
1
1
7
(29.05) = 4.15 lakh 22 4.4 crore = 44000000
5
38. No. of girls studying in engineering discipline
44. Required ratio
12
100
2000 240
5.5 5 0.5
5.3 2
39. It is obvious from pie-charts.
40. Number of students in Commerce discipline 1
ie,1 : 4
21 4
4800 1008
100 45. Average number of units sold by all the companies
together
Number of girls in Commerce discipline
1
24 (650 + 300 + 150 + 450 + 300 + 400)
2000 480 6
100
Number of boys = 1008 – 480 = 528 1
2250 375
Hence, required % 6
46. Hence, it is maximum for company A
528
100 52% 47. Average number of units produced by all
1008
companies together
Numbers, Algebra and Data Interpretation 1.17
= (x + y + z)(x2 + y2 + z2 – xy – yz – zx) a1 a 2 a a2 9
A+B = + = 1 = 1.
= 9[(x + y + z)2 – 3(xy + yz + zx)] 9 9 9 9
= 9(81 – 33) = 432. 9. If Anil has to give 1 rupee he needs a bag with
`1. For 2 rupees he had two bags with `1 each or
x 2 y 2 z2 `2. bag. To have minimum bags, he has a bag
3. with ` 2. Now with the two bags he can give `3.
yz zx xy
So next he will require a bag with `4. With these
... (since, x + y + z = 0; x3 + y3 + z3 = 3xyz) three he can give `5. `6 and `7 and next bag will
be one containing `8 and so on. Thus he would
x3 y 3 z 3 3xyz have bags with `1. 2. 4. 8. 16. 32. Sum of which
3.
xyz xyz is 63 and remaining 37 can be put in the last bag.
So total number of bags is 7.
4. Worst scenario is when other four get equal
number of votes. 10. 2050 2071 2095
Let the winning candidate get x votes. (23 89 + 3) (23 90 + 1) (23 91 + 2)
= (23[89 90 + 89 + 90 3] + 3) (23 91 + 2)
261
x > = (23 K + 3) (23 91 + 2)
5
= (23 [K 91 + K 2 + 91 3] + 6)
x > 52 .2
= 23 L + 6 (where K and L are constants).
x = 53
Hence remainder is 6.
5. Number = (555 + 445) × (555 – 445) × 2 + 30
11. L.C.M. of 27, 35, 25 and 21 = 4725
= (555 + 445) × 2 × 110 + 30
Number = 4725 – 3 = 4722.
= 220000 + 30 = 220030.
12. L.C.M. of 12, 21 and 35 = 420
6. x + (x + 36) + y = 100
Number required = 420 + 6 = 426.
2x + y = 64
13. Number is 120K + 1
Therefore y must be even prime which is 2
= ((13 9 + 3) K + 1)
2x + 2 = 64 x = 31.
= (13 9)K + 3 K + 1,
Third prime number
which is divisible by 13.
= x + 36 = 31+ 36 = 67.
3K + 1 is divisible by 13.
7. Given numbers are 1, 3, 5.........99.
K = 4. Number = 481.
This is an A.P with a = 1, d = 2.
1.18 Numbers, Algebra and Data Interpretation
a 16. Total income for 15 days = 15 7 = `105
14. =3
1 r Total income for first 5 days = 5 6 = `30
3 – 3r = a Total income for last 9 days = 9 8 = `72
Income on sixth day
a2
Also, =6 = 105 – (30 + 72)
1 r2
= 105 – 102 = `3
a a
. =6 17. (GM) = HM AM
2
1 r 1 r
8 8 64
a HM = 12.8
=2 5 5
1+ r
18. Total marks of 15 students = 15 145 = 2175.
2 + 2r = a
Average marks of 15 students (excluding two
3 – 3r = 2 + 2r lowest scores) = 13 (145 + 5) = 1950.
1 Total of two lowest scores = 2175 – 1950 =
r =
5 225
3 12 Given that two scores are consecutive multiples
a = 3= of 9
5 5
9x + 9x + 9 = 225
15. Since d = –2, maximum sum of the series will be
sum of all the positive terms. 18x = 216
Least positive term of the series is 2 or 0 (both 216
giving equal sum) x =
18
Let, 2 be the nth term
216
2 = 60 + (n – 1) (– 2) Lowest score = 9x = 9 = 108.
18
n = 30
Required sum = 30/2 (60 + 2) = 930
CHAPTER
2 Percentage and Its Applications
The term percentage is quite frequently used in our To find % decrease of a number:
day to day life especially to avoid the fractions less
than 1. Instead of treating the complete entity as 1, total decrease
% decrease = 100
we treat as 100, and take the ratios accordingly. Initial value
PERCENTAGE
Per Cent: This term means for every hundred. A
fraction whose denominator is 100 is called percentage Initial value Final value
= 100
and numerator of the fraction is called rate per cent. Initial value
It is denoted by the symbol %. e.g., Cost of a bike last year was ` 19000. Its cost this
To find % equivalent of a fraction. year is ` 17000. Find the % decrease in its cost.
Express fraction with the denominator 100 and
numerator is the required answer. 19000 17000
% decrease = 100
11 19000
11
e.g., = 100 = 45 5 %
24 24 6 2000
To find fraction equivalent of‘a %': Divide‘a' by 100. = 100 10.5%
19000
7 Note:
21
e.g.,
7
21 % =
8 = 175 = 7 (1) If price of a commodity increases by r%, then
8 100 800 32 reduction in consumption, so as not to increase
To increase a number by a given %:
r
100 rate expenditure is 100 %
Multiply the number by the factor 100 r
100 (2) If price of a commodity decrease by r%, then the
e.g., increase 20 by 15%, increase in consumption so as not to decrease
100 15 115 r
20 = 20 23 expenditure is 100 %
100
i.e 100 r
100
or 20 1.15 = 23 (3) If A's income is r% more than that of B, then B's
To decrease a number by a given %: r
income is less than that of A by 100 %
100 r
100 rate
Multiply the number by the factor (4) If A's income is r% less than that of B, then B's
100 income is more than that of A by
100 20 r
e.g., decrease 30 by 20%, i.e 30
100
100 r
100 % .
80 Example: If the production of rice went up from 250
= 30 24 MT in 2005 to 300 MT in 2006, then the percentage
100
or 30 0.8 = 24 increase in rice production from 2005 to 2006 is
calculated as follows:
To find the % increase of a number:
Actual increase = 300 – 250 = 50MT
total increase
%increase = 100 Percentage increase:
initial value
Final value Initial value
Actual increase from 2005 to 2006
100 100
=
Initial value Actual production of rice in 2005
e.g., Population of a village in 1980 was 3000 and in 50
1990 was 3200. Find the % increase. 100 20%
250
3200 3000
% increase = 100 I. If the increase on a value of 350 is 15% the new
3000 quantity is 1.15 × 350 = 402.5 (where 1.15 = 1 +
0.15, 0.15 being the decimal equivalent of 15%)
2.2 Percentage and Its Applications
II. If the production in 2005 is given as 400 MT and Solution:
the increase from 2004 to 2005 is given to be 25%
3 1 18 9
then the production in 2004 will be equal to The ratio of the monthly salaries 3 : 4 :
5 2 5 2
400
= 320 MT (where 1.25 = 1 + 0.25, 0.25 being (18)(2) : (9)(5) = 4 : 5
1.25
Let the monthly salary of A in December 2004
the decimal equivalent of 25%) = 4x
III. Similarly, if there is a decrease of 12% on a quantity His monthly salary in January 2005 would be = 5x.
of 225, then the new quantity will be equal to 225 Required percentage increase in his salary
× 0.88 (where 0.88 = 1 – 0.12, 0.12 being the
decimal equivalent of 12%). If the production in 5x 4x
(100)% 25%
2005 is given as 400MT and it is a decrease of 13% 4x
from 2004, then the production in 2004 will be Example: There are three numbers. The first and
400 the second numbers are 50% less and 60% less
equal to (where 0.87 = 1 – 0.13, 0.13 being respectively than
0.87
the decimal equivalent of 13%). the third. What percentage of the first number is the
On the basis of percentage increase, we can write second?
down how many times the old value gives the new Solution:
value. For example, if the percentage increase is Let the third number be 100
100%, we can conclude that the new value is 2
50
times the old value and if the percentage increase First number = 100 1 = 50
is 300% then the new value is 4 times the old 100
value. If the percentage increase is 450% then the 60
new value is 5.5 times the old value. In general if Second number = 100 1 = 40
100
the percentage increase is p% then the new value
40 4
p it is th i.e. th or 80% of the first number.
is 1 times the old value. 50 5
100
Example: Ram got 30% in a test and failed by 10
Conversely, if we know how many times the old marks. If the pass marks in the test was 70, find the
value gives the new value, we can find out the maximum marks in it.
percentage increase in the old value to get the
Solution:
new value. For example, if the new value is 3 times
the old value, the percentage increase in the old Let the maximum mark in the test be M.
value to get the new value is 200%. If the new 30
value is 4.25 times the old value, then the Ram’s mark = M
100
percentage increase is 325%. In general if the new
value is k times the old value, then the percentage 30
increase is (k – 1) × 100% Pass mark = M + 10
100
Example: If 45% of a certain number is 990, then
find the value of 54% of that number. 30
M + 10 = 70 i.e., M = 200
Solution: 100
Let the number be x Example: The price of an article is decreased by 20%.
By what percentage must the consumption of it be
45 increased in order to retain the expenditure on it?
Given, x 990
100 Solution:
x 2200 Let the initial price be Rs. 100/gm
54 Let the initial consumption be 1 gm
required value = x = 1188
100 Initial expenditure = Rs. 100
Example: The ratio of the monthly salaries of A in 20
New price = 100 1
3 1 100
December 2004 and in January 2005 was 3 : 4 .
5 2 = Rs. 80/gm
Find the percentage increase in his salary. New expenditure = Rs 100
Percentage and Its Applications 2.3
100 PNR 1500 3 9
New consumption = gm S.I. = = ` 405
80 100 100
5 Compound Interest (C.I.)
= gm Money is said to be lent at compound interest when at
4
the end of a year or other fixed period, the interest
Consumption must increase by that has become due is not paid to the lender, but is
5 added to the sum lent, and the amount thus obtained
1 100% becomes the principal for the next year or period. The
4
25% process is repeated until the amount for the last period
1 has been found. The difference between final amount
Example: The length of a rectangle increases by 14% and the original principal is the compound interest
and the breadth by 8%. What is the consequent (C.I.).
percentage increase in area? In compound interest, interest is calculated on the
Solution: accrued interest also.
N
Let length and breadth of the rectangle be l and b R
Amount = P 1
Area of rectangle = lb 100
Length is increased by 14% Compound Interest = Amount – Principal
Note:
14 (1) The time period after which the interest is added
New length = 1 × 1 × = (1 + 0.141) = 1.141
100 each time to form a new principal is called
Breadth is increased by 8% conversion period. It may be annually, semi
annually or quarterly.
New breadth = b + 8% b
(2) In case, interest is paid semi annually (half yearly),
= b + 0.08 b = (1.08)b N is number of half years and R is rate percent per
Area of rectangle half year i.e.,
= 1.14l × 1.08b = (1.2312)lb Number of years 2 = Number of half years.
Increase in area = 1.2312lb – lb Rate percent per annum 2 = Rate percent per
i.e., 0.2312 lb half year.
(3) In case interest is paid quarterly, N is number of
23.12 quarters and R is rate percent per quarter.
Percentage increase = lb
100 Number of years 4 = Number of quarters
i.e., 23.12% Rate percent per quarter + 4 = Rate percent per
quarter.
SIMPLE INTEREST AND COMPOUND
(4) When rates are different for different years, say
INTEREST R1, R2, R3 percent of 1st, 2nd and 3rd years respectively
When a sum of money is lent by A to B, A is called then
lender (creditor), B the borrower (debtor). R R R
Amount = P 1 1 1 2 1 3
The sum lent is called principal (P). 100 100 100
Interest (I) is the extra money paid by the borrowed to x
the lender for the use of the money for a specified time. (5) In case the time is a fraction of a year, say y years,
z
The time for which the money is borrowed is called then
period (N). x
y R
R
The extra amount paid per 100 rupees in a year is Amount = P 1 1 z
called rate per cent per annum (R). 100 100
The sum of interest and principal is called Amount (A).
A=P+I Example: A certain sum of money at C.I. amounts to
Simple Interest (S.I.) Rs. 811.25 in 2 years and to Rs. 843.65 in 3 years. Find
the sum of money.
When interest is paid as it falls due, it is called simple
interest i.e., throughout the load period, interest is Solution:
charged on the original sum (principal) borrowed. n
PNR R
S.I. = Since, A P 1
100 100
e.g., Find interest to be paid on ` 1500 at 9% per annum
2
for a period of 9 years. R
811.25 P 1 … (1) and
Here P = 1500, R = 9%, N = 3 years 100
2.4 Percentage and Its Applications
3 Solution:
R
843.65 P 1 … (2)
100 100 100 2 100 3
2000 a
112.5 112.5
On dividing (2) by (1), we get: 112.5
843.65 R = Rs. 840 (approx.)
1
811.25 100 Population Formula: The original population of a
town is P and annual increase is r%, then
R n
1.04 1 R4 R
100 population in n years is 1 and if annual
100
Now, putting R = 4 into (1), we get decrease is r%, then population in n years is given
by a change of sign in the formula
2
4
n
R
811.25 P 1 P 750
100
i.e., 1
100
e.g., If annual increase in the population of a town is
The sum of money is Rs.750
4% and present population is 15625, what will be the
population in 3 years?
EQUAL ANNUAL INSTALLMENT TO PAY THE
Required population = 15625 (1.04)3 = 17576
DEBT (BORROWED) AMOUNT Hire Purchase: In a hire purchase plan, a customer
Let, the value of each equal annual installment = Rs. a. can make use of the goods while paying for them.
Rate of interest = R% p.a. The amount paid at the time of purchase is called
down payment. The remainder is paid in equal
Number of installments per year = n. installments and each is the monthly installment. The
Number of years = T. difference between total amount to be paid and the
Total number of installments = n × T. cash price is called installment charge.
Borrowed amount = B. Then, Monthly Installment
Amount to be paid – down payment
=
100 100
2
100
nT
Number of instalments
a B
100 R 100 R 100 R
e.g., If a transistor is available at ` 400 each or ` 100
down payment and ` 70 per month for 5 months, find
Example: What annual installment will be required total amount paid for it, and the installment charge
to repay a borrowed amount of Rs.1,32,400 in 3 years Amount paid = 100 + 70 5 = ` 450
at 10% per annum compounded annually? Installment charge = 450 – 400 = ` 50
Solution: PROFIT AND LOSS
Let each annual installment be Rs. a. Cost Price (CP): The price for which an article is
By using the formula, bought is called its cost price.
Selling Price (SP): The price at which an article is
100 100 100
2 3
sold is called its selling price.
132400 a
100 10 100 10 100 10 Profit (Gain): The difference between selling price
and cost price is called profit. For profit, selling price
10 10 2 10 3 should be greater than cost price.
132400 a Loss: The difference between cost price and the selling
11 11
11 price is called loss. When cost price is greater than
10 10 100 the selling price, there is a loss.
132400 a 1
11 11 121 Profit and loss is generally represented as a percent
of the cost price, unless otherwise stated.
10 331
132400 a Overhead charges: If an individual has to spend
11 121 some money on transportation etc., then this extra
11 121 expenditure is called overhead charges.
a 132400
10 331 Marked price (MP): The price on the label is called
a Rs. 53,240 marked price or list price.
Example: A loan of Rs. 2000 is to be paid back in 3 Discount: The reduction made on the‘marked price'
equal annual installments. How much is each of an article is called discount. When no discount is
Installment to the nearest whole rupee, if the interest given,‘selling price' is the same as‘marked price'.
1
is compounded annually at 12 % p.a.?
2
Percentage and Its Applications 2.5
List of Formulae (100 15)
(1) Profit = SP – CP ... (SP > CP) SP2 200 Rs. 170.
100
(2) Loss = CP – SP ... (CP > SP) Total S.P. received = S.P1 + S.P2
Profit SP CP
(3) % Profit = 100 = 100 = 230 + 170 = Rs. 400.
CP CP Total C.P. = 200 + 200 = 400.
Loss CP SP
(4) % Loss = 100 = 100 Net result: No profit, No Loss.
CP CP
Profit % CP Note: If two items are SOLD, each at rupees S, one
(5) Profit = at a gain of X% and other at a loss of X%, then the net
100
result is always a loss.
Loss % CP
(6) Loss =
100
100+Profit %
(7) SP = CP
100
100 Loss%
(8) SP = CP
100
(9) SP = Marked Price – Discount
100
(10) CP = SP
100+Profit %
100
(11) CP = SP
100 Loss %
Discount
(12) Discount % = 100
Marked Price Example: Two articles were sold at Rs. 100 each. After
Some important point: selling it was realized that on one, a profit of 10% was
(1) If two items are sold, each at ` X, one at a made, and on the other, a loss of 10% was made. What
gain of p% and the other at a loss of p%, there is the net result?
p2 Solution:
is an overall loss given by % . The absolute
100 Item 1 Item 2
2p2 x Selling Price Rs. 100 Rs. 100
value of the loss is given by
1002 p2 Profit %10 -
(2) If CP of two items is the same and % Loss and
% Gain on the two items are equal, then net Loss % - 10
loss or net profit is zero.
100 100
(3) Buy x get y free i.e., if x + y articles are sold at Cost Price
cost price of x articles, then 1.1 0.9
x = 90.90 = 111.11
percentage discount = 100 .
xy Total S. P. received = 100 + 100 = Rs. 200
(4) By using false weight, if a substance is sold at Total C. P. = 90.90 + 111.11 = Rs. 202.01
cost price, then 200 202.01
100 Gain% Loss% = 100
overall gain % = 202.01
100
2.01
True Scale or Weight 100 1%
= 202.01
False Scale or Weight
The same calculation can be done by a very simple
(5) In case of successive discount a% and b%,
formula
ab
effective discount = a b %.
100 X2 2X 2 S
Loss % = and Value of loss = .
Example: Two shirts were having a cost price of Rs. 100 1002 X 2
200 each. One was sold at a profit of 15% and the
Where X is the percentage profit and loss made on
other was sold at a loss of 15%. Find the net profit or
each of the items and S is the common selling price
loss.
received on both. In case of discounts being offered,
Solution: the price on which the discount was offered is known
(100 15) as the marked price and the price that is finally received
SP1 200 Rs. 230. is known as the selling price.
100
2.6 Percentage and Its Applications
Example: A merchant purchases an item for Rs. 500. defrauds to the tune of 25% in selling. Find his overall
He marks the item at a price of Rs. 700 but allows a % gain.
discount of 10% on cash payment. What is the total Solution:
profit in terms of amount and percentage made by The milkman defrauds 20% in buying and also defrauds
the merchant? 25% in selling, so his overall % gain will be
Solution:
(100 20%)(100 25%)
C.P. = Rs. 500, M. P. = Rs. 700. 100 % 50%
100
10 If selling price of X articles is equal to the cost price
Hence S.P. = 700 1 = Rs. 630
100 of Y articles, then the net profit percentage is given
Thus, profit = Rs. 630 – Rs. 500 = Rs. 130. YX
by 100
X
130
Profit % = × 100 = 26%. Example: The cost price of 20 pens is equal to the
500
selling price of 25 pens. What is the net loss
If a trader uses a false scale for selling his goods, then percentage?
the overall gain made by him in this process will be Solution:
Error Here, cost price of 20 pens = selling price of 25 pens. So
Gain% = 100
True Value Error 20 25
the net loss percentage = × 100 = –20%
25
Example: A milkman claims to sell milk at the cost
price but uses a measure of 800 ml instead of a litre. Here minus sign indicates the loss.
Find the net profit made by him.
PARTNERSHIP
Solution:
It is an association of two or more persons who invest
Using the formula given above their money in order to carry on a certain business.
200 200 A partner who manages the business is called working
Gain % = 100 100 25 partner and the one who simply invests the money is
1000 200 800 called sleeping partner.
If a tradesman defrauds (by means of a false balance If capitals of the partners are invested for the same
or otherwise) to the tune of X% in buying and also time, then partnership is called simple, and if invested
defrauds to the tune of X% in selling, his overall for different periods, the partner is called compound.
percentage gain will be If period of investment is the same for each partner,
then profit or loss is divided in the ratio of their
(100 common gain %)2 investments.
100 %
100 e.g., (i) If A and B are partners in a business, then
Example: A trader defrauds the seller by 10% when Investment of A Profit of A
he purchases goods from him, and while selling the Investment of B Profit of B
same to a customer, he defrauds once again by 10%. Investment of A Loss of A
Find the net gain made by the trader. or
Investment of B Loss of B
Solution: (ii) If A, B and C are partners in a business, then
The required answer is Investment of A : Investment of B :
Investment of C
(100 10)2 12100
100 % 100 = Profit of A : Profit of B : Profit of C
100 100
OR Loss of A : Loss of B : Loss of C
121 100 21% Monthly Equivalent Investment: It is the product
If a tradesman defrauds (by means of a false balance of the capital invested and the period for which it is
invested.
or otherwise) to the tune of X% in buying and also
defrauds to the tune of Y% in selling, his overall If period of investment is different, then profit or
loss is divided in the ratio of their Monthly Equivalent
percentage gain will be
Investment.
(100 X%)(100 Y%) e.g., If A and B are partners in a business, then,
100 %
100 Monthly Equivalent Investment of A
Example: A milkman defrauds by means of a false Monthly Equivalent Investment of B
measure to the tune of 20% in buying and also
Percentage and Its Applications 2.7
Profit of A Loss of A of (Investment x time period) calculated for each
= = partner.
Profit of B Loss of B
Investment of A Period of Investment of A A started a business with a capital of Rs. 10000. Four
i.e.,
Investment of B Periof of Investment B months later, B joined him with a capital of Rs. 5000.
What is the share of A in a total profit of Rs. 2000 at
Profit of A Loss of A
= = the end of the year?
Profit of B Loss of B
Solution:
If A, B and C are partners in a business, then
Monthly Equivalent Investment of A Profit of A Amount No. of months
: Monthly Equivalent Investment of B Profit of B Amount No. of months
: Monthly Equivalent Investment of C = Profit of A
: Profit of B : Profit of C 10000 12 3
.
or 5000 8 1
Monthly Equivalent Investment of A : Monthly
Equivalent Investment of B 3
profit share of A = × 2000 = Rs. 1500.
: Monthly Equivalent Investment of C = Loss of A 31
: Loss of B : Loss of C
Example: A, B and C enter into a partnership. A
Example: A and B together invested Rs. 12000 in a
contributes Rs. 320 for 4 months, B contributes Rs.
business. At the end of the year, out of a total profit
510 for 3 months and C contributes Rs. 270 for 5
of Rs. 1800, A’s share was Rs. 750. What was the
investment of A? months. If the total profit is Rs. 208, then find the
profit share of each of the partners.
Solution:
Since profits are shared in the ratio of their Solution:
investments A’s Profit : B’s Profit : C’s Profit
= 320 × 4 : 510 × 3 : 270 × 5
A's investment Profit share of A
= 1280 : 1530 : 1350 = 128 : 153 : 135
B's investment Profit share of B
(Money invested by A & B for the same period) 128
profit of A = × 208
750 750 5 128 153 135
1800 750 1050 7 128
208 Rs. 64
5 416
Investment of A = 12000 = Rs. 5000
57
153
Example: In a business A, B and C invested Rs.380, profit of B = × 208
128 153 135
Rs. 400 & Rs. 420 respectively. Divide a new profit of
Rs. 180 among the partners.
153
Solution: 208 Rs. 76.50
416
A’s Profit. B’s Profit. C’s Profit
= A’s investment : B’s investment : C’s investment 135
= A’s investment : B’s investment : C’s investment profit of C = × 208
128 153 135
= 380 : 400 : 420 = 19 : 20 : 21
135
19 208 Rs. 67.50
Profit share of A = × 180 = Rs. 57. 416
60
STOCKS AND SHARES
20 Face value: The price of a share is printed on the
Profit share of B = × 180 = Rs. 60. share certificate is called face value of the share.
60
(Nominal value).
21 Dividend: Every shareholder of a company is entitled
Profit share of C = × 180 = Rs. 63. to a proportionate share of the profits of the company.
60 The amount of profits on each share is called dividend.
Example: If the partners invest different amounts Dividend is expressed as a percentage of the face
and for different period of time, then the profits at value.
the end of the year are shared in the ratio of products
2.8 Percentage and Its Applications
Market Value: In the market, selling price of the Note:
share may be different from its face value. Thus, (1) Income per share = Rate of dividend
market value of a share is fluctuable price.
Nominal value of 1 share.
Shares:In the market, three types of shares are
(2) Annual income = Income per share
available :
Number of shares.
(i) Premium share: If market price of a share is
(3) Number of shares
more than nominal value, then it is called
premium share. Nominal value of all the shares
=
(ii) Par share: If market price of a share is equal to Nominal value of one shares
the nominal value, then it is called par share.
(iii) Discount share: If market price of a share is
less than the nominal value, then it is called a Market value of the shares (Investment)
=
discount share. Market value of one share
Types of shares: There are two types of shares:
Total Dividend
(i) Preference shares: On these shares a fixed rate =
Dividend per share
of dividend is paid to share holders irrespective of
whether the company makes any profits or (4) Rate of interest on the Invest (yield)
whether it runs into loss. In case the company is
not able to pay the agreed dividend to preference Total income 100
=
shareholders, the dividend on cumulative Total investment
preference shares goes on accumulating and is paid (5) When stoke/share is purchased, brokerage is
as and when the company can do so. added to cost price.
(ii) Equity shares: Profit of the company that is left (6) When stock/share is sold, brokerage is subtracted
over after its distribution to the preference from selling price.
shareholders is distributed among the equity share
(7) In most problems, face value of a share is taken
holders. This dividend on equity share may
to be `, 100 unless specified.
sometimes be more or sometimes less than the
dividend on the preference share.
Stock
In order to meet expenses of a certain plan, the
Government of India sometimes raises a load from
the public at a certain fixed rate of interest. Bonds or
promissory notes called stocks of a fixed value are
used for sale to the public.
Percentage and Its Applications 2.9
EXERCISE
MCQ TYPE QUESTIONS dividend is 5% per annum?
1. The Manager (M) of the company along with his (a) 4.28% (b) 7.14%
Assistant Manager (AM) and a Business Analyst 1 5
(BA) invest ` 55000, ` 45000 and ` 60000 (c) 8 % (d) 5 %
3 6
respectively in a partnership firm for six months.
7. A rejects 0.08% of the meters as defective .How
The Manager being the highest authority among
many will he examine to reject 2?
the three gets one-tenth of the total profit and
Assistant Manager being second highest (a) 2500 (b) 1250
1 (c) 625 (d) 5000
authority gets th of the remaining and the 8. (a% of b) + (b% of a) is equal to
20
remaining profit is divided among the three in (a) a% of b (b) 2% of ab
the ratio of their investments. If the total profit (c) 20% of ab (d) 2% of 100 ab
was ` 40000, who gets the minimum share and 9. Ryan's stock of goods is worth $9462, which is
how much? 15% more than Harland's, and 15% less than
(a) BA, ` 11400 (b) AM, ` 11419 Lambert's. What is the value of stock carried by
(c) AM, ` 12500 (d) None of these Harland and Lambert respectively?
2. In a business partnership of three people A, B (a) $ 8218, $ 10032 (b) $ 1419, $ 10881
and C, B's share in total profit is ` 250 per ` 2000. (c) $ 8042, $ 10881 (d) $ 8228, $ 11132
At this profit his return on investment is 15%. If 10. Numerator of a fraction increases by 25%. What
B earns total of ` 900. what is the total investment
can be said about the resultant fraction?
made by A and C in business? Assume that each
of A, B and C have invested their money for the (a) It is 67% less than original.
entire year and profits are shared in the ratio of (b) It is equal to the original fraction.
their investments.
(a) ` 48000 (b) ` 42000 (c) It is 67% more than the original fraction.
(c) ` 6000 (d) Data insufficient (d) The change cannot be determine.
3. Abhay and Bimal started a business in 11. Which of the following statement/s is/are true?
partnership. Abhay invested a capital of ` 50000.
I. If two numbers are such that one is 25% more
Bimal agrees to pay Abhay an interest on half
the capital at 8% per annum. Also Bimal receives than the second, then sum of the two numbers
a salary of ` 500 per month for managing the is nine times that of the difference of the two
business. After six months Abhay withdraws numbers.
` 20000 and Bimal pitches in an equal amount.
Bimal continues to pay Abhay interest at 8% per II. If 56% of a number A is 24% of another number
annum for the balance amount. At the end of the B then 49% of A is 21% of B.
year they divide the profit equally and found that Codes:
total earning of Bimal in the year was exactly
thrice that of Abhay. The total profit ? (a) Only I (b) Only II
(a) ` 7200 (b) ` 6800 (c) Only I and II (d) None of these
(c) ` 5400 (d) ` 4800 12. In the Annual General Meeting of the Rotary club a
4. How much stock must be bought in 3 percent chairman is to be selected for which A and B is the
1 3 only two candidates. After the first round, each of
89 in order that by selling it at 91 a profit of
8 8 them gave a speech on their policy and work plan
` 45 may be made? for the coming year after which another round of
(a) 20 (b) 30 (c) 40 (d) 50 election was held. The number of votes not cast for
5. Which is the better investment (the security being A increased by 25% in the second round over those
equal), 3 percents perpetually at par, or 5 percents
not cast for him in the first round and A lost second
at 124, the latter stock being redeemable in
20 years at par? (Simple interest to be reckoned). round to B by twice as many votes as that by which
(a) 3% at par he had won in the first round. If total 260 people
(b) 5% at 124 voted each time what was the percent decrease in
(c) Both are same votes received by A in the second round assuming
(d) Cannot be determined that no vote was void in any of the rounds?
6. What rate percent per annum does a man get by (a) 18% (b) 25% (c) 21.43% (d) 30%
buying `100 share (` 70 paid-up) at ` 60 when
2.10 Percentage and Its Applications
13. A watermelon weighs 5000 gm. 99% of its weight (c) ` 150
is water. It is kept in a drying room and after (d) Cannot be determined
some time it turns out that it is only 98% water 18. By selling 5 dozen mangoes for ` 156, it was found
by weight. What is its weight now? 3
that th of the outlay was gained. What
(a) 2500 gm 10
(b) 4500 gm should the retail price per mango be in order to
gain 60%?
(c) 4950 gm
(a) ` 4 (b) ` 2 (c) ` 3.2 (d) ` 4.2
(d) None of these
19. A man sells sugar to a tradesman at a profit of
14. A sum of ` 5000 is divided into two parts A and B 20% but the tradesman becoming bankrupt pays
such that simple interests at the rate of 20% p.a. only 80 paise in the rupee. How much percentage
on A and B after 2 and 3 years respectively are does the man gain or lose by his sale?
equal. Which of the following is false about (a) 2.5% (b) 3% (c) 4% (d) 5.2%
A and B? 20. A trader allows a discount of 5 percent to his
(a) Ratio of A to B is 2 : 3 customers. What price should he mark on a article
(b) Ratio of A to B is 3 : 2 the cost price of which is ` 800 so as to make
clear profit of 25 percent on his outlay?
(c) A exceeds B by ` 1000
(a) ` 1000 (b) ` 1053
(d) Total interest earned on A and B is ` 2400
(c) ` 1200 (d) ` 1123
15. Which amongst the following two offers is the
21. A camera costing `550 is marked to be sold at a
better one? Consider with respect to interest
price, which gives a profit of 30%. What will be
accrued. its selling price in a sale when 20% is taken off
I. Investing an amount compounded annually at the marked price?
1% per annum for 100 years. (a) ` 600 (b) ` 572
II. Investing the amount compounded annually (c) ` 635 (d) ` 605
at 100% per annum for 1 year. 22. Buy two and get one free. Which of the following is
Codes: true?
(a) First offer (a) The person is selling at a loss.
(b) Second offer (b) The person is selling at a profit.
(c) Both are same (c) The loss or gain depends on the profit from sale
(d) Cannot be determined of two articles.
16. The population of rate in a locality x increases by (d) The person is selling at the cost price.
20% in one year. Observing this, the pest control
Directions (Q.23–25): Refer to the data below and
committee decided to use a special kind of
answer the questions that follow.
pesticide‘xyz' which effectively kills 160 rats in
3 months. Just after 2 years, what is the net A profit-maximizing producer has the option of
increase or decrease in the population of rats if, discriminating between markets A and B (i.e., charging
initially the population of rats is 3200 and different prices in markets A and B for the same product)
pesticide is used effectively? for a product X. Demand for the product in each of the
(a) Increase of 128 rats. markets A and B is given by
(b) Decrease of 128 rats. QA = 42 – 0.2 PA
(c) Neither an increase nor a decrease in the QB = 100 – 0.8 PB
population. Total cost in any market is given by
(d) None of these. C = 1000 + 5Q, where Q is quantity sold in that market.
17. A man travels from A to B to buy goods which he Consider two cases:
can get 10% cheaper in B than in A. If expenses
Case 1 in which she discriminates between the markets
of the journey are ` 15 and he makes a clear saving
and
of ` 10, how much does he pay for the goods?
Case 2 in which she does not discriminate.
(a) ` 225
(b) ` 200
Percentage and Its Applications 2.11
23. What is the approximate ratio of the price in market 7. A school has only three classes which contain
A to price in market B in Case 1? 40,50,60 students respectively. The pass percent of
(a) 1.65 (b) 2.00 these classes are 10, 20 and 10 respectively. Then
(c) 2.25 (d) 2.50 pass percent in the school will be______
24. What is the ratio of the total quantity of product X 8. For a sphere of radius 10cm, the numerical value
sold in Case 1 to the total quantity sold in Case 2? of surface area is ______ percent of the numerical
value of its volume.
(a) 0.50 (b) 1
9. At an election, there are two candidates only, a
(c) 1.10 (d) 1.05
candidate who gets 43 per cent of the votes is rejected
25. What is the difference in total profit in Case 1 and by a majority of 420 votes. Then total number of
the total profit in Case 2? votes recorded assuming that there was no void
(a) ` 289 (b) ` 829 (c) ` – 289 (d) ` – 829 vote are______
NUMERICAL TYPE QUESTIONS 10. If price of wheat be raised 30%, then by _____
percent a householder must reduce his consumption
1. A and B enter into a partnership. A puts in the whole of the article so as not to increase his expenditure?
capital of ` 45000 on the condition that the profits
11. A fraction in reduced form is such that when it is
will be equally divided after which B will pay A
squared and then its numerator is reduced by
interest on half the capital at 10% p.a. and receive 1
` 60 per month from A for carrying on the concern. 33 % % and denominator is reduced to 20%, its
3
The yearly profit in rupees, if B's income is half of result is twice the original fraction. Sum of the
A's income is ______ numerator and the denominator is _____
2. A, B and C started a business by investing 12. A person borrows two equal sums at the same
` 72000, ` 27000 and ` 81000. It was decided that time at 5 and 4 percent respectively and finds
servant's salary ` 2000 per month should be given that if he repays the former sum with simple
from the profit. If profit is ` 25000 in the first month interest on certain date 6 months before the latter,
and ` 20000 in the second month and average profit he will have to pay in each case the same
of remaining 10 months is ` 21000, then share of amount, viz., ` 1100. The amount borrowed in
C is _______ rupees is ______
3. Three friends started a business of renting out air 13. Effective annual rate corres-ponding to a
conditioners by investing ` 20000, ` 24000 and nominal rate of 6 percent per annum, payable
` 16000, respectively. C gets 20% of total profit for half-yearly is ______
repair and maintenance of the air conditioner. If in 14. A tradesman puts two prices on his goods, one
a particular year, C gets ` 487.50 less than total for ready money and the other for 6 month's
earnings of the other two, then total profit for the 1
credit interest being calculated at 12 % per
year in ` is ________ 2
annum.If credit price on an article be ` 26.56,
4. A person holds forty ` 500 shares in a concern, which then its cash pric e in rupees is _______
pays dividend at the rate of 6 percent per annum.
15. An article costing `9000 is sold at a discount
When the shares are at ` 675, he sells out and invests which is equal to the simple interest on ` 3000
half the proceeds in 4 percent stock at 90. With the for N months. If rate of discount is same as the
other half he buys a house. For which he receives rate of interest, the N will be_______months
an annual rent of ` 1440 subject to a deduction of 16. A lent B ` 900 for a certain time at a certain
25P per rupees for repairs and taxes. Alteration in rate percent per annum, which is equal to the
rupees in his annual income will be ________. square root of the number of the months of the
5. A person invests ` 34539 in three percent at 87. time. After the time B wanted to return the
After receiving one year's dividend, he sells out at money, but A, instead of taking the interest
89. He then invests the whole in railway stock paying which amounted to `8 asked for a certain sum
5 percent at 115.The difference in his income will of money at the same rate for a certain number
be_______ of years which is equal to the square root of
the rate percent. Sum of money in rupees so
6. Difference of two numbers is 1660.
that none of them should be the loser will
If 7.5 % of one number is 12.5% of the other number,
be_______ (Take simple interest)
then two numbers are______
2.12 Percentage and Its Applications
17. A shopkeeper sells a microwave at a discount 22. Three shepherds A, B and C rented a pasture
on its marked price of ` 2500. But in a clearance for a year. A grazed 22 sheeps for 4 months, B
sale he gives an additional discount of 20% thus grazed 16 sheeps for 8 months and C grazed
selling it at ` 1500. Then first disc ount 32 sheeps for 6 months. If C's share of rent is
was_____% `600, then total rent in rupees for the year
18. A sports goods shop owner has 6 packets of balls was_____
each containing the same number of balls. If 23. Three bachelors, Amar, Akbar and Anthony
he sells each ball at ` 8 he gains ` 96. If there rented a house for a year. But, Amar left after
were 1 ball less in each packet and he sells each 4 months, Akbar stayed for 8 months and only
ball at ` 9 then he gains ` 90 in the transaction. Anthony stayed for the entire year. If the annual
rent was ` 6000, then share of Akbar is______
The cost price of each packet of balls in rupees
is ______ 24. Ram, Shyam and Ghanshyam invest `2000,
` 10000 and ` 5000 to set up a stall. Ram gets
19. O n e mer c han t c or r ec tly c alcu lates h is
15% of the total profit for running the stall. Ram
percentage profit on the cost price; another
gets ` 500 less than Shyam and Ghanshym
wrongly calculates it on the selling price. The
together, then total profit of that year is_______
difference in actual profits in rupees if both
1 25. x, y and z start a joint venture wherein they
claim to make 17 % profit on goods sold at make an annual profit of ` 3600. x invested one-
2
` 3760 is______ third of the capital for one-fourth of the time, y
20. A manufacturer marks his goods in advance at invested one-fourth of the capital for one-half
80 percent more than the cost price, but he of the time, while z invested the remainder of
allow s 15 artic les to the dozen and also the capital for the entire year. Then share of x
10 percent discount for cash. The profit on his in the profit is_____.
outlay does he obtain from a customer who pays
cash is _____%
21. A trader buys a certain amount of goods worth
` 22520. He decides to make a profit of 5.36%
on the sale of goods worth ` 5000 and increase
the profit percent by 3.14% for sales upto `15000
and then increase the profit percent for the sale
of remaining lot such that he is able to make a
profit of 25% on the sale of the full lot. Then
profit in rupees that he makes on the third lot
of goods is______
ANSWERS
MCQ Type Questions
1. (b) 2. (b) 3. (a) 4. (a) 5. (b) 6. (d) 7. (a) 8. (b) 9. (d) 10. (c)
11. (c) 12. (c) 13. (a) 14. (a) 15. (a) 16. (c) 17. (a) 18. (c) 19. (c) 20. (b)
21. (b) 22. (c) 23. (a) 24. (b) 25. (a)
Numerical Type Questions
1. 9180 2. 103950 3. 2812.50 4. 480 5. 345.2 6. (4150, 2490)
1 1
7. 13 8. 30 9. 3000 10. 23 11. 8 12. 1000
3 13
13. 6.09 14. 25 15. 4 16. 600 17. 25 18. 48
19. 98 20. 29.6 21. 4512 22. 1275 23. 2000 24. 1000
25. 480
Percentage and Its Applications 2.13
EXPLANATIONS
MCQ TYPE QUESTIONS x 6000
Bimal's earning = 6000 + – 1200
2
1. The ratio of their investments is 11 : 9 : 12. x 6000 x 6000
3 1200 = 4800 +
2 2
1
So, Manager gets (40000) = 4000 + his share x
10 3x
3600 + – 9000 = 4800 + – 3000
Remaining amount = 36000. 2 2
1 3x x
Asstt.Manager gets (36000) = 1800 + his share – = 7200;
20 2 2
Remaining amount = 36000 – 1800 = 34200 x =` 7200
11 3 1 2
M's share = 34200 + 4000 4. Profit per stocks = 91 – 89 = ` 2
32 8 8 8
= 11756.25 + 4000
45
= ` 15756.25 ` 15756 = 20 stocks
1
2
9 4
AM's share = 34200 + 1800
32 5. Income from a stock at 3% in 20 years at par
= 9618.75 + 1800 100
= 3 20 60
= ` 11418.75 ` 11419 100
12 Income from a stock at 5% at 124 in 20 years
BA's share = 34200 = ` 12825.
32
100
2. B's share in the profit = 250 per 2000 = 5 20 = 80.64.
If B gets ` 900, then 124
6. He invests ` 60 and gets ` 100 share
18
total profit = 2000 = 7200 Income = ` 5
5
B's return on investment = 15% 5 5
Required % = 100(0.70) = 5.83% = 5 %
60 6
900
100 = 15 7. Let number of meters to be examined be x.
B's investment
Then 0.08% of x = 2.
B's investment = 6000
Hence profits are shared in the ratio of their 0.08
x =2
investments. 100
7200
Total investments = 6000 = 48000 2 100 2 100 100
900 x = = = 2500
0.08 8
Investment made by A and C = 48000 – 6000
= 42000. a b ab ab 2ab
8. b a =
3. Interest paid by Bimal to Abhay for first six months 100 100 100 100 100
50000 8 1 2
= = ` 1000 = ab = 2% of ab.
2 100 2 100
For last six months 9. Ryan's stock = 1.15 Harland's stock
30000 20000 8 1
= Harland's stock =
9462
= $8228
2 100 2 1.15
For = ` 200 Ryan's stock = 0.85 Lambert's stock
Total salary paid to Bimal = 12 500 = ` 6000
Let total profit be ` x 9462
Lambert's stock = = $11132.
0.85
x 6000
Abhay's earing = 1200 +
2
2.14 Percentage and Its Applications
x 10b
10. Let the fraction be . – 260 = 520 – 4b
y 4
Let x1 = x + 0.25x = 1.25x 10b
4b + = 520 + 260
y1 = y – 0.25x = 0.75x 4
x1 1.25x 5 x b = 120
= =
Hence first round, A gets 140 and B gets 120.
y1 0.75x 3 y
In second round A gets 110 and B gets 150.
5 x x % decrease in votes received by A
–
3 y y 2
Increase = 100 = 100 = 66.67% 140 110
x 3 = 100 = 21.43%
140
y
13. Non-water matter is 1% of 5000 = 50 gm.
Thus, resultant fraction is more than the original After drying, this matter becomes 2% of the weight
fraction by 67%. Thus III is true. of the water-melon. Hence weight of the
11. I. Let first number be x. watermelon now is 2500 gm.
Second number = 1.25x
Sum = x + 1.25x = 2.25x 14. Let A = ` x and B = ` (5000 – x)
Difference = 1.25x – x = 0.25x
Also, 9 0.25x = 2.25x Then,
x 2 20
=
5000 x 3 20
100 100
Thus I is true.
2x = 15000 – 3x
56 24
II. A= B 5x = 15000
100 100 x = 3000
49 A = x = ` 3000
Multiplying by on both the sides.
56 and B = 5000 – x
= 5000 – 3000
49 56 49 24
A= B = ` 2000
56 100 56 100
A 3000 3
49 21 = =
A = B B 2000 2
100 100
Thus, (a) is not true and (b) is true.
49% of A = 21% of B.
A – B = 3000 – 2000 = ` 1000.
Thus, II is true.
Thus, (c) is true.
(d) The total interest
12. In the first round, let A get‘a' votes and B get‘b'
votes. 3000 2 20 2000 3 20
=
Then, a + b = 260 100 100
i.e., a = 260 – b ... (i) = ` 2400.
In second round, B gets Thus, (d) is true.
125 5b
15. First investment gives a return
b = 100
100 4 1
P 1 P
5b 100
and A gets 260 –
4 100
1
First term is P 1 = X,
5b 5b 100
Also, – 260 = 2(a – b)
4 4 such that 2P < X 3P
10b Hence return X – P lies between P and 2P
– 260 = 2(260 – b – b) i.e., 2P – P < X – P 3P – P.
4
i.e., P < X – P 2P.
Percentage and Its Applications 2.15
Whereas second offer. If SP = CP of 1 article, there is not profit no loss.
1 Thus, only (c) is true.
100
= Return = P 1 P 23. Following two key points need to be noted :
100
(A) Producer is profit maximizing which implies
= 2P – P = P. she will set prices that maximize the profit
Alternatively function.
Option of investing at 1% for 100 years is obviously (B) Quantity demanded in market A is independent
better than investing at 100% for 1 year, because of quantity demanded in market B.
total compound interest for 100 years will be CASE 1:
atleast 100% of the total amount (1% for each In this case, producer charges different prices in
year), plus whatever the compounding effect will markets A and B. This implies that she will
have.
maximize profits individually.
16. Growth rate of rat population in 3 months
Equations can be re-written as
3 PA = 210 – 5QA
= 20 = 5%.
12 P B = 125 – 1.25 QB
Increase in first 3 months = 3200 1.05 = 3360 Profit in Market A = A
Also, net decrease in 3 months = 160 = Revenue – Cost
Rat population = 3360 – 160 = 3200 = QA PA – CA
In the same way, after every 3 months, rat = QA (210 – 5QA) – (1000 + 5QA)
population remains the same. At the maximum profit level, differential A with
Hence, even after 3 8 months i.e., 2 years, the respect to QA is 0
population is maintained or
17. Let CP in A be ` x 210 – 10QA – 5 = 0
CP in B = ` 0.9x QA = 20.5, PA = 107.5
0.9x + 15 + 10 = x and total revenue in market A
0.1x = 25 = 20.5 107.5 = ` 2203.75
x = 250
Similarly
Price in B = 250 0.9 = ` 225
Profit in Market B = B
18. Let CP of 5 dozen mangoes be ` x.
= Revenue Cost
SP = ` 156 and Gain = 0.3x
= QB PB – CB
156 – x = 0.3x
= QB (125 – 1.25 QB) – (1000 + 5QB)
x = 120
At maximum profit level, differ-ential of B with
SP of 60 mangoes = 120 1.6 = 192 respect to QB is 0
SP per mango = ` 3.2
or 125 – 2.5QB – 5 = 0
19. Let CP be ` x ; SP = 1.2x
QB = 48, PB = 65
But he gets only 1.2x 0.8 = 0.96x
and total revenue in market B
Loss = 0.04x
= 48 65 = ` 3120
Loss = 4 %
Using the above, we get
20. Selling price = 800 1.25 = ` 1000
PA 107.5
1000 ~ 1.65.
Marked price = ` = 1053 PB 65
0.95
21. CP = ` 550 24. Data for 1 is available above. Consider case 2. In
Marked price = 1.3 550 = ` 715 this case, prices across the two markets are the
same. Hence, quantity equations can be added to
Sales price = 0.8 715 = ` 572.
give
22. Profit or loss of the sale depends upon how much Q = 142 – P
profit the person is earning on the sale of two
where Q is total quantity and P is price in both
articles.
markets
If SP > CP of 1 article, there is a profit.
P = 142 – Q
If SP < CP of 1 article, there is a loss.
2.16 Percentage and Its Applications
= Profit = Q (142 – Q) – (1000 + 5Q) (6x + 5x) – 7.75x = 487.50
At the maximum profit level, differential of with 3.25x = 487.50
respect to Q is 0 x = 150
Q = 68.5, P = 73.5 Total profit = 18.75 150 = ` 2812.50.
and Total Revenue = 5034.75 4. Selling price of 40 shares
Given this, we note that quantities sold in both = 675 40 = ` 27000
Case 1 and Case 2 are the same. Hence, the ratio Income from half the proceeds
is exactly 1.
25. Profit Difference between case 1 and case 2 13500
= 4 = ` 600
= Revenue Difference (since quantity sold and 90
hence, cost is the same) Income from house = 1440 0.75 = 1080
Profit difference = 2203.75 + 3120 – 5034.75 Present total income = 1080 + 600 = ` 1680
= 289
6
Earlier Income = 40 500 = ` 1200
100
NUMERICAL TYPE QUESTIONS
Increase in income = `480
1. Interest on ` 22500 = 0.1 22500 = `2250
Charges for managing the concern 34539
5. One years dividend = 3 = ` 1191
= 60 12 = ` 720 87
If yearly profit is ` x, then 34539
Selling Price = 89 = ` 35333
A's Share 87
B's share =
2 Number of railway stocks bought
35333
x 1x = = 307.24
– 2250 + 720 = 720 2250 115
2 22
Income from railway stocks
x = ` 9180. = 307.24 5 = ` 1536.2
2. Total annual profit Difference in income = 345.2
= 25000 + 20000 + 21000 10 6. Let two numbers be x and y.
= ` 255000 7.5% of x = 12.5% of y
Actual profit 75x = 125 y
= 255000 – (2000 12) = ` 231000 3x = 5y
Total investment
5
= 72000 + 27000 + 81000 = `180000 x = y..
3
72000 Now x – y = 1660
A's share = 231000 = ` 92400
180000
5
3 y – y = 1660
B's share =
27000
231000 = ` 34650
180000
2
81000 3 y = 1660
C's share = 231000 = ` 103950.
180000
y = 2490
3. 80% of the total profit is divided in the ratio
x = 2490 +1660 = 4150.
20000 : 24000 : 16000 = 5 : 6 : 4
Hence numbers are 4150, 2490.
80% of total profit = 5x + 6x + 4x = 15x
7. Number of passed candidates
15x
Total profit = = 18.75x 10 20 10
80% = 40 + 500 + 60
100 100 100
Share of C in profit = 4x + 20% of 18.75
= 4 + 10 + 6 = 20
= 4x + 3.75x = 7.75x
Total students in school
Share of A in profit = 5x
= 40 + 50 + 60 =150
Share of B in profit = 6x
Percentage and Its Applications 2.17
Required percentage
5
4
20 40 1 2 = P 110
1100 = P 1
= 100 = = 13 % 100 100
150 3 3
8. Surface area = 4
22 2
r P = ` 1000
7 2
3
3 4 22 3 3 13. 1 1.0609
100
= r = Volume
r3 7 r Effective annual rate = 6.09%
where r = 10 cm 14. Let cash price be `P
So we have
P 12.5 0.5
3 3 26.56 = P +
S= V= 100 % of V = 30 % of V 100
10 10 P ` 25.
So surface area is 30 % of volume. 15. Let R be the rate of interest.
9. Difference in % of votes = 57 – 43 = 14%
9000 R
14% is represented by 420 Discount =
100
100 3000 R T
Total number of votes = 420 = 3000. Interest =
14 100
30 By the given information:
10. % reduction in consumption = 100 %
100 30 9000 R 3000 R T
=
100 100
30 1
= 100 % = 23 %
130 13 1
T = years = 4 months.
3
a
11. Let the fraction be 16. Let N be the number of years
b
2 R% = N 12
2
a
3 = 2
a a
0 if a 0, 900 12N N
48 =
b2 1 b b 100
5 Squaring both sides
a
2 48 48 16 4 64
then =2 N² = =
b 3/5 9 9 12 9 3 27
a 3
= R = 12
4
b 5 4%
3
a + b = 8.
1 P42
P 5 T 48 =
2 100
12. 1100 = P +
100 P = ` 600
P4T 17. Let first discount be x%
1100 = P +
100 100 20 100 x
2500 = 1500
1 100 100
P 5 T
2 P4T
= 80 100 x
100 100 2500 = 1500
100 100
5
5T – = 4T
2 1500 100 100
100 – x =
5 80 2500
T= years 100 – x = 75;
2
x = 25
2.18 Percentage and Its Applications
Alternatively, 22. A's Monthly Equivalent Rent = 22 4
ab B's Monthly Equivalent Rent = 16 8
Successive discount = a b %
100 C's Monthly Equivalent Rent = 32 6
L1 L2 1 1
X Y ;
2
L1 L2 1 1
Speed of the slower train X Y .
2
Example: Two trains 110 m and 88 m long
respectively are running in same direction. The first
L1 L 2 runs at the rate of 35.2 kmph and the second at the
T seconds rate of
X
44 kmph. How long will they take to cross each other?
(3) If two trains of lengths L 1 and L 2 meters
respectively, are moving in the same direction Solution: It is clear that the trains will cross each
with a speed of X and Y m/sec (where X > Y), other when they have travelled a distance equal to
then (X-Y)m/sec is called Relative Speed and the sum of their lengths = 110 + 88 = 198 m. Since
time taken by faster train to pass the slower train they are moving in the same direction, we can find
the relative speed = 44 – 35.2 = 8.8 km/hr.
L1 L2
seconds. Time required = distance/speed
XY
198 18
(4) If two trains of lengths L 1 and L 2 meters sec ond 81sec
respectively, are moving in opposite directions 8.8 5
(towards each other or away from each other) Example: A train in motion, 66 m long overtakes a
with a speed of X and Y m/sec, then (X + Y)m/sec train 88 m long travelling @ 30 kmph in the opposite
is called Relative Speed and time taken by the direction in 0.168 min. The speed of the first train is
trains to pass each other. (1) 60 kmph (2) 25 kmph
(3) 45 kmph (4) 36 kmph
Speed, Time and Work 3.3
Average speed for total journey (UP + DOWN) (3) Race Course: It means the path or ground on
which races are run.
Upstream Rate Downstream rate
(4) Finishing Point: The point where the race
Man 's rate in still water finishes is called the winning post or finishing
Total Journey time (tup + tdown) point or a goal.
(5) Dead Heat Race: A race in which no one is the
Man 's rate in still water Total dis tan ce winner because all the runners reach the winning
Upstream Rate Downstream rate post at the same time, is called a dead heat race.
Example: A man rows 10 km upstream and back again (6) Winner: The person who first reaches the
to the starting point in 55 min. If the speed of stream finishing point is called the winner.
is 2 kmph, then find the speed of rowing in still water. (7) Winner’s Time: The time taken by the winner
Solution: Let x be the speed of rowing in still water. to complete the race is called winner’s time. If A
and B both start from the same place, then
y = speed of stream = 2 kmph.
winner’s distance = Length of the race.
55 (8) Loser’s time: The time taken by the loser to
Total time T h.
60 complete the race is called loser’s time.
Hence, Linear Races
Suppose A & B are two contestants in race. If before
Speed in still water Total dis tan ce the start of the race, A is at the starting point and B is
Total Time =
Upstream Rate Downstream rate ahead of A by 10 meters, then A is said to have given
B a start of 10 meters. To cover a race of 300 meters
55 x 2 10 55 2 in this case, A will have to cover a distance of 300
(x 22 ) 2 x 10
60 (x 2)(x 2) 60 meters and B will have to cover (300 – 10) = 290 meters
only.
11x2 – 240x – 44 = 0
Suppose A & B are two contestants in race. At the
(x – 22)(11x + 2) = 0 end of the race, if A is at the finishing point and B is x
x = 22, since (–)ve value of x is not admissible. meters away from the finishing point, and then A is
Total Distance = Downstream distance + Upstream said to have beaten B by x meters in a race.
distance = 2 x any one side distance. (1) A gives B a start of x meters implies that, if
the distance between the starting point and
Example: A man who can swim 48 m/min in still finishing point is L meters, A covers L meters
water swims 200m against the current and 200 m with while B covers L – x meters.
the current. If the difference between these two times
is 10 min, then find the speed of the current in m per
min.
Solution: Let the speed of stream be x kmph. The From the figure given above, it is clear that A
equation becomes starts at point P, but B starts at R at the same
moment. For example in a 100 meters race, A
200 200
10 gives B a start of 10 meters means, while A run
48 x 48 x 100 meters, B runs 90 meters.
200(48 + x) – 200(48 – x) =10[48x2 – 22] (2) A beats B by x meters implies that, if the
x2 + 40x – 2304 = 0 distance between the starting point and finishing
point is L meters, A wins the race by covering L
On solving it we get x = 32 and x = –72 (not acceptable) meters while B covers L – x meters only.
i.e. speed of stream is 32 m/min.
Starting point P Finishing point Q
LINEAR AND CIRCULAR RACES
L-X R X
The terms given below are commonly used in this
topic, and a clear understanding will help the student (3) A gives B a start of t seconds, implies that A
get a good grip on the subject. starts the race t seconds after B starts from the
starting point.
(1) Race: A contest of speed between participants is
called a race. (4) A beats B by t seconds, implies that, A and B
start together from the starting point, but A
(2) Starting Point: The point from where a race
reaches the finishing point t seconds before B
begins is called the starting point.
reaches.
3.6 Speed, Time and Work
(5) St ar t di st ance is the distance between the two They will be first together again after an interval of
contestants at start if they are not starting from time which is required to complete a round with the
the same position. relative speed.
(6) If A & B are starting from the same point, A beats Example: Assuming the data given in previous
B by ‘x’ meters or ‘t’ seconds means, B runs ‘x’ example, when will they be together supposing they
meters in ‘t’ seconds. walk in opposite directions?
Example: In a km race, A beats B by 40 meters or Solution:
7 seconds. Find A’s time over the course. Relative speed =150 + 70 = 220 m/min.
Solution: Here B runs 40 meters in 7 seconds. So, they will be together after 1200/220
1000 7 = 60/11 minutes.
B runs 1000 m in 40
= 175 seconds. We hope that the concept of circular race is clear to
you. If not, please go through the following example.
Hence, A’s time over the course –(175 – 7) = 168 sec
It will help you to understand the concept thoroughly.
Short-cut: Example: Along with A and B, C is also running across
By formula, the circular track at the rate of 130 m/min in the same
direction. When will all the three people meet (use
Winner 's time Beat time Start time data of previous example)
Loser dis tan ce Beat dis tan ce Start dis tan ce 1) at the starting point?
2) at any other point on the track?
7
A’s time = 960 = 168 seconds. 3) At what distance (nearest) from the starting point
40 do the three meet?
If a race ends in a dead heat, then beat time = 0 and Solution:
beat distance = 0. (1) Time for 1 round by A = 8 min.
Circular Races Time for 1 round by B = 120/7 min.
When two or more persons start from the same place Time for 1 round by C = 120/13 min.
at the same time and travel round a circle in the same So, all the three people will meet at the starting
direction or in opposite directions, then they will be point after the time.
first together at the starting point again after an = The LCM of 8, 120/7 and 120/13 i.e.120 minutes.
interval of time which is the LCM of the times in which (2) A gains 80 meters per minute over B.
each of them makes one complete round. So, they will met every 1200/80 = 15min
Example: A and B walk around a circular path of A gains 20 meters per minute over C.
circumference 1200 meters. A walks @ 150 m/min and So they will meet every 1200/20 = 60min
B @ 70 m/min. If they start from the same point and So, all the three people will meet after the time
walk in the same direction, then when will they be which is equal to the LCM of 15 and 60 i.e. after
first together again at the starting point? 60 minutes or 1 hour.
Solution: Time for 1 round by A = 1200/150 (3) They are meeting after 60 minutes from the start.
= 8 minutes and for B Distance covered by A in 60 minutes
= 150 × 60 = 9000m = 600m from the starting
= 1200/70 = 120/7 minutes.
point.
Time taken to meet again at the starting point Distance covered by B in 60 minutes
= LCM of 8 and 120/7 = 120 minutes = 2 hrs. = 70 × 60 = 4200 m = 600m from the starting
They will be first together again after an interval of point.
time which is the LCM of the times in which one of Distance covered by C in 60 minutes
the persons gains one complete round over each of = 130 × 60 = 7800m = 600m from the starting
the others. point.
Example: Assuming the data given in previous So, the three people will meet at 600 m from the
example, when will they be together again anywhere starting point.
else on the course. Example: Now, if we assume that C starts running
Solution: Now, in 1 minute, A gains over B(150 – 70) in the opposite direction, then the three will once
= 80 meters. again meet at the starting point after the same time
interval. But can they meet anywhere else? If Yes,
To gain 1200 meters over B, A will take 1200/80 × 1 after what time interval and what minimum distance
= 15 min. from the starting point?
Speed, Time and Work 3.7
Solution: C starts running in the opposite direction, Example: Anish can finish a piece of work by himself
as shown in the figure below. in 14 days. Then calculate the amount of work done
by him in 1 day and 11 days.
Solution: The amount of work done by Anish, working
alone in 14 days = 1 unit of work. So, the amount of
work done by Anish, working alone in 1 day = 1/14
unit of work and in 11 days = 11/14 units of work.
Note: If A is twice as good a workman as B, then A
will take half the time B takes to finish a piece of
work.
Example: Suresh can finish a piece of work by himself
in 42 days. Mahesh, who is 1/5 times more efficient as
Suresh, requires X days to finish the work by working
all by himself. Then what is the value of X?
Solution: Suresh, working alone 42 days = 1 unit of
The three will once again meet at the starting point
work.
after the same time interval i.e. 2 hours.
Mahesh is 1/5 time more efficient than Suresh. So
Now A & C will meet after 1200/280 = 30/7 minutes
Mahesh is 6/5 times as efficient as Suresh. Hence
from the starting point. B & C will meet after 1200/200
Mahesh should require 5/6th of the time, the time
= 6 minutes from the starting point.
taken by Suresh.
So, in this case the three people will meet after the
Therefore time taken by Mahesh = 5/6 × 42 = 35 days.
time which is equal to the LCM of 30/7 and 6 i.e. after
30 minutes from the starting point. Measurement Of Work
Distance covered by A in 30 minutes Work can be measured by many units. Some of them
are
=150 × 30 = 4500m = 900m from the starting point.
(1) Man-Days
Distance covered by B in 30 minutes
A certain number of men working for acertain
=70 × 30 = 2100m = 900m from the starting point.
number of days.
Distance covered by C in 30 minutes
Example: 5 men can finish a piece of work in 13 days.
=130 × 30 = 3900m = 300m from the starting point.
This implies that the work is of 5 × 65 Man-days, i.e.
The minimum distance at which they will meet = 300 if one man alone works to finish the given work then
m from the starting point. he is going to take 65 days to finish it and if 65 men
Note: Throughout this chapter, units of quantities work together, then the work would be finished in
have a crucial importance. e.g. while solving a problem 1 day.
if we take distance in meters, we should take speed in (2) Man-Hours
m/sec and time in seconds. If proper units are not
A certain number of men working for acertain
used and conversions not effected, then you’ll find
number of hours.
yourself in trouble.
Example: 3 men can finish a piece of work in 13 hours.
WORK & TIME This implies that the work is of 3 × 13 = 39 Man-
In most of the problems based on Work and Time, hours, i.e. if one man alone works to finish the given
either the amount of time taken to finish a given job work then he is going to take 39 hours to finish it and
or the amount of work done is to be calculated. if 39 men work together, then the work would be
Unless otherwise specified, the amount of work done finished in 1 hour.
is generally taken as unity (1). Also, if it is given that Some others are, Man-minutes, Machine-hours
a person X can finish a job in D days, then it implies etc.
that X alone can do the job in D days. Note: If A, B and C can finish a piece of work in X, Y
If a man can do a piece of work in N days (or hours or and Z days respectively, while working alone and
any other unit of time), then the work done by him in together they require M days to finish the work, then
one day will be 1/N of the total work. the amount of work done by A is M/X, B is M/Y & C is
M/Z.
3.8 Speed, Time and Work
Concept Of Work Equivalence (13) Three persons, staring at the same time and from
the same point along a circular path, will be
In order to solve certain types of questions, there is a together for the first time after the start at a
very logical method that can be used. time which is equal to LCM of the time taken by
This is the WORK EQUIVALENCE method which the fastest to gain a complete round over each
works on a simple premise-make the LHS equal to of the other two.
the RHS on the basis of SAME UNITS in terms of 1
(14) A overtakes B th of xth round means, when A
which work can be measured. n
Some Important Points: 1
has completed x – rounds, B has completed
(1) ‘A gives B a start of x meters’, implies that, if n
distance between starting point and finishing 1
point is L meters, then A covers L meters while (x – 1) – n round.
B covers L – x metres.
e.g., In a 100 metre race. A gives B a start of A s Speed
10 metres means, while A runs 100 metres, B Also,
Bs Speed
runs 100 –10 = 90 metres.
(2) ‘A beats B by x metres’, implies that, if distance Rounds completed by A in a given time
between starting point and finishing point is L =
Rounds completed by B in the same time
meters, then A wins the race by covering L
metres, while B covers L – x metres only. e.g., A overtakes B in the middle of the 4th round
(3) ‘A gives B a start of t seconds’, implies that A implies, when A has completed 3 1 2 rounds, B
starts the race t seconds after B starts from the A s Speed 31
starting point. has completed 2 1 2 rounds. 2 = 12
Bs Speed 2 2
(4) ‘A beats B by t seconds’, implies that, A and B (15) If a can do a work in ‘a’ number of days, then in
start together from the starting point, but A th
reaches the finishing point t seconds before B one day 1 work is done.
finishes. a
Note: (iii) and (iv) both imply that B takes t seconds 1 th
more than A to finish the distance. Conversely, if a man does of a work in 1
a
(5) A beats B by ‘x’ metres or ‘t’ seconds means, B 1
runs ‘x’ metres in ‘t’ seconds. day, then he can complete the work in 1 = a
a
(6) Winner’s distance = Length of the race. days.
(7) Distance covered by loser = Winner’s distance (16) If A is ‘x’ time as good a workman as B, then he
– (Beat distance + Start distance)
1 th
(8) Time taken by winner = Time taken by loser will take of the time taken by B to do the
– (Beat time + Start time) x
same work.
Winner's Time Loser's time (17) If A and B can do a piece of work in ‘x’ and ‘y’
(9) =
Loser's Distance Winner's distance days respectively. then working together, they
will take
Beat time + Start time
= xy
Beat distance + Start distance days to finish the work and in one day..
x+y
(10) If a race ends in a dead heat, then beat time = 0 th
and beat distance = 0. x+ y
They finish part of the work.
(11) Two persons staring at the same time and from xy
the same point along a circular path will be (18) To compare the work done by different people,
together again for the first time. When faster first find the amount of work each can do in the
gains one complete round over the other, then same time.
time taken by faster person to complete one
round over the other (19) If number of men to do a job is changed in the
ratio a : b, then time required to do the work
Length of race course will be in the ratio b : a, assuming amount of
=
Relative Speed work done by each of them in the given time is
the same, or they are identical.
(12) Two persons, starting at the same time from the
same point along a circular path, will be together (20) If two men A and B together can finish a job in
again for the first time at the same starting point, ‘x’ days and if A working alone takes ‘a’ days
at a time which is LCM of the time taken by more than A and B working together and B
each to complete a round. working alone takes ‘b’ days more than A and B
working together, then x = ab .
Speed, Time and Work 3.9
(21) To do a piece of work, the number of men Just divide (2) by (1) to get the answer.
employed and the number of days required to
8C/200 = 6/3
do the work are in inverse proportion. Also, the
number of men employed and the hours worked 8C = 2 × 200
per day are in inverse proportion. C = 400/8 = 50 cows
Example: A group of soldiers can completely destroy
Hence, there were 50 cows in the second group.
an enemy bunker in 7 days. However 12 soldiers fell
ill. The remaining now can do the job in 10 days. Find PIPES AND CISTERNS
the original group strength. A pipe connected with a cistern is called an inlet, if it
Solution: Here, first of all, let us see how WORK fills the cistern.
can be defined. It is obvious that work can be measured A pipe connected with a cistern is called an outlet, if it
a “destruction of the enemy bunkers.” empties the cistern.
In the first case, let us say that there were S numbers filling or emptying a cistern can be considered as work
of soldiers in the group. So they had to work for done.
7 days for the work which we call W.
Some Important Points:
S×7=W …(1)
(1) If an inlet pipe fills a cistern in ‘a’ hours, then
Now 12 fell ill and the remaining did the work in
10 days. Hence the new equations 1th
part is emptied in 1 hour..
(S – 12) × 10 = W …(2) a
Just compare the two equations to get the answer. Similary, if an outlet pipe empties a cistern in
S × 7 = (S – 12) × 10
1th
‘a’ hours, then part is emptied in 1
7S = 10S – 120 a
120 = 3S hour.
(4) If an inlet pipe fills a cistern in ‘m’ hours and an Solution: The net part emptied in 1 hour
outlet pipe empties the cistern in ‘n’ hours, then
the net part filled in 1 hour when both the pipes 1 1 1
.
4 6 12
1 1 n–m
are opened is – i.e., and the
m n mn { Time required to empty the full tank is 12 hours.}
Two pipes A and B can fill a cistern in X and Y hours
mn
cistern will get filled in hours. respectively, while working alone. If both the pipes
n – m
are opened together, then the time taken to fill the
For the cistern to get filled, m < n. If m > n, the
XY
cistern will never get filled. cistern will be .
XY
In general,
Example: Two pipes A and B can fill a cistern in
Net part filled of a cistern = (Sum of work done
4 and 5 hours respectively, while working alone. If
by inlets) – (Sum of work done by outlets)
both the pipes are opened together, then find the time
(5) If an inlet pipe fills a cistern in ‘a’ minutes, takes taken to fill the cistern.
‘x’ minutes longer to fill the cistern due to a
Solution: Pipe A can fill the cistern in 4 hours and
leak in the cistern, then the time in which the
Pipe B can fill the cistern in 5 hours.
leak will empty the cistern is given by a
When the pipes are opened together, the time taken
a
1 + . 4 5 20
x to fill the cistern will be hrs.
45 9
(6) If two pipes A and B can fill a cistern in ‘x’ min-
Three pipes A, B and C can fill a cistern in X, Y and Z
utes and if A alone can fill it in ‘a’ minutes more
hours respectively, while working alone. If all the
than ‘x’ minutes and B alone can fill it in ‘b’ min-
three pipes are opened together, the time taken to
utes more than ‘x’ minutes, then x = ab .
X Y Z
Example: If an inlet pipe can fill a tank in 4 hours fill the cistern will be .
XY YZ ZX
and an outlet pipe empties the full tank in 5 hours,
then what is the net part filled in 1 hour when both Example: Three pipes A, B and C can fill a cistern in
the pipes are opened? 10, 12 and 15 hours respectively, while working alone.
If all the three pipes are opened together, then find
Solution: The net part filled in 1 hour when both the
the time taken to fill the cistern.
pipes are opened = (1/4) –(1/5) = (1/20).
Solution: The pipes can fill the cistern in 10, 12 and
So, time required to fill the tank = 1/(1/20) = 20 hrs.
15 hours respectively.
Example: An inlet pipe can fill the tank in 6 hours
When the three pipes are opened together, the time
and an outlet pipe can empty the tank in 4 hours.
taken to fill the cistern
When the tank is full and both the pipes are open,
find the new part emptied in 1 hour. 10 12 15
4hrs.
10 12 12 15 10 15
Speed, Time and Work 3.11
EXERCISE
(a) 100 m (b) 125 m
MCQ TYPE QUESTIONS (c) 150 m (d) 250 m
1. A barrel full of beer has 2 taps, one midway which 1
draws a litre in 6 minutes and the other at the 7. Everyday Sachin exercises for 2 hours. He
2
bottom which draws a litre in 4 minutes. The exercises by walking, jogging and running. His
lower tap is normally used after the level of beer speeds while walking, jogging and running in the
in the barrel is lower than midway. The capacity ratio 1 : 2 : 4. Which of the following statements
of the barrel is 36 litres. A new assistant opens are true?
the lower tap when the barrel is full and draws
out some beer. As a result the lower tap has been I. If he spends equal time on the three activities,
used 24 minutes before the usual time. For how then he runs twice the distance that he jogs.
long was the beer drawn out by the new assistant? II. If he runs at 8 km per hour and distance
(a) 10 mins. (b) 8 mins. covered by running and jogging together is
(c) 16 mins. (d) 32 mins. 12 km, then he walks a distance of 1 km in
2. Pipe A can fill a cistern in 36 minutes and B in 1 hour.
48 minutes. If both the pipes are opened together, III. If the time for which he walks, jogs and runs
when should pipe B be closed so that the cistern is in the ratio 2 : 2 : 1 and he covers a total
may be just full in 24 minutes? distance of 10 km, then his speeds while
(a) 8 mins. (b) 9 mins. walking, jogging and running are 2 km/hr,
(c) 12 mins. (d) 16 mins. 4 km/hr and 8 km/hr res-pectively.
3. Pipes P, Q and R are attached to a tank and each Codes:
can act as either an inlet or outlet pipe. Pipes P, (a) I and II (b) II and III
Q and R respectively take 8, 10 and 12 hours to
fill the empty tank or empty the full tank. In the (c) I and III (d) I, II and III
first hour, pipes P and R work as inlet and Q work 8. A monkey climbing up a greased pole ascends
as outlet. In the second hour, pipes P and Q work 10 metres and slips down 2 metres in alternate
as inlet and pipe R as outlet. In the third hour minutes. If the pole is 64 metres high, how long
pipes Q and R work as inlet and pipe P as outlet will it take him to reach the top?
and the process goes on like this. When will the
(a) 16 min
cistern be filled?
(a) In the 8th hour. (b) In the 9th hour. (b) 14 min and 48 sec
(c) In the 10th hour. (d) In the 11th hour. (c) 12 min
4. There are 2 inlet pipes and an outlet pipe. The (d) 14 min
efficiency of one of the inlet pipes is double than 9. A train travelling at 10 m/sec from A to B at
that of the other. Also, the efficiency of the outlet 7 a.m. meets a train leaving B at 7:20 a.m. and
pipe is half that of the lesser efficient inlet pipe. 1
The empty tank gets filled in 16 hours when all coming to A at a speed times faster than the
3
the pipes are opened. How many hours will be first train. If the distance from A to B is 68 km.
taken to fill the empty tank when the ‘lesser’ then, at what distance from A will the two trains
efficiency inlet pipe is plugged and the rest kept meet?
opened?
(a) 72 km (b) 36 km
1 2
(a) 33 hours (b) 26 hours (c) 60 km (d) 50 km
3 3
(c) 100 hours (d) 80 hours 10. Two trains A and B start from stations X and Y
towards each other. B leaves station Y half an
5. A train overtakes 2 persons walking at 2 km/hr
and 4 km/hr respectively in the same direction hour after train A leaves station X. Two hours
and completely passes them in 9 seconds and after train A has started, the distance between
10 seconds. Length of the train and its speed in 19
km/hr respectively are trains A and B is th of the distance between
30
(a) 75 m, 18 km/hr (b) 80 m, 21 km/hr stations X and Y. How much time would it take
(c) 60 m, 20 km/hr (d) 50 m, 22 km/hr each train (A and B) to cover the distance X to Y,
6. A policeman goes after a thief who has a 100 m if train A reaches half an hour later to its
start. The policeman runs 1 km. in 6 mins. and destination as compared to B?
the thief 1 km in 10 mins. How far did the thief (a) 8 hrs, 6 hrs (b) 5 hrs, 4 hrs
go before he was overtaken? (c) 10 hrs, 9 hrs (d) 9 hrs, 8 hrs
3.12 Speed, Time and Work
11. Two men, A and B, run a 4 km race on a circular without him. Meera has recently got a contract
1 to paint the walls and ceiling of a room of
course of km. If their speeds are in the ratio dimensions 10 ft by 15 ft and a height of 10 ft.
4
of 5 : 4, how often does the winner pass the other? She had promised to do the job within 9 days.
(a) Once (b) Twice The ______ number of days does she need to
(c) Thrice (d) Four times employ Ashok?
12. A and B start from the same point to run in 2. 3 pipes can fill a reservoir in 10, 15 and 20 hours
opposite directions round a circular path 550 yards respectively. If the three taps are opened one after
in length, A giving B a start of 100 yards. They another in the given order, with a certain fixed
pass each other when A has run 250 yards. Who time gap between them, the reservoir fills in
will come first to the starting point and at what 5 hours. The time gap will be _____ hr.
distance will they be apart? 3. Two pipes A and B can fill a tank in 8 hours. If
(a) A, 20 yards (b) B, 10 yards only pipe A is open, then it would take 4 hours
(c) B, 20 yards (d) A, 10 yards longer to fill the tank. The _______ hours it would
13. Two runners run in the same direction along a take if only pipe B is open.
circular track 2 km long. The faster runner Directions (Q.4 – 5): Refer to the data below and
overtakes the slower one every 30 minutes. What answer the questions that follow.
are their speeds, if faster one completes a round
one minute sooner than the other? The boiler tank in a chemical factory holds 105 litres.
5 tanks each having one-fifth the capacity of the boiler
(a) 24 km/hr and 20 km/hr
tank fill in ‘hard water’ at same rates in the boiler
(b) 44 km/hr and 28 km/hr
tank in 2 hours. The outlet of the two of smaller tanks
(c) 20 km/hr and 18 km/hr work as inlet pipes and other two work as outlet and
(d) 20 km/hr and 16 km/hr the fifth tank fill in the main ‘boiler’ at half its efficiency.
14. To do certain work alone ‘A’ takes 4 hrs, ‘B’ takes 4. The number of hours taken to fill in the main
5 hrs and ‘C’ takes 6 hrs. How long would A and B boiler when all the pipes are opened at once
together take to do a work which C can do in is ________
1
4 hrs.? 5. Only three pipes are working, two at their full
2
efficiency and the third one with half of its
(a) 2 hrs 10 mins. (b) 2 hrs 30 mins. efficiency and all three are acting as an inlet. If
(c) 1 hr 20 mins. (d) 1 hr 40 mins. all the three are opened alternately with the latter
15. A can do piece of work in 8 days which B and C pipe being first to be opened to fill the boiler such
can do in 12 and 16 days respectively. If they work that each smaller tank fills ‘hard water’ in the
on alternate days, then which of the following is main boiler for equal time, then number of hours
true? taken to fill the boiler to half its capacity is______
(a) If A started the work on the first day followed
6. A contract is to be completed in 46 days and 117
by B and c, then the work will be finished on
men were set to work, each working 8 hours a
the 10th day.
day. After 33 days, 4/7 of the work is completed.
(b) If B started the work on the first day followed
by C and A, then the work will be finished on The _______ number of additional men may be
the 11th day. employed so that the work may be completed in
(c) If C started the work on the first day followed time, each man now working 9 hours a day?
by A and B, then the work will be finished on 7. A train travelling at 42 km/hr. passes a cyclist
the 11th day. going in the same direction in 9 secs.; if cyclist
(d) None of these had been going in the opposite direction, the train
would have passed him in 5 secs. Length of the
NUMERICAL TYPE QUESTIONS
train is______ metres
1. Meera paints walls for a living. She sometimes
8. A man rows upstream 13 km. and downstream
has Ashok to work with her. Meera takes 2 days
28 km. taking 5 hrs. each time. Velocity of the
to paint a 10 ft by 10 ft wall. Ashok takes 3 days
current in km/hr_______ is
to do the same job. Meera has to pay Ashok for
his work on a per day basis, so she’d rather do
Speed, Time and Work 3.13
Directions (Q. 9 – 11): Refer to the data below and Vijay bought the new car Palio and filled exactly
answer the questions that follow. 10 litres in his car. He goes at a speed of 40 km/hr for
The variation in the speed of a car on a particular first 20 km and then at a speed of 60 km/hr for next
60 km and driving at a constant speed reaches his
day at the respective times is shown in the table below:
destination which was 115 km away from his starting
s(km/hr) t(hr) point.
0 11.00 am 14. The time in hours taken by Vijay if he reaches
his destination in minimum possible time and
40 11.30 am
finishes all the fuel.
50 1.00 pm
15. In the above question, if Vijay had travelled at a
85 1.30 pm
constant average speed rather than changing his
10 3.30 pm speed, taking the same minimum time, then the
10 4.30 pm approximate saving in the fuel consumed is ______
9. The distance travelled by the car from 11 a.m. to litres
1 p.m. is _______km 1
10. If car maintains the speed it has at 4.30 p.m. then 16. In a 2 km race on a circular course of of a km.
4
th
at _______ time will the car cover the same A overlaps B in the middle of his 6 round.
distance as it had covered from 11.00 a.m. to Then_____ distance (in km) will A win at the same
1.00 p.m.? rate of running
11. Average speed of the car from 11.00 a.m. to 3
17. A runs 1 times faster than B. A gives B a start
4.30 p.m. is________ kmph 8
of 120 metres. _______ metre must the winning
12. A cheetah chase a deer which is 100 m ahead.
post be so that it may be a dead heat.
The time in which the deer takes 10 leaps the
cheetah takes only 6 leaps. In one leap, the deer 18. The work done by a man, a woman and a child is
covers 1 m while the cheetah covers 2 m. In in the ratio of 3 : 2 : 1. There are 20 men,
______ leaps would the cheetah catch up the deer. 30 women and 36 children in a factory. Their
weekly wages amount of ` 780, which is divided
13. A hare pursued by a hound who is 50 of her own
in the ratio of work done by the men, women and
leaps before him. When the hare takes 4 leaps,
children. The wages of 15 men, 21 women and
the hound takes 3. In one leap, the hare goes
3 3 30 children for 2 weeks is _______
1 metres and the hound 2 metres. In how 19. A can do a work in 6 days and B in
4 4
_______ leaps will the hound overtake the hare. 8 days. With the help of a boy, the three complete
the work in 3 days and get ` 200. The boy’s share
Directions (Q. 14 – 15): Refer to the graph below in rupees, if the money is distributed in the ratio
and answer the questions that follow.
of the work done is _______
The engine of the new car Palio brought by Fiat in
the market follows certain mileage characteristics as 20. A can finish a job in 12 days and B when working
shown in the diagram below: at twice his efficiency finishes a job in 9 days.
The ______ number of days will they take if they
15
work for two days alternately working at their
14 standard rate given that A starts first.
13
12
M i le a g e k m /l it r e
11
10
9
8
7
6
5
40 45 60 80
S p e e d k m /h r
3.14 Speed, Time and Work
ANSWERS
MCQ Type Questions
1. (c) 2. (d) 3. (c) 4. (b) 5. (d) 6. (c) 7. (c) 8. (b) 9. (b) 10. (c)
11. (c) 12. (d) 13. (a) 14. (d) 15. (c)
1
1. 6 2. 3. 16 4. 20 5. 6 6. 81 7. 75 8. 1.5 9. 77.5
2
1 4
10. 12.15 a.m. 11. 39.3 12. 300 13. 210 14. 2 15. 2 16.
3 11
17. 440 18. 1170 19. 25 20. 14
EXPLANATIONS
MCQ TYPE QUESTIONS 1 1 1 15 12 10 37
= = =
1. Top tap is operational till 18 litres is drawn out. 8 10 12 120 120
Time after which the lower tap is usually open 120 3 27
Hence they will take = 9 hours.
= 18 6 = 108 minutes. 37 37
Time after which it is open now Thus, tank will be filled in 10th hour.
= 108 – 24 = 84 minutes. 4. Let number of hours taken by the outlet and the
84 2 inlet pipes be 4x, 2x and x respectively. So in 1
Litres drawn = = 14 litres hour the part of empty tank filled is
6
18 – 14 = 4 litres 1 1 1 1
were drawn by the new assistant. =
x 2x 4x 16
Time = 4 4 = 16 minutes. 5 1
Alternatively =
4x 16
Upper tap was not used for 24 minutes. x = 20
24 Hence, inlet pipe with higher efficiency fills the
Litres drawn = = 4 litres. tank in 20 hours.
6
Beer was drawn for 4 4 = 16 minutes. So in one hour, the desired pipes fill
2. Let pipe B be turned-off after x minutes. 1 1 3
= th of the tank.
x 4x 4x
1 1 7x
Part filled in x minutes = x
36 48
= . Tank gets filled in
144
4x 80 2
24 x = hours = 26 hours.
Part filled in 24 – x minutes = 3 3 3
36
5. In each case, train has to travel its own length,
7x 24 x to pass each man.
=1
144 36 If x km/hr is speed of the train, then
7x 96 4x Length = Relative speed Time
=1
144 9 10
x = 16 minutes = (x – 2) = (x 4)
60 60 60 60
3. In a cycle of 3 hours : pipes P, Q and R are working 9x – 18 = 10x – 40
as inlet pipes for 2 hours each and they are
working as outlet pipes for an hour each. x = 22 km/hr.
So part of tank filled in 3 hours. 9
Length = (22 – 2)
1 1 1 1 1 1 3600
= 2x
8 10 12 8 10 12
Speed, Time and Work 3.15
20 9
Thus, III is true.
= km = 50 m. 8. In 1 minute, monkey ascends 10 metres but he
3600
takes 1 minute to slip down 2 metres. Thus, at
1 1 4 the end of 2 minutes, net ascending of the monkey
6. Policeman gains = = km/min is =10 – 2 = 8 metres.
6 10 60
Thus, to have a net ascending of 8 metres, process
2 of ascending and then slipping happens once. So,
= 66 m/min.
3 64
to cover 64 metres, above process is repeated
Hence to gain100 m, time required 8
or 8 times.
100 1 It is clear that in 8 such happenings, the monkey
= = 1 min.
2 2 will slip 7 times, because 8th time, he will ascend
66
3 to the top. Thus, in climbing 7 times and slipping
Hence thief has gone ahead by 7 times, he covers (7 8) or 56 metres.
Time taken to cover 56 metres
1 1
1 1000 metres = 150 m. 56 2
2 10 = = 14 minutes
7. I. Ratio of speed = 1 : 2 : 4 8
If equal time is spent on each of these, then Remaining distance = 64 – 56 = 8 metres
ratio of distance = 1 : 2 : 4 Time taken to ascend 8 metres
So, Sachin runs twice the distance that he jogs. 8 4
= min = min.
Thus, I is true. 10 5
II. He runs at 8 km/hr and therefore walks at 2 4
km/hr. Total time taken = 14 minutes + min
5
Hence II is not true. = 14 mins. 48 sec.
III.Ratio of speed= 1 : 2 : 4 9. If they meet t hrs after 7 a.m,
Ratio of time = 2 : 2 : 1
18 4 1
Ratio of distance 10 t + 10 18 t – = 68
=21:22:41=2: 4: 4 5 5 3 3
If Sachin covers a total distance of 10 km, then 36t + 48t – 16 = 68
he t = 1 hr.
2 They meet at a distance of
Walks for 10 = 2 km,
244 10 60 60 = 36000 m
4 i.e., 36 km from A.
Joggs for 10 = 4 km.
244 10. Two equations are
4 1 1
Runs for 10 = 4 km. T 1 – T2 = + =1 ...(i)
244 2 2
1 5 where T1 and T2 are time taken by trains A and B
Total time taken is 2 hours = hours.
2 2 to cover the whole distance
2 5 2 1.5 11
Thus, he walks for = 1 hour.. and + = ...(ii)
2 21 2 T1 T2 30
2 5
jogs for = 1 hour.. Solving equations (i) and (ii) ,we get
221 2
T1 = 10 hrs. and T2 = 9 hrs.
1 5 1
runs for = hour.. 11. When A runs 5 rounds, B runs 4 rounds (ratio of
221 2 2 speeds)
So speed while, A passes B each time A has run 5 rounds or
2 1 5 1
Walking = = 2 km/hr 5 = km. = 1 km.
1 4 4 4
4 1
Jogging = = 4 km/hr 1 km is contained in 4 km.
1 4
4 3 times.
Running = = 8 km/hr.. HenceA passes B thrice.
1
2
3.16 Speed, Time and Work
12. When A has run 250 yards, B should have run Ratio of time taken
550 – 250 = 300 yards. 1 9
= : = 10 : 27
B runs 300 – 100 = 200 yards in the same time 6 20
taken by A to run 250 yards. If C takes 27 hrs. (A + B) will take 10 hrs.
Ratio of the speeds of A and B If C takes 4 1 2 hrs. (A + B) will take
= 250 : 200 = 5 : 4
10 9
If speed of A is 5x yards per minute, then speed 1 hr. 40 mins.
at B = 4x yards/min. 2 27
Time taken by B to reach the starting point 15. I. Work done by A, B and C together in 3 days
450 112.5 1 1 1 13
= = min. + + = th of the work.
4x x 8 12 16 48
Time taken by A to reach the starting point Thus, in 9 days, total work done will be
550 110 13 39
= = min. 3 = th of the work
5x x 48 48
2.5 9
A wins by min Work remaining = 1 –
39
= th of the
x 48 48
2.5 6
Distance travelled by B in min. work is left of which A will finish th of
x 48
2.5 work on 10th day and B will finish
= (4x) = 10 yards remaining work on 11th day.
x Thus, I is false
Hence B is 10 yards behind A. 39
13. Let n1 and n2 be the speeds in km/hr and let n1 be II. Work done by A, B, C together in 9 days =
48
the greater speed i.e., n1 > n2. 39
Work left = 1 –
2 2 1 48
= + .
n2 n1 60 9
= th of the whole.
n1n2 = 120(n1 – n2) 48
Faster runner covers (n1 – n2) km. more than the 1
On the 10th day B will finish th of the
slower runner in 1 hour. 12
Hence to cover one round of 2 km, he will take whole.
2 1 9 1
or hour.. Work left = –
n1 – n 2 2 48 12
2 1 5
= = th of the whole.
n1 – n 2 2 48
n1 – n2 = 4
1
n1 = 4 + n2 On 11th day C will finish th of the work
16
n2 (4 + n2) = 120 4
5 1
Work left = –
n 22 + 4n2 – 480 = 0 48 16
n2 = 20 or – 24 (rejected) 2
= th of the whole
n1 = 4 + n2 = 24 km/hr 48
Hence speeds are 24 km/hr and 20 km/hr which will be done by A on the 12th day. Thus,
Alternatively work will be finished on the 12th day. Thus, II
Options may also be substituted to find the is false.
result. 39
14. Work done by A and B in 1 hr III. Work done by A, B and C in 9 days =
48
1 1 9 th Work left after 9 days
+ =
= part.
4 5 20 39 9
Ratio of work done by (A + B) =1– = th of the whole.
48 48
9 1
and C= :
20 6
Speed, Time and Work 3.17
13 216.25
8. =5 Average speed = = 39.3 kmph.
Speed upstream 5.5
28 12. While deer makes 10 leaps, cheetah makes 6 leaps.
and =5
Speed downstream Distance covered in 10 leaps by deer
= 10 1 = 10m
13
Speed upstream = km/hr.. Distance covered in 6 leaps by cheetah
5
= 6 2 = 12 m
28 Hence in six leaps, cheetah gains
and Speed downstream = km/hr..
5 12 – 10 = 2 m over the deer.
1 28 13 Cheetah has to gain 100 m over the deer.
Speed of river =
2 5 5 So, total leaps required by cheetah
6
1 15
=
= 1.5 km/hr = 100 = 300.
2 5 2
3 175
9. Graph of speed vs time is plotted as shown. 13. 50 leaps of the hare 50 1 = = 87.5 m.
Area under the graph and time axis gives the 4 2
distance. 175
So, hound should gain m over the hare.
90
2
80 C 3
70 When hound travels 3 2 m, then
60
Speed 50 B
4
km/hr 40 A X Y 3
30
hare travels 4 1 m.
20 4
10 Z D E
P Q
Hence in 3 leaps of the hound the hound gains
0 O R S T
11.00 11.30 1.00 1.30 3.30 4.30 33 28 5
am am pm pm pm pm – = metres
4 4 4
Required distance
= A( OAP) + A( BXA) + A ( APQX) 175
1 1 1 3 3 Number of leaps required = 2 3
= 40 + 10 + 40 5
2 2 2 2 2
4
30
= 10 + + 60 = 77.5 km. 175 4
4 = 3 = 210 leaps.
2 5
10. Distance covered are the same.
Speed in the latter is maintained at 10 kmph. 20
14. Fuel required for first 20 km = = 2.5 litres.
77.5 = 10 x 8
x = 7.75 hours. 60
Fuel required for next 60 km = = 4 litres.
Hence, clock time will be 4.30 + 7 hours 45 mins 15
= 12.15 a.m. Petrol left = 10 – 6.5 = 3.5 litres.
Now, he has to travel = 115 – (60 + 20)
total distance travelled
11. Average speed = = 35 km in 3.5 litres.
total time taken
So he can go at a speed of either 45 km/hr or
Distance travelled from 1.00 pm to 4.30 p.m.
80 km/hr as mileage is 10 km/litre in both cases.
= A(BQRY) + A(BYC) + A(CZD) But since he reaches in minimum time, he travels
+ A(ZRTE) at 80 km/hr
1 1 1 1
= 50 + 35 + (2) 75 20 60 35 155
2 2 2 2 Time taken= + + =
40 60 80 80
+ 3 10
50 35 75
= + + 75 + 30 =1+ hours
2 4 80
= 130 + 8.75 = 1 hours 56 minutes and 25 seconds.
= 138.75 km. 15. Total distance travelled = 115 km
Total distance travelled = 77.5 + 138.75 155
Total time taken = hours
= 216.25 km. 80
Speed, Time and Work 3.19
3 4 4
3:4 and 4 : 5 = . = 528 × = 704
4 5 3
The number of boys to be decreased
3 3 5 15
= 1170 – 704 = 466.
4 4 5 20
Example:
4 4 4 16
If a : b = 3 : 4 and b : c = 6 : 13, then find a : b : c.
5 5 4 20
Solution:
16 15 4 3
Since or, , The best way to solve such questions is to make b
20 20 5 4 common in the two ratios.
So, 4 : 5 > 3 : 4.
Thus, we can write a : b = 9 : 12 and b : c = 12 : 26.
Note: A ratio is said to be in its simplest form if the Now that b is equal in both the ratios, we can write
HCF of the antecedent and the consequent is 1. the same as
Example: a:b:c
Divide 2400 in the ratio 3 : 5. 9 : 12
Solution: 12 : 26
The first part is 3 units and the second part is 5 units. Thus, we can write a : b : c = 9 : 12 : 26.
The total of both the parts = 3 units + 5 units = 8
Using formula directly, we can get
units.
a : b : c = (3 × 6) : (4 × 6) : (4 × 13) = 9 : 12 : 26
Ratio, Proportion and Mixtures 4.3
PROPORTION
9a 2 b x
When ratio of two terms is equal to the ratio of two 3
x 2 9a 2 b 25b3
x 25b
other terms, then these four terms are said to be in
proportion, i.e., if a : b = c : d, then a, b, c and d are
in proportion. x 15ab2
a, b, c and d are called first, second, third and fourth VARIATION
proportionals respectively. The terms a and d are Most of us would still remember statements like “The
called extremes, while b and c are called means. distance travelled varies directly as the speed of the
If a : b = c : d, then ad = bc i.e., product of extremes is vehicle.” These and similar statements have precise
equal to product of means. mathematical meanings and they represent a specific
Continued Proportion : type of function called variation functions.
a b The three general types of variation functions are
When = , then a, b and c are said to be in
b c direct variation, inverse variation and joint variation.
continued proportion and b is called geometric mean
Direct Variation
or mean proportional between a and c.
If two quantities X & Y are related such that any
Also, b2 = a c, b= ac increase or decrease in ‘Y’ produces a proportionate
a b c d increase or decrease in ‘X’ or vice versa, then the
If = ....., then a, b, c, d, e … are said to
b c d e two quantities are said to be in direct proportion.
be in continued proportion. X is directly proportional to Y is written as X Y or
Concept of continued proportion is useful in the X = KY.
following situation.
X
(i) If four quantities a, b, c and d and a : b, b : c and In other words X : Y = = K. Here K is a constant
Y
c : d, are know, then we can find a : d as
whose value for a particular variation is same.
a a b c
Consider X1 = KY1 and X2 = KY2, dividing the two we
d b c d
(ii) If a, b, c and d are four quantities and X1 Y1
get .
a : b, b : c, c : d are known, then we can find X2 Y2
a : b : c : d. Thus, the chances of your success in the test are
e.g., If a : b = 2 : 3; b : c = 4 : 5, c : d = 7 : 9, directly proportional to the number of hours of sincere
find a : b : c : d. work devoted every day.
a : b = 8 : 12, b : c = 12 : 15 (LCM of 3 and 4 is 12) Example:
a : b : c = 8 : 12 : 15 If X Y and x = 9 when y = 30, then find the relation
a : b : c = 56 : 84 : 105, c : d = 105 : 135 (LCM of 7 1
between x and y. Find x when y = 7 and y when
and 15 is 105) 2
a : b : c : d = 56 : 84 : 105 : 135 x = 6.
Example: Solution:
Find the fourth proportional to 12a , 9a b and 6ab .
2 2 2
3 3
Let x = ky, then 9 = k(30), k = , i.e. x = y.
Solution: 10 10
Let x be the fourth proportional, then 1 3 1 1
When y = 7 ,x= 7 2 .
2 10 2 4
12a 2 6ab2 6ab 2 9a 2 b 9 3
x ab 3
9a 2 b x 12a 2 2 When x = 6, x = y y = 20.
10
Example: Example:
Find the mean proportional between 9a b and 25b . 2 3
Different sizes of the car have different models. The
Solution: weight of a car model varies directly as the cube of
its length. The weight of a car model of length 3 cm
Let x be the mean proportional.
is 10 gm. What is the weight of a car model of length
12 cm?
4.4 Ratio, Proportion and Mixtures
Solution: Solution:
Let W gm be the weight of a car model and L cm be
As a bc 2 or a = kbc2 (where k is a constant)
its length.
770 k(15)(7) 2
W L3 or W = kL3 (where k is a constant)
22 22 2
10 10 3 k i.e. a bc . When c = 3, a = 132.
10 k(3)3 . k i.e. W = L 21 21
27 27
22
10 132 (b)(3)2 b 14
When L = 12, W = (12)3 = 640 gm. 21
27
Note:
The required weight is 640 gm.
1. If A B and B C, then A C
Inverse Variation
2. If A C and B C, then (A B) C (AB) C
Here two quantities X & Y are related such that, any
increase in X would lead to a decrease in Y or any A A
decrease in X would lead to an increase in Y. Thus 3. If A BC, then B and C.
C B
the quantities X & Y are said to be inversely related
4. If A B and C D, then AC BD .
1
and X is inversely proportional to Y is written as X 5. If A B, then An Bn.
Y
6. If A B and A C, then A (B – C) and A (B + C)
k
or X = or XY = k (Constant) 7. If A B, then AP BP where P is any quantity,
Y constant or variable.
Thus, X1Y1 = X2Y2 MIXTURES AND ALLIGATIONS
Or the product of two quantities remains constant. Simple Mixture : When two different ingredients are
Thus, the chances that you will be able to cheat in a mixed together, it is called simple mixture.
test are inversely proportional to the smartness of Compound Mixture : When two or more simple
the invigilator. mixtures are mixed together to form another mixture,
Example: it is called compound mixture.
If y varies inversely as x, and y = 3 when x = 2, then Alligation Rule
find x when y = 21. Alligation rule states that “When different quantities
Solution: of the same or different ingredients, of different costs
(one cheap and other dear) are mixed together to
1 k
y or y (where k is a constant) produce a mixture of a mean cost, then ratio of their
x x quantities is inversely proportional to the difference
k 6 in their cost from the mean cost.”
then 3 = , k = 6 i.e. y = Quantity of Cheap
2 x
Quantity of Dear
6 2
When y = 21, 21 = ,x= Price of Dear Mean Price
x 7 =
Joint Variation Mean Price Price of Cheap
(a) A varies jointly as B and C and is denoted by Application of Alligation Rule :
A BC Or A = kBC (where k is a constant). (1) To find mean value of a mixture when prices of
(b) A varies directly as B and inversely as C and is two or more ingredients, which are mixed
B kB together and the proportion in which they are
denoted by A Or A = (where k is a mixed are given.
C C
(2) To find proportion in which ingredients at given
constant).
prices must be mixed to produce a mixture at a
(c) If A varies as B when C is constant, and if A varies given price.
as C when B is constant then A varies as BC when
B and C both vary. Some Important Results :
(1) When x1 quantity of ingredient A of cost C1
A BC Or A = kBC (where k is a constant).
and x2 quantity of ingredient B of cost C2 are
Example: Given, a varies as b when c is constant,
and as c2 when b is constant. If a = 770, then b = 15 & mixed, the,cost of the mixture,
c = 7,and when c = 3 & a = 132 find b. C1x1 C2 x 2
Cm=
x1 x2
Ratio, Proportion and Mixtures 4.5
(4) When a mixture of three ingredients A, B and C parts of the mixture). As we can see, the mixture 1 is
strong in milk as the dearer mixture, mixture 2 as
is given, take any two ingredients such that cost
the cheaper mixture and the final mixture as the mean
of the mixture is between costs of the two chosen
mixture.
ones and find the ratio. Once again, take two
more ingredients and find their ratio. Then find 5 3
combined ratio. This will give infinite number of Quantity of cheaper 6 4 1
solutions. Quantity of dearer 3 2 1
(5) If a vessel contains ‘a' litres of liquid A, and ‘b' 4 3
litres be withdrawn and replaced by liquid B, then Hence, both the types of mixtures will have to be mixed
if ‘b' litres of mixture be withdrawn and replaced in the ratio of 1:1.
by liquid B, and operation repeated ‘n' times in Two mixtures of same ingredients mixed (Compound
all, then mixture)
Mixture 1 has ingredients (A and B) in a : b.
Liquid A left after n th operation
Initial quantity of liquid A in vessel Mixture 2 has same ingredients (A and B) in x: y.
n quantity of ingredient A X
a b .
quantity of ingredient B Y
a
=
a b
n
Now ‘M’ unit of Mixture 1 and ‘N’ units of Mixture 2
1
a are mix ed to for m a r esultant mixtu re w ith
ingredients (A and B) in the ratio qA:qB
Liquid A left after n th operation
Case 1:
Liquid B left after n th operation
When qA and qB are to be found out. By alligation rule
a b
n
in the resultant mixture,
a Quantity of ingredient A q A
= n
a b Quantity of ingredient B q B
1
a
Example: In what ratio should tea @ 35 per kg be a x
mixed with tea @27 per kg so that mixture may cost M N
ab x y
Rs .30 per kg?
b y
M N 8
a b xy
4.6 Ratio, Proportion and Mixtures
Then, amount of ingredient A in the resultant mixture Q1
(M N)
qA (Q1 Q 2 )
(M N)
(q A q B ) Amount of 2nd mixture in the resultant mixture
Amount of ingredient B in the resultant mixture
Q2
qA (M N)
(M N) (Q1 Q 2 )
(q A q B )
Example: A tea merchant buys two kinds of tea, the
Case 2: price of the first kind being twice that of the second.
When M and N are to be found out (i.e. amount of He sells the mixture at Rs.14/kg there by making a
mixtures) profit of 40%. If the ratio of the first to second kind of
tea in the mixture is 2 : 3, then find the cost price of
Consider the quantity of any ingredients (Say, A) in
each kind of tea.
all the three mixtures, i.e. quantity of ingredients a
in Solution:
a 14 100
Mix 1 = , i.e a out of (a + b) The cost of mixture = Rs.10kg.
ab 140
Ratio in which the cheaper and dearer is mixed = 3:2.
x qa
Mix 2 = , in the resultant mix = Let the price of cheaper tea be Rs. x/kg and dearer
xy q A qB tea be rs.2x/kg.
By alligation rule we get , in the resultant mixture, Applying the alligation rule, we get
x qA 3 2x 10
x y q q 2 10 x
Quantity of 1st mixture Q A b
1 30 – 3x = 4x – 20
Quantity of 2nd mixture Q2 q A a
7c = 50
q A qB a b
1 2
Then, amount of 1st mixture in the resultant mixture x and cost of dearer tea = Rs. 14
7 7
EXERCISE
MCQ TYPE QUESTIONS How much alcohol was originally poured from the
first vessel into the second?
1. A cask of wine when fully filled holds 10 litres.
2 litres of wine is removed and filled with water. (a) 9 litres
Then 4 litres in the solution is replaced with (b) 10 litres
water. Then, 6 and 8 litres respectively. At the (c) 12 litres
end of the 4th operation, the ratio of wine to water (d) 12.5 litres
is
3. What is the ratio compounded of x – y : x + y and
4! 8! y2 + xy : x2 – xy?
(a) (b)
(5)4 (10)4 (a) y2 : x2 (b) xy : 1
(c) y : x (d) x: y
4! 8!
(c) 4 (d)
5 4! (10)4 8! 4. If ratio of two natural numbers x and y is ‘a' and
that of y and x is ‘b', then value of ‘a + b' is
2. A 20 litre vessel is filled with alcohol. Some of the
(a) greater than 2 (b) lesser than 1
alcohol is poured out into another vessel of an
equal capacity, which is then completely filled by (c) greater than 1 (d) data insufficient.
adding water. Then mixture obtained is poured 5. The charges of a goldsmith is partly fixed and
2 partly variable with the amount in grams of gold.
into the first vessel to capacity. Then 6 litres
3 The charge is ` 300 for 20 grams of gold and
is poured from the first vessel into second. Both ` 550 for 45 grams of gold. Find the charge of
vessels now contain an equal amount of alcohol. work for 100 grams of gold.
Ratio, Proportion and Mixtures 4.7
(a) ` 900 (b) ` 1000 transferred to the second and first bag respectively.
(c) ` 1100 (d) ` 9000 The ratio of mangoes in bag 1 to bag 2 now is
Directions (Q.6–7) : Refer to the data below and (a) 2 : 1 (b) 2 : 2
answer the questions that follow. (c) 4 : 5 (d) 5 : 4
The ratio of white collar to blue collar employees in a NUMERICAL TYPE QUESTIONS
firm is 8 : 3. Also ratio of male employees to female
1. Five litres are drawn from a cask full of wine and
employees is 7 : 4. It is observed that 60% of the white
it is then filled with water. Five litres of the
collar employees are males.
mixture are drawn and the cask is again filled
6. What is the ratio of female white collar employees with water. The quantity of wine now left in the
to male blue collar employees? cask to that of the water in it is in the ratio
(a) 16 : 11 (b) 24 : 11 361 : 39. The cask hold ______ litres.
(c) 3 : 1 (d) Data insufficient 2. In a mixture of wheat and barley, wheat is 60%.
7. If there are 48 female blue collar workers, what To 400 kg of the mixture a quantity of barley is
is the difference between male white collar 1
added and then the wheat is 53 %. The_____kg
employees and female blue collar employees? 3
of barley are added?
(a) 288 (b) 240
3. Two jars of capacity 3 and 5 litres are filled with
(c) 220 (d) Data insufficient
mixtures of alcohol and water. In the smaller jar
8. In a chemical experiment, two NaOH solution 25% of the mixture is alcohol and in the larger
bottles are used. Bottle A contains salt and water 25% of the mixture is water. The jars are emptied
in the ratio 7 : 3 and bottle B contains salt and into a 9 litre cask and remaining volume is filled
water in the ratio 4 : 3. In what proportion should up with water. Then percentage of alcohol in the
the quantities be taken from A and B to give the cask will be ________
2 : 1 NaOH solution?
4. A man buys milk at ` 5 a litre and after adding
(a) 2 : 1 (b) 10 : 7 water, sells it at ` 6 a litre, thereby making a
(c) 20 : 7 (d) 1 : 2 1
profit of 33 %. The proportion of water to milk
9. A triswitch mixer grinder operates in three stages 3
of speed in revolutions per min i.e., rpm. The in the mixture is_______
speed of the motor with some load in the second 5. The percentage of alcohol in two wine tanks is
stage varies as a square of the speed in stage 20 and 40 respectively and rest is grape juice. If a
1 and that in stage 3 varies as a cube of that in wine glass is filled to its half by the tank 1 and
stage 2. If mixer motor makes 50 revolutions in 2 in the ratio 2 : 3 and the rest is filled again by
3 minutes in stage 1, its speed in stage 2 and 3 pure alcohol, then the ratio of the grape juice to
with the same load is 2500 and 12500 rpm alcohol in the wine glass is _______
respectively. What will be the speed of motor (in 6. In ______ ratio must a grocer mix two varieties
rpm) in stage 3, if at the same load motors speed of tea worth ` 60 a kg and ` 65 a kg so that by
in rpm is 25 revolutions in 6 minutes in stage 1? selling the mixture at ` 68.20 a kg he may gain
3125 55 10%?
(a) rpm (b) rpm
7. A container contains 40 lit of milk. From this
16 210
container 4. lit of milk was taken out and replaced
510
(c) rpm (d) None of these by water. This process was repeated further two
25
times. The ______litres of milk is now contained
10. Two bags have certain number of mangoes. If
by the container?
half the mangoes from bag 1 are transferred to
8. A merchant has 1000kg of sugar,part of which he
bag 2, the ratio of mangoes in bag 1 to bag 2 gets
sells at 8% profit and the rest at 18% profit. He
reversed. Now one-third and two-third of the
gains 14% on the whole. The quantity sold at 18%
mangoes from the first and the second bag are
profit is_____ kg
4.8 Ratio, Proportion and Mixtures
9. Volumes in the proportion 3 : 4 : 7 of three different substances are mixed together. The densities of equal
volume of the substances are in the ratio 5 : 2 : 6 respectively. The weight of the third substance contained
in 130 lbs. of the mixture is _______ lbs
10. In a bag, there are 50 paise coins, 25 paise coins and rupee coins, proportional to the numbers
1
3, 4 and 2 . If they amount to ` 210, then number of rupee coins is______
2
ANSWERS
MCQ Type Questions
1. (c) 2. (b) 3. (c) 4. (c) 5. (c) 6. (a) 7. (b) 8. (c) 9. (b) 10. (a)
EXPLANATIONS
MCQ TYPE QUESTIONS 2 x
In C1 20 x (x) = 10
Wine left 3 20
1. Stage
Original quantity Solving we get, x = 10 litres
10 Hence10 litres of alcohol was initially poured from C1
0
10 to C2.
8
1 x y y 2 xy y
10 3. Required ratio = = =y:x
x y x 2 xy x
8 6
2
10 10 x y
4. Given : = a and =b
8 6 4 y x
3
10 10 10
4 3 21 4! x y
4
8
6
4
2
= 4
If 1 , then 1,
10 10 10 10 5 4
5 y x
4! x y
4 1.
Wine 5 4! y x
= = 4
Water 1 4
4! 5 4!
Hence option (c) is correct.
5
5. Total charge = fixed + variable
2.
T C = m + kx
where m and k are constants.
(1) (2)
300 = m + k 20
C1 C2 Also, 550 = m + k 45
Operation1 20 litres alcohol Empty Solving for m and k; we get
Operation 2 (20 – x) alcohol x litres alcohol m = 100; k = 10.
(20 – x) litres Hence, for 100 grams,
water
total charge = 100 + 10 100 = ` 1100.
Operation 3 (20 – x) litres (20 – x) litres Q. 6 and 7.
alcohol + xlitres mixture
Using given data, we can draw following table
mixture i.e.,
x2 Male Female
20 litres alcohol
White Collar 4.8x 3.2x 8x
1
Now rd of C1 poured into C2 Blue Collar 2.2x 0.8x 3x
3
Quantity of alcohol in C1 = C2 = 10 litres 7x 4x 11x
Ratio, Proportion and Mixtures 4.9
Now, all the questions can be answered. 10. Let bag 1 and 2 have
Female white collar employess 3.2x 16
6. = = x mangoes y mangoes
Male blue collar employees 2.2x 11
x x
7. Since 0.8x = 48 Step 1 y
2 2
x = 60 x x
Required difference = 4.8x – 0.8x But : y y : x
2 2
= 4.8 60 – 0.8x = 240 2 2 1 1
Step 2 y x y x
8. Let 10x be taken from bottle A and 7y be taken 3 3 3 3
from bottle B.
2 2 x y
7 4 y x 2
Total salt taken = 10x 7y 3 = 3
10 7 Required ratio = 3
1 1 x y
y x 1
= 7x + 4y 3 3 3
Total water taken = 3x + 3y. =2:1
7x 4y 2
= NUMERICAL TYPE QUESTIONS
3x 3y 1
7x + 4y = 6x + 6y 1. Let quantity of wine be a, i.e., capacity of the
cask be ‘a'
x = 2y
2
5
x
=
2 1
y 1 Quantity of wine a
i.e., = 2
Quantity of water 5
10x 1 1
Ratio of quantities = a
7y 2
5
10 2 20 1
361 a
= = =
7 1 7 2
39 5
1 1
9. Let speed of the motor in stage 1 be x rpm. Then a
its speed in stage 2 be (x2) = ax2 (say) and in stage 2
361 361 5
2
5
3 be (ax2)3 = b.a3x6 (say) 1 = 1
39 39 a a
50
Given that x= ...(i) 5 361 19
3 1 a = 400 = 20
ax2 = 2500 ...(ii)
5 1
ba3x6 = 12500 ...(iii) =
Substituting value of x in (ii), we get a 20
a = 100 litres
2500
a= 2
32 = 9 2. Quantity of wheat in 400 kg of mixture
(50) = 0.6 400 = 240 kg
Again, substituting values of a and x, we get
Quantity of barley = 400 – 240 = 160 kg
12500 1
b= 3
= 10 4 Let x kg of barley be added to 400 kg of the mixture
(2500) 125
25 1
53
Given : x=
6
rpm
240
= 3 = 160 = 8
160 x 46
2 140 7
10 4
6
25 3
Speed in stage 3 = (9)3
125 6
240 7 = 8 160 + 8x
4
10 (25 5) (5 25 ) 4
x = 50 kg
= (3)6
5 25 (2)6 (3) 6 3. Alcohol content in 1st jar = 0.25 3 = 0.75 litre
Alcohol content in 2nd jar = 0.75 5 = 3.75 litres
5 254 55
= = rpm Total alcohol in 9 litre cask = 0.75 + 3.75
26 104 210
= 4.5 litres
4.10 Ratio, Proportion and Mixtures
Percentage of alcohol in the cask 7. Amount of milk left after 3 operations
4.5 3
= 100 = 50%
= 40 1 –
4
40
9
4. Cost price of milk = ` 5/litre
100 9 9 9
Cost price of mixture = 6 = ` 4.5/litre = 40
1 10 10 10
133
3 = 29.16 lit
Quantity of water 5 4.5 1
= = 8. Profit on 1st part 8% Profit on 2nd part 18%
Quantity of milk 4.5 0 9
i.e., Milk Water Mean Profit 14%
` 5.0 `0
4 6
`. 4.5
Ratio of 1st and 2nd parts = 4 : 6 = 2 : 3.
` 0.5 3
Quantity of 2nd part = 1000 kg = 600 kg.
Milk : Water = 9 : 1 5
3 4 7
9. Ratio of volumes = : :
5. Alcohol Grape juice Alcohol : Grape 14 14 14
Tank 1 20% 80% 1:4 5 2 6
Ratio of densities = : :
Tank 2 40% 60% 2:3 13 13 13
Ratio of grape juice to alcohol in half filled wine 3 5 4 2 7 6
Ratio of weights = : :
glass 14 13 14 13 14 13
4 3 = 15 : 8 : 42
2 3 ... (Mass = Volume Density)
= 5 5 = 17
1 2 8
2 3 Weight of the third substance =
42
130 = 84 bs.
5 5 65
So, half capacity of glass = 17 + 8 = 25
10. Let number of 50p coins be 3x, number of 25p
So, full capacity = 25 2 = 50
1
Now rest 25 parts has to be filled with alcohol. coins be 4x and number of rupee coins be 2 x.
2
So, ratio of the grape juice to alcohol 1 1 1
2 x 3x 4x = 210
= 17 : (8 + 25) = 17 : 33. 2 2 4
Alternatively, 5 3
x x x = 210
As the wine glass is filled to half by taking two 2 2
parts from tank 1 and three parts from tank 2 5x = 210
and second half is equivalent to five parts which x = 42
is pure alcohol,
5
percentage concentration of grape juice Number of rupee coins = 42 = 105.
2
2 80 3 60 5 10
= = 34% 13
235 Number of goats = 416 × = 169
Hence ratio of grape juice to alcohol 32
= 34 : 66 = 17 : 33. Total number of hens = 65 + 65 = 130
6. S.P of 1 kg of mixture = ` 68.20, Gain =10% Total number of ducks = 91 + 182 = 273
Total number of goats = 169 + 169 = 338
100
S.P of 1 kg of mixture = ` 68.2 = ` 62. The new ratio of hens: ducks: goats = 130: 273: 338
110 = 10: 21:26.
Cost of 1 kg tea of Cost of 1 kg tea of
1st kind 2nd kind
60 65
Mean Price ` 62
3 2
Required ratio = 3 : 2.
5
CHAPTER
Permutations-Combinations,
Elementary Statistics and Probability
PERMUTATIONS-COMBINATION Number of ways of dividing 2p items into two equal
Permutations
2p !
Each of the arrangements which can be made by groups of p each is where two groups have
taking some or all of a number of items is called a p!2
Permutation. Permutation implies “arrangement” or distinct identity.
that “order of the items” is important.
Number of ways of dividing 2p items into two equal
Permutations of three items a, b and c taken two at a
time are ab, ba, ac, ca, cb and bc. Since the order in which
groups of p each is
2p ! where two groups do not
2! p!
items are taken is important, ab and ba are counted as 2
two different permutations. The words “permutation”
and “arrangement” are synonymous and can be used have distinct identity.
interchangeably.” Number of ways in which (p + q + r) things can be
Number of permutations of n things taking r at a time divided into three groups containing p, q and r things
is denoted by nPr (and read as “nPr")
Combinations respectively is
p q r !
p!q! r!
Each of the groups or selections which can be made
by taking some or all of a number of items is called Number of circular arrangements of n distinct items
Combination. In combinations, order in which the is (n – 1)! If there is a DIFFERENCE between
items are taken is not considered as long as the specific clockwise and anticlockwise arrangements and (n –
things are included. 1)!/2 if there is NO DIFFERENCE between clockwise
Combinations of three items a, b and c taken two at a and anticlockwise arrangements.
time are ab, bc and ca. Here, ab and ba are not For x1 + x2 + x3 + – – – – – – + xn = s where s 0,
considered separately because the order in which a and number of positive integral solutions (when s n) is
b are taken is not important but it is only required that s–1 n–1
C and number of non-negative integral solution
a combination including a and b is what is to be counted. is n + s – 1Cn – 1.
Words “combination” and “selection” are synonymous.
NOTES:
Number of combinations of n things taking r at a time
is denoted by nCr (and read as “nCr") The Number of Diagonals in an
N–Sided Regular Polygon
n n! n n!
Pr Cr An n-sided regular polygon has n vertices. Joining
n r ! , r! n r !
any two vertices we get a line of the polygon which
Number of ways in which n things may be arranged are nC2 lines, n of them are sides. Hence diagonals
taking them all at a time, when p of the things are n(n 3)
exactly alike of one kind, q of them exactly alike of are nC2 =
2
another kind, r of them exactly alike of a third kind,
The following points are useful in solving the
n!
and the rest all distinct is problems.
p!q!r!
1) nC0 = nCn = 1
Number of ways of selecting one or more items from 2) nC1 = nCn–1 = n
n given items = 2n – 1
3) nCr = nCn–r
Number of ways of dividing (p + q) items into two
4) if nCr = nCs then r = s r n = r + s
groups of p and q items respectively is
p q ! . 5) nCr + nCr–1 = n+1
Cr
p!q!
5.2 Permutations-Combinations, Elementary Statistics and Probability
Example: Solution :
Find the number of 4 –digit numbers that can be A pack of 52 cards contains 4 different suits. Number
formed using the digits 0.2, 5, 7, 9, 6 where a digit of ways of drawin g fou r cards each
can be repeated more than once. = 13C1.13C1.13C1.13C1 = 134
Solution: ELEMENTARY STATISTICS
Among the given digits 0, 2, 5, 7, 9, 6 zero cannot be If you find ‘elementary' statistical formulae
in thousands place 2, 5, 7, 9, 6 are to be used for the meaningless, or merely wish to refresh our
thousands place of a four digit number, which can be
understanding,
done in 5 ways.
The term ‘statistic' can refer to several rather different
Since the repetition of digits is allowed, for each of
things.
the hundreds, tens and the units place, all the given
digits can be used which can be done in 6(6)(6) (6) or A statistic summarizes or represents a set of
216 ways. The total number of 4 digit numbers information - most commonly as a single number.
that can be formed is 5(216) or 1080 The term statistic is used both for the value and for
the mathematical ‘function' (usually an equation) used
Example:
to obtain that value.
If nP4 = 7920, then find nC4
Many functions are available to summarize
Solution:
information. For example, a salesman could equally
n
P4 = n(n – 4)(n – 2)(n – 3) truthfully provide the most typical cost as ‘on average'
= 10(4)(198) or give the maximum ('up to...') or the minimum
= 8(9)(10)(11) (‘from...') just $ 300. The 'average', ‘maximum' and
'minimum' are all statistics.
n 11 n 1
n =11 P4 C4 P4 Note, summary statistics of a sample are often used
4! as estimates for the population at large - for instance
when you are told ‘the average man has 1.8 children'
1
7920 330 that result was found in a sample of men - it is usually
4!
impossible to check every man.
Example: Statistics are also used to describe how sets of results
In how many ways can the letters of the word varied, or to infer how they are liable to vary, or to
RECRUDESCENCE be arranged? infer how their summary statistic might be expected
Solution: to vary. Inferential statistics are variously used to
The given word contains 4E’s ,2R’s, 3C’s ,1U, 1D, indicate how reliable an outcome is, or the probability
1S,1N. Here the total numbers of letters =13 it occurred by simple chance - given a simple
(hopefully plausible) set of assumptions.
13!
Number of arrangements 1!2!3! When used in the plural, ‘statistics' also describes
‘the study of the collection, organization, analysis,
Example: interpretation and presentation of data'.
Find the number of ways in which six boys and five Humans, of course, use non-numerical summaries
girls can be seated in a row such that each girls has a all of the time. For example when you say ‘cats are
boy sitting on either side. smaller than dogs' you are probably describing the
Solution: average situation - however some people assume
There are 6 boys and 5 girls you mean every cat is smaller than every dog.
Possible seating arrangement : BGBGBGBGBGB Humans also use non-numerical estimates of
Total number of sitting arrangements = 5!6! probability, using a simple scale, ranging from
Example: impossible to certain. Research shows most people
divide that scale into surprisingly few levels - seldom
Find the number of ways of drawing four cards, all of
more than 7 - and have problems in dealing with very
different suits from pack of 52 playing cards.
small probabilities.
(Diamonds, spades, hearts and clubs are the suits in
a pack)
Permutations-Combinations, Elementary Statistics and Probability 5.3
Statistics is a mathematical science including methods where di = xi – a is deviation from the assumed mean.
of collecting, organizing and analyzing data in such a e.g., Find the Arithmetic mean of 42, 46, 48, 50, 55,
way that meaningful conclusions can be drawn from 60, 70
them. In general, its investigations and analyses fall Let assumed mean be 50
into two broad categories called descriptive and
inferential statistics. xi di
Descriptive statistics deals with the processing of data 42 42 – 50 = – 8
without attempting to draw any inferences from it. 46 46 – 50 = – 4
The data are presented in the form of tables and graphs.
The characteristics of the data are described in simple 48 48 – 50 = – 2
terms. Events that are dealt with include everyday 50 50 – 50 = 0
happenings such as accidents, prices of goods, business,
55 55 – 50 = 5
incomes, epidemics, sports data, population data.
Inferential statistics is a scientific discipline that uses 60 60 – 50 = 10
mathematical tools to make forecasts and projections 70 70 – 50 = 20
by analyzing the given data.
i1 di = 50 +
7
Measures of Central Tendency in Elementary 21
x = 50 + = 53.
Statistics 7 7
Average of a distribution is the value of the variable Weighted Arithmetic Mean: If values x1, x2, ...... xn
which is representative of the entire distribution. are assigned weights w1, w2, ...... wn respectively, then
i1 w i x i
The following are the measures of central tendency. n
x =a+
n
5.4 Permutations-Combinations, Elementary Statistics and Probability
Properties of Arithmetic Mean
(i) Algebraic sum of the deviations of a set of 1
H= ...(i)
1 1 n
values from their arithmetic mean is zero.
n i1 xi
f x
n n n
i 1
i i X = f
i1
i xi X fi
i 1
In case of a frequ ency distribution
xi fi i 1,2,....., n
1 n
= NX NX X fi xi = 0
N i1 1
H= ...(ii)
(ii) The sum of the squares of the deviations of a 1 n
fi
set of values is minimum when taken about
N i1 xi
mean.
e.g., Find Harmonic mean of 1, 0.5, 10.
(iii) Mean of the composite series: If X 1 , X 2 ,.....X K
are the mean of K series of sizes n1, n2,.....nK 1 1
respectively then the mean X of the
xi
xi x 3.1
i
composite series is given by 1 1 3 30
H.M. =
n1 X 1 n2 X 2 ..... nK X K 0.5 2 3.1 31
X= .
n1 n2 ..... nK 10 0.1
(ii) Geometric Mean: Geometric mean of a set of This is an important result and is generally
n observations is the nth root of their product. used to find average speed when equal
distances are covered at different speeds.
1 n
G anti log fi log xi
(when different distances are covered in the
N i1 same time, arithmetic mean is used to
determine the average speed).
e.g., Geometric mean of 2, 3 and 4 is 3
234 e.g., A person travels from A to B at 6 kmph
3
= 24 = 2. 3 3 and returns at 4 kmph, find his average speed.
Average Speed= Harmonic Mean of 6 and 4
Geometric mean of a and b is ab 2 6 4 48
e.g., Find Geometric mean of 9 and 4. = = 4.8 kmph
64 10
G.M. = 9 4 = 6 Note: This concept will again be covered in
Geometric mean is generally used to time, speed and distance.
calculate rate of growth. Weighted Harmonic Mean: Weighted
If geometric mean of one group of ‘a' numbers Harmonic Mean of 3 numbers x1, x2 and x3 with
i 1 w i
3
is ‘x' and that of another group of ‘b' numbers
is ‘y', then weights w1, w2 and w3 respectively is
3 w
Geometric mean of the combined groups i 1 x i
1 i
= (xa + yb) This is used to calculate average speed when
ab
different distances are covered at different
Propertie of Geometric Mean speeds.
If G1 and G2 are the geometric means of two e.g., A cyclist covers his first 5 km at an
series of sizes n1 and n2 respectively, then the average speed of 10 kmph, another 3 km at 8
geometric mean G of the combined series is kmph and last 2 km at 5 kmph. Find his
given by average speed during the journey.
Average Speed= Weighted HM
n1 log G1 n2 log G 2
log G 532
n1 n2 = = 7.84 kmph
5 3 2
(iii) Harmonic Mea n:Har monic mean of a 10 8 5
observations is the reciprocal of the AM of the (iv) Median and Other Partition Values
reciprocals of the given values. Thus, harmonic
Median of a distribution is the value of the
mean H of n non-zero obser vations
variable which divides it into equal parts.
x1, x2, ....., xn is
Permutations-Combinations, Elementary Statistics and Probability 5.5
Note: Example:
(i) Probability [P(E)] of the happening of an event When two coins are tossed together, find the
E is called probability of success and probability probability of getting exactly two tails?
Solution:
P E of non-happening of the event is called
Number of favorable outcomes = 1(TT)
probability of failure.
Total outcomes = 4(HH, HT, TH, TT)
(ii) If P(E) = 1, then event is called a certain event
and if P(E) = 0, then event is called impossible 1
Required probability
event. 4
(iii)Instead of saying that the chance of happening of Example: I f
an event is m/n, we can also say that odds in favour two dice are tossed, the probability that the sum is 8
of the event are m (to n – m). Similarly, if m/n is
is
probability of the occurrences of an event, since
(n – m)/n is probability of non-occurrence of the Solution:
event, we can also say that the odds against the Two dice are rolled; the number of possibilities is 36.
event are (n – m) to m.
The favorable cases are (2, 6), (6, 2), (3, 5), (5, 3),
Addition Theorem on Probability (4, 4)
If there are two sets A and B, we know that the
number of elements in A B is given by 5
Required probability
n(A B) = n(A) + n(B) – n(A B). 36
A similar relationship exists in probability theory Example:
(called Addition Theorem of Probability) as Suppose, seven coins are tossed then the probability
P(A B) = P(A) + P(B) – P(A B), where A and B are of getting atleast one head is
two events. Solution:
If A and B are mutually exclusive events (i.e, A and B Probablity of getting at least one head
are disjoint sets), then P (A B) = 0 = 1 – (probability of getting no heads).
If there are two mutually exclusive events A and B, 1 1 127
then P (A B) = P(A) + P(B). 1 1
7 128 128
2
Independent Events
Example: A and B play a game where each is asked
Two events, E1 and E2 are said to be independent, if
to selected a number from 1 to 8. If both of them
occurrence of the event E2 is not affected by the select the same number, then they win a prize. Find
occurrence or non-occurrence of the event E1. the probability that they will not win the prize.
If A and B are two independent events, then Solution:
P(A B) = P(A) P(B) The total number of ways of selecting numbers for
Example: both A and B is = 8(8) = 64
If dice is rolled, then find the probability of getting a They win the prize if they select same numbers.
number divisible by 2. The total possibilities are 8, i.e. (1, 1), (2, 2),
Solution: (3, 3), (4, 4), (5, 5), (6, 6), (7, 7), (8, 8)
When a dice is rolled n(S) = 6
Probablity of winning prizes 8 1
Favorable outcomes are {2, 4, 6} i.e, 3 64 8
Required probability = 3/6 = 1/2
Example: 1 7
The probability of not getting a prize 1 .
When a dice is rolled, what is the probability of 8 8
getting a composite number?
Solution:
Number of favorable outcomes = 2({4, 6})
Total outcomes = 6
2 1
Required probability
6 3
5.8 Permutations-Combinations, Elementary Statistics and Probability
EXERCISE
MCQ TYPE QUESTIONS 7. All of them are from different suits?
4 13
Directions (Q. 1 – 2) : 13 3
If A and B are two possible events of an experiment (a) 52 (b)
C3 52
such that P (A B) = 0.6 and P(A) = 0.3 then find C3
P (B) given that
1. A and B are mutually exclusive events
(c)
39
C3
(d)
134
7 4 52 52
(a) (b) C3 C3
10 10
8. Two of them are number cards from the same
3 1 suit and the remaining number card is from a
(c) (d)
10 2 different suit?
2. A and B are independent events 9
C 2 9 C1 9
C 2 27 C1
4 9 3 3
(a) (b) (c) (d) (a) 52 (b) 52
7 10 10 7 C C
3 3
(a)
4 13
C3 (b)
13
C3
3
(a) (5!)2 (b)
5!2
52 52 3!
C3 C3
5! 2
(c) 5! 3! 2! (d)
2
4 13 C3 13
C3 2!2 3!
(c) 52
(d) 52
C3 C3
Permutations-Combinations, Elementary Statistics and Probability 5.9
Directions (Q. 13 – 14) : 20. Two red pencils, three black pencils and two white
A man has 12 friends whom he wants to invite for lunch. pencils are to be arranged in a row such that
In how many ways can he invite 1. No two adjacent pencils are of the same colour
13. At least one of them? and
(a) 4096 (b) 4095 2. Pencils at the two ends of the row are of same
(c) 2047 (d) 2048 colour.
14. At least 10 of them? (a) 12 (b) 8 (c) 9 (d) 10
(a) 66 (b) 140
(c) 153 (d) None of these NUMERICAL TYPE QUESTIONS
15. In how many ways can 6 prizes be distributed
among 3 boys, if each boy is eligible to receive Directions (Q. 1 – 2) :
one or more prizes? The odds against an event are 4 to 5 and the odds in
(a) 36 (b) 63 favour of another independent event are 3 to 7. The
(c) 6C3 (d) 6P3 probability that
Directions (Q. 16 – 17) : 1. Exactly one of them will occur is _______
Find the number of ways of dividing 16 different books 2. Neither of them will occur is _______
equally
16. Among 4 boys
Directions (Q. 3 – 5) :
16! 16!
Three persons Shiva, Jagan and Rohit aim at a target.
(a)
4!4
(b) 4! 3
Their respective probabilities of hitting the target are
2/3, 5/7 and 3/8. The probability that
16!
3. None of them hits the target is _______
(c)
4!5 (d) (4!)4
4. At least two of them hit the target is _______
17. Into 4 parcels (Parcels are not distinct) 5. Exactly one of them hits the target si _______
16! 16!
6. If three consecutive letters are selected at random
(a)
4!
4
(b) 4! 3
from the English alphabet, then the probability
16! that at least one letter is a vowel is _______
(c)
4!5 (d) (4!)4 7. Eight unbiased coins are tossed together. The
probability that the number of heads is equal to
18. How many linear arrangements can be made the number of tails is _______
using all the letters of the word ILLUSTRATE
which begin with S and end with a vowel? 8. In a biased coin, Head occurs three times as
frequently as tail occurs. If the coin is tossed 3
8!
times, then the probability of getting two heads
(a)
2!
2 (b) 8!
is _______
8! Directions (Q. 9 – 10) :
8!
Two cards are drawn at random from a well shuffled
(c)
2!
(d)
2!3 pack of cards. Given that both are black, the
19. From a group of 10 professors and 6 assistant probability that the cards have
professors, a management institute desires to 9. Same honour on them is_______
send to delegation of 8 persons consisting of 5 10. Different honours and belong to different suits
professors and 3 assistant professors to the IIMs is _______
annual meet. If Prof. Balamurali, a science Directions (Q. 11 – 13) :
professor refuses to be in the delegation if Three mountaineers Akil, Nikil, and Sunil are climbing
assistant Prof. Sheshdari, an arts professor is up a mountain with their respective probability of
included in the delegation, then in how many 2 5 4
ways can the delegation be formed? reaching the summit being , and , respectively..
3 8 7
(a) 9C4 4C3 The probability that
(b) 9C5 4 C2 11. None of them reach the summit is _______
12. Exactly two of them reaches the summit
(c) 10C5 6C3 – 9C4 5C2
is _______
(d) 9C4 4C3 + 9C5 4C2 13. Atleast two of them reach the summit is _______
5.10 Permutations-Combinations, Elementary Statistics and Probability
Directions (Q. 14 – 15) : 18. Nine points are marked on a straight line and 10
There are 12 points in a plane of which 4 are on a points are marked on another line which is
straight line and no three of the other points lie on a parallel to the first line. The_______ number of
straight line. triangles can be formed with these points as
vertices?
14. The _______number straight lines can be formed
by joining these points 19. The _______ number of positive integers less
than 100,000 and divisible by 125 can be formed
15. The _______number of triangles can be formed using the digits 0, 1, 2, 5 and 8, if repetition is
by joining these points allowed?
16. The _______ number of non-negative integral 20. There is an unlimited supply of identical red, blue,
solutions does the equation x1 + x2 + x3 + x4 = 10 and green coloured balls. In _______ number of
have? ways can 12 balls be selected from the supply.
17. A certain number of students of a school 21. Number of positive integral solutions of the equation
participated in the chess tournament of their a + b + c = 15 is _______
annual sports meet. Each player played 1 game
22. If six unbiased coins are tossed together, then
against each of the other player. It was found that
probability that the number of heads exceeds the
in 66 games both the players were girls, and in
number of tails is_______
240 games one was a girl and the other was a
boy. The number of games in which both the
players were boys is _______
ANSWERS
MCQ Type Questions
1. (c) 2. (d) 3. (d) 4. (c) 5. (d) 6. (a) 7. (b) 8. (c) 9. (b) 10. (b)
11. (c) 12. (d) 13. (b) 14. (d) 15. (a) 16. (a) 17. (c) 18. (b) 19. (c) 20. (d)
5
54 C2 .7 C3
= =`9 Required probability = 12
6 C5
Henceto make an average profit of ` 7 per throw, 10 35 175
Raju must pay ` 2 (9 – 7) for each time to throw = =
792 396
the die.
10. Total number of coins in bag = 6 + 5 + 4 = 15.
4. 7 letters can be arranged in 7 addressed envelopes
5 coins can be drawn from 15 coins in 15C5 ways.
in 7! ways.
When 4 coins are one rupee coins and fifth coin is
n(S) = 7! a two rupee coin the amount will be minimum.
We can arrange 7 letters into corresponding Number of favourable cases = 4C4 . 45C1 = 5
7 addressed envelopes in only one way.
Probability that the amount will be minimum is
1
Required probability = . 5 5
7! P(E) = 15 =
5. If six letters can be placed in their corresponding C5 3003
addressed envelopes, then seventh letter is also
in the correct envelope. 5 2998
P(E) = 1 – P(E) = 1 =
Hencerequired probability is 0. 3003 3003
6, 7 and 8. Odds in favour of the required event = 5: 2998
Three cards can be drawn from 52 cards in
52
C3 ways 11. G6 G1
n(S) = 52c3
3
6. Since we have 4 suits i.e., Diamonds, Spades,
Clubs and Hearts and each suit contains 13 cards. G2
G5
Three cards can be selected from 13 cards in 13C3
ways.
G3
Number of favorable outcomes = 4. 13C3 G4
8. Each suit contains 9 number cards. 2 number the remaining 5 places in 5! ways.
cards can be drawn from 9 cards in 9 C2 ways.. 3!
Third card can be selected from the remaining Hence, required number of ways
27 number cards in 27C1 ways.
Hence total number of favorable outcomes is 5! 5! 5!
2
= =
4(9C2) (27C1) 2!2! 3! 2!2 3!
Required probability =
4 9C 2 27
C1 13. Number of ways of inviting at least one friend is
52 212 – 1 = 4095
C3
9. Total number of balls in the bag = 5 + 7 = 12. 14. Number of ways of inviting at least 10 friends
5 balls can be drawn from 12 balls in 12C5 ways = 12C10 + 12C11 + 12C12 = 66 + 12 + 1 = 79
Number of ways of drawing 3 red balls and 15. Each prize can be distributed in 3 ways, so,
2 white balls from 5 white balls and 7 red balls is 5C2. 6 prizes can be distributed in 36 ways.
7
C3
5.12 Permutations-Combinations, Elementary Statistics and Probability
P A B = P A P( B ) =
5 7
. =
7
9 10 18 P A B C = P(A) P(B) P(C)
= P A B = P( A ).P(B) =
2 5 3
. . =
5
3 7 8 28
Permutations-Combinations, Elementary Statistics and Probability 5.13
Hencerequired probability
8. Given : P(H) = 3P(T)
= P A BC
+ P A BC We know
P(H) + P(T) = 1
+ P A B C + P A B C 3P(T) + P(T) = 1
1
25 1 5 5 50+12+15+30 107 P(T) =
= = = 4
84 14 56 28 168 168
5. Exactly one of them hits the target P(H) = 3
4
A B C A B C A B C Since the coin is tossed 3 times, two heads may
occur in 1st and 2nd trial, 2nd and 3rd trial or 1st and
denotes that exactly one of them hits the target. 3rd trial.
P AB C = P(A) . P( B ). P( C ) Required probability
3 3 1 1 3 3 1 3
=
2 2 5 5 4 4 4 4 4 4 4 4
= =
3 7 8 42
9 9 9 27
= =
1 2 3
P AB C = P A .P B .P C = . . =
1
3 7 8 28
64 64 64 64
9. Two black cards can be selected in 26c2 ways.
A pack contain 8 black honours.
1 5 5
P A B C = P(A).P(B) P(C) = . . =
25
3 7 8 168 Hence number ways of selecting two same honours
is 4.
Hencerequired probability is
4
4
= P A B C P A B C P A B C Required probability = =
26c 2 325
5 25 1 20 25 26 51 17 10. Let one card is king, then second card can be any
= = = =
42 168 28 168 168 56 of the other 3 cards from the other suit and it can
be selected in 4 3 = 12 ways. Since there are
6. Three consecutive letters can be selected from
four honour cards, hence
English alphabet in 24 ways.
n(S) = 24 12
Required probability =
Since, no two consecutive letters are vowels, the 325
number of ways that among three letters one 11, 12 and 13
letter is vowel is 13. Probability that Akil reach the summit
= P(A) = 2/3
13
Required probability =
favorable out come HHHHTTTT.
P N = 3/8
8!
This can be arranged in ways. Probability that Sunil reach the summit
4!4!
4
8 7 6 5 4! P(S) =
n(E) = = 70 7
4 3 2 1 4!
35 n E 70
P S =
3
7
Required probability = n S = =
2 128 11. Probability that none of them reaches the summit
= P A N S = P A P N P S
5.14 Permutations-Combinations, Elementary Statistics and Probability
6
CHAPTER
Miscellaneous
LINEAR EQUATIONS three cases which can be described geometrically.
An equation where the maximum power of any (Here we assume that the coefficients of x and y in
variable is unity (one) is a linear equation. A linear each equation are not both zero.)
equation is of the form: 1. The system has exactly one solution:
AX + BY + C = O Here the lines corresponding to the linear
Where A and B are coefficient and C is a constant equations intersect in one point as shown in fig.
below.
The equation is called LINEAR because the graph of
the equation on the X-Y Cartesian plane is a straight
line. The sets of values of X & Y satisfying any
equation(s) are called its solution(s).
Consider the equation 2x + y = 4. Now, if we substitute
x = –2 in the equation, we obtain 2. (–2) + y = 4 or –4
+ y = 4 or y = 8. Hence (–2, 8) is a solution. If we
substitute x = 3 in the equation, we obtain 2.3 + y = 4
or 6 + y = 4 or y = –2. Hence (3, –2) is a solution. The
following table lists six possible values for x and the
corresponding values for y, i.e. six solutions of the
equation.
x 2 1 0 1 2 3
y 8 6 4 2 0 2
The system a1X + b1Y = c1 and a2X + b2Y = c2 has
If we plot the solutions of the equation 2x + y = 4 a1 b1
which appear in the table above then we see that they a unique solution, if .
all lie on the same line. We call this line the graph of a2 b2
the equation since it corresponds precisely to the 2. The system has no solution: Here the lines
solution set of the equation. corresponding to the linear equations are parallel
as shown in fig. below.
GEOMETRY AND MENSURATION (5) Isosceles triangle: Triangle with two of its sides
equal and consequently the angles opposite the
STRAIGHT LINE equal sides are also equal.
Parallel lines: - Two straight lines are parallel if (6) Scalene triangle: Triangle with none of the sides
they lie on the same plane and do not intersect equal to any other side.
however far produced. Properties (General)
Transversal :- It is a straight line that intersects (1) Sum of the length of any two sides of a triangle
two parallel lines. When a transversal two parallel has to be always greater than the third side.
lines then
(2) Difference between the lengths of any two sides
(1) Corresponding angles are equal, ? (that is : For of a triangle has to be always lesser than the third
the figure below) side.
(2) Alternate interior angles are equal, that is (Refer (3) Side opposite to the greatest angle will be the
figure below.) 4 = 6; 5 = 3 greatest and the side opposite to the smallest angle
(3) Alternate exciter or angles are equal, that is 2 = 8; the smallest.
1=7 (4) The exterior angle is equal to the sum of two
(4) Interior angles on the same side of transversal interior angles not adjacent to it.
add up to 180°, that is 4 + 5 = 33 + 6 = 180° ACD = BCE = A + B
Polygons AREA:
Polygons are plane figures formed by a closed series (1) Area = 1/2 base × height or 1/2bh.
of rectilinear (straight) segments. Height = Perpendicular distance between the base
Example: and vertex opposite to it
Triangle, Rectangles. (2) Area s(s a)(s b)(s c) (Heros’s formula)
Polygons can broadly be divided into two types:
abc
(a) Regular polygons: Polygons with all the sides and Where S (a, b and c being the length of
2
angles equal.
the sides
(b) Irregular polygons: Polygons in which all the sides
(3) Area= r × S (where r is in radius)
or angles are not of the same measure.
(4) Area = 1/2 × product of two sides × sine of the
Triangles ( ) included angle.
A triangle I a polygon having three sides. Sum of all = 1/2 ac sin B
the angles of a triangle =180° = 1/2 ab sin B
Types: = 1/2 bc sin B
(1) Acute angle triangle: Triangles with all the angles
acute (less than 90°)
(2) Obtuse angle triangle: Triangles with one of the
angles obtuse (more than 90°)
Note: We cannot have more than one obtuse angle
triangle.
(3) Right angle triangle: Triangle with one of the (5) Area = abc/4R
angles equal to 90° Where R = circum radius
(4) Equilateral triangle: Triangle with all sides equal.
All the angles in such a triangle measure 60°
6.4 Miscellaneous
CONGRUENCY OF TRIANGLES B Q
Two triangles are congruent if all the sides of one are
And AC = PR
equal to the corresponding sides of another, It follows
that all the angles of one are equal to the So ABC PQR
corresponding angles of another. The notation for
congruency is
1. SAS congruency: If two sides and an included
angle of one triangle are equal to two sides and
an included angle of another, the two triangles
are congruent. (See figure below.)
Here,
AB = PQ
BC = QR
4. SSS congruency: If three sides of one triangle
And B Q
are equal to three sides of another triangle, the
So ABC PQR two triangles are congruent. In the figure below:
AB = PQ
BC = QR
AC = PR
ABC PQR
2h a
(3) R (circum radius)
3 3 In the triangle ABC,
h a ABC ~ DBA ~ DAC
(4) R (in radius)
3 2 3 (1) ABC ~ DBA
Properties
AB/BC = DB/BA
(1) The in centre and circum centre lies at a point
that divides the height in the ratio 2 : 1 AB2 = DB × BC
(2) The circum radius is always twice the in radius . c2 = pa
[R = 2r] (2) ABC ~ DAC
(3) Among all the triangles that can be formed with a AC/BC = DC/AC
given perimeter, the equilateral triangle will have
the maximum area. AC2 = DC × BC
(4) An equilateral triangles in a circle will have the b2 = qa
maximum area compared to other triangles inside (3) DBA ~ DAC
the same circle.
DA/DB = DC/DA
ISOSCELES TRIANGLE
DA2 = DB × DC
b
Area 4a 2 b2 AD2 = pq
4
6.6 Miscellaneous
IMPORTANT TERMS OF TRIANGLE 3. Perpendicular Bisectors: A line that is a
Important terms with respect to a triangle perpendicular to a side and bisects it is the
perpendicular bisector of the side.
1. Median: A line joining the mid- point of a side of
triangle to the opposite vertex is called a medium.
In the figure the three medians are PG, QF and
RE where G, E and F are mid-point of their
respective sides
(1) A median divides a triangle into two parts of equal
area.
(2) The point where the three medians of a triangle
meet is called the centroid of the triangle.
(3) The centroid of a triangle divides each median in
the ratio 2:1
i.e., PC : CG = 2:1 = QC : CF = RC : CE (1) The point at which the perpendicular bisectors of
the sides meet is called the circumcentre of the
triangle.
(2) The circumcentre is the centre of the circle that
circumscribes the triangle. There can be only one
such circle.
(3) Angle formed by any side at the circumcentre is
two times the vertical angle opposite to the side.
This is the property of the circle where by angles
formed the property of the circle where by angles
formed by an arc at the centre are twice that of
Important formula with respect to a median the angle formed by the same arc in the opposite
2 × (median)2 + (1/2 the third side)2 = Sum of the arc. Here we can view this as:
squares of other two sides
QCR = 2QPR (When we consider arc QR and its
QR
2 opposite arc QPR)
2(PG)2 2 4. In center:
2
(1) The lines bisecting the interior angles of a triangle
2. Altitude/ Height: A perpendicular drawn from
are the angle bisectors of that triangle.
any vertex to the opposite side is called the altitude
(In the figure, AD BF and CE are the altitudes of (2) The angle bisectors meet at a point called the in
the triangles) centre of the triangle.
(1) All the altitudes of a triangle met at a point called (3) The in centre is equidistant from all the sides of
the orthocenter of the triangle. the triangle.
(2) The angle made by any side at the orthocenter
and the vertical angle make a supplementary pair
(i.e., they both add up to 180°). In the figure below:
A BOC 180 C AOB
Trapezium of Square
2. Rectangles:
(1) Diagonals are equal and bisect each other at right
A rectangle is a parallelogram with all angles 90°
angles.
(1) Area = Base × Height = b × h
(2) Side is the diameter of the inscribed circle.
6.8 Miscellaneous
circumradius a / 2
(3) Area of a sector r 2
360
(where is the angle between two radii)
= (1/2)r × length (arc xy)
( r / 180 = length arc xy)
4. Trapezium: 1 r
r
A trapezium is a quadrilateral with only two sides 2 360
parallel to each other.
(1) Area = 1/2 × sum of parallel sides × height = 1/2
(AB + DC) × h. For the figure below.
(2) Median = 1/2 × sum of the parallel sides*(median
is the line equaidstant from the parallel sides)
For any line EF parallel to
EF
P (AB) Q (DC) (4) Segment: A sector minus the triangle formed by
AD the two radii is called the segment of the circle.
Area = Area of the sector–Area
1
OAB r 2 r 2 sin
360 2
Properties of Trapezium
(1) If the non-parallel sides are equal then diagonals
will be equal too.
CIRCLES
(1) Area = r 2
(5) Perimeter of segment = length of the arc + of
(2) Circumference = 2r = (r = radius) segment AB
(3) Area = 1/2 × circumference × r
Arc 2r 2r sin
360 2
It is a part of the circumference of the circle. The
bigger one is called the major arc and the smaller one r
2r sin
the minor arc. 180 2
(6) Congruency:
(1) Length (ArcXY) 2r
360 Two circles can be congruent if and only if they
have equal radii.
Miscellaneous 6.9
5. Pyramid 1
100–3
A pyramid is a solid which can have any polygon as its 1003
base and its edges converge to a single apex. Its This shows that for any non-zero negative integers a,
dimensions are defined by the dimensions of the
polygon at its base and the length of its lateral edges 1
a –m
which lead to the apes. The Egyptian pyramids are am
example of pyramids. where m is the positive integer and a m is the
(1) Slant surface of a pyramid = 1/2* Perimeter of multiplicative inverse of a–m.
the base* slant height Laws of Exponents
(2) Whole surface of a pyramid = Slant surface + area If we have a and b as the base and m and n as the
of the base exponents, then
area of the base * am × an = am+n
(3) Volume of a pyramid height (am)n
3
6. Cone am
am n , m n
A cone is a solid which has a circle at its base and a an
slanting lateral surface that converges at the apex. ambm = (ab)n
Its dimensions are defined by the radius of the base? a0 = 1
(r) the height (h) and the slant height (I) A structure
a1 = a
similar to a cone is used in ice cream cones
Use of Exponents to Express Small Numbers in
(1) Curved surface of a cone = rl where l is the Standard Form:
height Sometimes we need to write the numbers in very small
(2) Whole surface of a cone rl r 2 or large form and we can use the exponents to
represent the numbers in small numbers.
r 2 h 1. Standard form to write the natural numbers
(3) Volume of a cone = like xyz000000......
3
Step 1: First of all count the number of digits from
7. Sphere
left leaving only the first digit.
Is a solid in the form of a ball with radius r.
Step 2: To write it in exponent or standard form, write
(1) Surface area of a sphere = 42 down the first digit.
Step 3: If there are more digits in the number then
4 3 put a decimal after the first digit and then write down
(2) Volume of a sphere r
3 the other digits until the zero comes. And if there are
POWER AND EXPONENTS: no digits after the first digit then skip this step.
The exponents tell us that how many times the number Step 4: Now place a multiplication sign and then write
should be multiplied. It is called the Exponential form. down the counted digits in the first step as the exponent
Exponential formis written like this: to the base number 10.
9
9
10
2 × 10
Exponent 2.000000000
1 2 34 5 67 8 9
2,000,000,000
Base Example:
Express 1730000000000 in exponent form.
Power Solution:
Here 10 is the base and 9 is the exponent and this In standard form, the number 1730000000000 will be
complete number is the power. We pronounce it as 10 written as 1.73 × 1012.
raised to the power 9. The exponent could be positive 2. Standard form to write decimal numbers like
or negative. 0.00000.....xyz.
This tells us that the number 10 will be multiplied 9 Step 1: First of all count the number of digits from
times, like, 10 × 10 × 10 × 10 × 10 × 10 × 10 × 10 × 10 the decimal point to the last digit.
Powers with Negative Exponents Step 2: If there is only one digit after the zeros then
The exponents could be negative also and we can simply write down that digit. Place a multiplication
convert them in positive by the following method. sign and write down the counted digits in step-1 with a
negative sign as the exponent to base number 10.
Miscellaneous 6.11
Step 3: If there are two or more non-zero digits at the Characteristic and Mantissa
end of the number. Then, write down the digits followed 1. The integral part of a logarithm is called the
by a decimal point after the first digit and the other characteristic and the fractional part is called
non-zero digits. mantissa.
Step 4: Now calculate the number of digits in the first
log N = integer + fraction (+ve)
step and minus the number of digits appearing after 10
EXERCISE
a b 1
MCQ TYPE QUESTIONS 9. If loge log e a log e b , then:
2 2
1. If one root of a quadratic equation is 3 + 2 , b
then quadratic equation is (a) a = b (b) a =
2
(a) x2 – 6x – 7 = 0 (b) x2 + 6x + 7 = 0
b
2
(c) x + 6x – 7 = 0 (d) x2 – 6x + 7 = 0 (c) 2a = b (d) a
3
2. Three consecutive positive integers such that
square of their sum exceeds the sum of their 10. If 2 log (x + 1)– 10g (x2 – 1)= log2, then x equals
squares by 214, are (a) 1 (b) 0 (c) 2 (d) 3
(a) 5, 6, 7 (b) 10, 11, 12 11. The number log 27 is:
(c) 9, 10, 11 (d) 7, 8, 9 (a) an integer (b) a rational number
3. A and B attempt to solve a quadratic equation of (c) an irrational number (d) a prime number
the form ax 2 + bx + c = 0. A starts with a wrong 12. If y = 21/logx (8), then x is equal to:
value of b and gets the roots as –3 and –5. B (a) y (b) y2
starts with a wrong value of C and gets the roots 3
as 6 and 2. The correct roots are (c) y (d) none of these
(a) 3, 4 (b) 3, 5 13. If log05 sin x = 1 – log05 cos x, then number of
solution of x [–2, 2] is:
(c) 4, 6 (d) 2, 3
(a) 1 (b) 2 (c) 3 (d) 4
4. What is the value of
14. If a, b, c are in G.P. then log axx, logbx x, logcx x are in:
1 (a) GP (b) HP
?
1 (c) AP (d) none of these
5
1 15. Consider the following statements:
5
5 ..... 1. Solution of the inequality log5(x2 – 11 x + 43) < 2
is (0, 2)
5 29 5 – 29
(a) (b) (log x 2log x )
2 9
(a) 26 (b) 24 (c) 22 (d) 20 18. If 4 log 3 3 9log 2 10log x 83 , ( x R), then x is:
(a) 4 (b) 9
7. If log3 log4 x > 0, then
(c) 10 (d) none of these
(a) x > 1 (b) x > 4
19. Which of the following is not true?
(c) x > 64 (d) none of these
1 1
8. log 1/4 (a2 – 1)< log1/2 (a + 1)2 then: (a) 2
log 3 log 4
(a) a < 1
(b) log3 5 is an irrational number
(b) a < – 1
10
(c) a > 1 (c) log 8 x x 16
3
(d) none of these (d) If logx (a2 + 1)< 0, (a 0)then 0 < x < 1
Miscellaneous 6.13
(a) 2 6 (b) 3 6
(c) 4 6 (d) 6 6
40. In the above figure, AD is the bisector of BAC,
AB = 6 cm, AC = 5 cm and BD = 3 cm. Find DC,
(a) a = 84°, b = 21°, c = 48°
(b) a = 48°, b = 20°, c = 50°
(c) a = 72°, b = 24°, c = 54°
(d) a = 64°, b = 28°, c = 45°
36. In the following figure, it is given that O is the
centre of the circle and AOC 140. Find
(a) 11.3 cm (b) 2.5 cm
ABC
(c) 3.5 cm (d) 4 cm
41. ABCD is a trapezium in which AB is parallel to
DC, AD = BC, AB = 6 cm, AB = EF and DF = EC.
If two lines AF and BE are drawn so that area of
DF
ABEF is half of ABCD. Find .
CD
(a) 110° (b) 120°
(c) 115° (d) 130°
37. In a triangle ABC, point D is on side AB and point
E is on side AC, such that BCED is a trapezium.
DE : BC = 3:5. Calculate the ratio of the area of
1 1
ADE and the trapezium BCED (a) (b)
4 3
(a) 3:4 (b) 9:16
(c) 3:5 (d) 9:25 2 1
(c) (d)
5 6
Miscellaneous 6.15
42. The diagram represents the area swept by the 7. A band of workers unloaded a certain number of
wiper of a car. With the dimensions given in the boxes from their ship. The number of workers were
figure, calculate the shaded area swept by the between 50 and 60 (both inclusive). Each worker
wiper. handled exactly 11 boxes, and each box was handled
by exactly 7 workers. Exactly_______number of
boxes were there?
8. If ax = b,by = c ,cz = a, then value of xyz is _______
9. The value of
1 1 1
+ +
xb xc xa xc xb xa
1 1 1
xa xa xb xb
xc xc
is _______
(a) 102.67 cm (b) 205.34 cm
10. A palm tree was 90 cm high, when it was planted.
(c) 51.33 cm (d) 208.16 cm
It grows by an equal number of cm each year, and
at the end of the seventh year it was one ninth
NUMERICAL TYPE QUESTIONS taller than at the end of the sixth year. The height
1. When x5 + x4 + 5x2 – 3 is divided by (x + 2), the of tree in cm at the end of the twelfth year
remainder will be_______ is _______
2. Pressure of a gas is inversely proportional to the 11. A bottle and its cork together cost ` 1.10, and the
volume. If pressure of oxygen is 84 atm when the bottle costs ` 1.00 more than its cork. The price
volume is 1000 cm3, then volume when the of the bottle is ______
pressure is 28 atm will be_______ cm3 12. The value of is (100 – 1) (100 – 2) (100 – 3) ..........
3. The value of ‘a’ will make x2 – ax + 25 a perfect (100 + 1) (100 + 2) (100 + 3) is _______
square is _______
4. The value of 3 + 3 if and are roots of the 1
2
equation : 2x2 – 7x – 4 = 0 is _______ 4
3
5. Sum of Anita’s and Sunita’s age is 43 years. 5
13. The value of is _______
7 1
11 years hence, Anita’s age will be times 2
6 1
3
Sunita’s age then. Sunita’s present age is___ years 1
1
6. In a school trip, the students were accommodated 4
in two rooms A and B. If 5 students are shifted
from room A to room B then there will be equal
number of students in both the rooms. But if 5 14. Three integers, a, b and c, are such that b = 3.
students from room B is shifted to room A, then c
the resulting number of students in room A is b
The fraction is in its lowest form. The_______
double the number of students left in room B. a
is the possible value of a.
Total number of students are _______
6.16 Miscellaneous
ANSWERS
MCQ Type Questions
1. (d) 2. (a) 3. (b) 4. (a) 5. (b) 6. (a) 7. (b) 8. (b) 9. (a) 10. (d)
11. (c) 12. (c) 13. (b) 14. (b) 15. (b) 16. (c) 27. (c) 18. (c) 19. (c) 20. (c)
21. (c) 22. (a) 23. (a) 24. (d) 25. (a) 26. (b) 27. (b) 28. (d) 29. (c) 30. (a)
31. (a) 32. (a) 33. (b) 34. (a) 35. (a) 36. (a) 37. (b) 38. (a) 39. (a) 40. (b)
41. (b) 42. (a)
Numerical Type Questions
511
1. 1 2. 3000 3. 10 4. 5. 19 6. 60 7. 88 8. 1 9. 1
8
10. 450 11. 1.05 12. 0 13. 1 14. 104
EXPLANATIONS
MCQ TYPE QUESTIONS 4. Let
1
x=5+
1. If one root = 3 + 2 , then other root = 3 – 2
1
Sum of roots = 3 + 2 + 3 – 2 = 6 5 1
Product of roots 5
5 .....
= (3 + 2 )(3 – 2 ) = 9 – 2 = 7 1 5x + 1
Quadratic equation is i.e., x = 5 + =
x x
x2 – (Sum of roots) x + Product of roots = 0 x2 = 5x + 1
i.e., x2 – 6x + 7 = 0 x2 – 5x – 1 = 0
2. Let consecutive positive integers be x – 1, x and
5± (–5) 2 4(1)(1)
x+1 x =
[x – 1 + x + x + 1]2 2
= (x – 1)2 + x2 + (x + 1)2 + 214 5 ± 25 4 5 ± 29
= =
(3x)2 = x2 + 1 – 2x + x2 + x2 + 2x + 1 + 214 2 2
9x2 = 3x2 + 216 Since, all the terms are positive. The answer has
6x2 = 216 5 + 29
to be .
x2 = 36 ; x = 6 2
But x is a positive integer, hence x = 6
So, numbers are 5, 6, and 7. 5. Equation (ii) is c(a + b) = 23, a prime number.
3. Equation formed by A with the roots –3 and –5 is So, 2 factors must be 1, 23.
x2 – (–8x) + 15 = 0 Since a, b are positive, therefore a + b > 1
i.e., x2 + 8x + 15 = 0 c = 1; a + b = 23
But value of B is wrong. Put c = 1; b = 23 – a in (i)
Hence coefficient of x is wrong. a2 – 22a + 21 = 0
Equation formed by B with roots 6 and 2 is a = 1 or 21
x2 – 8x + 12 = 0 b = 22 or 2
But the value of C is wrong. Solution sets are (1, 22, 1); (21, 2, 1).
hence constant term is wrong. 6. First day the number of idlis he ate be x
So, correct equation is Second day the count is (x+6)
x2 – 8x + 15 = 0. Third day ----------------------- (x+12)
Hence, real roots are 3 and 5. Fourth day --------------------(x+18)
Miscellaneous 6.17
Fifth day -----------------------(x+24)
1 1
Total is5x + 6(1+2+3+4) =100 14. log ax x log ax x
log x ax 1 log x a
5x + 60 =100
x=8 If a, b, c are in G.P
Day Idlis logx a, logx b, logx c are in AP
1 8 1 + logx a, 1 + logx b, 1 + logx C are in AP
2 14 1 1 1
3 20 , , are in HP..
1 log x a 1 log x b 1 log x c
4 26
logax x, logbx x logcx x are in HP.
5 32
So, on fourth day, number of idlis the man ate 15. (1) alog5 (x2 – 11x + 43) < 2
were 26. and x2 – 11x + 43 > 0
7. We have log3 log4 x > 0 x2 – 11x + 43 < 52
log4 x > 1 x > 4 2
11 51
x>4 and x 0
2 4
8. log1/4 (a2 – 1) < log(1/4) (a + 1)2
a2 – 1 > (a + 1)2 x2 – 11x + 18 < 0
a2 – 1 > a2 + 1 + 2a and (x – 2) (x – 9) < 0
2a + 2 < 0 a < – 1 solution is (2, 9)
a b 1 (2) For domain |x, – 1| 0, – 1
9. log log e a log e b Now, |x – 1| = 1, x – 1 = ± 1
2 2
x = 0, 2
ab
= ab x = 0 is not in the domain and x = 2 satisfies
2 the given equation.
a b 2 ab = 0 If x – 1 > 0 i.e., x > 1 then the given equation
becomes
a = bab
10. By definition x 1, = 1 4 1
2 log3 x – 7 x 81,
Given equation can be written as log 3 x 3
( x 1)2 x 1 1
log = log 2 2 x 3 But being less than 1 is not valid.
x2 1 x 1 3
11. Let x = log 27
Hence, x = 2, 81
2x = 7
16. log cos x sin x 2 sin x cos2 x
which is only possible for irrational number.
12. y = 21/log x(8) sin x 1 – sin2 x
sin2 x + sin x – 1 0
y = 2log 8x y 2log 2
3
x
2
x = y3 sin x 0
1 5
13. log 0.5 sin x + log 0.5 cos x = 1 2 4
log 0.5 sin x cos x = 1 Also by definition of logarithm
1 sin x > 0, cos x > 0, cos x 1
sin x cos x = sin 2 x 1
2
1 5 5 1
sin x + 0 sin x
n 2 2 2
2 x n ( 1) n
x (1) n
2 2 4
17. log2 xy 6 xy 26
Since log0.5 sin x and log0.5 cos x are real.
sin x and cos x must lie in first quardrant x y
Now xy
2
x , 2 x y
4 4 23 x y 24 x y 16
Hence, the number of solutions is 2. 2
6.18 Miscellaneous
n
18. 2
4 log 3 31 / 2 9 log 2 2 = 10 log x
83
3
1
4 4 106
41/2 + 92 = 10log x83 Now,
1
2 + 81 = 10log x83 4
83 = 10log x83 n
3 1 6
x = 10 1 = 1 (10 )
4 4
1 1 6
19. (a) log log = log 3 log 4 = log 12 > 2
n
3 = 10
4 4
3 4
12 > 2
3 6
(b) log3 5 is an irrational number n log10 log 10 10 log10 4
4
x 8 n(0.47712 – 2 0.30103) = – 6 – 2 (0.30103)
10 10 / 3
(c) log 8 x 25
3 6 60206
x = 32 n= 53
0 12494
(d) log x ( a2 1) 0, a 0 a2 1 1 22. Given that, xlog x2 = 2 = log3 (x + y)
x + y = 9 and x2 + y2 = 65
So, log is negative
x = 8, y = 1 or x = 1, y = 8
Hence, base is (0, 1).
But x 1
20. From the given relation, we have x = 8, y = 1
a = y1 – log ax = z1 – log ay 23. loga nlogb n + logb nlogc n + logc n logan
loga a = (1 – loga x) loga y 1 1 1
=
and loga a = (1 – loga y) loga z log n a log n b log n b log n c log n c log n a
loga y(1 – loga x) = 1 1
and loga z (1 – loga y) = 1 log m n log m
n
1
loga y = 1 log x log n c log n a log n b
a =
log n a log n b log n c
1
and loga z =
1 log a y log a n log b n log c n
log n (abc)
1 1 = =
loga z = log n a log n b log n c log abc n
1 log a y 1
1 24. Here, 2 log10 x – logx (10)–2 = 2 log10 x + 2 log x10
1 log a x
1 log a x 1
= = 2 log10 x 2
log a x log10 x
1 1
Now = log a x 1
1 log a z 1 log a x = 2 log10 x ...(i)
1 log10 x
log a x
1 Using AM GM we get
logax = 1 log
a z
1
1 log10 x 1/2
x = a 1 log z log10 x 1
log 10 x
a 2 log10 x
n
3
2 1
3 3 4 1
21. Series 1 ... n terms log10 x 2
4 4 3
1 log 10 x
4
2 2 log10 x – logx (0.01) 4
3 3 1
and 1 ... 4 Least value is 4.
4 4 3
1
4
Miscellaneous 6.19
26. 3M + 4G + 5W = 750 ...(i) 32. By the rule of chords, cutting externally, we get
6M + 9G + 10W = 1580 ...(ii) (9 + 6)6 = (5 + x)5
Now, if we look at the equation (i) and multiply it 90 = 25 + 5x
by 2, we get: 5x = 65
6M + 8G + 10W = 1500 ...(iii) x = 13 cm
From equation (ii) and (iii) we get, G = 80 area
33. In radius =
If we subtract the cost of 4 guavas from equation semi perimeter
(iii) we would get:
24
6M + 4G + 10W = 1500 – 320 = 1180 = cm
12
27. Let, the two digits number be 10x + y 34. ADBC is a cyclic quadrilateral as well its four
x+y=9 ...(1) vertices are on the circumference of the circle.
Also, the opposite angles of the cyclic quadrilateral
1 are supplementary.
x (10x + y) – 6 ...(2)
6 Therefore, ADB = 180 – 48° = 132°
Solving Eq (1) and (2) we get, 35. POS = QOR
x = 5 and y = 4 (Vertically opposite angles)
Therefore, Number = 54 So a = 4b
28. ABC ~ CDE SOT + TOQ + QOR = 180°
(sum of angles on a line = 180°)
AC BC AB
Then 4b + 2c = 180°
CD CE DE
84 + 2c = 180°
Then
AC 4
AC 6.4 cm and 2c = 96°
4.8 3 c = 48°
So a = 84°, b = 21° c = 48°
BC 4
= BC = 5.6 cm 36.
4.2 3
29.
140
ADC 70 (because the angle subtended
2
BC = ED = 6 m by an arc on the circumference is half of what it
AB = AC – BC = 11 – 6 = 5 m subtends at the centre.) ABCD one cyclic
CD = BC = 12 m quadrilateral So ABC = 180° – 70 = 110°
Then by Pythagoras theorem : (because opposite angles of a cyclic quadrilateral
AC2 = AB2 + BE2 AE = 13 m are supplementary.)
30. x = 35° because angles subtended by an arc 37.
anywhere on the circumference are equal.
31. By the rule of tangents, we know :
62 = (5 + n) 5
36 = 25 + 5x
11 = 5x
x = 2.2 cm
6.20 Miscellaneous
8 r 2
R 2
360 360
O O
360
212 72
102.67 cm
O
7 = 3 + 3 + 3(–2) 7
3
1 1 1
9. + +
2 2 xb xc xa xc xb xa
1 + + 1 + + 1 + +
xa xa xb xb xc xc
343 = 3 + 3 – 21
8
xa xb xc
343 = + +
8
+ 21 = 3 + 3 xa + xb + xc xa + xb + xc xa + xb + xc
511
= 3 + 3 =
(xa + xb + xc)
=1
8 xa + xb + xc
5. Let Anita’s present age be ‘x’ years and that of
10. Let tree grows x cm each year
Sunita by ‘y’ years.
Height of the tree at the end of the sixth year
x + y = 43 ...(1)
= (90 + 6x) cm
7
and (x + 11) = (y + 11) Growth in seventh year,
6
6x + 66 = 7y + 77 1
x = (90 + 6x) cm
6x – 7y = 11 ...(2) 9
Multiplying (1) by 6. 2x
x = 10 + x = 30
6x + 6y = 258 ...(3) 3
Subtracting (3) from (2), Hence height of tree at the end of the twelfth
–13y = –247 year = (90+12 × 30) = 450cm.
y = 19 years. 11. Let B = price of the bottle
6. Let the number of students in room A be x and C = price of the cork
number of students in room B be y. Given : B + C = ` 1.10 ...(1)
x–5 =y+5 and B – C = ` 1.00 ...(2)
x – y = 10 ...(1) From equations (1) and (2)
Also, x + 5 = 2(y – 5) B = ` 1.05
x – 2y = –15 ...(2) C = ` 0.05.
Solving (1) and (2) we get, 12. (100 – 1) (100 – 2) (100 – 3).........
x = 35 and y = 25 (100 – 100) (100+1) (100+2) (100+3)
x + y = 60 (100 – 1)(100 – 2)(100 – 3).........
7. Let P be the number of workers, and let C be the (0) (100 + 1) (100 + 2) (100 + 3) = 0.
number of boxes. Then, by counting the number
of boxes handling in 2 different ways yields 1
2
11P = 7C. 19 5
2
Since P and C are integers, it follows that P must 13. 5 = 19 =1
be divisible by 7. 1 5
2 2
Since 50 P 60, 3
4 19
P must equal 56 5
56
C = 11 = 88 b
7 14. = 3 b = 3c, i.e., b is divisible by 3.
c
8. ax = b
(cz) x = b ....[ since cz = a] b
Also, cannot be reduced,
(by) xz = b ....[since by = c] c
i.e., a is not divisible by 3.
xyz =1
SOLVED PAPER – 2016
1. Based on the given statements, select the 6. The Venn diagram shows the preference of the
student population for leisure activities.
appropriate option with respect to grammar and
usage. Read Watch TV
Statements : books
13 12 19
I. The height of Mr. X is 6 feet. 7
II. The height of Mr. Y is 5 feet. 44 17
10. A wire of length 340 mm is to be cut into two If the amounts invested in the companies, P and
parts. One of the parts is to be made into a square Q, in 2006 are in the ratio 8 : 9, then the amounts
and the other into a rectangle where sides are in received after one year as interests from
the ratio of 1:2. What is the length of the side of companies P and Q would be in the ratio:
the square (in mm) such that the combined area (a) 2 : 3 (b) 3 : 4
of the square and the rectangle is a MINIMUM?
(c ) 6 : 7 (d ) 4 : 3
(a) 30 (b) 40
17. Today, we consider Ashoka as a great ruler
(c) 120 (d) 180
because of the copious evidence he left behind in
11. If I were you, I __________ that laptop. It’s much the form of stone carved edicts. Historians tend
too expensive. to correlate greatness of a king at his time with
(a) won’t buy (b) shan’t buy the availability of evidence today. Which of the
following can be logically inferred from the above
(c) wouldn’t buy (d) would buy
sentences?
12. He turned a deaf ear to my request. What does
(a) Emperors who do not leave significant
the underlined phrasal verb mean?
sculpted evidence are completely forgotten.
(a) ignored (b) appreciated
(b) Ashoka produced stone carved edicts to
(c) twisted (d) returned ensure that later historians will respect him.
13. Choose the most appropriate set of words from (c) Statues of kings are a reminder of their
the options given below to complete the following greatness.
sentence.
(d) A king’s greatness, as we know him today, is
_________ _________ is a will, _________ is a way. interpreted by historians.
(a) Wear, there, their 18. Fact 1 : Humans are mammals.
(b) Were, their, there Fact 2 : Some humans are engineers.
(c) Where, there, there Fact 3 : Engineers build houses.
(d) Where, their, their If the above statements are facts, which of the
14. (x % of y) + (y % of x) is equivalent to _______. following can be logically inferred ?
(a) 2 % of xy (b) 2 % of (xy/100) I. All mammals build houses.
(c) xy % of 100 (d) 100 % of xy II. Engineers are mammals.
15. The sum of the digits of a two digit number is 12. III.Some humans are not engineers.
If the new number formed by reversing the digits (a) II only (b) III only
is greater than the original number by 54, find
(c) I, II and III (d) I only
the original number.
19. A square pyramid has a base perimeter x, and
(a) 39 (b) 57
the slant height is half of the perimeter. What is
(c) 66 (d) 93 the lateral surface area of the pyramid?
16. Two finance companies, P and Q, declared fixed (a) x2 (b) 0.75 x2
annual rates of interest on the amounts invested
(c) 0.50 x2 (d) 0.25 x2
with them. The rates of interest offered by these
companies may differ from year to year. Year-wise 20. Ananth takes 6 hours and Bharath takes 4 hours
annual rates of interest offered by these to read a book. Both started reading copies of the
companies are shown by the line graph provided book at the same time. After how many hours is
below. the number of pages to be read by Ananth, twice
P Q that to be read by Bharath? Assume Ananth and
Bharath read all the pages with constant pace.
9.5 10 (a ) 1
9 9
8 8 8 8
7 7.5 (b) 2
6.5 6.5 6
4
(c) 3
(d ) 4
2000 2001 2002 2003 2004 2005 2006
SOLVED PAPER – 2016 3
21. Which of the following is CORRECT with respect Which of the following inference(s) is/are logically
to grammar and usage? valid and can be inferred from the above passage?
Mount Everest is ____________. (i) His friends were always asking him to help
(a) the highest peak in the world them.
(b) highest peak in the world (ii) He felt that when in need of help, his friends
would let him down.
(c) one of highest peak in the world
(iii) He was sure that his friends would help him
(d) one of the highest peak in the world
when in need.
22. The policeman asked the victim of a theft, “What
(iv) His friends did not help him last week.
did you______ ?”
(a) (i) and (ii) (b) (iii) and (iv)
(a) loose (b) lose
(c) (iii) only (d) (iv) only
(c) loss (d) louse
28. Leela is older than her cousin Pavithra. Pavithra’s
23. Despite the new medicine’s ______________ in
brother Shiva is older than Leela. When Pavithra
treating diabetes, it is not ______________widely.
and Shiva are visiting Leela, all three like to play
(a) effectiveness --- prescribed chess. Pavithra wins more often than Leela does.
(b) availability --- used Which one of the following statements must be
(c) prescription --- available TRUE based on the above?
(d) acceptance --- proscribed (a) When Shiva plays chess with Leela and
Pavithra, he often loses.
24. In a huge pile of apples and oranges, both ripe
and unripe mixed together, 15% are unripe fruits. (b) Leela is the oldest of the three.
Of the unripe fruits, 45% are apples. Of the ripe (c) Shiva is a better chess player than Pavithra.
ones, 66% are oranges. If the pile contains a total
(d) Pavithra is the youngest of the three.
of 5692000 fruits, how many of them are apples?
(a) 2029198 (b) 2467482 1 1 c 1
and r and s , the value of
a b
29. If q
(c) 2789080 (d) 3577422 r s q
abc is ____ .
25. Michael lives 10 km away from where I live.
Ahmed lives 5 km away and Susan lives 7 km (a) (rqs)–1 (b) 0
away from where I live. Arun is farther away than (c) 1 (d ) r + q + s
Ahmed but closer than Susan from where I live. 30. P, Q, R and S are working on a project. Q can
From the information provided here, what is one finish the task in 25 days, working alone for 12
possible distance (in km) at which I live from hours a day. R can finish the task in 50 days,
Arun’s place? working alone for 12 hours per day. Q worked 12
(a) 3.00 (b) 4.99 hours a day but took sick leave in the beginning
(c) 6.02 (d) 7.01 for two days. R worked 18 hours a day on all days.
What is the ratio of work done by Q and R after 7
26. A person moving through a tuberculosis prone days from the start of the project?
zone has a 50% probability of becoming infected.
However, only 30% of infected people develop the (a) 10:11 (b) 11 :10
disease. What percentage of people moving (c) 20:21 (d) 21:20
through a tuberculosis prone zone remains 31. The chairman requested the aggrieved
infected but does not show symptoms of disease? shareholders to _________ him.
(a) 15 (b) 33 (a) bare with (b) bore with
(c) 35 (d) 37 (c) bear with (d) bare
27. In a world filled with uncertainty, he was glad to 32. Identify the correct spelling out of the given
have many good friends. He had always assisted options :
them in times of need and was confident that they (a) Managable (b) Manageable
would reciprocate. However, the events of the last
(c) Mangaeble (d) Managible
week proved him wrong.
4 SOLVED PAPER – 2016
33. Pick the odd one out in the following : Which of the following can be logically inferred
13, 23, 33, 43, 53 from the above paragraph?
(a) 23 (b) 33 (a) Many students have misconceptions regarding
various engineering disciplines.
(c) 43 (d) 53
(b) Men with advanced degrees in mechanical
34. R2D2 is a robot. R2D2 can repair aeroplanes. No
engineering believe women are well suited to
other robot can repair aeroplanes. Which of the
be mechanical engineers.
following can be logically inferred from the above
statements? (c) Mechanical engineering is a profession well
suited for women with masters or higher
(a) R2D2 is a robot which can only repair
degrees in mechanical engineering.
aeroplanes.
(d) The number of women pursuing higher
(b) R2D2 is the only robot which can repair
degrees in mechanical engineering is small.
aeroplanes.
38. Sourya committee had proposed the
(c) R2D2 is a robot which can repair only
establishment of Sourya Institutes of Technology
aeroplanes.
(SITs) in line with Indian Institutes of Technology
(d) Only R2D2 is a robot. (IITs) to cater to the technological and industrial
4y needs of a developing country.
35. If 9 y 6 3, then y2 is ________.
3 Which of the following can be logically inferred
1 from the above sentence?
(a ) 0 (b) Based on the proposal,
3
1 (i) In the initial years, SIT students will get
(c) (d) Undefined degrees from IIT.
3
36. The following graph represents the installed (ii) SITs will have a distinct national objective.
capacity for cement production (in tonnes) and (iii) SIT like institutions can only be established
the actual production (in tonnes) of nine cement in consulation with IIT.
plants of a cement company. Capacity utilization (iv) SITs will serve technological needs of a
of a plant is defined as ratio of actual production developing country.
of cement to installed capacity. A plant with
(a) (iii) and (iv) only (b) (i) and (iv) only
installed capacity of at least 200 tonnes is called
a large plant and a plant with lesser capacity is (c) (ii) and (iv) only (d) (ii) and (iii) only
called a small plant. The difference between total 39. Shaquille O’ Neal is a 60% career free throw
production of large plants and small plants, in shooter, meaning that he successfully makes 60
tonnes is ________. free throws out of 100 attempts on average. What
is the probability that he will successfully make
300 Installed Capacity Actual Production
exactly 6 free throws in 10 attempts?
250
250 220 (a) 0.2508 (b) 0.2816
Capacity/production(tonnes)
230
200 200
200 180 190 190 190 (c) 0.2934 (d) 0.6000
160
150 160 160 150
140 40. The numeral in the units position of
150
120 120
100 211870 + 146127 3424 is _________.
100
41. Out of the following four sentences, select the
50 most suitable sentence with respect to grammar
0 and usage.
1 2 3 4 5 6 7 8 9
Plant Number (a) I will not leave the place until the minister
does not meet me.
37. A poll of students appearing for masters in
engineering indicated that 60% of the students (b) I will not leave the place until the minister
believed that mechanical engineering is a doesn't meet me.
profession unsuitable for women. A research (c) I will not leave the place until the minister
study on women with masters or higher degrees meet me.
in mechanical engineering found that 99% of such
(d) I will not leave the place until the minister
women were successful in their professions.
meets me.
SOLVED PAPER – 2016 5
42. A rewording of something written or spoken is 47. Indian currency notes show the denomination
a__________. indicated in at least seventeen languages. If this
(a) paraphrase (b) paradox is not an indication of the nation's diversity,
nothing else is.
(c) paradigm (d) paraffin
Which of the following can be logically inferred
43. Archimedes said, “Give me a lever long enough from the above sentences?
and a fulcrum on which to place it, and I will move
(a) India is a country of exactly seventeen
the world.”
languages.
The sentence above is an example of a ________
(b) Linguistic pluralism is the only indicator of a
statement.
nation's diversity.
(a) figurative (b) collateral
(c) Indian currency notes have sufficient space
(c) literal (d) figurine for all the Indian languages.
44. If ‘relftaga' means carefree, ‘otaga' means careful (d) Linguistic pluralism is strong evidence of
and ‘fertaga' means careless, which of the India's diversity.
following could mean ‘aftercare'? 48. Consider the following statements relating to the
(a) zentaga (b) tagafer level of poker play of four players P, Q, R and S.
(c) tagazen (d) relffer I. P always beats Q
45. A cube is built using 64 cubic blocks of side one II. R always beats S
unit. After it is built, one cubic block is removed III.S loses to P only sometimes
from every corner of the cube. The resulting
IV. R always loses to Q
surface area of the body (in square units) after
the removal is _______. Which of the following can be logically inferred
from the above statements?
(a) 56 (b) 64
(i) P is likely to beat all the three other players
(c) 72 (d) 96
(ii) S is the absolute worst player in the set
46. A shaving set company sells 4 different types of
(a) (i) only (b) (ii) only
razors, Elegance, Smooth, Soft and Executive.
Elegance sells at Rs. 48, Smooth at Rs. 63, Soft (c) (i) and (ii) (d) neither (i) nor (ii)
at Rs. 78 and Executive at Rs. 173 per piece. The 49. If f(x) = 2x + 3x – 5, which of the following is a
7
table below shows the numbers of each razor sold factor of f(x) ?
in each quarter of a year. (a) (x3 + 8) (b) (x – 1)
Quarter (c) (2x – 5) (d) (x + 1)
Elegance Smooth Soft Executive
\product 50. In a process, the number of cycles to failure
Q1 27300 20009 17602 9999 decreases exponentially with an increase in load.
At a load of 80 units, it takes 100 cycles for failure.
Q2 25222 19392 18445 8942
When the load is halved, it takes 10000 cycles for
Q3 28976 22429 19544 10234 failure. The load for which the failure will happen
Q4 21012 18229 16595 10109 in 5000 cycles is _____.
Which product contributes the greatest fraction (a) 40.00 (b) 46.02
to the revenue of the company in that year? (c) 60.01 (d) 92.02
(a) Elegance (b) Executive
(c) Smooth (d) Soft
6 SOLVED PAPER – 2016
ANSWERS
1. (c) 2. (b) 3. (c) 4. (c) 5. (b) 6. (d) 7. (a) 8. (d) 9. (c) 10. (b)
11. (c) 12. (a) 13. (c) 14. (a) 15. (a) 16. (d) 17. (d) 18. (b) 19. (d) 20. (c)
21. (a) 22. (b) 23. (a) 24. (a) 25. (c) 26. (c) 27. (b) 28. (d) 29. (c) 30. (c)
31. (c) 32. (b) 33. (b) 34. (b) 35. (c) 36. (120) 37. (a) 38. (c) 39. (a) 40. (7)
41. (d) 42. (a) 43. (a) 44. (c) 45. (d) 46. (b) 47. (d) 48. (a) 49. (b) 50. (b)
EXPLANATIONS
General Aptitude 7. Social science disciplines had a pre-colonial origin.
12
1. Height
8. 11 1
Height 10 2
6 Feet
5 Feet
X Y
(Statement I) (Statement II)
Mirror image of 1 : 20 is 10 : 30
10 : 30 was the time two and quarter hour back
Hence from the given figure Mr. X is taller than
so time now will be 12 : 45
Mr. Y by 1 foot.
9. M
4. (9 inches)1/2 = (0.25 yards)1/2,
Solving we get 9 inch = 0.25 yards
km 5 2 km
10
(since 1 inch = 0.028 yard) (Starting point)
5. M is twice as efficient as E but worked for half as O 10 km
5 2 km
many days. So in this case they will do equal work
5 km
5 km
if their shifts had same timings. But M’s shift is
2 2 km
for 6 hours, while E’s shift for 12 hours. Hence, E
4k 2 2 km
will do twice the work as M. m
N O
Ratio of contribution of M : E in work is 1 : 2.
From the given figure
6. Given Venn diagram is
Read Watch TV MN = (OM)2 (ON)2
books
13 12
7
OM = 5 2 5 2 2 5 7 2
44 17
ON = 10 5 2 2 2 10 3 2
15
MN = (5 7 2)2 (10 3 2)2
Play sports
The number of students who like to read books
or play sports = 25 98 70 2 100 18 60 2)
= 13 + 12 + 44 + 7 + 15 + 17 20.61 km
= 108 14. (x% of y) + (y% of x)
x y 2 xy
y x = = 2% of xy
100 100 100
Option (a) is correct.
SOLVED PAPER – 2016 7
s
45 15 b
ac
= 5692000 s sabc s1
100 100
= 384210 abc = 1
Option (c) is correct
34 85
Ripe type of apples = 5692000 30. Q can finish the task = 25 days, 12 hrs/day = 300 hrs
100 100
= 1644988 1
th
1 hr =
Total number of apples 300
= 384210 + 1644988 R can finish the task = 50 days, 12 hrs/day
= 2029198 = 50 12 = 600 hrs
25. 1
1 hr = th
600
Q working hours 7 2 12 60 hrs
After 7 days, the ratio of work done by Q and R 39. The question clearly explains that shaquille
Q : R makes 60 free throws out of 100
60 126 60
Hence, Probability of free throw = 0.6
300 600 100
20 : 21 And probability of NOT free throw
33. The given number 33 is odd one out, because the = 1 – 0.6 = 0.4
remaining numbers are prime number. So required probability of exactly 6 throws in
35. Given, 9y 6 = 3 10 attempts will be given by
9y = 3 otaga careful
fertaga careless
1
y = taga care
3
aftercare taga____.
4y
Now y2 = Option (c) matches i.e. tagazen.
3
put y = 1, 45.
4
we get = 1
3
4 unit
1
=
3 4 unit
1 4 unit
and also put y = Required Surface Area = (6 4) + (8 6) + (3 8)
3
= 24 + 48 + 24
1 4 3 1
we get, = = 96 units.
9 9 9 3
1 46. Elegance (27300 + 25222 + 28976 + 21012) 48
So, the required value is .
3 Executive (9999 + 8942 + 10234 + 10109) 173
36. From the given graph large plants which are Smooth (20009 + 19392 + 22429 + 18229) 63
having installed capacity of at least 200 tonnes Soft (17602 + 18445 + 19544 + 16595) 78
are 1, 4, 8 and 9. Executive has the highest revenue.
So, the total production of large plants 48. (i) is followed.
= 160 + 190 + 230 + 190 (ii) is not followed.
= 770 49. f(x) = 2x7 + 3x – 5
Now the remaining plants with installed capacity Putting x = 1
is less then 200 tonnes are 2, 3, 5, 6 and 7.
f(1) = 2 + 3 – 5 = 0
So, total production of small plants
So, (x – 1) is the factor of f(x).
= 150 + 160 + 120 + 100 + 120
= 650
Then difference = 770 – 650
= 120
SOLVED PAPER – 2017
1. What is the value of x when From this what can one conclude?
x 2 2 x 4 (a) The centre operates on a first-come-first-
16 3
81 144 ? served basis, but with variable service times
25 5
depending on specific customer needs.
(a ) 1
(b) Customers were served in an arbitrary order
(b) – 1 since they took varying amounts of time for
(c) – 2 service completion in the centre.
(d) Cannot be determined (c) Since some people came out within a few
2. There was no doubt that their work was thorough minutes of entering the centre, the system is
Which of the words is closest in meaning to the likely to operate on a last-come-first-served
underlined word above? basis.
(a) pretty (b) complete (d) Entering the centre early ensured that one
(c) sloppy (d) haphazard would have shorter service times and most-
people attempted to do this.
3. Four cards lie on a table. Each card has a number
printed on the one side and a color on the other. 7. The points in the graph below represent the halts
The faces visible on the cards are 2, 3 red and of a lift for duration of 1 minute, over a period of
blue. 1 hour.
Proposition. If a card has an even value on one
side then its opposite face is red.
The cards which MUST be turned over to verify
the above porposition are
(a) 2, red (b) 2, 3, red
(c) 2, blue (d) 2, red, blue
4. The event would have been successful if you _____
able to come.
Which of the following statements are correct?
(a) are (b) had been
(i) The elevator never moves directly from any
(c) have been (d) would have been non-ground floor to another non-ground floor
5. Two dice are thrown simultaneously. The over the one hour period.
probability that the product of the numbers (ii) The elevator stays on the fourth floor for the
appearing on the top faces of the dice is a perfect longest duration over the one hour period.
square is
(a) only (i) (b) only (ii)
(a) 1/9
(c) Both (i) and (ii) (d) Neither (i) nor (ii)
(b) 2/9
8. A map shows the elevations of Darjeeling,
(c) 1/3 Gangtok, Kalimpong, Pelling and Siliguri.
(d) 4/9 Kalimpong is at a lower elevation than Gangtok.
6. Bhaiclung was observing the pattern of people Pelling is at a lower elevation than Gangtok
entering and leaving a car service centre. There Pelling is at a higher elevation than Siliguri.
was a single window where customers were being Darjeeling is at a higher elevation than Gangtok.
served. He saw that people inevitably came out Which of the following statements can be inferred
of the centre in the order that they went in. from the paragraph above?
However, the time they spent inside seemed to (i) Pelling is at a higher elevation than
vary a lot some people came out in a matter of Kalimpong
minutes while for others it took much longer.
(ii) Kalimpong is at a lower elevation than
Darjeeling
2 SOLVED PAPER – 2017
(iii) Kalimpong is at a higher elevation than 15. Rahul, Murali, Srinivas and Arul are seated
Siliguri around a square table. Rahul is sitting to the
(iv) Siliguri is at a lower elevation than Gangtok. left of Murali. Srinivas is sitting to the right of
Arul. Which of the following pairs are seated
(a) Only (ii)
opposite each other?
(b) Only (ii) and (iii)
(a) Rahul and Murali
(c) Only (ii) and (iv)
(b) Srinivas and Arul
(d) Only (iii) and (iv)
(c) Srinivas and Murali
9. P, Q, R, S, T and U are sealed around a circular
(d) Srinivas and Rahul
table. R is seated two places to the right of Q. P
is seated three places to the left of R. S is seated 16. “The hold of the nationalist imagination on our
opposite U. If P and U now switch seats which of colonial past is such that anything inadequately
the following must necessarily be true. or improperly nationalist is just not history.”
(a) P is immediately to the right of R Which of the following statements best reflects
the author’s opinion?
(b) T is immediately to the left of P
(a) Nationalists are highly imaginative
(c) T is immediately to the left of P or P is
immediately to other right of Q. (b) History is viewed through the filter of
nationalism
(d) U is immediately to the right of R or P is
immediately to the left of T. (c) Our colonial past never happened
10. Budhan covers a distance of 19 km in 2 hours by (d) Nationalism has to be both adequately and
cycling one fourth of the time and walking the properly imagined.
rest. The next day he cycles (at the same speed ( x y) x y
as before) for half the time and walks the rest (at 17. The expression is equal to
2
the same speed as before) and covers 26 km in (a) the maximum of x and y
2 hours. The speed in km/h at which Budhan
walks is (b) the minimum of x and y
(a ) 1 (b) 4 (c ) 1
20. A test has twenty questions worth 100 marks in 26. An air pressure contour line joins locations in a
total. There are two types of questions. Multiple region having the same atmospheric pressure.
choice questions are worth 3 marks each and The following is an air pressure contour plot is a
essay questions are worth 11 marks each. How geographical region. Contour lines are shown at
many multiple choice questions does the exam 0.05 bar intervals in this plot.
have ?
R
(a) 12 (b) 15
0.65 0.9
(c) 18 (d) 19 0.7
5
S
0.9
0.95
21. Choose the option with words that are not
synonyms.
P 0.9
(a) aversion, dislike (b) luminous, radiant
(c) plunder, loot (d) yielding, resistant 0.8
0.8 Q
22. There are five buildings called V, W, X, Y and Z 0.75
in a row (not necessarily in that order). V is to
the West of W. Z is to the East of X and the West 1 2 3
of V. W is to the West of Y. Which is the building If the possibility of a thunderstorm is given by
in the middle ? how fast air pressure rises or drops over a region,
(a ) V which of the following regions is most likely to
(b) W have a thunderstorm ?
(c ) X (a ) P (b) Q
(d) Y (c ) R (d) S
23. There are 3 red socks, 4 green socks and 3 blue 27. The number of roots of ex + 0.5x2 – 2 = 0 in the
socks. You choose 2 socks. The probability that range [–5, 5] is
they are of the same colour is (a ) 0
(a) 1/5 (b) 1
(b) 7/30 (c ) 2
(c) 1/4 (d) 3
(d) 4/15 28. There are three boxes. One contains apples
24. Saturn is _____ to be seen on a clear night with another contains oranges and the last one
the naked eye. contains both apples and oranges. All three are
(a) enough bright known to be incorrectly labelled. If you are
(b) bright enough permitted to open just one box and then pull out
and inspect only one fruit, which box would you
(c) as enough bright
open to determine the contents of all three boxes?
(d) bright as enough
(a) The box labelled ‘Apples’
25. “We lived in a culture that denied any merit to
literary works, considering them important only (b) The box labelled ‘Apples and Oranges’
when they were handmaidens to somethings (c) The box labelled ‘Oranges’
seemingly more urgent – namely ideology. This (d) Cannot be determined
was a country where all gestures, even the most
29. X is a 30 digit number starting with the digit 4
private, were interprated in political terms.”
followed by the digit 7. Then the number X3 will
The author’s belief that ideology is not as have
important as literature is revealed by the word :
(a) 90 digits
(a) ‘culture’
(b) 91 digits
(b) ‘seemingly’
(c) 92 digits
(c) ‘urgent’
(d) 93 digits
(d) ‘political’
4 SOLVED PAPER – 2017
30. In the summer, water consumption is known to 36. “If you are looking for a history of India, or for an
decrease overall by 25%. A Water Board official account of the rise and fall of the British Raj, or
states that in the summer household for the reason of the cleaving of the subcontinent
consumption decreases by 20%, while other into two mutually antagonistic parts and the
consumption increases by 70%. effects this mutilation will have in the respective
Which of the following statements is correct? sections, and ultimately on Asia, you will not find
it in these pages; for though I have spent a
(a) The ratio of household to other consumption
lifetime in the country, I lived too near the seat
is 8/17.
of events, and was too intimately associated with
(b) The ratio of household to other consumption the actors, to get the perspective needed for the
is 1/17. impartial recording of these matters”.
(c) The ratio of household to other consumption Here, the word ‘antagonistic’ is closest in meaning
is 17/8. to
(d) There are errors in the official’s statement. (a) impartial (b) argumentative
31. Some tables are shelves. Some shelves are chairs. (c) separated (d) hostile
All chairs are benches. Which of the following
37. A contour line joins locations having the same
conclusions can be deduced from the preceding
height above the mean sea level. The following is
sentences?
a contour plot of a geographical region. Contour
(i) At least one bench is a table lines are shown at 25 m intervais in this plot.
(ii) At least one shelf is a bench
(iii)At least one chair is a table
(iv)All benches are chairs
(a) only (i) (b) only (ii)
(c) only (ii) and (iii) (d) only (iv)
32. I _______ made arrangements had I _______
informed earlier.
The path from P to Q is best described by
(a) could have, been (b) would have, being
(a) Up-Down-Up-Down
(c) had, have (d) had been, been
(b) Down-Up-Down-Up
33. 40% of deaths on city roads may be attributed to
(c) Down-Up-Down
drunken driving. The number of degrees needed
(d) Up-Down-Up
to represent this as a slice of a pie chart is
38. There are 3 Indians and 3 Chinese in a group of
(a) 120 (b) 144
6 people. How many subgroups of this group can
(c) 160 (d) 212 we choose so that every subgroup has at least
34. She has a sharp tongue and it can occasionally one Indian?
turn ________. (a) 56 (b) 52
(a) hurtful (b) left (c) 48 (d) 44
(c) methodical (d) vital 39. S, T, U, V, W, X, Y and Z are seated around a
circular table. T’s neighbours are Y and V. Z is
35. Trucks (10 m long) and cars (5 m long) go on a
seated third to the left of T and second to the
single lane bridge. There must be a gap of at least
right of S. U’s neighbours are S and Y; and T and
20 m after each truck and a gap of at least 15 m W are not seated opposite each other. Who is third
after each car. Trucks and cars travel at a speed to the left of V?
of 36 km/h. If cars and trucks go alternately. What
(a ) X
is the maximum number of vehicles that can use
(b) W
the bridge in one hour?
(c) U
(a) 1440 (b) 1200
(d ) T
(c) 720 (d) 600
SOLVED PAPER – 2017 5
49. “Here, throughout the early 1820s, Stuart (a) Start’s commander-in-chief was moved by this
continued to fight his losing battle to allow his demonstration of his prejudice
sepoys to wear their caste- marks and their own (b) The Europeans were accommodating of the
choice of facial hair on parade, being again sepoys’ desire to wear their caste-marks
reprimanded by the commander-in-chief. His
(c) Stuart’s losing battle’ refers to his inability to
retort that “A stronger instance than this of
succeed in enabling sepoys to wear cast-marks
European prejudice with relation to this
country has never come under my observations (d) The commander-in-chief was exempt from the
“had no effect on his superiors” European prejudice that dictated how the
sepoys were to dress.
According to this paragraph, which of the
statements below I most accurate?
ANSWERS
1. (b) 2. (b) 3. (a) 4. (b) 5. (b) 6. (a) 7. (d) 8. (c) 9. (c) 10. (d)
11. (c) 12. (d) 13. (c) 14. (d) 15. (d) 16. (b) 17. (b) 18. (d) 19. (a) 20. (b)
21. (d) 22. (a) 23. (d) 24. (b) 25. (c) 26. (c) 27. (c) 28.(a) 29. (a) 30. (d)
31. (b) 32. (a) 33. (b) 34. (a) 35. (a) 36. (d) 37. (c) 38. (a) 39. (a) 40. (d)
41. (d) 42. (b) 43. (a) 44. (d) 45. (a) 46. (8) 47. (c) 48. (b) 49. (c)
SOLVED PAPER – 2017 7
E XPLANATIONS
x 2 2 x 4 8. K < G, P < G, P > S, G < D
16 3
1. 81 = 144 if K < G and G < D
25 5
2 x 4 2 x 4
Hence, K < D
4 5 144
5 = If S < P and P < G
3 81
Hence, S < G
2 x 4
4 144 Hence, (ii) and (iv) are correct.
3 =
81 9.
x 2
4 144
3 =
81
x
4 16 12
3 9 = 9
x
4 3
3 =
4
x 1
4 4
3 =
3
10. Let C = Speed by cycling (km/h)
x =–1
W = Speed of walking (km/h)
3. Card with visible face 2 should have red on the Now, by according to questions,
other side.
2 3
Card with visible face 3 may have any color on C W2 = 19
4 4
other side.
C + 3W = 38 ...(i)
Card with visible face red should have even
2 2
number on other side. and, C W = 26
2 2
Card with visible face blue may have any number
C + W = 26 ...(ii)
on other side.
By solving equations (i) and (ii) we get,
Hence, to verify the above proposition, cards with
W = 6 km/h
face 2 and Red must be turned.
11.
5. Favourable outcomes = {(1, 1), (1, 4), (2, 2), (3, 3),
(4, 1), (4, 4), (5, 5), (6, 6)}
No. of favourable outcomes = 8 Each digit can be filled in 7 ways as 0, 5 and 9 is
not allowed, so each of these places can be filled
Total outcomes = 36
by 1, 2, 3, 4, 6, 7, 8 so required probability is
8 2 K
Probability = 7
36 9 10 or 0.7K
7. The elevators has moved from 2nd to 5th floor
between time 25 min to 30 min. fC 7K
= = 0.7K
TC 10K
Elevation stayed at 4th floor for
12. y 162 = Perfect cube
= 3 + 4 + 3 + 2 + 3 + 2 + 2 = 19 min
Option, y = 24 23 3 (2 81) Not perfect cube
Elevation stayed at ground floor for
y = 27 33 (2 34) Not perfect cube
=1+2+1+1+1+1+2+1+2+1+1+1 y = 32 25 2 34 Not perfect cube
+1+2+1+1+1 y = 36 22 32 2 34
= 21 min. = 23 36 = (2 32)3
Hence, (i) and (ii) both are false. is a perfect cube Hence the answer is, y = 36
8 SOLVED PAPER – 2017
13. Correct option is; Wished 18. As there are 4 people A, G, N, S and four colours
After Rajender Chola retured from his voyage so without any restriction total ways have to be
to Indonesia, he wished to visit the temple in 4 4 = 16.
Thanjavur. Now, Arun dislikes Red and Shweta dislikes
white so 16 – 2 = 14 ways.
Both are events of past. Use of past perfect form
is unwarranted as it reflects part of past. Therefore correct answer should be option (d).
14. Besides money Alternate solution:
Research in the work place reveals the people Only one option is less than 16.
works for many reasons b es i de s mo n ey. Therefore correct answer should be option (d).
Besides conveys the meaning of ‘in addition’ 19. Out of six people. 3 place definitely occupied by
Beside means ‘next to’ right handed people as at least 2 women are
there so these two will sit adjacently. Now as only
15. Following seating arrangement can be drawn
on seat is left it will be occupied by a left handed
man because on right side of this seat is sitting
an right handed man.
Probability that 2 socks choosen are of both Red 35. According to question,
or Both Green or Both blue Length of Truck + gap required = 10 + 20 = 30 m
3 2 4 3 3 2 and
Same colour =
10 9 10 9 10 9 Length of Car + gap required = 5 + 15 = 20 m
6 12 6 Alternative pairs of Truck and Car needs
=
90 90 10 30 + 20 = 50 m
24 Let ‘n’ be the number of repetition of (Truck
=
90 + Car) in 1 hour (3600 secs)
4 Given speed = 36 km/hr
=
15 = 10 m/sec
Option (d) is correct.
50m n
24. Saturn is bright enough to be seen on a clear night = 36 km/hr
3600 secs
with the naked eye.
Option (b) is correct 50n
m/sec = 10 m/sec
3600
25. The author’s belief that ideology is not as
important as literature is revealed by the word 36000
n=
‘urgent’ in above paragraph. 50
Option (c) is correct. = 720 (Trucks + Cars)
27. ex + 0.5x2 – 2 = 0 So, 720 (Trucks + Cars) passes
Differention w.r.t to ‘x’ we get = (720 2)
= 1440 vehiclesEnd
ex + x = 0
36. Antagonist is a adversary or one who opposes/
Hence ‘x’ has total number of root which lies
contends against another. Hence the closest
in the range [– 5,5] is ‘2’.
meaning to word “antagonistic” is hostile.End of
30. Let H is household consumption and P be other
37. Contour lines can be observed to cross region with
consumption
height
Now according to question, 550 – 500 – 575 – 550
H 0.8 + P 1.7 = (H + P) 0.75
From this it is concluded that ratio is negative. Down UP Down
31. From given condition Down-Up-Down satisfies.d of Solution
38. Subgroups containing only Indians
T S C = 3C1 + 3C2 + 3C3
=3+3+1=7
B
Only conclusion (ii) follows and Subgroups containing one Indian and rest
Chinese
32. Use of conditional sentence based on past
participle form. = 3C1[3C1 + 3C2+ 3C3]
I could have made arrangements had I been = 3[3 + 3 + 1]
informed earlier. = 21
40 and Subgroups containing two Indian and
33. 40% of 360 = 360 remaining Chinese
100
= 144 = 3C2[3C1 + 3C2 + 3C3]
So, the angle subtended on pie chart will be 144 = 21
34. Hurtful means causing pain or suffering or and Subgroups containing three Indian and
something that is damaging or harmful. The remaining Chinese
expression ‘sharp tongue’ defines a bitter or = 3C3[3C1 + 3C2 + 3C3]
critical manner of speaking. =7
10 SOLVED PAPER – 2017
47.
100 1
Median = = 50.5
2
From cumulative column,
Median salary = 30000
SOLVED PAPER – 2018
GENERAL APTITUDE
S E T -1 8. To pass a test, a candidate needs or answer at
1. “When she fell down the _______. She received least 2 out of 3 questions correctly. A total of
many _________ but little help.” The words that 6,30,000 candidates appeared for the test.
best fill the blanks in the above sentence are Question A was correctly answered by 3,30,000
candidates. Question B was answered correctly
(a) stairs, stares (b) stairs, stairs
by 2,50,000 candidates. Question C was answered
(c) stares, stairs (d) stares, stares
correctly by 2,60,000 candidates. Both questions
2. “In spite of being warned repeatedly, he failed to A and B were answered correctly by 1,00,000
correct his ___________ behaviour.” The word candidates. Both questions B and C were
that best fills that blank in the above sentence is answered correctly by 90,000 candidates. Both
(a) rational (b) reasonable questions A and C were answered correctly by
(c) errant (d) good 80,000 candidates. If the number of students
3. For 0 x 2, sinx and cosx are both decreasing answering all questions correctly is the same as
functions in the interval __________. the number answering none, how many
candidates failed to clear the test?
(a) 0, (b) , (a) 30,000 (b) 2,70,000
2 2
(c) 3,90,000 (d) 4,20,000
3
(c) , (d) , 2 1
2 2 9. If x2 + x – 1 = 0 what is the value of x 4 ?
x4
4. The area of an equilateral triangle is 3 . What
(a) 1 (b) 5
is the perimeter of the triangle?
(c) 7 (d) 9
(a) 2 (b) 4
10. In a detailed study of annual crow births in India,
(c) 6 (d) 8
it was found that there was relatively no growth
5. Arrange the following three-dimensional objects during the period 2002 to 2004 and a sudden spike
in the descending order of their volumes: from 2004 to 2005. In another unrelated study, it
(i) A cuboid with dimensions 10 cm, 8 cm and 6 cm was found that the revenue from cracker sales in
(ii) A cube of side 8 cm India which remained fairly flat from 2002 to
(iii)A cylinder with base radius 7 cm and height 2004, saw a sudden spike in 2005 before declining
7 cm again in 2006. The solid line in the graph below
refers to annual sale of crackers and the dashed
(iv) A sphere of radius 7 cm
line refers to the annual crow births in India.
(a) (i), (ii), (iii), (iv) (b) (ii), (i), (iv), (iii) Choose the most appropriate inference from the
(c) (iii), (ii), (i), (iv) (d) (iv), (iii), (ii), (i) above data.
6. An automobile travels from city A to city B and
Annual sale of crackers in India
(a) There is a strong correlation between crow 7. The price of a wire made of a superalloy material
birth and cracker sales. is proportional to the square of its length. The
(b) Cracker usage increases crow birth rate. price of 10 m length of the wire is Rs. 1600. What
(c) If cracker sale declines, crow birth will decline. would be the total price (in Rs.) of two wires of
lengths 4 m and 6 m?
(d) Increased birth rate of crows will cause an
increase in the sale of crackers. (a) 768 (b) 832
S E T -2 (c) 1440 (d) 1600
1. “The driver applied the ________ as soon as she 8. What of the following function(s) in an
approached the hotel where she wanted to take accurate description of the graph for the range(s)
a ________.” indicated?
The words that best fill the blanks in the above y
sentence are 3
(a) brake, break (b) break, break 2
(c) brake, brake (d) break, brake
1
2. “It is no surprise that every society has had codes –3 –2 –1 0 1 2 3 x
of behavior; however, the nature of these codes –1
is often _________.”
–2
The word that best fills the blank in the above
sentence is –3
(a) unpredictable (b) simple (i) y = 2x + 4 for –3 x – 1
(c) expected (d) strict (ii) y = x – 1 for –1 x 2
3. Hema’s age is 5 years more than twice Hari’s (iii) y = x–1 for –1 x 2
age. Suresh’s age is 13 years less than 10 times (iv) y = 1 for 2 x 3
Hari’s age. If Suresh is 3 times as old as Hema.
(a) (i), (ii) and (iii) only
How old is Hema?
(a) 14 (b) 17 (b) (i), (ii) and (iv) only
(c) 18 (d) 19 (c) (i) and (iv) only
4. Tower A is 90 m tall and tower B is 140 m tall. (d) (ii) and (iv) only
They are 100 m apart. A horizontal skywalk 9. Consider a sequence of number a1, a2, a3, ...., an
connects the floors at 70 m in both the towers. If 1 1
a taut rope connects the top of tower A to the where an , for each integer n > 0.
n n2
bottom of tower B, at what distance (in meters) What is the sum of the first 50 terms?
from tower A will the rope intersect the skywalk?
1 1 1 1
5. The temperature T in a room varies as a function (a) 1 (b) 1
of the outside temperature T0 and the number of 2 50 2 50
persons in the room p, according to the relation 1 1 1 1 1
(c ) 1 (d) 1
T = K(p + T0), where is K are constants. What 2 51 52 51 52
would be the value of given the following data?
10. Each of the letters arranged as below represents
T0 p T a unique from 1 to 9. The letters are positioned
25 2 32.4 in the figure such that (A × B × C), (B × G × E) and
30 5 42.0 (D × E × F) are equal. Which integer among the
(a) 0.8 (b) 1.0 following choices cannot be represented by the
(c) 2.0 (d) 10.0 letters A, B, C, D, E, F or G?
5. As 1.5 m in tall person is standing at a distance of (iii) It is known that a witness can correctly
3 m from a lamp post. The light from the lamp at identify the cab colour only 80% of the time.
the top of the post casts her shadow. The length Which of the following options is closest to the
of the shadow is twice her height. What is the probability that the accident was caused by a blue
height of the lamp post in meters?
cab?
(a) 1.5 (b) 3
(a) 12% (b) 15%
(c) 4.5 (d) 6
(c) 41% (d) 80%
6. Leila aspires to buy a car worth Rs. 10,00,000
10. A coastal region with unparalleled beauty is home
after 5 years. What is the minimum amount in
Rupees that she should deposit now in a bank to many species of animals. It is dotted with coral
which offers 10% annual rate of interest, if the reefs and unspoilt white sandy beaches. It has
interest was compounded annually? remained inaccessible to tourists due to poor
connectivity and lack of accommodation. A
(a) 5,00,000 (b) 6,21,000
company has spotted the opportunity and is
(c) 6,66,667 (d) 7,50,000 planning to develop a luxury resort with
7. Two alloys A and B contain gold and copper in the helicopter service to the nearest major city
ratios of 2 : 3 and 3 : 7 by mass, respectively. Equal airport. Environmentalists are upset that this
masses of alloys A and B are melted to make an would lead to the region becoming crowded and
alloy C. The ratio of gold to copper in alloy C is polluted like any other major beach resorts.
.
Which one of the following statements can be
(a) 5 : 10 (b) 7 : 13 logically inferred from the information given in
(c) 6 : 11 (d) 9 : 13 the above paragraph?
8. The Cricket Board has long recognized John’s (a) The culture and tradition of the local people
potential as a leader of the team. However, his will be influenced by the tourists.
on-field Temper has always been a matter of
(b) The region will become crowded and polluted
concern for them since his junior days. While this
due to tourism.
aggression has filled stadia with die-hard fans, it
has taken a toll on his own batting. Until recently, (c) The coral reefs are on the decline and could
it appeared that he found it difficult to convert soon vanish.
his agression into big scores. Over the past three (d) Helicopter connectivity would lead to an
seasons though, that picture of John has been increase in tourists coming to the region.
replaced by a cerebral, calculative and successful
batsman-captain. After many years, it appears S E T -5
that the team has finally found a complete captain. 1. Going by the ______ that many hands make light
Which of the following statements can be logically work, the school ______ involved all the students
inferred from the above paragraph? in the task.”
(i) Even as a junior cricketer, John was (a) principle, principal
considered a good captain. (b) principal, principle
(ii) Finding a complete captain is a challenge. (c) principle, principle
(iii) Fans and the Cricket Board have differing (d) principal, principal
views on what they want in a captain.
2. “Her ____ should not be confused with miserliness;
(iv) Over the past three seasons John has she is ever willing to assist those in need.”
accumulated big scores.
The word that best fills the blank in the above
(a) (i), (ii) and (iii) only (b) (iii) and (iv) only sentence is
(c) (ii) and (iv) only (d) (i), (ii), (iii) and (v) (a) cleanliness (b) punctuality
9. A cab was involved in a hit and run accident at
(c) frugality (d) greatness
right you are given the following data about the
cabs in the city and the accident. 3. Seven machines take 7 minutes to make 7 identical
toys. At the same rate, how many minutes would
(i) 85% of cabs in the city are green and the
it take for 100 machines to make 100 toys?
remaining cabs are blue.
(a) 1 (b) 7
(ii) A witness identified the cab involved in the
accident as blue. (c) 100 (d) 700
SOLVED PAPER – 2018 5
4. A rectangle becomes a square when its length 10. Consider the following three statements:
and breadth are reduced by 10m and 5 m, (i) Some roses are red
respectively, during this process, the rectangle (ii) All red flowers fade quickly
loses 650 m2 of area. What is the area of the
(iii) Some roses fade quickly
original rectangle in square meters?
Which of the following statements can be logically
(a) 1125 (b) 2250 inferred from the above statements?
(c) 2924 (d) 4500 (a) If (i) is true and (ii) is false, then (iii) is false.
5. A number consists of two digits. The sum of the (b) If (i) is true and (ii) is false, then (iii) is true.
digits is 9. If 45 is subtracted from the number, (c) If (i) and (ii) are true, then (iii) is true.
its digits are interchanged. What is the number? (d) If (i) and (ii) are false, then (iii) is false.
(a) 63 (b) 72
S E T -6
(c) 81 (d) 90
1. “Since you have gone off the ______, the ______
6. For integers a, b and c, what would be the sand is likely to damage the car”. The words that
minimum and maximum values respectively of best fill the blanks in the above sentence are
a + b + c if log |a| + log |b| + log |c| = 0 (a) course, coarse (b) course, course
(a) –3 and 3 (b) –1 and 1 (c) coarse, course (d) coarse, coarse
(c) –1 and 3 (d) 1 and 3 2. “A common misconception among writers is that
7. Given that a and b are integers and a + a2b3 is sentence structure mirrors thought; the more _____
odd, which one of the following statements is the structure, the more complicated the ideas”.
correct? (a) detailed (b) simple
(a) a and b are both odd (c) clear (d) convoluted
(b) a and b are both even 3. The three roots of the equation f(x) = 0 are x =
{–2, 0, 3}. What are the three values of x for which
(c) a is even and b is odd f(x – 3) = 0?
(d) a is odd and b is even (a) –5, –3, 0 (b) –2, 0, 3
8. From the time the front of train enters a platform, (c) 0, 6, 8 (d) 1, 3, 6
it takes 25 seconds for the back of the train to
leave the platform, while travelling at a constant 4. For what values of k given below is
k 2 2 and
speed of 54 km/h. At the same speed, it takes 14 k 3
seconds to pass a man running at 9 km/h in the integer?
same direction as the train. What is the length of (a) 4, 8, 18 (b) 4, 10, 16
the train and that of the platform in meters, (c) 4, 8, 28 (d) 8, 26, 28
respectively? 5. Functions, F(a, b) and G(a, b) are defined as
(a) 210 and 140 (b) 162.5 and 187.5 follows:
(c) 245 and 130 (d) 175 and 200 F(a, b) = (a – b)2 and G(a, b) = |a – b|, where |x|
represents the absolute value of x. What would
9. Which of the following functions describe the be the value of G(F(1, 3), G(1, 3))?
graph shown in the below figure.
(a) 2 (b) 4
y (c) 6 (d) 36
3 6. An e-mail password must contain three
2 characters. The password has to contain one
numeral from 0 to 9, one upper case and one
1
lower case character from the English alphabet.
–3 –2 –1 0 1 2 3x How many distinct passwords are possible?
–1 (a) 6,760 (b) 13,520
–2 (c) 40,560 (d) 1,05,456
–3 7. In a certain code AMCF is written as EQGJ and
NKUF is written as ROYJ. How will DHLP be
(a) y x 1 2 (b) y x 1 1 written in the code?
(a) RSTN (b) TLPH
(c) y x 1 1 (d) y x 1 1
(c) HLPT (d) XSVR
6 SOLVED PAPER – 2018
8. A class of twelve children has two more boys than (a) 35.75 (b) 40.25
girls. A group of three children are randomly (c) 43.75 (d) 46.25
picked from this class to accompany the teacher
10. P, Q, R and S crossed a lake in a boat that can
on a field trip. What is the probability that the
group accompanying the teacher contains more hold a maximum of two persons, with only one
girls than boys? set of oars. The following additional facts are
available
325
(a) 0 (b) (i) The boat held two persons on each of the three
864 forward trips across lake and one person on
525 5 each of the two return trips.
(c) (d)
864 12 (ii) P is unable is row when someone else is in
9. A designer uses marbles of four different colours the boat.
for his designs. The cost of each marble is the (iii)Q is unable to row with anyone else except R.
same, irrespective of the colour. The table below
(iv) Each person rowed for at least one trip.
shows the percentage of marbles of each colour
used in the current design. The cost of each (v) Only one person can row during a trip.
marble increased by 25%. Therefore, the designer Who rowed twice?
decided to reduce equal numbers of marbles of (a) P (b) Q
each colour to keep the total cost unchanged.
(c) R (d) S
What is the percentage of blue marbles in the
new design?
Blue Black Red Yellow
40% 25% 20% 15%
ANSWERS
GENERAL APTITUDE
S E T -1 3 2
4. Area of equilateral triangle = a
1. (a) 2. (c) 3. (b) 4. (c) 5. (d) 4
6. (a) 7. (c) 8. (d) 9. (c) 10. (a) 3 2
3 = a
4
EXPLANATIONS a2 = 4
1. Stares - To look at someone for a long time a=2
Stairs - A construction designed to bridge a large Now perimeter of equilateral triangle
vertical distance by dividing it into smaller = 3a = 3 × 2 = 6
vertical distances, called steps 5. (i) Volume of cuboid = (8 × 10 × 6) cm3 = 480 cm3
2. Errant means misbehaving, exhibiting
(ii) Volume of cube = (8 × 8 × 8) cm3 = 512 cm3
inappropriate behaviour / offending conduct.
3. (iii) Volume of cylinder = r 2 h
cos x
22
7 7 7 7 cm3 = 1078 cm3
3
4 3 4 2
2 (iv) Volume of Sphere r 7 cm3
0 x 3 3
2 = 1436.75 cm3
sin x Hence, the descending order of their volumes is
From the curve it is clear that sin x and cos x (iv), (iii), (ii), (i).
both are decreasing in the interval ,
2
SOLVED PAPER – 2018 7
2(S1 S2 ) 1 1
2
6. Average speed = x4 = x2
S1 S2 Now, 2
x4 x2
2(60 90) = (3)2 – 2
= km/h
(60 90) =9–2
2 60 90
= 72 km / h =7
150
7. Number of parallelogram = 4C2 × 5C2 ANSWERS
4! 5! S E T -2
=
2! 2! 3! 2! 1. (a) 2. (a) 3. (d) 4. 22.22 5. (b)
54 6. (c) 7. (b) 8. (b) 9. (c) 10. (b)
= 6
2
= 60 EXPLANATIONS
100,000 3. Let the age of Hari be ‘x’
8. A B
(3,30,000) (2,50,000) Hema age = 5 + 2x
and Suresh age = 10 x – 13
Now Suresh is 3 time old as Hema
150000 + y 100000 – y 60000 + y (10x – 13) = 3(5 + 2x)
10x – 13 = 15 + 6x
y
4x = 28
80000 – y 90000 – y
x =7
80,000
90,000 Hema’s age = 5 + 2x
= 5 + 14
90000 + y C
(2,60,000) = 19
4. B
140 m
6,30,000 = 2y + 1,50,000 + 100000 + 80000 20 m E
M
90 m
1 1 1 1 –1 –1 –1 –1 –1 –1
49 51 50 52 So, 1440 must be answer.
All like terms will cancel out and we will be left 6. Male (M) + Female (F) = 300
with 8 2
Total money = M 750 F 1000 ...(i)
1 1 1 9 3
= 1
2 51 52 = 6000 M 6000 F = 6000 M + F
10. A×B×C=B×G ×E 9 9 9
=D×E×F From equation (i) we get,
= 72 6000 300
Total money = 2,00,000
8×9×1=9× 2× 4 9
7. px = q x log p = log q
=3×4×6
and qy = r y log q = log r
= 72 and z2 = p z log r = log p
Any of A, B, C, D, E, F, G cannot be 5.
log q log r log p
xyz = =1
log p log q log r
SOLVED PAPER – 2018 9
3 7 3 7
Option c is maximum value.
ANSWERS
So, male attended the party 80 – 40 = 40
S E T -4
Ratio of male to female who attended the party
1. (b) 2. (a) 3. (b) 4. (d) 5. (b)
M : W = 40 : 40
6. (b) 7. (b) 8. (c) 9. (a) 10. (b)
=1:1
9. E EXPLANATIONS
3. Rule of divisibility by 3 : Sum of all digits should
be divisible by 3
D C 715 ? 423
p F
q 7 + 1 + 5 + 4 + 2 + 3 = 22
B So, the next number after sum 22 which are
A divisible by 3 are 24, 27, 30 etc.
E + F = ? So, 22 + 2 = 24
+ q + E = 180 ....(i) 22 + 5 = 27
and + + F = 180 ....(ii) But according to given condition 2 is right answer.
and + + p + q = 360 ....(iii) 4. The given series is in G.P,
Adding equations (i) and (ii) and equating it to (iii) 1
we get where a = 1 and r =
4
+q+E+++F=++p+q a
Now, in G.P, sn =
E+F=p– 1 r
10. Four green, two red face 1 1 4
=
4 2 1 3/4 3
P(G) = 1
6 3 4
1 5. A
q(R) = 1 – P(G) =
3
n=7 D
Now from options
3 4 1.5
P(G = 3) = 7 C3
2 1
From option (a),
3 4
B E C
35 23 35 23 3 3
= Let ‘AB’ be the height of lamp post
3 7 3 7
4 3 Here ABC and DEC are similar triangles,
2 1
7
From option (b), P(G = 4) = C4
3 3 AB BC
=
35 2 4
35 2 4 DE EC
= AB 6
37
3 7 =
5 2 1.5 3
7 2 1
From option (c), P(G = 5) = C5 AB
3 3 =2
1.5
21 2 = 42 2
5 4
AB = 3 meter
3 7 3 7
Hence height of lamp post is 3 m.
10 SOLVED PAPER – 2018
6. P:A
EXPLANATIONS
(10)5 : (11)5
Machine time M t
(10)6 (10)5 3. = 1 1
P= Number of toys T1
(11)5
M2 t2
11 =
(10) T2
=
(11)5 77 100 t2
= 620921.323 621000 =
7 100
7. A B t2 7 min
G:C G:C
4. Let the initial length of rectangle is l and breath
2:3 3:7 is b. Now if length is reduced by 10 m and breath
Now making the mass equal of alloy A and B. is reduced by 5 m then it becomes square.
A B l – 10 = b – 5
G:C G:C [ In square length = breadth]
4:6 3:7 l–b=5
Alloy of C contain Given, initial, Area (A) = l × b
G : C = 4 + 3 : 6 + 7 = 7 : 13 Now according to question,
8. Statement (i) is not true as nowhere it is (l – 10) (b – 5) = A – 650
mentioned that John was a captain in junior team. lb – 10b – 5l + 50 = A – 650
The introductory line emphasizes on the board
recognizing John’s potential (Latent quality/ A – 10b – 5l = A – 700
possibility) as leader of the team. 10b + 5l = 700
Statement (iii) also manipulates the facts 10b + 5 (b + 5) = 700
mentioned in the argument. 15b + 25 = 700
The 3rd statement of the argument while this 15b = 700 – 25
aggression has filled stadia with die-hard fans does
not indicate fans expectations from John as a 15b = 675
caption. b = 45 m
Statement (ii) The concluding statement of the l = 45 + 5 = 50 m
para suggests that finding a completer captain is Area of original rectangle (A) = (45 × 50)m2
a tough task as it look John many years to become
= 2250 m2.
a successful and calculative batsman - captain.
5. Let two digit number be xy.
Statement (iv) can be explicity concluded from
the last 4 lines of the para. Now sum of digit is 9
9. Probability that accident was caused by blue cab x+y=9 ....(i)
= 0.15 × 0.8 = 0.12 = 12% and when ‘45’ is subtracted from number then
10. (A) is beyond the scope of given information option digit gets interchanged
(c) can also be discarded on the same grounds. 10x + y – 45 = 10y + x
The argument deals with the coastal region x –5=y
becoming crowded and polluted because of the x–y=5 ....(ii)
upcoming luxury resort. Option (b) precisely
underlines the theme of the para. Now solving equation (i) and (ii) we get
x = 7 and y = 2
ANSWERS Hence the number is 72.
S E T -5 6. log a + log b + log c = 0
1. (a) 2. (c) 3. (b) 4. (b) 5. (b) It is possible only,
when a, b and c all are equal to 1.
6. (a) 7. (d) 8. (d) 9. (b) 10. (c)
a, b, c may be 1, 1, 1 respectively.
SOLVED PAPER – 2018 11
Now for minimum value of all three will be 9. Here from graph for x = 1, y = – 1
negative. Checking the same from options.
minimum value = – 3 From option (a)
and maximum value of all three will be positive. for x = 1
maximum value = 3
y = 1 1 2
7. From option (a),
=2–2=0
Let a = 1, b = 3
Option (a) is wrong
a + a2b3 = 1 + 27
From option (c)
= 28 (Even)
for x=1
Option (a) is wrong
y = x 1 1
From option (b)
Let a = 2, b = 4 = 1 1 1 1
a + a2b3 = 2 + 4 64 Option (c) is wrong.
= 258 (Even) 10.
From option (c), Roses
Let a = 2, b = 1 Fade
Red quickly
From option (d),
for x = 1
Case-I
y = x 1 1
Fade
= 1 1 quickly
Option (c) is wrong.
Roses/Red
Now from option (b),
for x=1
y = x 1 1
Case-II
= 1 1 1 Roses
=–1 Fade
Option (b) is correct.
8. Train speed (ST) = 54 km/h
Time = 25 sec for travelling length of train and
length of platform
Man speed (SM) = 9 km/h
Case-III
Speed of train to man = 45 km/h
Time = 14 sec ANSWERS
So, length of train = time × speed S E T -6
5 1. (a) 2. (d) 3. (d) 4. (c) 5. (a)
= 14 45
18 6. (c) 7. (c) 8. (b) 9. (c) 10. (c)
Length of train (LT) = (35 × 5 m) = 175 m
Length of platform (L) + length of train (LT) EXPLANATIONS
= speed × time 3. f(x) = 0
5 for, x = {–2, 0, 3}
= 54 25 15 25 375m
18 f(–2) = 0, f(0) = 0, f(3) = 0
Length of platform (L) = 375 – 175 = 200 m f(x – 3) = 0
12 SOLVED PAPER – 2018
20 Administrators
Garden
N E
20
Researchers Entrance
40
(a) 0 to 15 W S Y
(b) 31 to 45
Garden
(c) 46 to 60 WR X
(d) 16 to 30
SOLVED PAPER – 2019 3
(c) S E T -3
WR
Q. 1 to Q. 5 : Carry One Mark Each.
Garden
N
1. The passengers were angry _______ the airline
E
staff about the delay.
(a) towards (b) on
Entrance
(c) with (d) about
W S 2. The missing number in the given sequence
Y
343, 1331, ________, 4913 is
Garden
(a) 4096 (b) 3375
WR X Z
(c) 2744 (d) 2197
(d) 3. Newspapers are a constant source of delight and
WR X recreation for me. The _______ trouble is that I
read ______ many of them.
Garden
WR
(a) 60 (b) 80
(c) 120 (d) 90
9 . The police arrested four criminals – P, Q, R and 5. I am not sure if the bus that has been booked will
S. The criminals knew each other. They made be able to _____ all the students.
the following statements :
(a) deteriorate (b) sit
P says ‘‘Q committed the crime.’’
(c) accommodate (d) fill
Q says ‘‘S committed the crime.’’
Q. 6 to Q. 10 : Carry Two Marks Each.
R says ‘‘I did not do it.’’
6. Given two sets X = {1, 2, 3} and Y = {2, 3, 4}, we
S says ‘‘What Q said about me is false.’’ construct a set Z of all possible fractions where
Assume only one of the arrested four committed the numerators belong to set X and the
the crime and only one of the statements made denominators belong to set Y. The product of
above is true. Who committed the crime? elements having minimum and maximum values
(a) R (b) P in the set Z is _________.
(c) Q (d) S 1 3
(a) (b)
10. ‘‘A recent High Court judgement has sought to 12 8
dispel the idea of begging as a disease –– which 1 1
leads to its stigmatization and criminalization –– (c) (d)
8 6
and to regard it as a symptom. The underlying
7. Consider five people-Mita, Ganga, Rekha, Lakshmi
disease is the failure of the state to protect citizens
and Sana. Ganga is taller than both Rekha and
who fall through the social security net.’’
Lakshmi. Lakshmi is taller than Sana. Mita is
Which one of the following statements can be taller than Ganga.
inferred from the given passage?
Which of the following conclusions are TRUE?
(a) Begging is an offence that has to be dealt with
1. Lakshmi is taller than Rekha
firmly
2. Rekha is shorter than Mita
(b) Beggars are lazy people who beg because they
are unwilling to work 3. Rekha is taller than Sana
(c) Beggars are created because of the lack of 4. Sana is shorter than Ganga
social welfare schemes (a) 3 only (b) 1 only
(d) Beggings has to be banned because it adversely (c) 2 and 4 (d) 1 and 3
affects the welfare of the state
4 SOLVED PAPER – 2019
8 . How many integers are there between 100 and 3. It would take one machine 4 hours to complete a
1000 all of whose digits are even? production order and another machine 2 hours
(a) 60 (b) 100 to complete the same order. If both machines
work simultaneously at their respective constant
(c) 90 (d) 80
rates, the time taken to complete the same order
9. An award-winning study by a group researchers is _______ hours.
suggests that men are as prone to buying on
(a) 2/3 (b) 7/3
impulse as women but women feel more guilty
about shopping. (c) 4/3 (d) 3/4
Which one of the following statements can be 4. When he did not come home, she _______ him
inferred from the given text? lying dead on the roadside somewhere
(a) Many men and women indulge in buying on (a) concluded (b) pictured
impulse (c) notice (d) looked
(b) All men and women indulge in buying on 5. The strategies that the company _______ to sell
impulse its products ________ house-to-house marketing.
(c) Few men and women indulge in buying on (a) uses, include (b) use, includes
impulse (c) uses, including (d) used, includes
(d) Some men and women indulge in buying on Q. 6 to Q. 10 : Carry Two Marks Each.
impulse
6. ‘‘Indian history was written by British historians–
10. The ratio of the number of boys and girls who extremely well documented and researched, but
participated in an examination is 4 : 3. The total not always impartial. History had to serve its
percentage of candidates who passed the purpose: Everything was made subservient to the
examination is 80 and the percentage of girls who glory of the Union Jack. Latter-day Indian scholar
passed is 90. The percentage of boys who passed presented a contrary picture.’’
is _________.
From the text above, we can infer that :
(a) 90.00
Indian history written by British historians ____.
(b) 80.50
(a) was well documented and not researched but
(c) 55.50 was always biased
(d) 72.50 (b) was not well documented and researched and
S E T -4 was sometimes biased
Q. 1 to Q. 5 : Carry One Mark Each. (c) was well documented and researched but was
sometimes biased
1. Five different books (P, Q, R, S, T) are to be
arranged on a shelf. The books R and S are to (d) was not well documented and researched and
be arranged first and second, respectively was always biased
from the right side of the shelf. The number of 7. Two design consultants, P and Q, started working
different order in which P, Q and T may be from 8 AM for a client. The client budgeted a total
arranged is _________. of USD 3000 for the consultants. P stopped working
(a) 2 when the hour hand moved by 210 degrees on the
clock. Q stopped working when the hour hand moved
(b) 120
by 240 degrees. P took two tea breaks of 15 minutes
(c) 6 each during her shift, but took no lunch break. Q
(d) 12 took only one lunch break for 20 minutes, but no
2. The boat arrived _________ dawn. tea breaks. The market rate for consultants is USD
(a) on 200 per hour and breaks are not paid. After paying
the consultants, the client shall have USE _____
(b) at
remaining in the budget.
(c) under
(a) 000.00 (b) 433.33
(d) in
(c) 166.67 (d) 300.00
SOLVED PAPER – 2019 5
(c)
Panel (a)
(d)
9. Four people are standing in a line facing you. They Panel (b) Panel (c)
are Rahul, Mathew, Seema and Lohit. One is an (a) 33.43
engineering, one is a doctor, one a teacher and (b) 35.43
another a dancer. You are told that:
(c) 34.43
1. Mathew is not standing next to Seema
(d) 30.43
2. There are two people standing beween Lohit
and the engineer
6 SOLVED PAPER – 2019
T = 40 20 2 ; then Q P
S
P 32 16 2 ; S 38 19 2 ; T 40 20 2
P E S T 32 10 38 40 120.
5.
R T
Female
Male
T is son/daughters of S.
S is the aunt of T is false.
10. Let, Side of smaller square = x cm
Side of larger square = (x + 5) cm
Area of small square = x2
H denotes height of the pole from the bottom. Area of larger square = (x + 5)2
Given,
In ABC ;
(x + 5)2 = 4x2 [ Area of the large square is four
H times the area of the smaller square]
tan 45 5
6 x 2 25 10x 4x 2
H
3x2 10x 25 0
1 5 tan 45 1
6 5
x 5; which is not possible.
H 6 H 30 m. 3
5
Side if larger square 10 cm.
7. Given,
S E T -2
Play only cricket 105
Play only hockey 70 ANSWERS
Play only football 50 1. (b) 2. (c) 3. (b) 4. (a) 5. (c)
Play both cricket and hockey 25
6. (b) 7. (d) 8. (c) 9. (a) 10. (c)
Play both hockey and football 15
8 SOLVED PAPER – 2019
Total USD Paid = (6.5 200 + 7.67 200) Total number of literates in 2011
= (1300 + 1534) = 2834 = 30 + 30 = 60
Remaining amount left Percentage increase in the total number of
= 3000 – 2834 166 (approx) literates from 2001–2011
9 . According to the given data;
60 46 14
= 100 % 100 % 30.43%
46 46
S E T -5
ANSWERS
1. (c) 2. (d) 3. (a) 4. (c) 5. (d)
EXPLANATIONS
Mathew must be an Engineer.
3. From prime factorisation of
10. Let us assume, that population = 100 [2001 – 2011]
165 = 3 5 11
165 = 15 11
From panel (a); From panel (b & c) So the two numbers whose sum is 26 and
Percentage of male Number of males multiplication is 165 is 15 and 11
literates in 2001 = 50% in 2001 = 60 Now, difference = 15 – 11 = 4
Percentage of female Number of females 5. Since, we know that
literates in 2001 = 40% in 2001 = 40
360 = 12 hours
Percentage of male Number of males
literates in 2011 = 60% in 2011 = 50 12
10 = hours
Percentage of female Number of females 360
literates in 2011 = 60% in 2011 = 50
12 225
225 = hours
360
Number of male literates in 2001
= 7.5 hours
50
= 60 30 6. Let the first part = x
100
Number of male literates in 2011 So second part = 100,000x
60 According to question,
= 50 30
100 [0.1x + 0.12(100,000 – x)] – [0.12x + 0.1(1000,000 – x)]
Number of female literates in 2001
= 120
= 40 40 16
100 0.2x – 0.24x + 1200 – 1000 = 120
Total number of literates in 2001 –0.04x = –1880
= 30 + 16 = 46 x = 47000
Number of female literates in 2011
x 47000 47
60 =
= 50 30 100,000 x 53000 53
100
SOLVED PAPER – 2019 11
8.
15%
P = 20% = 120% 1.15 600
0%
P = 25% = 150 150
0%
R = 25% = 150 150
From the above family chart of ‘M’ and ‘N’, we 40%
S = 25% = 150 210
can conclude ‘W’ is the wife of P.
T = 5% = 30 ?
Now, employees at skill level T in 2016,
= 690 – (120 +150 + 150 + 210)
= 60
SOLVED PAPER – 2020
GENERAL APTITUDE
S E T -1 (c) Modern educationists want to develop and
Q 1 – Q 5 carry one mark each assess the theory of multiple intelligences.
1. It is a common criticism that most of the (d) Modern educationists insist that the teaching
academicians live in their _____., so, they are curriculum and evaluation needs to be
not aware of the real life challenges. multidimensional
(a) homes (b) ivory towers 7. Five friends P, Q, R, S and T went camping. At
(c) glass palaces (d) big flats night, they had to sleep in a row inside the tent.
P, Q and T refused to sleep next to R since he
2. His hunger for reading is insatiable. He reads snored loudly. P and S wanted to avoid Q as he
indiscriminately. He is most certainly a/an _____ usually hugged people in sleep.
reader. Assuming everyone was satisfied with the
(a) all-round (b) precocious sleeping arrangements. What is the order in
(c) voracious (d) wise which they slept?
3. Select the word that fits the analogy: (a) RSPTQ
Fuse : Fusion :: Use: _______. (b) SPRTQ
(a) Usage (b) User (c) QRSPT
(c) Uses (d) Usion (d) QTSPR
8. Insert seven numbers between 2 and 34, such
4. If 0, 1, 2, ......., 7, 8, 9 are coded as O, P, Q ....., V,
that the resulting sequence including 2 and 34 is
W, X, then 45 will be coded as _______. an arithmatic progression. The sum of these
(a) TS (b) ST inserted seven numbers is _______.
(c) SS (d) SU (a) 120 (b) 124
5. The sum of two positive numbers is 100. After (c) 126 (d) 130
subtracting 5 from each number, the product of 9. The unit’s place in 26591749110016 is _______.
the resulting numbers is 0. One of the original (a) 1 (b) 3
numbers is _______ .
(c) 6 (d) 9
(a) 80 (b) 85 10. The total expenditure of a family, on different
(c) 90 (d) 95 activities in a month, is shown in the pie-chart.
Q 6 – Q 10 carry two marks each The extra money spent on education as compared
to transport (in percent) is _______ .
6. The American psychologist Howard Gardener
expounds that human intelligence can be
subcategorised into multiple kinds, in such a way Health (5%)
that individuals differ with respect to their 5%
relative competence in each kind. Based on this Transport (10%)
10%
theory, modern educationists in sist on Household Items (15%)
25%
prescribing multi-dimensional curriculum and
15% Eduction (15%)
evaluation parameters that enable development
and assessment of multiple intelligences. Leisure (10%)
20% 15%
Which of the following statements can be inferred House rent (20%)
from the given text? 10%
Other (25%)
(a) Howard Gardner insists that the teaching
curriculum and evaluation needs to be multi-
dimensional
(a) 5 (b) 33.3
(b) Howard Gardner wants to develop and assess
(c) 50 (d) 100
the theory of multiple intelligences
2 SOLVED PAPER – 2020
200 100
200
300 200
1 b 2
2 2 n 2
0 (a) (b a ) (b a)
d 200 4
0 100 (b) [(b2 – a2) – n (b – a)2]
100 e
2 2 n 2
(c) (b a ) (b a)
100 4
f
(d) [(b 2 a 2 ) n(b a)2 ]
SOLVED PAPER – 2020 3
9. Two straight lines are drawn perpendicular to 5. If P, Q, R, S are four individuals, how many teams
each other in X-Y plane. If and are the acute of size exceeding one can be formed, with Q as a
angles the straight lines make with the X-axis, member?
then + is ________. (a) 5 (b) 6
(a) 60° (b) 90° (c) 7 (d) 8
(c) 120° (d) 180° Q 6 – Q 10 Carry two marks each.
10. The total revenue of a company during 2014-2018 6. Non-performing assets (NPAs) of a bank in India
is shown in the bar graph. If the total expenditure is defined as an asset, which remains unpaid by
of the company in each year is 500 million rupees, a borrower for a certain period of time in terms
then the aggregate profit or loss (in percentage) of interest. Principal, or both. Reserve Bank of
on the total expenditure of the company during
India (RBI) has changed the definition of NPA
2014-2018 is ________.
thrice during 1993 - 2004 in terms of holding
period of loans. The holding period was reduced
900
by one quarter each time. In 1993, the holding
800
Revenue (in millions rupe es)
(d) Growed
1
4. I do not think you know the case well enough to
1/m
have opinions. Having said that, I agree with your X
other point.
What does the phrase “having said that” mean in m
X
the given text? 0 x
1
(a) as opposed to what I have said
(b) y
(b) despite what I have said
(c) in addition to what I have said
1
(d) contrary to what I have said
m
5. Define [x] as the greatest integer less than or X
equal to x, for each x (–, ). If y = [x], then
area under y for x [1, 4] is ________ . 1/m
X
(a) 1 (b) 3
0 x
(c) 4 (d) 6 1
6 SOLVED PAPER – 2020
(c) y 10. The bar graph shows the data of the students who
appeared and passed in an examination for four
schools P, Q, R and S. The average of success
1
rates (in percentage) of these four schools is _____.
Performance of schools P, Q, R and S
m
X 800
700
700
1/m
0 X x 600
600
1
Number of studens
500
(d) y 500
455
400 Appeared
400
330 Passed
300 280
240
1 200
1/m 100
X
0
School P School Q School R School S
m
X
(a) 58.5% (b) 58.8%
(c) 59.0% (d) 59.3%
0 x
1
EXPLANATIONS ANSWERS
1. “Within” Preposition that is used to express 1. (c) 2. (b) 3. (c) 4. (b) 5. (c)
something that occurs inside a particular period 6. (b) 7. (c) 8. (b) 9. (a) 10. (c)
of time.
Here, because Sentance is in ‘present perfect
continuous tense’ “For is”used when we talk about
EXPLANATIONS
a period of time. 1. Subject + Verb + agreement
2. Here, we have to note that the confunction used
is ‘but’. ‘But’ is used for introducing an idea which is are
contrasts with the statement that has been 2. Prohibited from parking means "to officially refuse
already said. allow something"
3. (i) Flyer : Relation is Verb : Noun 3. Do : Undo : Trust : Distrust
(ii) One who cooks is a cook, and one who flies The obvious analogical relationship is do and undo
any aircraft is a flyer. are antonyms just as trust and distrust are
5. • Cost of 1-a-C-2 : 200 + 200 + 100 + 100 + antonyms.
100 = 700 4. Plunged means plummet especially (prices,
• Cost of 1-f-e-2 : 100 + 100 + 100 + 200 + temperature, etc.) to decrease suddenly and quickly.
200 = 700 5. When team size is more than 1 then
• Cost of 1-b-2 : 300 + 100 + 200 = 600 Case 1 : When team size = 2, then Q is fixed,
• Cost of 1-f-b-2 : 100 + 100 + 100 +200 = 500 only 41 member to be selected among P, R & S
Hence, 1-f-b-2 is having minimum cost. 3C = 3 ways
1
6. According to given passage GST is imposed on Case 2 : When team size = 3, then Q is fixed,
the usage or supply of goods and services hence, only 2 members to be selected among P, R & S
option (d) is correct. 3C
2 = 3 ways
7. As given that
Case 3 : When team size = 4, then Q is fixed,
P = 31, Q = 9 = 32, R = 27 = 33, S = 81 = 34,
only 3 members to be selected among P, R & S
T = 243 = 35 3C = 1 ways
Hence, Q + S = 9 + 81 = 90. 3
Case 4 : When team size 5; That Q Not possible.
8. Unpainted area = Area of annular ring – Area of
n blue color circles So, total possible ways = 3 + 3 + 1 = 7 ways
Area of annular ring = b2 – a2 Hence, possible combinations are :
i.e. (b2 – a2) PQ, RQ, SQ, PRQ, PSQ, RSQ, PQRS.
b a
2 6. Based on the given paragraph the holding period
Area of 1 blue circle = of loans was reduced by one quater in 2004.
2
2
Therefore after the third revision equal to one
b a quarter days of the year.
Hence, area of n blue circles = n
2 1
= 360 = 90 days.
Unpainted area = (b2 a 2 ) (b a)2 4
4 7. 26 23,22 18,17,16 11,10,9,8
9. Since, and are acute angles hence it will form Z, WV, RQP, KJIH
the interior angles of straight lines + = 180° –
90° = 90° YX UTS ONML GFED
8 SOLVED PAPER – 2020
r
2
r
Average number of students in school (Q) in 5 years 6. The correct statement about crowd funding is to
4000 7000 8000 7000 5000 fund raised through voluntary contributions on
= = 6200 web-based platforms.
5
The difference of the number of students enrolled
in school P and Q = (6200 – 4600) = 1600 7. P, Q, R, S
The ratio of the average number of the student
enrolled in school P to the average of the diffrence
of the number of student enrolled in school P and 8. S = 8 + 88 + 888 + ... n
Q then, 8
4600 23 = (9 99 999 .....n)
Required ratio = = 23 : 8 9
1600 8
8 1
= (10 1) (1002 1) ... (10n 1)
9
S E T -5
8 1
= (10 1002 ...100 n ) n
ANSWERS 9
8 10 (10 n 1) 8
1. (c) 2. (a) 3. (c) 4. (b) = 9 n
10 1 9
5. (d) 6. (d) 7. (d) 8. (d)
9. (a) 10. (c) 80 8
= (10n 1) n
81 9
EXPLANATIONS 9. Graph of y = xm and y = x1/m for m > 1
E F Number of candidtes
3 who passed the exam
10. Success rate =
Number of appeared
C candidates
2 D
A
So, Average success rate,
1 B
280 330 455 240
0 500 600 700 400 = 0.59 or 59%
X 4
1 2 3 4
The total area, A = (1 × 1) + (2 × 1) + (3 × 1) Correct option is (c).
=1+2+3=6
SOLVED PAPER – 2021
GENERAL APTITUDE
SET 1 (a) 2 (b) 4
Q. 1 to Q. 5 : Carry One Mark Each. (c) 6 (d) 8
1. Getting to the top is __________ than staying on Q. 6 to Q. 10 : Carry Two Marks Each.
top.
6. Statement: Either P marries Q or X marries Y
(a) more easy (b) much easy Among the options below, the logical NEGATION
(c) easiest (d) easier of the above statement is:
2. (a) P does not marry Q and X marries Y.
(b) Neither P marries Q nor X marries Y.
(c) X does not marry Y and P marries Q.
(d) P marries Q and X marries Y.
7. Consider two rectangular sheets, Sheet M and
Sheet N of dimensions 6 cm x 4 cm each.
Folding operation 1: The sheet is folded into half
by joining the short edges of the current shape.
Folding operation 2: The sheet is folded into half
by joining the long edges of the current shape.
The mirror image of the above text about the x- Folding operation 1 is carried out on Sheet M
axis is
three times.
(a) (b) Folding operation 2 is carried out on Sheet N
three times.
(c) (d)
The ratio of perimeters of the final folded shape
3. In a company, 35% of the employees drink coffee, of Sheet N to the final folded shape of Sheet M is
40% of the employees drink tea and 10% of the _______.
employees drink both tea and coffee. What % of (a) 13 : 7 (b) 3 : 2
employees drink neither tea nor coffee?
(c) 7 : 5 (d) 5 : 13
(a) 15 (b) 25
8.
(c) 35 (d) 40
4. and are two operators on numbers p and q
such that
p2 q 2 p2
p q and p q ;
pq q
If x y 2 2, then x
y
(a) (b) y
2
Five line segments of equal lengths, PR, PS, QS,
QT and RT are used to form a star as shown in
3y
(c) (d) 2y the figure above.
2
The value of , in degrees, is
5. Four persons P, Q, R and S are to be seated in a (a) 36 (b) 45
row, all facing the same direction, but not (c) 72 (d) 108
necessarily in the same order. P and R cannot 9. A function, , is defined by
sit adjacent to each other. S should be seated to
(p q)2 , if p q,
the right of Q. The number of distinct seating (p, q)
arrangements possible is: p q, if p q.
2 SOLVED PAPER 2021
The value of the expression 3. Two identical cube shaped dice each with faces
( (–3 2),(–2 3) numbered 1 to 6 are rolled simultaneously. The
is :
( ( 2 1)) probability that an even number is rolled out on
(a) –1 (b) 0 each dice is:
16 (a) 1/36 (b) 1/12
(c) (d) 16
3 (c) 1/8 (d) 1/4
10. Humans have the ability to construct worlds
entirely in their minds, which don't exist in the 4. and are two operators on numbers p and q
physical world. So far as we know, no other such that p q = p – q, and p q = p × q
species possesses this ability. This skill is so
important that we have different words to refer Then, (9 (6 7)) (7 (6 5))
to its different flavors, such as imagination,
(a) 40 (b) –26
invention and innovation.
Based on the above passage, which one of the (c) –33 (d) –40
following is TRUE? 5. Four persons P, Q, R and S are to be seated in a
(a) No species possess the ability to construct row. R should not be seated at the second position
worlds in their minds. from the left end of the row. The number of distinct
(b) The terms imagination, invention and seating arrangements possible is:
innovation refer to unrelated skills.
(a) 6 (b) 9
(c) We do not know of any species other than
humans who possess the ability to construct (c) 18 (d) 24
mental worlds. Q. 6 – Q. 10 carry two marks each
(d) Imagination, invention and innovation are
unrelated to the ability to construct mental 6. On a planar field, you travelled 3 units East from
worlds. a point O. Next you travelled 4 units South to
SET 2 arrive at point P. Then you travelled from P in
the North-East direction such that you arrive at
Q. 1 – Q. 5 carry one mark each
a point that is 6 units East of point O. Next, you
1. (i) Arun and Aparna are here. travelled in the North-West direction, so that you
(ii) Arun and Aparna is here. arrive at point Q that is 8 units North of point P.
(iii)Arun’s families is here. The distance of point Q to point O, in the same
(iv) Arun’s family is here. units, should be _____
Which of the above sentences are grammatically (a) 3 (b) 4
CORRECT? (c) 5 (d) 6
(a) (i) and (ii) (b) (i) and (iv) 7. The author said, “Musicians rehearse before their
(c) (ii) and (iv) (d) (iii) and (iv) concerts. Actors rehearse their roles before the
2. opening of a new play. On the other hand, I find
it strange that many public speakers think they
can just walk on to the stage and start speaking.
In my opinion, it is no less important for public
speakers to rehearse their talks.”
Based on the above passage, which one of the
following is TRUE?
(a) The author is of the opinion that rehearsing is
The mirror image of the above text about the x-
important for musicians, actors and public
axis is
speakers.
(a) (b) (b) The author is of the opinion that rehearsing is
less important for public speakers than for
(c) (d)
musicians and actors.
SOLVED PAPER 2021 3
9.
7.
(c)
(d)
In the figure shown above, each inside square is
formed by joining the midpoints of the sides of
the next larger square. The area of the smallest
3. For a regular polygon having 10 sides, the interior square (shaded) as shown, in cm2 is :
angle between the sides of the polygon, in (a) 12.50
degrees, is : (b) 6.25
(a) 396 (b) 324 (c) 3.125
(c) 216 (d) 144 (d) 1.5625
4. Which one of the following numbers is exactly 8. Let X be a continuous random variable denoting
divisible by (1113 + 1)? the temperature measured. The range of
temperature is [0, 100] degree Celsius and let
(a) 1126 + 1 (b) 1133 + 1 the probability density function of X be f(x) = 0.01
(c) 1139 – 1 (d) 1152 – 1 for 0 X 100.
5. Oasis is to sand as island is to The mean of X is ____________
Which one of the following options maintains a (a) (b) 5.0
similar logical relation in the above sentence? (c) (d)
6 SOLVED PAPER 2021
9.
The number of students passing or failing in an exam for a particular subject are presented in the bar
chart above. Students who pass the exam cannot appear for the exam again. Students who fail the exam
in the first attempt must appear for the exam in the following year. Students always pass the exam in
their second attempt.
The number of students who took the exam for the first time in the year 2 and the year 3 respectively, are
________
(a) 65 and 53 (b) 60 and 50 (c) 55 and 53 (d) 55 and 48
10. Seven cars P, Q, R, S, T, U and V are parked in a Which of the above sentences are grammatically
row not necessarily in that order. The cars T CORRECT?
and U should be parked next to each other. The (a) (i) and (ii) (b) (i) and (iii)
cars S and V also should be parked next to each (c) (iii) and (iv) (d) (ii) and (iv)
other, whereas P and Q cannot be parked next 2. Ms. X came out of a building through its front
door to find her shadow due to the morning sun
to each other. Q and S must be parked next to
falling to her right side with the building to her
each other. R is parked to the immediate right back. From this, it can be inferred that building
of V. T is parked to the left of U. is facing ____________
Based on the above statements, the only (a) North (b) East
INCORRECT option given below is : (c) West (d) South
(a) There are two cars parked in between Q and 3. In the above figure, O is the center of the circle
and, M and N lie on the circle.
V.
The area of the right triangle MON is 50 cm2.
(b) Q and R are not parked together.
What is the area of the circle in cm2 ?
(c) V is the only car parked in between S and R. M
(d) Car P is parked at the extreme end.
SET 5
Q. 1 to Q. 5 : Carry One Mark Each. N
1. Consider the following sentences: O
(i) After his surgery, Raja hardly could walk.
(ii) After his surgery, Raja could barely walk.
(iii)After his surgery, Raja barely could walk. (a) 2 (b) 50
(iv) After his surgery, Raja could hardly walk. (c) 75 (d) 100
SOLVED PAPER 2021 7
seated at either end of the row. P should not be Based on the above statements and conclusions,
seated adjacent to S. R is to be seated at the second which one of the following options is CORRECT?
position from the left end of the row. The number (a) Only conclusion I is correct
of distinct seating arrangements possible is: (b) Only conclusion II is correct
(a) 2 (b) 3 (c) Neither conclusion I nor II is correct
(c) 4 (d) 5 (d) Both conclusions I and II are correct
2. Consider the following sentences: 7. A box contains 15 blue balls and 45 black balls. If
(i) The number of candidates who appear for the 2 balls are selec ted randomly, with out
GATE examination is staggering. replacement, the probability of an outcome in
(ii) A number of candidates from my class are which the first selected is a blue ball and the
appearing for the GATE examination. second selected is a black ball, is ____________
(iii)The number of candidates who appear for the 3 45
(a) (b)
GATE examination are staggering. 16 236
(iv) A number of candidates from my class is 1 3
appearing for the GATE examination. (c) (d)
4 4
Which of the above sentences are grammatically 8.
CORRECT?
(a) (i) and (ii) (b) (i) and (iii)
(c) (ii) and (iii) (d) (ii) and (iv)
3. A digital watch X beeps every 30 seconds while
watch Y beeps every 32 seconds. They beeped
together at 10 AM. The ratio of the area of the inscribed circle to
The immediate next time that they will beep the area of the circumscribed circle of an
together is ___________ equilateral triangle is __________
(a) 10.08 AM (b) 10.42 AM 1 1
(a) (b)
(c) 11.00 AM (d) 10.00 PM 8 6
1 1
4. If 2; 3; 5; 10, (c) (d)
4 2
Then, the value of , is: 9. Consider a square sheet of side 1 unit. The sheet
2
SET 7
Q. 1 to Q. 5 : Carry One Mark Each.
1. The ratio of boys to girls in a class is 7 to 3.
Among the options below, an acceptable value (c) (d)
for the total number of students in the class is :
(a) 21 (b) 37
(c) 50 (d) 73 5. ____ is to surgery as writer is to ______.
2. A polygon is convex if, for every pair of points, P Which one of the following options maintains a
and Q belonging to the polygon, the line segment similar logical relation in the above sentence ?
PQ lies completely inside or on the polygon. (a) Plan, outline (b) Hospital, library
Which one of the following is NOT a convex (c) Doctor, book (d) Medicine, grammar
polygon ? Q. 6 to Q. 10 : Carry Two Marks Each
6. We have 2 rectangular sheets of paper, M and N,
of dimensions 6 cm × 1 cm each. Sheet M is rolled
to form an open cylinder by bringing the short
(a) (b) edges of the sheet together. Sheet N is cut into
equal square patches an assembled to form the
largest possible closed cube. Assuming the ends
of the cylinder are closed, the ratio of the volume
of the cylinder to that of the cube is ______.
(c) (d)
3
(a) (b)
2
3. Consider the following sentences :
9
(i) Everybody in the class is prepared for the (c) (d) 3
exam.
(ii) Babu invited Danish to his home because he 7. Items Cost Profit Marked Price
enjoys playing chess. (`) % (`)
Which of the following is the CORRECT P 5,400 -- 5,860
observation about the above two sentences ? Q --- 25 10,000
(a) (i) is grammatically correct and (ii) is
unambiguous Details of prices of two items P and Q are
presented in the above table. The ratio of cost of
(b) (i) is grammatically incorrect and (ii) is
unambiguous item P to cost of item Q is 3:4. Discount is
calculated as the difference between the marked
(c) (i) is grammatically correct and (ii) is
price and the selling price. The profit percentage
ambiguous
is calculated as the ratio of the difference between
(d) (i) is grammatically incorrect and (ii) is
selling price and cost, to the cost
ambiguous
Selling price Cost
Pr ofit % 100 .
Cost
4.
The discount on item Q, as a percentage of its
marked price, is _____.
A circular sheet of paper is folded along the lines
in the directions shown. The paper, after being (a) 25 (b) 12.5
punched in the final folded state as shown and (c) 10 (d) 5
unfolded in the reverse order of folding, will look 8. There are five bags each containing identical sets
like _____. of ten distinct chocolates. One chocolate is picked
form each bag.
The probability that at least two chocolates are
identical is ________.
(a) 0.3024 (b) 0.4235
(a) (b)
(c) 0.6976 (d) 0.8125
10 SOLVED PAPER 2021
7.
A jigsaw puzzle has 2 pieces. One of the pieces is shown above. Which one of the given options for the
missing piece when assembled will form a rectangle ? The piece can be moved, rotated or flipped to
assemble with the above piece.
(a) (b)
(c) (d)
8. The number of students in three classes is in the ratio 3 : 13 : 6. If 18 students are added to each class, the
ratio changes to 15 : 35 : 21.
The total number of students in all the three classes in the beginning was :
(a) 22 (b) 66 (c) 88 (d) 110
9.
350
296 300
300
240
250 210
200
200
150
100
100
50
0
Year 1 Year 2 Year 3
Number of units Not Profit
The number of units of a product sold in three different years and the respective net profits are presented
in the figure above. The cost/unit in Year 3 was ´1, which was half the cost/unit in Year 2. The cost/unit in
Year 3 was one-third of the cost/unit in Year 1. Taxes were paid on the selling price at 10%, 13% and 15%
respectively for the three years. Net profit is calculated as the difference between the selling price and the
sum of cost and taxes paid in that year.
The ratio of the selling price in Year 2 to the selling price in Year 3 is ____.
(a) 4 : 3 (b) 1 : 1 (c) 3 : 4 (d) 1 : 2
12 SOLVED PAPER 2021
10. Six students P, Q, R, S, T and U, with distinct heights, compare their heights and make the following
observations.
Observation I: S is taller than R.
Observation II: Q is the shortest of all.
Observation III: U is taller than only one student.
Observation IV: T is taller than S but is not the tallest.
The number of students that are taller than R is the same as the number of students shorter than ____.
(a) T (b) R (c) S (d) P
ANSWERS
Set 1
1. (d) 2. (b) 3. (c) 4. (b) 5. (c) 6. (b) 7. (a) 8. (a) 9. (b) 10. (c)
Set 2
1. (b) 2. (b) 3. (d) 4. (d) 5. (c) 6. (c) 7. (a) 8. (b) 9. (c) 10. (d)
Set 3
1. (c) 2. (b) 3. (c) 4. (c) 5. (b) 6. (c) 7. (d) 8. (a) 9. (c) 10. (a)
Set 4
1. (c) 2. (c) 3. (d) 4. (d) 5. (c) 6. (d) 7. (c) 8. (d) 9. (d) 10. (a)
Set 5
1. (d) 2. (d) 3. (d) 4. (d) 5. (c) 6. (c) 7. (a) 8. (c) 9. (c) 10. (d)
Set 6
1. (b) 2. (a) 3. (a) 4. (b) 5. (d) 6. (c) 7. (b) 8. (c) 9. (b) 10. (d)
Set 7
1. (c) 2. (a) 3. (c) 4. (a) 5. (c) 6. (c) 7. (c) 8. (c) 9. (d) 10. (d)
Set 8
1. (a) 2. (b) 3. (b) 4. (b) 5. (c) 6. (c) 7. (a) 8. (c) 9. (a) 10. (c)
SOLVED PAPER 2021 13
EXPLANATIONS
SET 1 4. As given that,
1. When the comparison is between two things we p2 q 2 p2
use the second degree of the adjective the degree p q and p q
from of easy are : easy easier easiest. pq q
Hence, comparative degree is easier. x2 y2
Now, L.H.S x y
2. The mirror image of TRIANGLE about the X-axis x.y
is
22
R.H.S x y 2 2
TRIANGLE 2
So, L.H.S = R.H.S
x 2 y 2 22
Top to Bottom will interchange (or) xy 2
symmetrical about X-axis. x2 + y2 = 2xy x2 + y2 – 2xy = 0
(x – y)2 = 0 x – y = 0 x y
Coffee C(35) T(40) Tea
5. According to the given information,
Possible way are
RQSP
25% 10 30%
% PQSR
RQPS
35% PQRS
QRSP
QPSR
C Coffee Percentage of employees drink Total number of ways = 2 + 2 + 2 = 6
Coffee = 35%
The number of distinct seating arrangements
T Tea Percentage of employees drink Tea possible is 6.
= 40% 6. As Given statement:
Percentage of employees drink both Tea Either P marries Q or X marries Y.
and Coffee = 10%
The logical negation of the Either P marries Q
Percentage of employees drink neither or X marries Y
Tea and Coffee = 100% – n(TUE) “above statement is Neither P marries Q nor X
= 100% – (25% + 10% + 30%) marries Y.”
= 35% Hence option (b) is correct.
7. Shortest side of folding:
2.
a a a
4 , 6 , 8 is integer,,
3
24 24
144 3
sq. units
4
SET 3
1. As given that,
r 2
Area of shaded region from the figure is . Current population of a city = 11, 02, 500
4
Increasing rate = 5%/Annum
Shaded area from the figure: The population 2 years ago = ?
r 2 1 Let us Consider, the population 2 years ago = x
= 2 r r
4 2 Using compound interest formula, we have
n
R
A = P 1
100
r 2 1
2 2
5
Total area = r 2 11,02,500 = x 1
100
As we know that, 100 100
11,02,500 × =x
105 105
Shaded area
Required probability = x 10,00,000
Totalarea
Hence, the population 2 years ago = 10,00,000.
p q
r 2 1 2. As Given that, 3
= 1
2 q p
r2
2
By Squaring on both sides;
16 SOLVED PAPER 2021
2
p q
=9
q p Triangle:
2 2
p q
2 = 9
q2 p2 2
1
p2 q2
Hence, 7
q2 p2
1
3. Add
1
Add
2
2 1
Add Add
4
3 1
Add
Radius of base circle(r) = 1 unit
6
Height of cone(h) = 1 unit
Add
5 2
Volume of cone = r h = 1 1 = unit3
3 3 3
Hence, the least number of squares added = 6.
9. From the bar graph, it is clear that except
4. Nostalgia (means excessively sentimental
Thursday, total 6 days one of the students spent
yearning for return to or of some past period) is
a minimum of 10% more than the other student.
to anticipation. (means visualization of a future
event or state) 10. In regular hexagon, each interior angle = 120°,
5. Wake pastense is woke and third form is woken So, if we cut corners of an equilateral triangle,
then the removed triangles are also equilateral
so, the verb forms are:
triangles.
V1 V2 V3
Wake Woke Woken 7
6. Green Green 1
Purple Purple
3 2
Black 4 6
Black 8 5 9
Let, side of regular hexagon = x units
Then, side of original equilateral triangle = 3x
No black is purple Some black are purple
3 3 2
7. The passage deduces that computers are present Area of regular hexagon = x
every where and they have both positive and 2
negative effects on humans.
3
8. Square: Area of equilateral triangle = (3x)2
4
3 3 2 3
2 Required ratio = x : (3x)2
1 2 4
3 3 2 9 3 2
= x : x
2 4
1
Solid core formed by revolving tringle about its 3
= 1: =2:3
short edge. 2
SOLVED PAPER 2021 17
SET 4 100
1. Here the subject of the verb is ‘who’ because we E[x] = x.f(x)dx 0 x 100
0
can not use ‘Which’ for people, ‘Who’ is the only 100
word which acts as a subject for the verb.
100
x2
2. If the square sheet is folded along the dotted line
= 0
x.(0.01)dx 0.01
2 0
by upon rotation, then the folded sheet will look 1
like as follows. = 100 100 50
200
9. From the given bar graph:
In year 2 : Total number of students who took
the exam = 60 + 5 = 65
Failed students = 0
But who appeared the exam for 1st time = 65 – 10
= 55.
In year 3 : Total number of students who took
3. As given that, the exam = 50 + 3 = 53
n = 10 Failed students = 5
As we know that
But who took the exam for 1st exam = 53 –5 = 48
The sum of interior angles for a regular
Hence option (d) is correct.
polygon = (n – 2) × 180°
10. According to the given information, possible
= (10 –2) × 180 = 8 × 180° = 1440
arrange:
8 180
So, Each interior side angle = 144.
10 T U P Q S V R
4. (an – bn) is divisible by (a + b), only when ‘n’ is
even U Q S V R P I
So, we take the option (d). I U O S V R P
1152 –1 = (1126)2 – 12
Statements : (b), (c), (d) are true and
= (1126 + 1) (1126 – 1)
Statements (A) is incorrect.
= (1126 + 1) [(1113)2 – (12)]
Hence, there is only one car between Q and V.
= (1126 + 1) [(1113) – 1] (1113 + 1)
1113 + 1 is divisor of 1152 – 1. SET 5
5. Oasis is a water pool amidst sand just as island is 1. When you say you could hardly do some thing
a piece of land amidst water. you are emphasing that it is very difficult for you
6. Deprivation of sleep during exams will have to do it.
negative effect on exams. So, considerable sleep Could + Adverb + Verb (I).
is necessary to well in exams. Hence, option (d) is the correct answer.
7. According to the question. 2. According to the questions:
Total number of squares = 6
Now, by using Pythagoras theorem, in each
square, we can find side length.
5
Side of smallest square becomes =
8
5 5 25
So, Area of smallest square = 3.125
8 8 8
8. As we know that, From the above figure, it is clear that building
is facing towards South .
E(X) = Mean of X = x.f x x dx Hence, option (d) is the correct answer.
18 SOLVED PAPER 2021
3. Given that,
20
Area of right triangle MON = 50 Number of executives in C5 = 10,000 × = 2000
100
According to given figure:
Number of management degree holders among
1 1
× ON × OM = 50 the executives in C2 = 500 × = 100
2 5
[ ON = OM = radius of circle = r(say)] And, Number of management degree holders
1 9
× r × r = 50 among the executives in C5 = 2000 × = 1800
2 10
r2 = 100 So, the total number of management degree
Area of circle with radius r = r2 holders among the executives in companies C5
and C5 together = 100 + 1800 = 1900.
= (100) = 100.
Hence, option (c) is the correct answer.
Hence option (d) is the correct answer.
10. As given that
4. As given expression =
Total block is (Q_R)
2 3(( 4 2) 4)
Total number of ways,
= +2 – 3 + ((4 2) × 4)
1 2 1 2 1
= 2 – 3 + (2 × 4) = 2 – 3 + 8 = 7 4 choices
Q R S
Hence option (d) is the correct answer.
(OR)
5. One can clearly infer that eating leafy vegetables
undoubtely lead a healthy style. 2 1 S
2 choices
7. From the options, the figure in option (b) is the Q R
missing piece. If the figure is rotated clockwise S2 1
then it will form a rectangular with the given 2 choices
Q R
piece in the picture.
(OR)
8. For farm P:
2 1
Number of hens = 65 2 choices
R Q S
Number of ducks = 91
Number of goats = 169
(OR)
For farm Q:
S R 2 Q 1
5 2 choices
Number of hens = 416 × = 65
32 (OR)
14 2 2 1
Number of ducks = 416 × = 182 4 choices
32 S R Q
Hence, total number of distinct seating
13
Number of goats = 416 × = 169 arrangement possible = 4 + 2 + 2 + 2 + 2 + 4 = 16.
32
SET 6
Total number of hens = 65 + 65 = 130
1. Positive arrangements are
Total number of ducks = 91 + 182 = 273
S R P T Q (OR) Q R P T Q (OR) S R T P Q
Total number of goats = 169 + 169 = 338
Total number of combinations possible is 3.
The new ratio of hens: ducks: goats = 130: 273: 338
2. (i) The first sentence informs us about the
= 10: 21:26.
number with is singular.
9. As given that,
(ii) A number of means several, some.
Total number of executives, across all companies
= 10,000 The second sentence takes about the students
which is plural word.
5 3. As given that,
Number of executives in C2 = 10,000 × = 500
100
Watch x beeps every 30 sec
SOLVED PAPER 2021 19
1
=1:4 =
4
9. Area of each face of the final folded shape.
(Right angle triangle)
6. E Entrepreneurs 1 1 11
= Box weight
W Wealthy 2 2 22
R Risk seekers
1
Statement-1 Statement-2 square units
8
F
W
W
E E
Total balls =
10. Due to worst pandemic the air travel industry is Volume of a cube = (1)3
facing a crisis is nothing but restarting the idea So, request ratio
from first sentence. 9
SET 7 9
1. Let us consider the number of boys = B 1
And total number of students = S Hence, option (c) is the correct answer.
So, we have 7. As given that
B 7 CPp = 5400
S B 3 MPP = 5860
7S 4
B Cost of Q(CPq) * CPp = 7200
10 3
Thus, S must be multiple of 10. Hence, answer is
50. Let, selling price ( S.P) of Q = S
2. If we choose two points P, Q such that P is the
top-left corner and Q is the bottom left corner
then the line joining P, Q will not lie on the
25 SPq 7200 * 100
7200
polygon. Hence, option (a) is the correct answer. SPq = 9000
3. Every body is prepared means they are ready for Discount on Q = MPq – SPq = 10000 – 9000 = 1000
the exam, the second sentence does not clearly
state whether babu will play with Danish or not. 1000
Discount% 100 = 10%
Babu loves playing chess the does not means 10000
danish knows how to play chess. So statement Discount on SPq i.e. 1000 is 10% of marked price
(i) is grammatically correct and (MPq) of Q i.e. 10, 000.
(ii) is ambiguous. So, answer is 10%.
Statement (ii) is ambiguous because we do not Hence, option (c) is the correct answer.
know who enjoys playing chees, Babu or Danish. 8. P (exactly 1 =) Probability that all the picked
chocolates are different:
So, statement (i) is grammatically correct.
Hence, option (c) is the correct answer.
10 * 9 * 8 * 7 * 6 0.3024
4. Unforhed in reverse order of folding, it’s look 105
like option (a)
Probability that at least two of the picked
chocolates are identical:
1 – (Probability that all the picked chocolates
are different)
1 – 0.3024 = 0.6976
M 1 cm
1 cm Case I Case II
6 cm
Micro Micro
P
N 1 cm 1 cm B B P
10. Superficially is the deciding key word which means 6. Listening to music improves exercise
apparently or seemingly. performance and reduces discomfort. Music as
positive effort only on few students during
As AOM are not addressing the core problems, learning.
they are superficial. 7. When the piece can be moved, rotated or flipped
Superficial means shallow, cursory mean lacking to assemble with the given piece is option (a)
in depth or solidity. Superficial implies a concern 8. The total number of students in three classes in
only with surface aspects or obvious features. A the beginning.
superficial analysis of the problem shallow is more Let us consider:
generally derogatory in implying lack of depth 3a + 13a + 6a = 22a = n
in knowledge, reasoning, emotions, or character. after addition of 18 student total number of
students.
Hence, option (d) is the correct answer.
15b + 35b + 21b = 71b
SET 8
3a + 18 + 13a + 18 + 6a + 18 = 22a + 54
1. ‘As well as’ is used in positive degree.
22a + 54 = 71b
2. The folded sheet is look like option (b).
n + 54 = 71b
n = 71b – 54
Let (b = 2)
So, n = 71 × 2 – 54
= 88
9. Let us consider, the selling price in year ‘2’ be ‘x’.
3. B Net profit = Selling price – (Cost + Taxes)
1 296 = x – (400 + 0.13x)
296 = 0.87x – 400
1 696
A 1 x 800
0.87
BX 1 So, selling price in year ‘2’ = Rs.800
cos In year 3, total cost = 300 × 1 = Rs.300
AB 3
Taxes paid = 0.15% of selling price.
4. As we know that
Now, let us consider, the selling price in year 3
(a – b)2 = a2 + b2 – 2ab be ‘y’
As given expression 210 = y – (300 + 0.15y)
2 2 210 = (0.85y – 300)
1 3
x x x 2 510
2 2 y= 600
0.85
So,
Selling price in year ‘3’ = 600
2
1 1 9 3 x 800 4
x 2 2(x) – x 2 – 2(x). So, .
2 2 4 2 y 600 3
= [x + 2] 10. I: S > R
II: Q < P, R, S, T, U
1 9
x 2 x x 2 3x [x 2] From III & IV observation
4 4
P>T>S>R>U>Q
[–2 + 2x] = [x + 2]
The number of students that are taller than ‘R’ is
x=4 same as the number of students shorter than S.
5. Pen is to write just as knife is to cut.